Vous êtes sur la page 1sur 294

PartI

1.VALIAOv. REPUBLIC
GRNo. 170757, 28November 2011
PERALTA,J.:

FACTS:

PetitionersValiao filed forthe registration their parcel of land


(Lot No. 2372) whichthey acquired fromtheir uncle in 1947. The
Solicitor General and privateoppositors, Zafra andYusay, opposed said
registration on the followinggrounds: (1) the land appliedfor has not
been declared alienable anddisposable; (2) there was no proof thatthe
applicants had in open, continuous,exclusive and notorious possession
of theland in question since June 12, 1945or prior thereto. In the
petitionersdefense, they alleged that the possessionof their uncle
since 1916 until 1966had been open, continuous anduninterrupted;
thus, converting the said land intoa private land.

Issue:Whether or not Lot No. 2372 is an alienable land of the


PublicDomain under theRegalian Doctrine.

HELD:
No,Lot No. 2372 isinalienable.
Underthe Regalian doctrine,all lands of the public domain
belong to the State. Alllands not appearing tobe clearly within private
ownership are presumed tobelong to the State.Accordingly, public
lands not shown to have beenreclassified or released asalienable
agricultural land or alienated to a privateperson by the State,remain
part of the inalienable public domain.

Propertyof the public domainis beyond the commerce of man


and not susceptible ofprivate appropriation andacquisitive prescription.
Occupation thereof in theconcept of owner no matterhow long cannot
ripen into ownership and beregistered as a title. To prove thatthe
<a></a>landsubject of an application for registration isalienable, the
applicant mustestablish the existence of a positive act of
thegovernment, such as apresidential proclamation or an executive
order. Theapplicant may also securea certification from the
government that the<a></a>landclaimed to have been possessed for
the required numberof y<a>earsis alienable and disposable.</a>

Inthis case, no such evidencewas offered by the petitioners to


show that the <a></a>landinquestion has been classified as alienable
and disposable<a></a>landof the <a></a>public domain. In the
absenceofincontrovertible evidence to prove that the subject property
isalreadyclassified as alienable and disposable, Lot 2372 is still an
inalienablepublicdomain.

2. Secretary of the DENR vs. YapG.R. 167707 October 8, 2008Sacay et.


al and Secretary of DENRG.R. 173775 Reyes, R.T., J.:Facts: There are
two consolidated cases in this petition.G.R. 167707On November 10,
1978, then President Ferdinand Marcos issued Proclamation No. 1801
declaring Boracay Island, among other islands, caves and peninsulas in
the Philippines, as tourist zones and marine reserves under the
administration of the Philippine Tourism Authority (PTA). President
Marcos later approved the issuance of PTA Circular 3-82 dated
September 3, 1982, to implement Proclamation No. 1801. Claiming
that Proclamation No. 1801 and PTA Circular No 3-82 precluded them
from filing an application for judicial confirmation of imperfect title or
survey of land for titling purposes, respondents-claimants Mayor Jose
S. Yap, Jr., Libertad Talapian, Mila Y. Sumndad, and Aniceto Yap filed a
petition for declaratory relief with the RTC in Kalibo, Aklan.In their
petition, respondents-claimants alleged that Proclamation No. 1801
and PTA Circular No. 3-82 raised doubts on their right to secure titles
over their occupied lands.They declared that they themselves, or
through their predecessors-in-interest, had been in open, continuous,
exclusive, and notorious possession and occupation in Boracay since
June 12, 1945, or earlier since time immemorial. On July 14, 1999, the
RTC rendered a decision in favor of respondents-claimants, The RTC
upheld respondents-claimants right to have their occupied lands titled

in their name. It ruled that neither Proclamation No. 1801 nor PTA
Circular No. 3-82 mentioned that lands in Boracay were inalienable or
could not be the subject of disposition. The Circular itself recognized
private ownership of lands. The trial court cited Sections 87 and 53 of
the Public Land Act as basis for acknowledging private ownership of
lands in Boracay and that only those forested areas in public lands
were declared as part of the forest reserve. On December 9, 2004, the
appellate court affirmed in toto the RTC decision, The CA held that
respondents-claimants could not be prejudiced by a declaration that
the lands they occupied since time immemorial were part of a forest
reserve.G.R. 173775On May 22, 2006, during the pendency of G.R. No.
167707, President Gloria Macapagal-Arroyo issued Proclamation No.
1064 classifying Boracay Island into four hundred (400) hectares of
reserved forest land (protection purposes) and six hundred twentyeight and 96/100 (628.96) hectares of agricultural land (alienable and
disposable). The Proclamation likewise provided for a fifteen-meter
buffer zone on each side of the centerline of roads and trails, reserved
for right-of-way and which shall form part of the area reserved for
forest land protection purposes. Petitioners-claimants contended that
there is no need for a proclamation reclassifying Boracay into
agricultural land. Being classified as neither mineral nor timber land,
the island is deemed agricultural pursuant to the Philippine Bill of 1902
and Act No. 926, known as the first Public Land Act. Thus, their
possession in the concept of owner for the required period entitled
them to judicial confirmation of imperfect title. Issue: Whether private
claimants (respondents-claimants in G.R. No. 167707 and petitionersclaimants in G.R. No. 173775) have a right to secure titles over their
occupied portions in Boracay.Held:NO. The Regalian Doctrine dictates
that all lands of the public domain belong to the State, that the State is
the source of any asserted right to ownership of land and charged with
the conservation of such patrimony. The doctrine has been consistently
adopted under the 1935, 1973, and 1987 Constitutions. All lands not
otherwise appearing to be clearly within private ownership are
presumed to belong to the State. Thus, all lands that have not been
acquired from the government, either by purchase or by grant, belong
to the State as part of the inalienable public domain. Necessarily, it is
up to the State to determine if lands of the public domain will be
disposed of for private ownership. The government, as the agent of the
state, is possessed of the plenary power as the persona in law to
determine who shall be the favored recipients of public lands, as well
as under what terms they may be granted such privilege, not excluding
the placing of obstacles in the way of their exercise of what otherwise
would be ordinary acts of ownership.A positive act declaring land as
alienable and disposable is required. In keeping with the presumption
of State ownership, the Court has time and again emphasized that
there must be a positive act of the government, such as an official

proclamation, declassifying inalienable public land into disposable land


for agricultural or other purposes. In fact, Section 8 of CA No. 141 limits
alienable or disposable lands only to those lands which have been
officially delimited and classified. In the case at bar, no such
proclamation, executive order, administrative action, report, statute, or
certification was presented to the Court. The records are bereft of
evidence showing that, prior to 2006, the portions of Boracay occupied
by private claimants were subject of a government proclamation that
the land is alienable and disposable. Absent such well-nigh
incontrovertible evidence, the Court cannot accept the submission that
lands occupied by private claimants were already open to disposition
before 2006.Matters of land classification or reclassification cannot be
assumed.

3(Gina)

FERNANDAARBIAS - versus - THE REPUBLIC OF THE PHILIPPINES,


G.R. No. 173808

September 17, 2008

CHICO-NAZARIO, J

Facts:
On 12March 1993, Lourdes T. Jardeleza (Jardeleza) executed a Deed of
Absolute Sale sellingto petitioner a parcel of unregistered land situated
at Poblacion, Estancia,Iloilo, for the sum of P33,000.00. According to
theDeed, the subject property was residential and consisted of 600
square meters,more or less.

Three years thereafter, on 17 June 1996,petitioner filed with the RTC a


verified Application for Registration of Titleover the subject property.
She attached to her application the Tracing Clothwith Blue Print copies,
the Deed of Absolute Sale involving the subjectproperty, the Surveyors

Certification, the Technical Description of the land,and Declaration of


Real Property in the name of petitioner and her spouse Jimmy.

Thereafter,the respondent filed an Opposition to petitioners


application for registrationof the subject property. In its Brief,
respondent questioned
the failure of petitioner to prove
thecontinuous, open, exclusive and notorious possession by
theirpredecessor-in-interest.

On 26June 2000, the RTC granted the petitioners application for


registration in thiswise:

As tothe issue that muniments of title and/or tax declarations and


taxreceipts/payments do not constitute competent and sufficient
evidence ofownership, the same cannot hold through (sic) anymore it
appearing from therecords that the muniments of titles as presented
by the herein applicant arecoupled with open, adverse and continuous
possession in the concept of anowner, hence, it can be given greater
weight in support of the claim forownership. The [herein petitioner] is a
private individual who is qualifiedunder the law being a purchaser in
good faith and for value. The adverse, open,continuous and exclusive
possession of the land in the concept of owner of the[petitioner]
started as early as in 1992 when their predecessors in interestfrom
Lourdes Jardeleza then to the herein [petitioner] without any
disturbanceof their possession as well as claim of ownership. Hence,
uninterruptedpossession and claim of ownership has ripen (sic) into an
incontrovertibleproof in favor of the [petitioner].

It wason the issue of possession, however, that the Court of Appeals


digressed fromthe ruling of the RTC. The appellate court found that
other than petitionersown general statements and tax declarations, no
other evidence was presented toprove her possession of the subject
property for the period required by law.Likewise, petitioner failed to
establish the classification of the subjectproperty as an alienable and
disposable land of the public domain. The Court ofAppeals reversed
and set aside the decision of the RTC.

Issue:
whether ornot the petitioner has proved that the subject land is an
alienable land.

Held:
No. Underthe Regalian doctrine, all lands of the public domain belong
to the State, andthe State is the source of any asserted right to
ownership of land and chargedwith the conservation of such patrimony.
This same doctrine also states thatall lands not otherwise appearing to
be clearly within private ownership arepresumed to belong to the
State. Hence, the burden of proof in overcoming thepresumption of
State ownership of lands of the public domain is on the personapplying
for registration. The applicant must show that the land subject of
theapplication is alienable or disposable.

Section14, paragraph 1 of Presidential Decree No. 1529 states the


requirementsnecessary for a judicial confirmation of imperfect title to
be issued. Theapplicant for registration under said statutory provision
must specificallyprove: 1) possession of the subject land under a bona
fide claim of ownershipfrom 12 June 1945 or earlier; and 2) the
classification of the land as analienable and disposable land of the
public domain.

In thecase at bar, petitioner miserably failed to discharge the burden of


proofimposed on her by the law. The Deed of Sale did not state the
duration of timeduring which the vendor (or her predecessors-ininterest) possessed the subjectproperty in the concept of an owner.
Well-settled is the rule that taxdeclarations and receipts are not
conclusive evidence of ownership or of theright to possess land when
not supported by any other evidence. The Survey Planand Technical
Description of the subject property submitted by petitionermerely plot
the location, area and boundaries thereof.

For theoriginal registration of title, the applicant must overcome the


presumptionthat the land sought to be rehgtered forms part of the
public domain. Unlesspublic land is shown to have been reclassified or

alienated to a private personby the State, it remains part of the


inalienable public domain. Indeed,occupation thereof in the concept of
owner, no matter how long, cannot ripeninto ownership and be
registered as a title. To overcome such presumption,incontrovertible
evidence must be shown by the applicant. Absent such evidence,the
land sought to be registered remains inalienable.

4(Zhon)

Case Number I.A.d.Alcantara vs. DENR


G.R. No. 161881, July 31, 2008

Facts:Petitioner is a lessee under FLGLA No. 542 , issued by the DENR,


of 923 hectares of public forest land (subject land) located in the
vicinity of Sitio Lanton, Barrio Apopong, General Santos City, since
1983.

The subject land, is being claimed as the ancestral land of the


indigenous B'laan and Maguindanao people, who maintain that they
and their predecessors have been cultivating, possessing and
occupying it since time immemorial.

On April 10, 1990, private respondents, representing the B'laan and


Maguindanao tribes, filed a complaint against petitioner before the
Commission on the Settlement of Land Problems (COSLAP) seeking the
cancellation of FLGLA No. 542 and the reversion of the land to the
indigenous communities.

In 1993, despite the pendency of the COSLAP case, and despite


opposition from private respondents, petitioner was able to renew
FLGLA No. 542 when it expired that year. The renewal given to
petitioner was for another 25 years, or until December 31, 2018.

On October 29, 1997, Congress passed Republic Act No. 8371, or the
Indigenous People's Rights Act (IPRA), which was intended to recognize
andpromote all the rights of the country's Indigenous Cultural
Communities/Indigenous Peoples (ICCs/IPs) within the framework of the
Constitution.

On August 3, 1998, the COSLAP rendered its decision, recommending


to DENR Sec., inter alia, cancellation of respondents renewed FLGLA
No. 542 and placing in possession the subject land to the petitioners in
order to start cultivation and plant crops for their food. COSLAP denied
petitioners MR. Upon appeal, the CA affirmed said decision in toto.

Petitioner filed a petition for review on certiorari before SC, docketed as


G.R. No. 145838. SC upheld the CA and the COSLAP, holding that a)
COSLAP had jurisdiction to decide the case; b) FLGLA No. 542 was
issued in violation of the law, and; c) the 923 hectares covered by
FLGLA No. 542 were ancestral land of the private respondents.

On July 29, 2002, the COSLAP issued a writ of execution of its decision,
ordering the DENR Sec. to implement the August 3, 1998 decision as
affirmed by SC. DENR Sec. ordered the DENR Director of Region 12
toconduct a review and investigation of FLGLA No. 542. The latter
found violations by petitioner of the terms of the FLGLA. On August 15,
2002, Sec. Alvarez cancelled FLGLA No. 542. Petitioners MR was
denied.

On November 26, 2002, Community Environment and Natural


Resources Officer (CENRO) Andrew B. Patricio Jr. sent a letter to
petitioner, advising him to vacate and remove all improvements in the
area within 10 days from receipt of the letter.

On November 27, 2002, CENRO Patricio issued an Installation Order,


which directed the immediate installation and occupation of the area
by the private respondents indigenous communities.

Upon appeal, the CA dismissed petitioners appeal, holding that issue


was already decided by SC in GR No. G.R. No. 145838. Hence, this
instant petition via Rule 45.

Issue: w/n petitioner has residual rights over subject property until the
expiration of FLGLA 542 (Dec. 31, 2018)

Held:No. Petitioner has no residual rights to speak of because such


right presupposes an existing right. In the case at bar, FLGLA No. 542
which is supposed to be the source of petitioners right was already
cancelled and held contrary to law.

Further, FLGLA 542, even if not cancelled, does not confer any right.
FLGLA No. 542 is a mere license or privilege granted by the State to
petitioner for the use or exploitation of natural resources and public
lands over which the State has sovereign ownership under the Regalian
Doctrine. Like timber or mining licenses, FLGA is a mere permit which,
by executive action, can be revoked, rescinded, cancelled, amended or
modified, whenever public welfare or public interest so requires. Thus,
a privilege or license is not in the nature of a contract that enjoys
protection under the due process and non-impairment clauses of the
Constitution. In cases in which the license or privilege is in conflict with
the people's welfare, the former must yield to the supremacy of the
latter, as well as to the police power of the State.

5(May Ann)

6(Rhea B.)
REPUBLIC OF THE PHILIPPINES VS CANDY MAKER, INC.GR.NO. 163766,
June 22, 2006Facts:On April, 29, 1999, Antonia, Eladia, and Felisa, all
surnamed Cruz, executed a Deed of Absolute Sale in favor of Candy
Maker, Inc. for a parcel of land located below the reglementary lake
elevation of 12.50m, about 900 meters away the Laguna de Bay.

Candy Maker, Inc. as applicant, filed an application with the MTC of


Taytay, Rizal for registration of its alleged title over the lot.The CENRO
of Antipolo City declared the land to fall within the alienable and
disposable zone. On the other hand, the Land Registration Authority
recommended the exclusion of lot no. 3138-B on the ground that it is a
legal easement and intended for public use, hence, inalienable and
indisposable. On July 2001, the Republic of thePhilippines, the LLDA
filed its opposition which alleged that the lot subject of the application
for registration may not be alienated and disposed since it is
considered part of the Laguna Lake Bed, a public land within, its
jurisdiction.Issue:Whether or not the property subject of the amended
application is alienable and disposable property of the State, and if so,
whether respondent adduced the requisite quantum of evidence to
prove its ownership over the property.Ruling:The property subject of
this application was alienable and disposable public agricultural land.
However, respondent failed to prove that it possesses registrable title
over the property. The statute of limitations with regard to public
agricultural lands does not operate against the statute unless the
occupant proves possession and occupation of the same after a claim
ofownership for the required number of years to constitute a grant
from the State. A mere casual cultivation of portions of the land by the
claimant does not constitute sufficient basis for a claim of ownership,
such possession is not exclusive and notorious as to give rise to
presumptive grant from the state. In light of the foregoing, the petition
of the Republic of the Philippines is granted.

7(Angel)

8. Cario vs Insular Government, 41 Phil 935

Facts: Carino, an Igorot, applied for the registration of a certain land


situated in the town of Baguio, Province of Benguet. He and his
ancestors had held the land as owners for more than 50 years, which
he inherited under Igorot customs. There was no document of title
issued for the land when he applied for registration. The government
contends that the land in question belonged to the state. Under the
Spanish Law, all lands belonged to the Spanish Crown except those

with permit private titles. Moreover, there is no prescription against the


Crown.

Issue: Wether the land in question belonged to the Spanish Crown


under the Regalian Doctrine? Held: No. Law and justice require that the
applicant should be granted title to his land.

The United States Supreme Court, through Justice Holmes declared:

It might perhaps, be proper and sufficient to say that when, as far as


testimony or memory goes, the land has been held by individuals
under a claim of private ownership, it will be presumed to have been
held in the same way from before the Spanish conquest, and never to
have been public land.

There is an existence of native title to land, or ownership of land by


Filipinos by virtue of possession under a claim of ownership since time
immemorial and independent of any grant from the Spanish Crown, as
an exception to the theory of jura regalia.

9. Cruzvs Secretary ofDENR


GR.No. 135385, Dec.6, 2000

FACTS:
Petitioners Isagani CruzandCesar Europa filed a suit for
prohibition and mandamus as citizensandtaxpayers, assailing the
constitutionality of certain provisions ofRepublicAct No. 8371,
otherwise known as the Indigenous Peoples Rights Act of1997(IPRA)
and
its
implementing
rules
and
regulations
(IRR).
The
petitionersassailcertain provisions of the IPRA and its IRR on the ground

that theseamount toan unlawful deprivation of the States ownership


over lands of thepublicdomain as well as minerals and other natural
resources therein, inviolation ofthe regalian doctrine embodied in
section 2, Article XII of the Constitution.

ISSUE:
Dotheprovisions of IPRA contravene the Constitution?

HELD:
No,theprovisions of IPRA do not contravene the Constitution. Examining
theIPRA,there is nothing in the law that grants to the ICCs/IPs
ownership overthenatural resources within their ancestral domain.
Ownership over thenaturalresources in the ancestral domains remains
with the State and therightsgranted by the IPRA to the ICCs/IPs over
the natural resources intheirancestral domains merely gives them, as
owners and occupants of the landonwhich the resources are found, the
right to the small scale utilizationofthese resources, and at the same
time, a priority in their largescaledevelopment and exploitation.

Additionally,ancestrallands and ancestral domains are not part of the


lands of the publicdomain. Theyare private lands and belong to the
ICCs/IPs by native title,which is a conceptof private land title that
existed irrespective of any royalgrant from theState. However, the right
of ownership and possession by theICCs/IPs of theirancestral domains
is a limited form of ownership and does notinclude the rightto alienate
the same.

10(Lou)

Secretaryof DENR vs. Yap (568 SCRA 164) G.R. No. 167707
(8October 2008)

Facts:On November 10, 1978, then President Ferdinand Marcos


issuedProclamation No. 1801 declaring Boracay Island, amongother
islands, caves and peninsulas in the Philippines, as touristzones and
marine reserves under the administration of the PhilippineTourism
Authority (PTA). President Marcos later approved the issuanceof PTA
Circular 3-82 dated September 3, 1982, toimplement Proclamation
No. 1801.

On July14, 1999, the RTC rendered a decision in favor ofrespondentsclaimants.


The RTC upheldrespondents-claimants right to have their occupied
lands titled in their name.The Office of the Solicitor General moved for
reconsideration but itsmotion was denied. The Republic then appealed
to the CA. On December9, 2004, the appellate court affirmed in toto
the RTC decision.The CA held that respondents-claimants could not be
prejudiced by a declarationthat the lands they occupied since time
immemorial were part of a forestreserve. Again, the OSG sought
reconsideration but it was similarlydenied. Hence, the present petition
under Rule 45.

G.R.No. 173775

On May22, 2006, during the pendency of G.R. No. 167707, President


GloriaMacapagal-Arroyo issued Proclamation No. 1064[26] classifying
BoracayIsland into four hundred (400) hectares of reserved forest land
(protectionpurposes) and six hundred twenty-eight and 96/100
(628.96) hectares ofagricultural land (alienable and disposable). The
Proclamation likewiseprovided for a fifteen-meter buffer zone on each
side of the center line ofroads and trails, reserved for right-of-way and
which shall form part of thearea reserved for forest land protection
purposes.

On November21, 2006, this Court ordered the consolidation of the two


petitions (G.R.No. 167707 and G.R. no. 173775) as they principally
involve thesame issues on the land classification of Boracay Island.

Respondents-claimants allegedthat they themselves, or through


their predecessors-in-interest, had been inopen, continuous, exclusive,
and notorious possession and occupation in Boracaysince June 12,
1945, or earlier since time immemorial. They declared theirlands for
tax purposes and paid realty taxes on them. Also they alleged thatthe
proclamation and its implementing circulars did not place Boracay
beyondthe commerce of men and was classified as a tourist zone, it
was susceptible ofprivate ownership. Under Section 48(b) of
Commonwealth Act (CA) No. 141,otherwise known as the Public Land
Act, they had the right to have the lotsregistered in their names
through judicial confirmation of imperfect titles.

TheOffice of the Solicitor General (OSG), opposed the


petition:Because Boracay Island was an unclassified land ofthe
public domain. It formed part of the mass of lands classified aspublic
forest, which was not available for disposition pursuant to Section
3(a)of Presidential Decree (PD) No. 705 or the Revised Forestry Code,
asamended. The OSG maintained that respondents-claimants
relianceon PD No. 1801 and PTA Circular No. 3-82 was
misplaced. Theirright to judicial confirmation of title was governed by
CA No. 141 and PD No.705. Since Boracay Island had not been
classified asalienable and disposable, whatever possession they had
cannot ripen intoownership.

Issue/s:W/N Proclamation No. 1801 and


82converted Boracay into an agricultural land.

PTA

Circular

No.

3-

Held:No.

Privateclaimants cannot rely on Proclamation No. 1801 as


basis for judicialconfirmation of imperfect title. The
proclamation did not convert Boracayinto an agricultural land.
ProclamationNo. 1801 or PTA Circular No. 3-82 did not convert the
whole ofBoracay into an agricultural land. There is nothing in the law or
theCircular which made Boracay Island an agricultural land.
Thereference in Circular No. 3-82 to private lands and areas declared

as alienableand disposable does not by itself classify the entire island


asagricultural.

Therefore,Proclamation No. 1801 cannot be deemed the positive


act needed toclassify Boracay Island as alienable and
disposable land. IfPresident Marcos intended to classify the island as
alienable and disposable orforest, or both, he would have identified the
specific limits of each, asPresident Arroyo did in Proclamation No. 1064.
This was not done inProclamation No. 1801.

TheWhereas clauses of Proclamation No. 1801 also explain the


rationale behind thedeclaration of Boracay Island, together with other
islands, caves andpeninsulas in the Philippines, as a tourist zone and
marine reserve to beadministered by the PTA to ensure the
concentrated efforts of thepublic and private sectors in the
development of the areas tourism potentialwith due regard for
ecological balance in the marine environment. Simplyput, the
proclamation is aimed at administering the islands for tourismand
ecological purposes. It does not address the areas alienability.

Privateclaimants continued possession under Act No. 926 does


not create a presumptionthat the land is alienable.
Privateclaimants also contend that their continued possession of
portionsof Boracay Island for the requisite period of ten (10)
yearsunder Act No. 926 ipso facto converted the island into
privateownership. Hence, they may apply for a title in their name.

Exceptfor lands already covered by existing titles, Boracay was


an unclassified landof the public domain prior to Proclamation
No. 1064. Such unclassifiedlands are considered public forest
under PD No. 705. TheDENR and the National Mapping and Resource
InformationAuthority certify that Boracay Island is an unclassifiedland
of the public domain.

WHEREFORE,judgment is rendered as follows:

1.The petition for certiorari in G.R. No. 167707 is GRANTED andthe


Court of Appeals Decision in CA-G.R. CV No. 71118 REVERSED AND
SETASIDE.

2.The petition for certiorari in G.R. No. 173775 is DISMISSED forlack of


merit.

SOORDERED.

Syllabus: Natural Resources; Public Lands; Regalian Doctrine;


Words and Phrases;The Regalian Doctrine dictates that all
lands of the public domain belong tothe State, that the State
is the source of any asserted right to ownership ofland and
charged with the conservation of such patrimony, a
doctrineconsistently adopted under the 1935, 1973 and 1987
Constitution; Prior toProclamation No.1064 of May 22, 2006,
Boracay Island had never been expresslyand administratively
classfified under any of the grand divisions of land.Boracay
was an unclassified land of the public Domain. --- The1935
Constitution
classified
lands
of
the
public
domain
into
agricultural,forest or timber. Meanwhile, the 1973 Constitution
provided the followingdivisions: agricultural, industrial or commercial,
residential, resettlement,mineral, timber or forest and grazing lands
and such other classes as may beprovided by law, giving the
government great leeway for classification. Thenthe 1987 Constitution
reverted to the 1935 Constitution classification with oneaddition:
national parks. Of these, only agricultural lands may be alienated.Prior
to Proclamation No. 1064 of May 22, 2006, Boracay Island had never
beenexpressly and administratively classified under any of these grand
divisions.Boracay was an unclassified land of the public domain. The
Regalian Doctrinedictates that all lands of the public domain belong to
the State, That theState is the source of any asserted right to
ownership of land and charged withthe conservation of such patrimony.
The doctrine has been consistently adoptedunder the 1935, 1973, and
1987 Constitutions.

TheRegalian doctrine was first introduced in the Philippines


through the Laws ofthe Indies and the Royal Cedulas, Which
laid the foundation that "alllands that were not acquired from
the Government, either by purchase or bygrant, belong to the
public domain." -- Ourpresent land law traces its roots to the
Regalian Doctrine. Upon the Spanishconquest of the Philippines,
ownership of all lands, territories andpossessions in the Philippines
passed to the Spanish Crown. The Regaliandoctrine was first
introduced in the Philippines through the laws of the Indiesand the
Royal Cedulas, which laid the foundation that "all lands that werenot
acquired from the Government, either by purchase or by grant, belong
to thepublic domain."

NOTES:
Under the Regalian doctrine, all lands not otherwise appearing to be
clearlywithin private ownership are presumed to belong to the State -unless publicland is shown to have been reclassified as alienable or
disposable to a privateperson by the State, it remains part of the
inalienable public domain. (Republicvs. Jacob, 495 SCRA
529(2006))
Underthe Regalian doctrine, all lands not otherwise appearing to be
clearly withinprivate ownership are presumed to belong to the State -applicants forconfirmation of imperfect title must therefore, prove the
following: (a) thatthe lands of the public domain; and, (b) that they
have been in open,continuous, exclusive, and notorious possession and
occupation of the sameunder a bona fide claim of ownership either
since time immemorial or since June12,1945 lies in the presumption
that the land applied for pertains to theState, and that the occupants
or possessor claim an interest thereon only byvirtue of their imperfect
title as continuous open and notorious possession.(Republicvs.
Candy Maker, Inc., 492 SCRA 272 (2006))

Historyof Public Land Dispostition

TheRegalian principle traces its roots:

Uponthe Spanish conquest of the Philippines, ownership of all


lands,territories and possessions in thePhilippines passed to the
Spanish Crown.The Regalian doctrine was first introduced in the
Philippines throughthe Laws of the Indies and the Royal Cedulas, which
laid thefoundation that all lands that were not acquired from the
Government, either bypurchase or by grant, belong to the public
domain.

The Lawsof the Indies was followed by the Ley Hipotecaria orthe
Mortgage Law of 1893. The Spanish Mortgage Law providedfor the
systematic registration of titles and deeds as well as possessoryclaims.

TheRoyal Decree of 1894 or the Maura Law partly amended the


SpanishMortgage Law and the Laws of the Indies. It established
possessoryinformation as the method of legalizing possession of
vacant Crown land, undercertain conditions which were set forth in said
decree.

UnderSection 393 of the Maura Law, an informacion posesoria


orpossessory information title, when duly inscribed in the Registry
ofProperty, is converted into a title of ownership only after the lapse of
twenty(20) years of uninterrupted possession which must be actual,
public, andadverse, from the date of its inscription. However,
possessory informationtitle had to be perfected one year after the
promulgation of the Maura Law, oruntil April 17, 1895. Otherwise, the
lands would revert to the State.

Privateownership of land could only be founded on royal concessions


which tookvarious forms, namely: (1) titulo real or royal grant;(2)
concesion especial or special grant; (3) composicioncon el estado or
adjustment title; (4) titulo de compra ortitle by purchase; and (5)
informacion posesoria or possessoryinformation title.

AmericanRule:

The Philippine Bill of 1902: lands of the public domain in the


Philippine Islands were classified into three (3) grand divisions, to
wit: agricultural, mineral, and timber or forest lands. The act
provided for, among others, the disposal of mineral lands by
means of absolute grant (freehold system) and by lease
(leasehold system). It also provided the definition by exclusion of
agricultural public lands.

On February 1, 1903, the Philippine Legislature passed Act No.


496, otherwise known as the Land Registration Act. The act
established a system of registration by which recorded title
becomes absolute, indefeasible, and imprescriptible. This is
known as the Torrens system.

On October 7, 1903, the Philippine Commission passed Act No.


926, which was the first Public Land Act. The Act introduced the
homestead system and made provisions for judicial and
administrative confirmation of imperfect titles and for the sale or
lease of public lands. It permitted corporations regardless of the
nationality of persons owning the controlling stock to lease or
purchase lands of the public domain. Under the Act, open,
continuous, exclusive, and notorious possession and occupation
of agricultural lands for the next ten (10) years preceding July 26,
1904 was sufficient for judicial confirmation of imperfect title.

On November 29, 1919, Act No. 926 was superseded by Act


No. 2874, otherwise known as the second Public Land Act. This
new, more comprehensive law limited the exploitation of
agricultural lands to Filipinos and Americans and citizens of other
countries which gave Filipinos the same privileges. For judicial
confirmation of title, possession and occupation en concepto
dueo since time immemorial, or since July 26, 1894, was
required.

After the passage of the 1935 Constitution, CA No.


141 amended Act No. 2874 on December 1, 1936. To this day,
CA No. 141, as amended, remains as the existing general law
governing the classification and disposition of lands of the public

domain other than timber and mineral lands, and privately


owned lands which reverted to the State.

Section 48(b) of CA No. 141 retained the requirement under Act


No. 2874 of possession and occupation of lands of the public
domain since time immemorial or since July 26, 1894. However,
this provision was superseded by Republic Act (RA) No. 1942,
which provided for a simple thirty-year prescriptive period for
judicial confirmation of imperfect title. The provision was last
amended by PD No. 1073, which now provides for possession
and occupation of the land applied for since June 12, 1945, or
earlier.

The issuance of PD No. 892 on February 16, 1976 discontinued


the use of Spanish titles as evidence in land registration
proceedings. Under the decree, all holders of Spanish titles or
grants should apply for registration of their lands under Act No.
496 within six (6) months from the effectivity of the decree on
February 16, 1976. Thereafter, the recording of all unregistered
lands shall be governed by Section 194 of the Revised
Administrative Code, as amended by Act No. 3344.

On June 11, 1978, Act No. 496 was amended and updated by PD
No. 1529, known as the Property Registration Decree. It was
enacted to codify the various laws relative to registration of
property. It governs registration of lands under the Torrens
system as well as unregistered lands, including chattel
mortgages.

11(Jess)
CENTRAL MINDANAO UNIVERSITY, G.R. No. 184869
Represented by Officer-In-Charge
Dr. Rodrigo L. Malunhao,

Petitioner
Vs
THE HONORABLE EXECUTIVE SECRETARY, THE HONORABLE
SECRETARY OF THE DEPARTMENT OF ENVIRONMENT AND
NATURAL
RESOURCES,
THE
CHAIRPERSON
AND
COMMISSIONERS
OF
THE
NATIONAL
COMMISSION
ON
INDIGENOUS PEOPLES, and THE LEAD CONVENOR OF THE
NATIONAL ANTI-POVERTY COMMISSION,
Respondents.

Facts
Central Mindanao University is a chartered educational institution
owned and run by the State. In 1958, the Presidential Proclamation
476, reserving 3401 hectares of lands of the public domain in Musuan ,
Bukidnon, as school site for CMU. CMU obtained title in its name over
3,080 hectares of those lands under OCT. Meanwhile, the government
distributed the remaining untitled lands to several tribes.
Forty five years later, President GMA issued PP 310 that takes 670
hectares from the CMUs registered lands for distribution belonging to
tribes in Barangay Musuan, Maramag, Bukidnon

On April 3, 2003, however, CMU filed a petition for prohibition against


respondents Executive Secretary, Secretary of the Department of
Environment and Natural Resources, Chairperson and Commissioner of
the National Commission on Indigenous Peoples (NCIP), and Lead
Convenor of the National Anti-Poverty Commission (collectively,
NCIP, et al) before the Regional Trial Court (RTC) of Malaybalay City
(Branch 9), seeking to stop the implementation of Presidential
Proclamation 310 and have it declared unconstitutional.

The NCIP, et al moved to dismiss the case on the ground of lack of


jurisdiction of the Malaybalay RTC over the action, pointing out that
since the act sought to be enjoined relates to an official act of the
Executive Department done in Manila, jurisdiction lies with the Manila
RTC. The Malaybalay RTC denied the motion, however, and proceeded

to hear CMUs application for preliminary injunction. Meanwhile,


respondents NCIP, et al moved for partial reconsideration of the RTCs
order denying their motion to dismiss.

On October 27, 2003, after hearing the preliminary injunction incident,


the RTC issued a resolution granting NCIP, et als motion for partial
reconsideration and dismissed CMUs action for lack of jurisdiction. Still,
the RTC ruled that Presidential Proclamation 310 was constitutional,
being a valid State act. The RTC said that the ultimate owner of the
lands is the State and that CMU merely held the same in its
behalf. CMU filed a motion for reconsideration of the resolution but the
RTC denied the same on April 19, 2004.This prompted CMU to appeal
the RTCs dismissal order to the Court of Appeals (CA) Mindanao
Station.[2]

In a March 14, 2008 decision,[4] the CA dismissed CMUs appeal for lack
of jurisdiction, ruling that CMUs recourse should have been a petition
for review on certiorarifiled directly with this Court, because it raised
pure questions lawbearing mainly on the constitutionality of
Presidential Proclamation 310. The CA added that whether the trial
court can decide the merits of the case based solely on the hearings of
the motion to dismiss and the application for injunction is also a pure
question of law.

Issue
Whether or not Presidential Proclamation 310 can distribute the land of
CMU given by PP 476 in 1958
Held: No
It did not matter that it was President Arroyo who, in this case,
attempted by proclamation to appropriate the lands for distribution to
indigenous peoples and cultural communities. As already stated, the
lands by their character have become inalienable from the moment
President Garcia dedicated them for CMUs use in scientific and
technological research in the field of agriculture. They have ceased to
be alienable public lands.

Besides, when Congress enacted the Indigenous Peoples Rights Act


(IPRA) or Republic Act 8371in 1997, it provided in Section 56 that
property rights within the ancestral domains already existing and/or
vested upon its effectivity shall be recognized and respected. In this
case, ownership over the subject lands had been vested in CMU as
early as 1958. Consequently, transferring the lands in 2003 to the
indigenous peoples around the area is not in accord with the IPRA.

12(Diane)

HEIRSOF MARIO MALABANAN, (Represented by Sally A.


Malabanan), Petitioners,vs. REPUBLIC OF THE PHILIPPINES,
Respondent.
G.R.No. 179987 April 29, 2009

Thepetition, while unremarkable as to the facts, was accepted by the


Court enbanc in order to provide definitive clarity to the applicability
and scopeof original registration proceedings under Sections 14(1) and
14(2) of theProperty Registration Decree

(PDNo. 1529)

FACTS:
On20 February 1998, Mario Malabanan filed an application for land
registrationcovering a parcel of land identified as Lot9864-A, Cad-452D, Silang Cadastre,situated in Silang Cavite. Malabanan claimed that
he had purchased the propertyfrom Eduardo Velazco, and that he and
his predecessors-in-interest had been in open,notorious, and
continuous adverse and peaceful possession of the land for
morethan thirty (30) years. Malabanan and Aristedes Velazco,
testified atthe hearing. Velazco testified that the property was

originally belonged to a22 hectare property owned by his greatgrandfather, Lino Velazco.
TheRepublic of the Philippines likewise did not present any evidence to
controvertthe application.

Malabananpresented evidence during trial a Certification dated 11 June


2001, issued bythe CENRO-DENR, which stated that the subject
property was verified to bewithin the Alienable or Disposable land
per Land Classification Map No.3013 established under Project No. 20-A
and approved as such under FAO 4-1656 onMarch 15, 1982.

On3 December 2002, the RTC rendered judgment in favor of


Malabanan. The Republicappealed to the Court of Appeals. CA reversed
the decision and dismissed theapplication of Malabanan. Malabanan
died while the case was pending with theCA; it was his heirs who
appealed the decision of the appellate court.

ISSUE:
Whetheror not Malabanan has acquired ownership over the subject
property under Section48(b) of the Public Land Act.

HELD:
No.Evidence of petitioners is insufficient to establish that Malabanan
hasacquired ownership over the subject property under Section 48(b)
of the PublicLand Act. There is no substantive evidence to
establish that Malabanan orpetitioners as his predecessors-ininterest have been in possession of theproperty since 12 June
1945 or earlier.

Neithercan petitioners properly invoke Section 14(2) as basis for


registration. Whilethe subject property was declared as alienable or
disposable in 1982, there isno competent evidence that is no longer
intended for public use service or forthe development of the national

evidence, conformably with Article 422 of theCivil Code. The


classification of the subject property as alienable and disposableland of
the public domain does not change its status as property of the
publicdominion under Article 420(2) of the Civil Code. Thus, it is
insusceptible toacquisition by prescription.

DOCTRINES:
CommonwealthAct No. 141 (Public Land Act) governed the
classification and disposition oflands of the public domain. The
President is authorized, from time to time, toclassify the lands of the
public domain into alienable and disposable, timber,or mineral lands.
Alienable and disposable lands of the public domain arefurther
classified according to their uses into (a) agricultural; (b)residential,
commercial, industrial, or for similar productive purposes;
(c)educational, charitable, or other similar purposes; or (d) reservations
fortown sites and for public and quasi-public uses.

Section11 of the Public Land Act acknowledges that public lands


suitable foragricultural purposes may be disposed of by confirmation
of imperfect orincomplete titles through judicial legalization.

Section48(b) of the Public Land Act, as amended by P.D. No. 1073,


supplies the detailsand unmistakably grants that right, subject to the
requisites stated therein:
Sec.48. The following described citizens of the Philippines, occupying
lands of thepublic domain or claiming to own any such land or an
interest therein, butwhose titles have not been perfected or completed,
may apply to the Court ofFirst Instance of the province where the land
is located for confirmation oftheir claims and the issuance of a
certificate of title therefor, under theLand Registration Act, to wit:
(b)Those who by themselves or through their predecessors in interest
have been in open,continuous, exclusive, and notorious
possession and occupation of alienable anddisposable lands of
the public domain, under a bona fide claim of acquisitionof
ownership, since June 12, 1945, or earlier, immediately
preceding the filingof the application for confirmation of title
except when prevented by waror force majeure. These shall be

conclusively presumed to have performed allthe conditions essential to


a Government grant and shall be entitled to acertificate of title under
the provisions of this chapter.

Twosignificant amendments were introduced by P.D. No. 1073. First,


the termagricultural lands was changed to alienable and disposable
lands of thepublic domain.

UnderSection 9 of the Public Land Act, agricultural lands are a mere


subset oflands of the public domain alienable or open to disposition.
Evidently, alienableand disposable lands of the public domain
are a larger class than onlyagricultural lands.

Second,the length of the requisite possession was changed from


possession for thirty(30) years immediately preceding the filing of the
application to possessionsince June 12, 1945 or earlier.

Itbears further observation that Section 48(b) of Com. Act No, 141 is
virtuallythe same as Section 14(1) of the Property Registration Decree.

SECTION14. Who may apply. The following persons may file in the
proper Court of FirstInstance an application for registration of title to
land,
whether
personallyor
through
their
duly
authorized
representatives:
(1)those who by themselves or through their predecessors-in-interest
have been inopen, continuous, exclusive and notorious possession and
occupation ofalienable and disposable lands of the public domain
under a bona fide claimof ownership since June 12, 1945, or earlier.
Notwithstandingthe passage of the Property Registration Decree and
the inclusion of Section14(1) therein, the Public Land Act has remained
in effect.

Bothlaws commonly refer to persons or their predecessors-in-interest


who have beenin open, continuous, exclusive and notorious
possession and occupation ofalienable and disposable lands of the
public domain under a bona fide claimof ownership since June 12,
1945, or earlier.

Theopening clauses of Section 48 of the Public Land Act and Section 14


of theProperty Registration Decree warrant comparison:

Sec.48 [of the Public Land Act]. The following described citizens of
thePhilippines, occupying lands of the public domain or claiming to
own any suchland or an interest therein, but whose titles have not
been perfected orcompleted, may apply to the Court of First
Instance of the province wherethe land is located for confirmation of
their claims and the issuance of acertificate of title therefor, under the
Land Registration Act, to wit:
xxx
Sec.14 [of the Property Registration Decree]. Who may apply. The
following personsmay file in the proper Court of First Instance an
application forregistration of title to land, whether personally or
through their dulyauthorized representatives:
Itis indeed the Public Land Act that primarily establishes the
substantiveownership of the possessor who has been in possession
of the propertysince 12 June 1945.

Section14(a) of the Property Registration Decree recognizesthe


substantive right granted under Section 48(b) of the Public Land
Act,as well provides the corresponding original registration procedure
for thejudicial confirmation of an imperfect or incomplete title.

Thereis another limitation to the right granted under Section 48(b).


Section 47 ofthe Public Land Act (amended by Rep. Act No. 9176 in
2002) limits theperiod within which one may exercise the right to seek
registration underSection 48.

Section47. The persons specified in the next following section are


hereby grantedtime, not to extend beyond December 31, 2020
within which to avail of thebenefits of this Chapter: Provided,
That this period shall apply only wherethe area applied for does not
exceed twelve (12) hectares: Provided, further,That the several periods
of time designated by the President in accordance withSection FortyFive of this Act shall apply also to the lands comprised in theprovisions
of this Chapter, but this Section shall not be construed asprohibiting
any said persons from acting under this Chapter at any time priorto the
period fixed by the President.
Thesubstantive right granted under Section 48(b) may be availed of
only until 31December 2020. The OSG has adopted the position that
for one to acquire theright to seek registration of an alienable and
disposable land of the publicdomain, it is not enough that the applicant
and his/herpredecessors-in-interest be in possession under a bona fide
claim ofownership since 12 June 1945; the alienable and disposable
character of theproperty must have been declared also as of
12 June 1945.

Followingthe OSGs approach, all lands certified as alienable and


disposable after 12June 1945 cannot be registered either under Section
14(1) of the PropertyRegistration Decree or Section 48(b) of the Public
Land Act as amended.

Discussedin Naguit. adopting the OSGs view, that all lands of the
publicdomain which were not declared alienable or disposable before
June 12, 1945would not be susceptible to original registration, no
matter the length ofunchallenged possession by the occupant. Such
interpretation renders paragraph(1) of Section 14 virtually inoperative
and even precludes the government fromgiving it effect even as it
decides to reclassify public agricultural lands asalienable and
disposable. The unreasonableness of the situation would even
beaggravated considering that before June 12, 1945, the Philippines
was not yeteven considered an independent state.
[T]hemore reasonable interpretation of Section 14(1) is that it merely
requiresthe property sought to be registered as already
alienable and disposable at thetime the application for
registration of title is filed.

Petitionersmake the salient observation that the contradictory


passages from Herbieto areobiter dicta since the land registration
proceedings therein is void abinitio in the first place due to lack of the
requisite publication of thenotice of initial hearing.

Theapplication therein was ultimately granted, citing Section 14(2).

Theevidence submitted by petitioners therein did not establish any


mode ofpossession on their part prior to 1948, thereby
precluding the applicationof Section 14(1). It is not even apparent from
the decision whether petitionerstherein had claimed entitlement to
original registration following Section14(1), their position being that
they had been in exclusive possession under abona fide claim of
ownership for over fifty (50) years, but not before 12 June1945.

TheCourt in Naguit offered the following discussion concerning


Section14(2)

Prescriptionis one of the modes of acquiring ownership under the Civil


Code. There is aconsistent jurisprudential rule that properties classified
as alienable publicland may be converted into private property by
reason of open, continuous andexclusive possession of at least thirty
(30) years.[[31]] With such conversion,such property may now fall
within the contemplation of private lands underSection 14(2), and
thus susceptible to registration by those who have acquiredownership
through prescription. Thus, even if possession of the alienablepublic
land commenced on a date later than June 12, 1945, and such
possessionbeing been open, continuous and exclusive, then the
possessor may have theright to register the land by virtue of Section
14(2) of the PropertyRegistration Decree.
Theobiter in Naguit cited the Civil Code provisions on prescription as
thepossible basis for application for original registration under Section
14(2).Specifically, it is Article 1113 which provides legal foundation for
the application.It reads:

Allthings which are within the commerce of men are susceptible of


prescription,unless otherwise provided. Property of the State or any
of its subdivisionsnot patrimonial in character shall not be the
object of prescription.

Thecritical qualification under Article 1113 of the Civil Code is


thus:[p]roperty of the State or any of its subdivisions not patrimonial
incharacter shall not be the object of prescription. The identification
whatconsists of patrimonial property is provided by Articles 420 and
421

Art.420. The following things are property of public dominion:


(1)Those intended for public use, such as roads, canals, rivers, torrents,
portsand bridges constructed by the State, banks, shores, roadsteads,
and others ofsimilar character;
(2)Those which belong to the State, without being for public use, and
are intendedfor some public service or for the development of the
national wealth.
Art.421. All other property of the State, which is not of thecharacter
stated in the preceding article, is patrimonial property
Itis clear that property of public dominion, which generally
includes propertybelonging to the State, cannot be the object
of prescription
Landsof the public domain, whether declared alienable and
disposable or not, areproperty of public dominion and thus
insusceptible to acquisition by prescription.

Article422 of the Civil Code states that [p]roperty of public dominion,


when nolonger intended for public use or for public service, shall form
part of thepatrimonial property of the State.

Accordingly,there must be an express declaration by the State


that the public dominionproperty is no longer intended for

public service or the development of thenational wealth or that


the property has been converted into patrimonial.
Shouldpublic domain lands become patrimonial because they are
declared as such in aduly enacted law or duly promulgated
proclamation that they are no longerintended for public service or for
the development of the national wealth, wouldthe period of
possession prior to the conversion of such public dominion
intopatrimonial be reckoned in counting the prescriptive
period in favor of thepossessors? - We rule in the negative.

Asthe application for registration under Section 14(2) falls wholly within
theframework of prescription under the Civil Code, there is no way that
possessionduring the time that the land was still classified as public
dominion propertycan be counted to meet the requisites of acquisitive
prescription and justifyregistration.

Section14(1) mandates registration on the basis of possession,


while Section 14(2)entitles registration on the basis of
prescription.Registration under Section 14(1) is extended
under the aegis of the PropertyRegistration Decree and the
Public Land Act while registration under Section14(2) is made
available both by the Property Registration Decree and the
CivilCode.

Registrationunder Section 48(b) of the Public Land Act as


amended by Rep. Act No. 1472 isbased on thirty years of
possession alone without regard to the Civil Code,while the
registration under Section 14(2) of the Property Registration
Decreeis founded on extraordinary prescription under the Civil
Code.

Whetherunder ordinary prescription or extraordinary prescription, the


period ofpossession preceding the classification of public dominion
lands as patrimonialcannot be counted for the purpose of computing
prescription. But after theproperty has been become patrimonial, the
period of prescription begins to runin favor of the possessor.

Oncethe possessor automatically becomes the owner of the converted


patrimonialproperty, the ideal next step is the registration of the
property under theTorrens system. It should be remembered that
registration of property is not amode of acquisition of ownership, but
merely a mode of confirmation of ownership.

(1)In connection with Section 14(1) of the Property Registration


Decree, Section48(b) of the Public Land Act recognizes and confirms
that those who bythemselves or through their predecessors in interest
have been in open,continuous, exclusive, and notorious possession and
occupation of alienable anddisposable lands of the public domain,
under a bona fide claim of acquisitionof ownership, since June 12,
1945 have acquired ownership of, and registrabletitle to, such lands
based on the length and quality of their possession.
(a)Since Section 48(b) merely requires possession since 12 June
1945 and doesnot require that the lands should have been
alienable and disposable during theentire period of possession,
the possessor is entitled to secure judicialconfirmation of his title
thereto as soon as it is declared alienable anddisposable, subject to the
timeframe imposed by Section 47 of the Public LandAct.
(b)The right to register granted under Section 48(b) of the Public
Land Actis further confirmed by Section 14(1) of the Property
RegistrationDecree.
(2)In complying with Section 14(2) of the Property Registration
Decree,consider that under the Civil Code, prescription is recognized as
a mode ofacquiring ownership of patrimonial property. However, public
domain landsbecome only patrimonial property not only with a
declaration that these arealienable or disposable. There must also be
an express government manifestationthat the property is already
patrimonial or no longer retained for public serviceor the development
of national wealth, under Article 422 of the Civil Code. Andonly when
the property has become patrimonial can the prescriptive period forthe
acquisition of property of the public dominion begin to run.
(a)Patrimonial property is private property of the government. The
person acquiresownership of patrimonial property by prescription
under the Civil Code isentitled to secure registration thereof under
Section 14(2) of theProperty Registration Decree.

(b)There are two kinds of prescription by which patrimonial property


may beacquired, one ordinary and other extraordinary. Under ordinary
acquisitiveprescription, a person acquires ownership of a patrimonial
property throughpossession for at least ten (10) years, in good faith
and with just title.Under extraordinary acquisitive prescription, a
persons uninterrupted adversepossession of patrimonial property for
at least thirty (30) years, regardlessof good faith or just title, ripens
into ownership.

13(April)

JOSE CHING AND CARIDAD CHING, petitioners,


vs.
Hon. ANTONIO Q. MALAYA, as Presiding Judgeof the CFI of
Laguna, Branch IV, Hon. MAXIMIANO C. ASUNCION, as
Presiding Judgeof the CFI of Laguna Branch II, and Spouses
CESAR ALVARADO and ARACELIALVARADO, respondents.
G.R. No.L-56449

August 31, 1987

CRUZ, J.:
-------------------------------------------------------------------(Thispetition for certiorari under Rule 65 of the Rules of Courtseeks a
reversal of the decision of the respondent court nullifying thejudgment
of the municipal court in a forcible entry case on the ground of lackof
jurisdiction.)
FACTS: The petitioners had alleged in their complaint for ejectment
thatthe private respondents had forced their way into the disputed
premises withoutany right whatsoever and had refused to vacate the

same despite repeateddemands. These demands were based on the


petitioners case that they were theowners of the said property, having
acquired it by virtue of a valid sale. Theprivate respondents, in their
answer, had challenged the claimed sale, arguingthat the property
belonged to them by right of inheritance. At any rate, theyhad argued,
as the basic question was one of ownership and not of merepossession,
the municipal court had no jurisdiction and should dismiss
thecomplaint.
Themunicipal court, affirming its jurisdiction, proceeded totrial and
thereafter rendered judgment ordering the private respondents
tovacate the disputed property. It also required them to pay the
petitioners backand current rentals at P 1,000.00 a month until actual
surrender of thepremises, as well as a P 3,000.00 attorneys fee plus
the costs of the suit. Onappeal, this decision was set aside by the
respondent judge, who held that themunicipal court had no
competence to resolve the case as it involved a questionof ownership.
The Supreme Court provided the pertinent provisions of R.A. No. 296
readas follows:
Sec. 88. Original jurisdiction incivil cases. Inall civil actions,
including those mentioned in Rules fifty-nine and sixty-two(now Rule 57
and 60) of the Rules of Court, arising in his municipality orcity, and not
exclusively cognizable by the Court of First Instance, thejustice of the
peace (now municipal judge) and the judge of a municipal court(now
city court) shall have exclusive original jurisdiction ....
Inforcible entry and detainer proceedings, the justice of the peace or
judge ofthe municipal court shall have original jurisdiction, but the said
justice orjudge may receive evidence upon the question of title therein,
whatever may bethe value of the property, solely for the purpose of
determining the characterand extent of possession and damages for
detention.

ISSUE:
1)Whetheror not the case at bar falls in the exception.
2)Whetheror not party may introduceevidence of ownership to prove
character of possession and amount of damagesfor unjust deprivation
thereof.
RULING:

1) No.
Afterexamining the facts, the Court finds that it does not come under
the exceptionto the rule.
Theproperty in question consists of a residential house and lot covered
by TCT No.T-85126 and registered in the name of petitioner Jose Ching
in the Registry ofDeeds of Laguna. The basis of the registration is a
deedof sale executed in his favor by Felix Carpio, the former owner,
who hadacquired it from Brigido Alvarado, Cesar Alvarados supposed
father. Therecord does not show that such registration has been
challenged since theissuance in 1978 of the said certificate of title,
which in the absence ofevidence to the contrary should be presumed
valid.
Thereis no encumbrance on the land, and there is no adverse claim or
notice of lis pendis annotated in thecertificate. Such registration, it
may be added, isbinding against the whole world unless
annulled for cause in proper cases.
TheCourt also provided that fact alone could not divest the municipal
court ofjurisdiction to continue trying the question of possession, more
so since thequestion of ownership was appropriately being litigated in
the annulment suit.Significantly, the deed of sale being challenged in
that action was differentfrom the contract involved in the exception

2) Yes.
TheCourt ruled that the fact that the petitioners themselves adduced
evidence ofownership over the property in question did not, as
claimed, have the effect ofdivesting the municipal court of its
jurisdiction. As permitted in the Section88 of R.A. No. 296: the plaintiff
in an ejectment case may introduce suchevidence for the purpose of
proving the character of his possession and theamount of damages he
is claiming for unjust deprivation of such possession. Thepetitioners
were only trying to prove their right to possession and damages
byestablishing their right of ownership.

14(Daniel Eblahan)

Legardav. Saleeby
GRno. L-8936, Oct 2, 1915
Johnson,J.:
Facts:
Consuelo Legarda and her husband owned a lot in Ermita
adjoining the lot of N.MSaleeby separated by a stone wall located on
her side. The wall was not a jointwall. On March 2 1906, Consuelo
petitioned the Court of Land Registration toregister their lot and the
court allowed them. In the original certificate andtitle issued to them,
provided by the Torrens system, the wall was included. OnMarch 25,
1912 N.M Saleeby also petitioned the Court of Land Registration
toregister his lot and was also allowed. The wall was also included in
hisoriginal certificate and title. The spouses discovered this error
andpetitioned the Court of Land Registration to adjust and correct it.
The courtdenied them because they did not object to the error when
the defendantregistered his lot.
Issue:
Whether N.M. Saleeby owns the wall and the land occupying said
wall
Held:
No, the Court disagreed with the reason of the lower court that
the plaintiffsdid not object to the registration of Saleeby. The Court
argued that shouldthis be the case, the plaintiffs would always be on
alert if another personregisters the property and should immediately
oppose. The Court declared thatif the holder of the certificate cannot
rest secure in the registered title,the purpose of the Torrens system is
defeated and nothing is gained by theregistration. The Torrens system
intended to quiet title, putting a stop to anyquestion of the legality of
the title.
In this case, the Court decided to award theplaintiffs the property
because they were the once who acquired and registeredit first and
complied with the requirements of the law. The defendant cannotraise
the defense that he is an innocent purchaser, because the
presumptionthat he has examined every instrument affecting the title
is irrebutable. Thedefendant should have taken notice that the
property was registered to theplaintiffs six years before he did. The

title, once registered, is notice tothe whole world. All persons must
take notice.
Significance:
The purpose of the Torrens system is to quiet the title of a land
registered under said system. Once registered it is notice to the whole
world and shall be conclusive upon and against all persons, including
the Government and all the branches thereof, whether mentioned by
name in the application, notice, or citation, or included in the general
description "To all whom it may concern."

15(Melodia)
________________________________________________________________________
__________________________
16(Mel)

DINAH C. CASTILLOvsANTONIO M. ESCUTIN


G.R. No. 171056

March 13, 2009

Facts:
Petitioner is a judgment creditor of Raquel K. Moratilla. Racquel, her
mother, Urbana Kalaw and sister, Perla Moratilla, co-owned Lot 13713,
15,000 square-meter, covered by Tax Declaration No. 00449.
When the petitioner verified the property, she found out that the
application of Summit Point Golf & Country Club, Inc. for conversion of
several agricultural landholdings, including Lot 13713, to residential,
commercial, and recreational uses was approved and the property was
not covered by a certificate of title, whether judicial or patent, or
subject to the issuance of a Certificate of Land Ownership Award or
patent under the Comprehensive Agrarian Reform Program.
Petitioner then proceeded to levy on execution Lot 13713. Before the
scheduled public auction sale, petitioner learned that Lot 13713 was
inside the Summit Point Golf and Country Club Subdivision owned by
Summit Point Realty and Development Corporation. She immediately
went to the Makati City office of Summit Realty to meet with its Vice

President, Orense. However, she claimed that Orense did not show her
any document to prove ownership of Lot 13713 by Summit Realty.
Petitioner bought Raquels 1/3 pro-indiviso share in Lot and was then
issued Tax Declaration No. 00942-A, indicating that she owned 5,000
square meters of Lot 13713, while Urbana and Perla owned the other
10,000 square meters.
When petitioner attempted to pay real estate taxes, she was shocked
to that, without giving her notice, her Tax Declaration No. 00942-A was
cancelled. Lot 13713 was said to be encompassed in and overlapping
with the 105,648 square meter parcel of land known as Lot 1-B, both in
the name of Francisco Catigbac. The reverse side of TCT No. 129642
bore three entries, reflecting the supposed sale of Lot 1-B to Summit
Realty.
In the supposed Deed of Absolute Sale in favor of Summit Realty by
Leonardo Yagin, as Catigbacs attorney-in-fact, it did not express the
desire of Summit Realty to purchase Lot 1-B or indicate its consent and
conformity to the terms of the Deed. There were also missing
information in the said Deed.
Petitioner asserted that Summit Realty was well-aware of Catigbacs
death, having acknowledged the same in LRC Case No. 00-0376, the
Petition for Issuance of New Owners Duplicate of TCT No. 181 In Lieu
of Lost One, filed by Summit Realty before the Regional Trial Court of
Lipa City. During the ex parte presentation of evidence in the latter
part of 2000, Orense testified on behalf of Summit Realty that
Catigbacs property used to form part of a bigger parcel of land, Lot 1
of Plan Psu-12014, measuring 132,975 square meters, covered by TCT
No. 181 in the name of Catigbac; after Catigbacs death, Lot 1 was
informally subdivided into several parts among his heirs and/or
successors-in-interest, some of whom again transferred their shares to
other persons; Summit Realty separately bought subdivided parts of
Lot 181 from their respective owners, with a consolidated area of
105,648 square meters, and identified as Lot 1-B after survey; despite
the subdivision and transfer of ownership of Lot 1, TCT No. 181
covering the same was never cancelled; and the owners duplicate of
TCT No. 181 was lost and the fact of such loss was annotated at the
back of the original copy of TCT No. 181 with the Registry of Deeds.
Subsequently, in an Order dated 3 January 2001, the RTC granted the
Petition in LRC Case No. 00-0376 and directed the issuance of a new
owners duplicate of TCT No. 181 in the name of Catigbac, under the
same terms and condition as in its original form.

Petitioner cast doubt on the acts undertaken by Summit Realty in


connection with Catigbacs property, purportedly without legal
personality and capacity. The Special Power of Attorney dated 6
February 1976 granted Yagin the right to sue on behalf of Catigbac, yet
it was Summit Realty which instituted LRC Case No. 00-0376, and Yagin
had no participation at all in said case. Likewise, it was not Yagin, but
Orense, who, through a letter dated 27 June 2001, requested the
cancellation of TCT No. 181 covering Lot 1 and the issuance of a new
certificate of title for Lot 1-B. Hence, it was Orenses request which
resulted in the issuance of TCT No. 129642 in the name of Catigbac,
later cancelled and replaced by TCT No. T-134609 in the name of
Summit Realty.
Petitioner questioned why, despite the cancellation of TCT No. 129642
in the name of Catigbac and the issuance in its place of TCT No. T134609 in the name of Summit Realty, it was the former cancelled title
which was used as basis for canceling petitioners Tax Declaration No.
00942-A. Tax Declaration No. 00949-A was thus still issued in the name
of Catigbac, instead of Summit Realty.
Summit Realty bought Lot 1-B measuring 105,648 square meters,
specifically covered by TCT No. 129642, both in the name of Catigbac.
As a result of such purchase, ownership of Lot 1-B was transferred from
Catigbac to Summit Realty. Summit Realty had every reason to believe
in good faith that said property was indeed owned by Catigbac on the
basis of the latters certificate of title over the same. Catigbacs right
as registered owner of Lot 1-B under TCT No. 181/No. 129642, was
superior to petitioners, which was based on a mere tax declaration.
Issue:
Whether petitioner was indeed unlawfully deprived of her 5,000 square
meter property.
Ruling:
As between Catigbacs title, covered by a certificate of title, and
petitioners title, evidenced only by a tax declaration, the former is
evidently far superior and is, in the absence of any other certificate of
title to the same property, conclusive and indefeasible as to Catigbacs
ownership of Lot 1-B. Catigbacs certificate of title is binding upon the
whole world, including respondent public officers and even petitioner
herself. The Court ruled that tax declarations and corresponding tax
receipts cannot be used to prove title to or ownership of a real property
inasmuch as they are not conclusive evidence of the same. Petitioner
acquired her title to the 5,000 square meter property from Raquel who,

it is important to note, likewise only had a tax declaration to evidence


her title. In addition, the Court of Appeals aptly observed that,
"curiously, as to how and when petitioners alleged predecessor-ininterest, Raquel K. Moratilla and her supposed co-owners acquired
portions of Lot 1 described as Lot 13713 stated in TD No. 00449,
petitioner had so far remained utterly silent."
A title is different from a certificate of title. Title is generally defined as
the lawful cause or ground of possessing that which is ours. It is that
which is the foundation of ownership of property, real or personal. Title,
therefore, may be defined briefly as that which constitutes a just cause
of exclusive possession, or which is the foundation of ownership of
property. Certificate of title, on the other hand, is a mere evidence of
ownership; it is not the title to the land itself. Under the Torrens
system, a certificate of title may be an Original Certificate of Title,
which constitutes a true copy of the decree of registration; or a
Transfer Certificate of Title, issued subsequent to the original
registration.

________________________________________________________________________
___________________________

17(Rocky)

Philippine National Bank vs.Court of Appeals, 153 SCRA 435


FACTS: Thesubject of the action is 30 parcels of land which they claim
to be the conjugalproperty of the spouses Donata Montemayor and
Clodualdo Vitug of which theyclaim a share of 2/11 of 1/2 thereof. They
assailed the mortgage to the PNB andthe public auction of the
properties as null and void. They invoked the case ofVitug vs.
Montemayor, L-5297 decided by this Court on Oct. 20, 1953 which is
anaction for partition and liquidation of the said 30 parcels of land
wherein theproperties were found to be conjugal in nature.
ISSUE: Doesthe presumption of conjugality of properties acquired by
the spouses duringcoverture provided for in Article 160 of the Civil

Code apply to propertycovered by a Torrens certificate of title in the


name of the widow?
HELD: The petitionis impressed with merit.
When the subject propertieswere mortgaged to the PNB they were
registered in the name of DonataMontemayor, widow. Relying on the
torrens certificate of title covering saidproperties the mortgage loan
applications of Donata were granted by the PNB andthe mortgages
were duly constituted and registered in the office of theRegister of
Deeds. In processing the loan applications of Donata Montemayor,the
PNB had the right to rely on what appears in the certificates of title
andno more. On its face the properties are owned by Donata
Montemayor, a widow.The PNB had no reason to doubt nor question
the status of said registered ownerand her ownership thereof.
The well-known rule in thisjurisdiction is that a person dealing with a
registered land has a right torely upon the face of the torrens
certificate of title and to dispense with theneed of inquiring further,
except when the party concerned has actual knowledgeof facts and
circumstances that would impel a reasonably cautious man make
suchinquiry.
The presumption applies toproperty acquired during the lifetime of the
husband and wife. In this case, itappears on the face of the title that
the properties were acquired by DonataMontemayor when she was
already a widow. When the property is registered in thename of a
spouse only and there is no showing as to when the property
wasacquired by said spouse, this is an indication that the property
belongsexclusively to said spouse. The PNB had a reason to rely on
what appears on thecertificates of title of the properties mortgaged.
Indeed, if the PNB knew ofthe conjugal nature of said properties it
would not have approved the mortgageapplications covering said
properties of Donata Montemayor without requiringthe consent of all
the other heirs or co-owners thereof. Moreover, when saidproperties
were sold at public auction, the PNB was a purchaser for value ingood
faith. So its right thereto is beyond question.
As correctly held by thelower court. Pragmacio and Maximo Vitug as
occupants and lessees of theproperty in question cannot now dispute
the ownership of their mother over thesame who was their lessor.

Hilario vs. City of Manila,19 SCRA 931

FACTS: Dr.Jose Hilario was the registered owner of a large tract of land
around 49hectares in area (Barrio Guinayang, San Mateo, Rizal). Upon
his death thisproperty was inherited by his son, Jose Hilario, Jr., to
whom a new certificateof title was issued. During the lifetime of
plaintiffs father, the Hilarioestate was bounded on the western side by
the San Mateo River. To prevent itsentry into the land, a bamboo and
lumber post dike or ditch was constructed onthe northwestern side.
This was further fortified by a stonewall built on thenorthern side. For
years, these safeguards served their purpose. However, in1937, a
great and extraordinary flood occurred which inundated the entire
placeincluding the neighboring barrios and municipalities. The River
destroyed thedike on the northwest, left its original bed and
meandered into the Hilarioestate, segregating from the rest thereof a
lenticular piece of land. Thedisputed area is on the eastern side of this
lenticular strip which now standsbetween the old riverbed site and the
new course. In 1945, the US Army opened asand and gravel plant
within the premises, and started scraping, excavating andextracting
soil, gravel and sand from the nearby areas along the River.
Theoperations eventually extended northward into the strip of land.
Consequently,a claim for damages was filed with the US War
Department by Luis Hidalgo, thethen administrator of Dr. Hilarios
estate. The US Army paid. In 1947, theplant was turned over to herein
defendants-appellants and appellee who tookover its operations.
On 22 October 22, 1949,plaintiff filed his complaint for injunction and
damages against the defendantsCity Engineer of Manila, District
Engineer of Rizal, the Director of PublicWorks, and Engr. Busuego, the
Engineer-in-charge of the plant. Subsequently,the Bureau of Mines and
Atty. Maximo Calalang were respectively allowed to jointhe litigation as
intervenors; as per issue of fees and penalties for materials(sand and
gravel) extracted. On 14 March 1954, defendants filed a petition
forinjunction against plaintiff and intervenor Calalang in the same case,
allegingthat the latter have fenced off the disputed area in
contravention of anagreement had between the latter and the Director
of Public Works wherein thedefendants were allowed to continue their
operations but subject to the finaloutcome of the pending suit. On 13
May 1954, plaintiff amended his complaintand impleaded as additional
defendants the City of Manila, the ProvincialTreasurer of Rizal, and
Engr. Eulogio Sese, the new Engineer-in-charge of theplant. Plaintiff
also converted his claim to one purely for damages directedagainst the
City of Manila and the Director of Public Works, solidarily, in theamount
of P1,000,000.00, as the cost of materials taken since 1949, as well
asthose to be extracted therefrom until defendants stop their
operations. On 21December 1956, the lower court rendered its
decision, ordering the City ofManila and Director of Public Works to pay
Hilario in solidum the sum ofP376,989.60 as cost of gravel and sand

extracted from the plaintiffs land,plus costs; and ordering the


Provincial Treasurer of Rizal to reimburseintervenor Calalang of P36.80
representing gravel fees illegally collected.None of the parties litigants
seemed satisfied with this decision and they allsought a
reconsideration of the same. On August 30, 1957, the lower
courtresolved the motions to reconsider with an order, holding that the
2/5 portionof the area in controversy to Hilario, and dismissing the case
against theBureau of Public Works insofar as money claims are
concerned without prejudiceto Hilario taking action against proper
party in such claim. Hilario andCalalang filed a second motion for
reconsideration, which the lower court denied.Hence, the appeal.
ISSUES: (a)Whether or not the northern two-fifths of the disputed area
belongs toplaintiff Hilario. (b) When a river, leaving its old bed,
changes its originalcourse and opens a new one through private
property, would the new riverbankslining said course be of public
ownership also?
HELD: (a) TheSupreme Court set aside the decision and orders
appealed from, and enteredanother judgment to the effect that the
City of Manila and the Director ofPublic Works, and his agent and
employees, are absolved of liability fromextracting materials from
subject property (of public domain); and the portionwithin the strip of
land question declared not part of public domain andconfirmed as part
of Hilarios private property. No Costs.
(b) From 1945 to 1949, thewest bank of the River extended westward
up to the secondary bank line; from1950 to 1952, this bank had
moved, with the River, to the east, its lateralborders running along a
line just 20 meters west of the camachile tree; andfrom 1953 to 1955,
the extremities of the west bank further receded eastwardbeyond the
camachile tree, until they lay just about 20 meters east of saidtree.
Evidence shows that the River floods with annual regularity during
therainy season. These floods can hardly be called accidental. The
Colegio deSan Jose case is not exactly in point. What was mainly
considered there wasArticle 74 of the Law of Waters relating to lakes,
ponds and pools. In thepresent case, none of these is involved.
Defendants cannot be accusedof unjustly profiting at plaintiffs
expense. They were not responsible for theshifting of the river. It was
due to natural causes for which no one can beblamed. Further,
defendants were extracting from public property then, underproper
authorization. The government, through the defendants, may have
beenenriched by chance, but not unjustly.

Director of Lands vs. Courtof Appeals, 179 SCRA 522


FACTS: Theland in question is situated in Obando, Bulacan. It adjoins
the Kailogan Riverand private respondent Valeriano have converted it
into a fishpond. In theirapplication in 1976, private respondents
claimed that they are the co-owners infee simple of the land partly
through inheritance and partly by purchase andthat; it is not within any
forest or military reservation. The Republic of thePhil., represented by
the Dir of the Bureau of Forest Development, opposed theapplication
on
the
principal
ground
that
the land
applied
for
is
WITHINTHEUNCLASSIFIED REGION of Obando, Bulacan and that such
area are denominated asFORESTLANDS-do not form part of the
disposable and alienable portion of thepublic domain. The Trial Court
ordered registration of the subject land infavor of the Valerianos. This
was affirmed by the CA which said in part thatsince the subject
property is entirely devoted to fishpond purposes, it cannotbe
categorized as part of forest lands.
ISSUE: Whetheror not the courts can reclassify the subject public land.
HELD: Courtscannot reclassify... its beyond their competence and
jurisdiction. Theclassification of public lands is an exclusive prerogative
of the Executive Departmentof the Government (Bureau of Forest
Development) and not of the Courts. In the absenceof such
classification, the land remains as unclassified land until it isreleased
therefrom and rendered open to disposition. Since the subject
propertyis still unclassified, whatever possession Applicants (Valeriano)
may have had,and, however long, cannot ripen into private ownership.
The conversion of thesubject property into a fishpond by Applicants
does not automatically renderthe property as alienable and disposable.
The recommendation of the DistrictForester for release of subject
property from unclassified region is not theultimate word on the
matter.

Ong vs. Republic, GR No.175746, 12 March 2008, 548 SCRA 160


FACTS: OnJuly 1, 1999, petitioner Charles L. Ong (petitioner) in his
behalf and as dulyauthorized representative of his brothers, namely,
Roberto, Alberto and Cesar,filed an Application for Registration of
Title[4] over Lot 15911 (subject lot)situated in Barangay Anolid,
Mangaldan, Pangasinan with an area of five hundredseventy four (574)
square meters, more or less. They alleged that they are theco-owners
of the subject lot; that the subject lot is their exclusive propertyhaving
acquired the same by purchase from spouses Tony Bautista and

AliciaVillamil on August 24, 1998; that the subject lot is presently


unoccupied; andthat they and their predecessors-in-interest have been
in open, continuous andpeaceful possession of the subject lot in the
concept of owners for more thanthirty (30) years.
After due notice andpublication, only respondent Republic of the
Philippines (respondent),represented by the Office of the Solicitor
General, opposed the application forregistration of title. Respondent
asserted that neither applicants nor theirpredecessors-in-interest have
been in open, continuous, exclusive and notoriouspossession and
occupation of the subject lot since June 12, 1945 or earlier asrequired
by Section 48(b) of Commonwealth Act No. 141, as amended
byPresidential Decree (P.D.) No. 1073; that applicants failed to adduce
anymuniment of title to prove their claims; that the tax declaration
appended tothe application does not appear genuine and merely
shows pretended possessionof recent vintage; that the application was
filed beyond the period allowedunder P.D. No. 892; and that the subject
lot is part of the public domain whichcannot be the subject of private
appropriation.
On January 16, 2002, thetrial court rendered a Decision in favor of
petitioner and his brothers.
Aggrieved, respondentappealed to
rendered the assailed Decision.

the Court

of

Appeals

which

In reversing the decision ofthe trial court, the Court of Appeals found
that the subject lot is part of thealienable and disposable lands of the
public domain. Thus, it was incumbentupon petitioner to prove that
they possessed the subject lot in the nature andfor the duration
required by law. However, petitioner failed to prove that heor his
predecessors-in-interest have been in adverse possession of the
subjectlot in the concept of owner since June 12, 1945 or earlier as
mandated bySection 14(1) of P.D. 1529. It noted that the earliest tax
declaration whichpetitioner presented is dated 1971. Consequently,
petitioner could not fairlyclaim possession of the land prior to 1971.
Neither was petitioner able toprove that he or his predecessors-ininterest actually occupied the subject lotprior to the filing of the
application. Thus, the trial court erred in grantingthe application for
registration of title over the subject lot.
ISSUE: (a)Whether or not petitioner, together with his brothers have
registrable ownershipover the real property and (b) whether or not the
findings and conclusions ofthe Court of Appeals that the subject real
property is a public land iscorrect.

HELD: Thepetition lacks merit. The law provides that applicants for
registration oftitle must prove that: (1) the subject land forms part of
the disposable andalienable lands of the public domain, and (2) that
they have been in open,continuous, exclusive and notorious possession
and occupation of the same undera bona fide claim of ownership since
June 12, 1945, or earlier. Theserequisites involve questions of fact
which are not proper in a petition forreview on certiorari.
There is no dispute that thesubject lot is classified as alienable and
disposable land of the publicdomain. The Report of the Bureau of Lands
stated that the subject lot is withinthe alienable and disposable zone.
This finding is, likewise, embodied in theReport of the Department of
Environment and Natural Resources CommunityEnvironment and
Natural Resources Office (DENR-CENRO) and the blue print Copyof the
plan covering the subject lot. However, petitioner failed to prove
thathe or his predecessors-in-interest have been in open, continuous,
exclusive andnotorious possession and occupation of the subject lot
since June 12, 1945 orearlier.
Further, as correctlypointed by the Court of Appeals, possession alone
is not sufficient to acquiretitle to alienable lands of the public domain
because the law requirespossession and occupation.

Director of Lands vs. C.A,129 SCRA 689


FACTS: Petitionersseek a review of the Decision and Resolution of the
Court of Appeals affirmingthe judgment of the Court of First Instance of
Bulacan, decreeing registrationof a parcel of land in private
respondents' favor. The land in question, issituated in Obando,
Bulacan, and has an area of approximately 9.3 hectares. Itadjoins the
Kailogan River and private respondents have converted it into
afishpond.
In their application forregistration, private respondents claimed that
they are the co-owners in feesimple of the land applied for partly
through inheritance in 1918 and partly bypurchase on May 2, 1958;
that it is not within any forest zone or militaryreservation; and that the
same is assessed for taxation purposes in theirnames.
The Republic of thePhilippines, represented by the Director of the
Bureau of Forest Developmentopposed the application on the principal
ground that the land applied for iswithin the unclassified region of
Obando, Bulacan, and that areas within theunclassified region are

denominated as forest lands and do not form part of thedisposable and


alienable portion of the pub
18 (Maribeth)

Bornales vs Intermediate Appellate Court, 166SCRA 519


FACTS:
In 1927, SixtoDumolong married to Isabel Marquez-Dumolong was
awarded a parcel of landdenominated as Lot 1318 and covered by
Original Certificate of Title No. 6161. Since1920 however, said spouses
have been living separately and that they had nochildren. Sixto then
cohabited with one Placida Dumolong and they had children,one of
which is Renito Dumolong.
In March 1978 aDeed of Extrajudicial Adjudication and Sale of Real
Property was executed byRenito and Isabel as settlement of the
conjugal estate of Sixto involving Lot1318 and its eventual sale. A
supposed thumbmarkof Isabel appeared in the document. Thedeed
was registered and pursuant thereto Transfer Certificate of Title No.T15856 was issued in the name of the vendees. The subject lot was
further sold to Spouses Bornales, the cultivators ofthe subject lot, and
eventually they were able to secure a TCT in their names.
On March 11,1980, Isabel filed an action for reconveyance and
damages against SpousesBornales, Placida, Renito and all the other
parties named in the ExtrajudicialAdjudication and Sale of Real
Property alleging forgery thereof. The lower court rendered judgement
in favorof Isabel and against all the defendants. Spouses Bornales were
declared purchasers in bad faith. On appeal, the appellate court
affirmed theappealed decision with modifications as it found that the
subject land was theexclusive property of Sixto who had other
illegitimate children surviving withRenito. It further denied the
hereinpetitioners motion for reconsideration, hence, the petition.
ISSUE: Whether
or
not
SpousesBornales
indefeasibilityof a certificate of title?

may

invoke

the

HELD: No. Having bought the land registered under the Torrenssystem
from the vendor who procured title thereto by means of fraud, they
cannotinvoke the indefeasibility of a certificate of title against Isabel to
theextent of her interest over the lot. TheTorrens system of land
registration should not be used as a means to perpetratefraud against

the rightful owner of real property. Registration, to be effective, must


be made ingood faith. It is a settled rule thatthe defense of
indefeasibility of a certificate of title does not extend to atransferee
who takes it with notice of the flaws in his transferors title.

19 (Ailyn)

20 (Jen)

21 (Bambi) - Hindi ko mahanap ang Galloy vs. CA, 173 SCRA 26

22 (Nelson)
HACIENDA BIGAA, INC.,
Petitioner,
versus
EPIFANIO V. CHAVEZ (deceased), substituted by SANTIAGO V.
CHAVEZ,
Respondent. -- G.R. No. 174160

Facts:
Hacienda Calatagan covered by TCT 722 with an area of
9,652.583 hectares was owned by Ayala y Cia and the Zobels, they

expanded the lot to cover 2000 hectares consisting, among others of


beach, foreshores and bay areas, and navigable waters(excess areas),
making it appear that these excess areas are part of the hacienda.
They sold the excess area to different third parties; one of them was
the Hacienda Bigaan(herein petitioner). Notwithstanding the prior
ruling of the court dated 1965 that renders Hacienda Calatagan as
public land including the expanded area outside the land, they still sold
it to another contending that the subject land is not included in the
prior decision(antecedent cases of Dizon, Ayala y Cia, and De los
Angeles).
Hacienda Bigaa also contended that the rulings in the antecedent
cases on the nullity of its subdivision titles should not apply to the
present case because the titles TCT Nos. 44695 and 56120 have not
been specifically declared void by court order and must be given
probative value. It likewise posits that Chavez failed to introduce
evidence before the MTC that the land subject matter of the suit is the
same land covered by the decision of the Supreme Court in the
antecedent cases.
On June 5, 1996, petitioner Hacienda Bigaa, a buyer or transferee of
the subject land, filed with the Municipal Trial Court (MTC) of
Calatagan, Batangas a complaint[for ejectment (forcible entry) and
damages with application for writ of preliminary injunction against
respondent Epifanio V. Chavez (Chavez), docketed as Civil Case No.
129. The complaint alleged that Chavez, by force, strategy and/or
stealth, entered on April 29, 1996 the premises of Hacienda Bigaa's
properties (subject land) by cutting through a section of the barbed
wire fence surrounding the properties and destroying the lock of one of
its gates, subsequently building a house on the property, and
occupying the lots without the prior consent and against the will of
Hacienda Bigaa.
Defendant (now respondent) Chavez alleged in his answer before the
MTC of Calatagan that his mother, Zoila de Chavez (who died intestate
on September 14, 1979) was a fishpond permittee/lessee
under Fishpond Permit Nos. F-4572-0 and F-24735 issued by the
Bureau of Fisheries on April 21, 1959 and June 3, 1966, respectively;
that the areas covered by the permits are the same parcels of land
which he presently occupies as Zoila's successor-in-interest and which
Hacienda Bigaa also claims.
Issue:
1. Whether or not Chavez has a better claim over the subject land.

2. Whether or not Hacienda Bigaas title carry no probative value.


Held:
1. NO. As framed above, the case before us inevitably brings to
memory the antecedent decided cases touching on the
ownership of the vast tract of land in Calatagan, Batangas,
covered by Transfer Certificate of Title (TCT) No. 722 in the
name/s of Ayala y Cia, Alfonso Zobel, Jacobo Zobel and Enrique
Zobel and/or Hacienda Calatagan the predecessors-in-interest of
petitioner Hacienda Bigaa. We ruled in the antecedent cases
of Dizon, Ayala y Cia, and De los Angeles, that: (1) all expanded
subdivision titles issued in the name of Ayala y Cia, the Zobels
and/or Hacienda Calatagan covering areas beyond the true
extent of TCT No. 722 are null and void because they cover
areas belonging to the public domain; (2) Ayala y Cia and the
Zobels of Hacienda Calatagan are mere usurpers of these public
domain areas; and that (3) these areas must revert to
the Republic.Significantly,
we
declared
in De
los
Angeles that the Republic, as the rightful owner of the
expanded areas portions of the public domain has the
right to place its lessees and permittees (among them
Zoila de Chavez) in possession of the fishpond lots whose
ownership and possession were in issue in the case.

1. Yes. The Court rejected this contention in light of our holding in


the Ayala y Cia and De los Angeles cases that apart from those
expressly litigated and annulled, all other subdivision titles over
the excess areas of Hacienda Calatagan must be nullified for
covering unregisterable lands of the public domain that must
revert to the Republic. To reiterate, lots and their titles
derived from the Ayalas and the Zobels TCT No. 722 not
shown to be within the original coverage of this title are
conclusively public domain areas and their titles will be
struck down as nullities. What could have saved Hacienda
Bigaa, as successor-in-interest of the Ayalas and the Zobels, is
competent evidence that the subdivision titles in its possession
do not fall within the excess areas of TCT No. 722 that are null
and void because they are lands of the public domain. Hacienda
Bigaa however failed to discharge this burden.
Significance:

The registration of lands of the public domain under Torrens


System, by itself cannot convert public land into private land.

23 (JM)

Director of Forestry vs. Munoz, 23 SCRA 1183 (L-24796, June


28, 1968)

FACTS:

PINAGCAMALIGAN INDO-AGRO DEVELOPMENTCORPORATION, INC


(Piadeco) is a company engaged in logging. It was given a Certificate
ofPrivate Woodland Registration sothat it can operate in a 72000
hectare.It also has a Titulo de Propriedad which it acquired in1894
under the Spanish regime.

In 1964, the NAWASA director orderedthe cancellation of Piadecos


certificate because it encroached beyond what wasallowed in the
certificate. It actually cut trees inthe Angat and Marikina watershed
area which was prohibited. The lower courtruled in favor of Piadeco.
Piadeco also had a settlement with Nawasa. Piadecosought to renew its
certificate but it was deniedby the Asst. Director of Forestry. The latter
ruled that the Spanish title isno longer recognized and should have
never been used to applyfor a Certificate.

ISSUE:

Whether or not Piadeco can claimownership over the property.

HELD:

No. The Spanish title it acquired cannotbe used to register for another
Certificate. There should be no question nowthat Forestry
Administrative Order 12-2 has the forceand effect of law. It was
promulgated pursuant to law. Section 1817, RevisedAdministrative
Code, empowers the Bureau of Forestry, with the approval of the
department head, to issue regulations deemed expedient or necessary
to secure the protection and conservation of thepublic forests in such
manner as to insure a continuedsupply of valuable timber andother
forest products for the future, and regulating the use and occupancy of
theforests and forest reserves, to the same end. It is an administrative
regulation germane to the objects and purposes of the law.

24 (Maris)

25 (Reg)

26 (Greg)

27 (Ed)

28 (LJ)

29 (Zax)

30 (Clathem)

31 (Lea)

32 (Rubie)

#19 Ailyn
Part 1

G.R. No. 77294

December 12, 1988

ANGELICA VIAJAR and CELSO VIAJAR,plaintiffs-appellants,

vs.

COURT OF APPEALS, LEONOR P. LADRIDO,LOURDES LADRIDO


IGNACIO, EUGENIO P. LADRIDO and L P. LADRIDO,defendantsappellees.

FACTS

The spouses Ricardo Y. Ladrido andLeonor P. Ladrido were the


owners of Lot No. 7511 of the Cadastral Survey ofPototan
situated in barangay Cawayan, Pototan, Iloilo. Spouses
Rosendo H. Teand Ana Te were also the registered owners of a
parcel of land described intheir title as Lot No. 7340 of the
Cadastral Survey of Pototan. On September 6,1973, Rosendo
H. Te, with the conformity of Ana Te, sold this lot to AngelicaF.
Viajar and Celso F. Viajar for P5,000. A Torrens title was later
issued inthe names of Angelica F. Viajar and Celso F. Viajar.
Later, Angelica F. Viajarhad Lot No. 7340 relocated and found
out that the property was in thepossession of Ricardo Y.
Ladrido. Consequently, she demanded its return butLadrido
refused. On February 15, 1974, Angelica F. Viajar and Celso F.
Viajarinstituted a civil action for recovery of possession and
damages
againstRicardo
Y.
Ladrido.
Summoned
to
plead,defendant Ladrido filed his answer with a counterclaim.
The Viajars filed theirreply to the answer. Subsequently, the
complaint was amended to implead RosendoH. Te as another
defendant. The Viajars sought the annulment of the deed
ofsale and the restitution of the purchase price with interest in
the event thepossession of defendant Ladrido is sustained.
Defendant Te filed his answer tothe amended complaint and he
counter claimed for damages. The Viajars answeredthe
counterclaim. During the pendency of the case, plaintiff Celso
F. Viajarsold his rights over Lot No. 7340 to his mother and coplaintiff, Angelica F.Viajar. For this reason, plaintiff Angelica F.
Viajar now appears to be the soleregistered owner of this lot.
On May 25, 1978, defendant Ladrido died. He wassubstituted
in the civil action by his wife, Leonor P. Ladrido, and their
children.
The facts admitted by the partiesduring the pre-trial show that
the piece of real property which used to be LotNo. 7340 of the
Cadastral Survey of Pototan was located in barangay
GuibuanoganPototan, Iloilo; that it consisted of 20,089 square
meters; that at the time ofthe cadastral survey in 1926, Lot
No. 7511 and Lot No. 7340 were separated bythe Suague
River; that the area of 11,819 square meters of what was Lot
No.7340 has been in the possession of the defendants; that
the area of 14,036square meters, which was formerly the river
bed of the Suague River percadastral survey of 1926, has also
been in the possession of the defendants;and that the
plaintiffs have never been in actual physical possession of
LotNo. 7340. On December 10, 1981, the trial court rendered

its decision in favorof the defendants Ladrido. Not satisfied


with the decision, the plaintiffsappealed to the Court of
Appeals. In thedecision appealed from, the Court of Appeals
held that the appeal of Viajar isnot impressed with merit.

ISSUE

Whetherthe registration of the land is not a mode of


acquiring ownership

RULING

TheCourt affirmed the decision of the lower court.

The rulethat registration under the Torrens System does not


protect the riparian owneragainst the diminution of the area of
his registered land through gradualchanges in the course of an
adjoining stream is well settled.

Thecontroversy in the present cases seems to be due to the


erroneous conceptionthat Art. 366 of the Civil Code does not
apply to Torrens registered land. Thatarticle provides that "any
accretions which the banks of rivers maygradually receive from
the effects of the current belong to the owners of theestates
bordering thereon." Accretions of that character are
naturalincidents to land bordering on running streams and are
not affected by theregistration laws. It follows that
registration does not protect the riparianowner against
diminution of the area of his land through gradual changes in
thecourse of the adjoining stream.

In C.N.Hodges vs. Garcia, 109 Phil. 133, The Court ruled:

It clearlyappearing that the land in question has become part


of defendant's estate as aresult of accretion, it follows that
said land now belongs to him. The factthat the accretion to his
land used to pertain to plaintiffs estate, which iscovered by a
Torrens Certificate of Title, cannot preclude him (defendant)
frombeing the owner thereof. Registration does not protect the
riparian owneragainst the diminution of the area of his land
through gradual changes in the courseof the adjoining stream.
Accretions which the banks of rivers may graduallyreceive
from the effect of the current become the property of the
owners of thebanks (Art. 366 of the Old Civil Code; Art. 457 of
the New). Such accretionsare natural incidents to land
bordering on running streams and the provisionsof the Civil
Code in that respect are not affected by the Registration Act.

Part 2

Non-registrability of navigablerivers

G.R. No. L-31271

April 29, 1974

ROMEO MARTINEZ and LEONOR SUAREZ,spouses, petitionersappellants,

vs.

HON. COURT OF APPEALS, SECRETARY andUNDERSECRETARY


OF PUBLIC WORKS & COMMUNICATIONS, respondentsappellees.

FACTS

The spouses Romeo Martinez andLeonor Suarez, now


petitioners-appellees, are the registered owners of two
(2)parcels of land located in Lubao, Pampanga. Both parcels of
land are fishponds.The disputed property was originally owned
by one Paulino Montemayor, whosecured a "titulo real" over it
way back in 1883. After the death ofPaulino Montemayor the
said property passed to his successors-in-interest,Maria
Montemayor and Donata Montemayor, who in turn, sold it, as
well as thefirst parcel, to a certain Potenciano Garcia. Because
Potenciano Garcia wasprevented by the then municipal
president of Lubao, Pedro Beltran, fromrestoring the dikes
constructed on the contested property, the former, on June22,
1914, filed Civil Case No. 1407 with the Court of First Instance
againstthe said Pedro Beltran to restrain the latter in his
official capacity frommolesting him in the possession of said
second parcel, and on even date,applied for a writ of
preliminary injunction, which was issued against saidmunicipal
president. The Court, by decision promulgated June 12, 1916,
declaredpermanent the preliminary injunction, which, decision,
on appeal, was affirmedby the Supreme Court on August 21,
1918. From June 22, 1914, the dikes aroundthe property in
question remained closed until a portion thereof was
againopened just before the outbreak of the Pacific War.

On April17, 1925. Potenciano Garcia applied for the


registration of both parcels ofland in his name, and the Court
of First Instance of Pampanga, sitting as landregistration
court, granted the registration over and against the opposition
ofthe Attorney-General and the Director of Forestry. Pursuant
to the Court'sdecision, original certificate of title No. 14318,
covering said parcels 1 and2 was issued to the spouses
Potenciano Garcia and Lorenza Sioson. These parcelsof land
were subsequently bought by Emilio Cruz de Dios in whose
name transfercertificate of title No. 1421 was first issued on
November 9, 1925. Thereafter,the ownership of these
properties changed hands until eventually they wereacquired
by the herein appellee spouses who hold them by virtue of
transfercertificate of title No. 15856.

To avoid any untoward incident,the disputants agreed to refer


the matter to the Committee on Rivers andStreams, by then
composed of the Honorable Pedro Tuason, at that time
Secretaryof Justice, as chairman, and the Honorable Salvador
Araneta and Vicente Orosa,Secretary of Agriculture and
National Resources and Secretary of Public Worksand
Communications, respectively, as members. This committee
thereafterappointed a Sub-Committee to investigate the case
and to conduct an ocularinspection of the contested property,
and on March 11, 1954, said Sub-Committeesubmitted its
report to the Committee on Rivers and Streams to the effect
thatParcel No. 2 of transfer certificate of title No. 15856 was
not a public riverbut a private fishpond owned by the herein
spouses.

On July 7,1954, the Committee on Rivers and Streams rendered


its decision restoring theownership to the exclusive possession
of spouses Martinez

Themunicipal officials of Lubao, led by Acting Mayor Mariano


Zagad, apparentlyrefused to recognize the above decision,
because on September 1, 1954, thespouses Romeo Martinez
and Leonor Suarez instituted Civil Case No. 751 beforethe

Court of First Instance of Pampanga against said Mayor Zagad,


praying thatthe latter be enjoined from molesting them in their
possession of theirproperty and in the construction of the
dikes therein. The writ of preliminaryinjunction applied for was
issued
against
the
respondent
municipal
Mayor,
whoimmediately elevated the injunction suit for review to the
Supreme Court, whichdismissed Mayor Zagad's petition on
September 7, 1953. With this dismissalorder herein appellee
spouses
proceeded
to
construct
the
dikes
in
the
disputedparcel of land.

Some four(4) years later, and while Civil Case No. 751 was still
pending the HonorableFlorencio Moreno, then Secretary of
Public
Works
and
Communications,
orderedanother
investigation of the said parcel of land, directing the
appelleesherein to remove the dikes they had constructed, on
the strength of theauthority vested in him by Republic Act No.
2056, approved on June 13, 1958,entitled "An Act To Prohibit,
Remove and/or Demolish the Construction ofDams. Dikes, Or
Any Other Walls In Public Navigable Waters, Or Waterways and
InCommunal Fishing Grounds, To Regulate Works in Such
Waters or Waterways And InCommunal Fishing Grounds, And
To Provide Penalties For Its Violation, And ForOther Purposes.
The
said
order
which
gave
rise
to
the
instant
proceedings,embodied a threat that the dikes would be
demolished should the hereinappellees fail to comply
therewith within thirty (30) days.

The spouses Martinez replied tothe order by commencing on


January 2, 1959 the present case, which was decidedin their
favor by the lower Court. Asagainst this judgment respondent
officials
of
the
Department
of
Public
Worksand
Communications took the instant appeal.

ISSUE

Whethernavigable rivers could be registered as private


property

RULING

TheCourt affirmed that the judgment of the Court of


Appeals appealed from is inaccordance with law . The abovementioned properties are parts of the publicdomain intended
for public use, therefore they are outside the commerce of
menand not subject to private appropriation.

In Ledesmav. Municipality of Iloilo, 49 Phil. 769, the Court held:

A simplepossession of a certificate of title under the Torrens


system does notnecessarily make the possessor a true owner
of all the property describedtherein. If a person obtains title
under the Torrens system which includes bymistake or
oversight, lands which cannot be registered under the
Torrenssystem, he does not by virtue of said certificate alone
become the owner of theland illegally included.

In Mercadov. Municipal President of Macabebe, 59 Phil. 592, it


was also said:

It isuseless for the appellant now to allege that she has


obtained certificate oftitle No. 329 in her favor because the
said certificate does not confer uponher any right to the creek
in question, inasmuch as the said creek, being ofthe public
domain, is included among the various exceptions enumerated
inSection 39 of Act 496 to which the said certificate is subject
by expressprovision of the law.

The sameruling was laid down in Director of Lands v. Roman


Catholic Bishop ofZamboanga, 61 Phil. 644, as regards public
plaza.

In Dizon,et al. v. Rodriguez, et al., G.R. No. L-20300-01 and


G.R. No. L-20355-56, April30, 1965, 20 SCRA 704, it was held
that the incontestable and indefeasiblecharacter of a Torrens
certificate of title does not operate when the landcovered
thereby is not capable of registration.

It is now clearthat the authorities cited by the appellants as to


the conclusiveness andincontestability of a Torrens certificate
of title do not apply here. The LandRegistration Court has no
jurisdiction over non-registerable properties, suchas public
navigable rivers which are parts of the public domain, and
cannotvalidly adjudge the registration of title in favor of a
private applicant.Hence, the judgment of the Court of First
Instance of Pampanga as regards theLot No. 2 of Certificate of
Title No. 15856 in the name ofpetitioners-appellants may be
attacked at any time, either directly orcollaterally, by the State
which is not bound by any prescriptive periodprovided for by
the Statute of Limitations (Article 1108, par. 4, new CivilCode).
The right of reversion or reconveyance to the State of the
publicproperties fraudulently registered and which are not
capable of privateappropriation or private acquisition does not
prescribe.

When it comes to registeredproperties, the jurisdiction of the


Secretary of Public Works &Communications under Republic
Act 2056 to order the removal or obstruction tonavigation
along a public and navigable creek or river included therein,
hasbeen definitely settled. The evidence submitted before the
trial court whichwas passed upon by the respondent Court of
Appeals shows that Lot No. 2 (PlanPsu 992) of Transfer
Certificate of Title No. 15856, is a river of the publicdomain.

The rulingthat a purchaser of a registered property cannot go


beyond the record to makeinquiries as to the legality of the

title of the registered owner, but may relyon the registry to


determine if there is no lien or encumbrances over the
same,cannot be availed of as against the law and the accepted
principle that riversare parts of the public domain for public
use and not capable of privateappropriation or acquisition by
prescription.

Part 3
NATURE OF RECLAIMED LANDS
G.R. No. 133250 July 9,2002
FRANCISCO I. CHAVEZ, PETITIONER,
AUTHORITY
AND
AMARICOASTAL
CORPORATION, RESPONDENTS

VS. PUBLIC ESTATES


BAY
DEVELOPMENT

FACTS
On
November
20,
1973,
the
government,
through
theCommissioner of Public Highways, signed a contract with
the
Construction
andDevelopment
Corporation
of
the
Philippines (CDCP) to reclaim certain foreshoreand offshore
areas of Manila Bay. On February 4, 1977, then President
FerdinandE. Marcos issued Presidential Decree No. 1084
creating PEA - tasked to reclaim land, including foreshoreand
submerged areas, and to develop, improve, acquire, x x x
lease and sellany and all kinds of lands. On the same date,
then President Marcos issuedPresidential Decree No. 1085

transferring to PEA the lands reclaimed in theforeshore and


offshore of the Manila Bay under the Manila-Cavite Coastal
Roadand Reclamation Project (MCCRRP).
OnJanuary 19, 1988, then President Corazon C. Aquino issued
Special Patent No.3517, granting and transferring to PEA the
parcels of land so reclaimed underthe Manila-Cavite Coastal
Road and Reclamation Project (MCCRRP) .Subsequently,on
April 9, 1988, the Register of Deeds of the Municipality of
Paraaque issuedTransfer Certificates of Title in the name of
PEA, covering the three reclaimedislands known as the
Freedom Islands located at the southern portion of
theManila-Cavite Coastal Road, Paraaque City. On April 25,
1995, PEA entered intoa Joint Venture Agreement (JVA) with
AMARI, a private corporation, to developthe Freedom Islands.
PEA
and
AMARI
entered
into
the
JVA
through
negotiationwithout public bidding. On June 8, 1995, then
President Fidel V. Ramos, throughthen Executive Secretary
Ruben Torres, approved the JVA.
OnNovember 29, 1996, then Senate President Ernesto Maceda
delivered a privilegespeech in the Senate and denounced the
JVA as the grandmother of all scams.As a result, the Senate
Committee on Government Corporations and PublicEnterprises,
and the Committee on Accountability of Public Officers
andInvestigations, conducted a joint investigation. Among the
conclusions of theirreport are: (1) the reclaimed lands PEA
seeks to transfer to AMARI under theJVA are lands of the public
domain which the government has not classified asalienable
lands and therefore PEA cannot alienate these lands; (2)
thecertificates of title covering the Freedom Islands are thus
void, and (3) theJVA itself is illegal.
OnDecember 5, 1997, then President Fidel V. Ramos issued
PresidentialAdministrative Order No. 365 creating a Legal Task
Force to conduct a study onthe legality of the JVA. The Legal
Task Force upheld the legality of the JVA,contrary to the
conclusions reached by the Senate Committees.
On April27, 1998, petitioner Frank I. Chavez (Petitioner for
brevity) as a taxpayer,filed the instant Petition for Mandamus
with Prayer for the Issuance of aWrit of Preliminary Injunction
and Temporary Restraining Order. Petitionercontends the
government stands to lose billions of pesos in the sale by PEA
ofthe reclaimed lands to AMARI. Petitioner prays that PEA
publicly disclose theterms of any renegotiation of the JVA,

invoking Section 28, Article II, andSection 7, Article III, of the


1987 Constitution on the right of the people toinformation on
matters of public concern. Petitioner assails the sale to
AMARIof lands of the public domain as a blatant violation of
Section 3, Article XIIof the 1987 Constitution prohibiting the
sale of alienable lands of the publicdomain to private
corporations. In a Resolution dated March 23, 1999, the
Courtgave due course to the petition and required the parties
to file theirrespective memoranda.
On March30, 1999, PEA and AMARI signed the Amended Joint
Venture Agreement (Amended JVA).On May 28, 1999, the Office
of the President under the administration of thenPresident
Joseph E. Estrada approved the Amended JVA.
Due tothe approval of the Amended JVA by the Office of the
President, petitioner nowprays that on constitutional and
statutory grounds the renegotiated contractbe declared null
and void.
ISSUE
WHETHERTHE Lands reclaimed from foreshore andsubmerged
areas also form part of the public domain and are also
inalienable
WHETHERTHE STIPULATIONS IN THE AMENDED JOINT VENTURE
AGREEMENT FOR THE TRANSFER TOAMARI OF CERTAIN LANDS,
RECLAIMED AND STILL TO BE RECLAIMED, VIOLATE THE
1987CONSTITUTION
RULING
The 1987 Constitution, like the 1935 and 1973 Constitutions
before it,has adopted the Regalian doctrine. The 1987
Constitution declares that allnatural resources are owned by
the State, and except foralienable agricultural lands of the
public domain, natural resources cannot bealienated.
UnderSection 2, Article XII of the 1987 Constitution, the
foreshore and submergedareas of Manila Bay are part of the
lands of the public domain, waters x x xand other natural
resources and consequently owned by the State. As
such,foreshore and submerged areas shall not be alienated,
unless they areclassified as agricultural lands of the public
domain. The mere reclamationof these areas does not convert

these inalienable natural resources of theState into alienable


or disposable lands of the public domain. There must be alaw
or presidential proclamation officially classifying these
reclaimed landsas alienable or disposable and open to
disposition or concession. Moreover,these reclaimed lands
cannot be classified as alienable or disposable if thelaw has
reserved them for some public or quasi-public use.
Landsreclaimed by the government are sui generis, not
available forsale to private parties unlike other alienable
public lands. Reclaimed landsretain their inherent potential as
areas for public use or public service.
Clearly,the Amended JVA violates glaringly Sections 2 and 3,
Article XII of the 1987Constitution. Under Article 1409 of the
Civil Code, contracts whose object orpurpose is contrary to
law, or whose object is outside the commerce of men,are
inexistent and void from the beginning. The Court must
perform its dutyto defend and uphold the Constitution, and
therefore declares the Amended JVAnull and void ab initio.

PART 4
VIII. PROCEEDINGS FORREGISTRATION OF LAND
A.

JUDICIAL REGISTRATION

h. Amendments. Need forpublication and notice if change are


substantial
i. DIRECTOR OF LANDS VS. IAC, 219 SCRA 399
G.R. No. 73246

March2, 1993

DIRECTOR OF LANDS AND DIRECTOR OF FORESTDEVELOPMENT,


petitioners,

vs.

INTERMEDIATE APPELLATE COURT AND J. ANTONIOARANETA,


respondents.
FACTS
The case is aboutthe registration of the Tambac Island in
Lingayen Gulf situated in theMunicipality of Bani, Pangasinan.
The initial application for registration wasfiled for Pacific
Farms, Inc. under the provisions of the Land RegistrationAct,
Act No. 496, as amended. The Republic of the Philippines, thru
theDirector of Lands opposed the application alleging that the
applicant, PacificFarms, Inc. does not possess a fee simple title
to the land nor did itspredecessors possess the land for at
least thirty (30) years immediatelypreceding the filing of
application. The opposition likewise specificallyalleged that the
applicant is a private corporation disqualified under the(1973)
new Philippine Constitution from acquiring alienable lands of
the publicdomain citing Section 11, Article 14. 2 The Director
of Forest Development alsoentered its opposition alleging that
the land is within the unclassified publicland and, hence,
inalienable. Other private parties also filed theiroppositions,
but were subsequently withdrawn.

In an amendedapplication, Pacific Farms, Inc. filed a


manifestation-motion to change theapplicant from Pacific
Farms,
Inc.
to
J.
Antonio
Araneta.
Despite
the
supposedamendment, there was no republication.Evidence
presented by the applicantinclude the testimony of Placido
Orlando, fishery guard of Pacific Farms, Inc.,who said he has
known the disputed land since he attained the age of reason
forsome forty (40) years now; that when he first came to know
the property it wasthen owned by and in the possession of
Paulino Castelo, Juan Ambrosio and JulioCastelo, and later on
the whole island was bought by Atty. Vicente Castelo whoin
turn sold it to J. Antonio Araneta.

While this case ispending, respondent filed an Omnibus Motion


for Substitution of privaterespondent. Apparently, Antonio
Araneta had assigned his rights to and interestin Tambac
Island to Amancio R. Garcia who in turn assigned his rights and
interest in the same property toJohnny A. Khonghun whose
nationality was not alleged in the pleadings.
On October 4, 1979,the trial court rendered a decision
adjudicating the subject property to J.Antonio Araneta. On
appeal to the then Intermediate Appellate Court, thedecision
of the lower court was affirmed on December 12, 1985.
ISSUE
Whether the lowercourt erred in adjudicating the land to the
applicant under the provisions ofPresidential Decree No. 1529,
otherwise known as the Property RegistrationDecree, despite
absence of any specific invocation of this law in the originaland
amended application.
RULING
The Court agree with petitioners that the amendment ofthe
application from the name of Pacific Farms Inc., as applicant,
to the nameof J. Antonio Araneta Inc., was a mere attempt to
evade disqualification. The1973 even the 1987 Philippine
Constitution prohibits private corporations orassociations from
holding alienable lands of the public domain except by
lease.Apparently realizing such prohibition, respondent
amended its application toconform with the mandates of the
law. However, the Court did not agree withpetitioners' position

that the absence of republication of an amendedapplication for


registration
is
a
jurisdictional
flaw.
Amendments
to
theapplication may be due to change in parties or substantial
change in theboundaries or increase in the area of the land
applied for. Neither the LandRegistration Act, as amended, nor
Presidential Decree No. 1529, otherwise knownas the Property
Registration
Decree,
requires
republication
and
registrationmay be allowed by the court at any stage of the
proceeding upon just and reasonableterms. On the other hand,
republication is required if the amendment is due tosubstantial
change in the boundaries or increase in the area of the
landapplied for.

PART 5

THE POWER TO CLASSIFY OR RECLASSIFY PUBLIC LANDSINTO


ALIENABLE AND DISPOSABLE LANDS BELONG TO THE
EXECUTIVE BRANCH OF THEGOVERNMENT

G.R. No. L-66866 June18, 1987

REPUBLIC OF THE PHILIPPINES, petitioner,


vs.
MINDA
DE
PORKAN,
SADIN
MARAUG,
GORGONIO
BERMUDEZ,LOLITA MACATINDOG, MEDORI DE PORKAN, JUAN
ARANGALI, ANTONINA ESTARES, REGISTEROF DEEDS OF DAVAO
DEL NORTE and the INTERMEDIATE APPELLATE COURT [Fourth
CivilCases Division], respondents.
FACTS

The family of Sadin dePorkan, father of Medori and Macampon


de Porkan, both native Muslims of La Paz,Carmen, Davao del
Norte, had been in actual possession as owner since
theSpanish colonial period of a tract of land planted with
coconuts situated insaid municipality. During the Tagum
Cadastral Survey of July 22, 1937, thistract of land, were
respectively allocated to Medori and Macampon de Porkan,the
predecessors-in-interest of the private respondents. For the
succeedingyears, there had been changes in the ownership of
the land within the de Porkanfamily which was approved by
the government until a problem arose at the timeof ownership
of Minda de Porkan and Lolita Macatindog.

On November 26, 1969, thePhilippine Fisheries Commissioner


ordered the Regional Director at Davao Cityto investigate the
conflict of fishpond applications between Viola C. Azurinand
Moonyeen R. Beleno. On May 26, 1972,while the fishpond
conflict
case
was
pending
investigation
by
the
PhilippineFisheries Commission [Davao City], Viola C. Azurin
filed with the Bureau ofLands a complaint for the
correction,amendment or cancellation of Homestead Patent of
Minda de Porkan and FreePatent of Lolita Macatindog situated
at La Paz, Panabo [now Carmen], Davao delNorte, alleging,
among others, that the patentees secured their patents
andtitles through fraud, misrepresentation and illegal
machinations.

Minda de Porkan andLolita Macatindog similarly claimed in


their separate answers that they werethe ones who first
protested, along with their Muslim relatives, the intrusionof
the families of the Azurins, Rodriguezes, and Belenos into their
"ancestrallands, which from time immemorial had been
occupied and cultivated by theirancestors and predecessors-ininterest.
After a joint hearing,the Court of First Instance of Davao del
Norte, Branch [Tagum] rendered itsdecision 41 on November
18, 1980 dismissing the complaints for cancellation oftitles and
upholding the validity of the patents/titles of Lolita
Macatindogand Minda de Porkan, as well as the titles of their
transferees
co-defendantsJuan
Arangali
and
Gorgonio

Bermudez, who were adjudged to be innocentpurchasers for


value and in good faith.

The Solicitor Generalappealed the decision of the lower court


to the then Intermediate AppellateCourt, now Court of
Appeals.
On
February
29,
1984,
the
Intermediate
AppellateCourt, now Court of Appeals, affirmed the decision of
the lower court.

ISSUE

Whetherthe executive branch of government has the


power to classify and reclassifypublic lands into alienable and
disposable lands

RULING

It is significant to notethat the tract of public land then


possessed, occupied, developed and plantedto coconuts by the
family of Sadin de Porkan and his predecessors-in-interest,all
native Muslims of La Paz, Carmen [formerly Panabo], Davao del
Norte, which,by virtue of its being part of the unregistered
lands, was included in theTagum Cadastral Survey of July 22,
1937 and formed part of the disposable oralienable
agricultural lands of the public domain referred to under
Section 6,par. [a] in relation to Section 9, par. [a] of the Public
Land Act [C.A. 141,as amended]. The nature and character of
said tract of public land, moreparticularly Lot No. 1099, as one
found inside an "agriculturalzone", and that of Lot No. 1546, as
one suitable for rice cultivation,which were categorically
stated in the separate investigation reports in 1953of Vicente
J. Villena, junior public land inspector of the Bureau of
Lands[Davao] is binding on the courts inasmuch as it is the

exclusive prerogative ofthe Executive Department of the


Government to classify public lands .

In the case of Mapa vs.Insular Government, 10 Phil. 175, this


court said that the phrase 'agriculturallands' as used in Act No.
926 means those public lands acquired from Spainwhich are
not timber or mineral. Whatever may have been the meaning
of the term'forestry' under the Spanish law, the Act of
Congress of July 1st, 1902,classifies the public lands in the
Philippine Islands as timber, mineral oragricultural lands, and
all public lands that are not timber or mineral landsare
necessarily agricultural public lands, whether they are used as
nipaswamps, manglares, fisheries or ordinary farm lands.

Since the disputed tractof public land is neither timber nor


mineral lands, the same is alienable oropen to disposition as
public agricultural lands, under Section 11, C.A. 141thru
homestead settlement or free patent.

Where, as in the instantcase, the possession of a public land


by Sadin de Porkan, father of Medori dePorkan, and their
predecessors-in-interest who were native Muslims of la
Paz,Panabo [now Carmen], Davao del Norte, dates back to the
time of the Spanishcolonial period, such possession of the said
tract of public land has attainedthe character and duration
prescribed by law as the equivalent of an expressgrant from
the Government. The mandate of the law itself is that the
possessors"shall be conclusively presumed to have performed
all the conditionsessential to a Government grant and shall be
entitled to a certificate oftitle" and by legal fiction, the land
ceases to be public and thus becomesprivate land.

Part 6
TAX DECLARATION
G.R. No. 177797, December 04, 2008
SPS. PEDRO TAN AND NENA ACERO TAN, PETITIONERS,
VS.
REPUBLIC OF THE PHILIPPINES, RESPONDENT
FACTS
The spouses Pedro Tan and Nena AceroTan were natural-born
Filipino citizens, who became Australian citizens on 9February
1984. They seek to have thesubject property registered in
their names. The subject property was declaredalienable and

disposable on 31 December 1925, as established by a


Certificationdated 14 August 2000 issued by the Department of
Environment and NaturalResources (DENR), Community
Environment and Natural Resources Office (CENRO),Cagayan
de Oro City. Prior to the spouses Tan, the subject property was
in thepossession of Lucio and Juanito Neri and their respective
spouses. Lucio and Juanito Neri had declared thesubject
property for taxation purposes in their names.The spouses Tan
acquiredthe subject property from Lucio and Juanito Neri and
their spouses by virtue ofa duly notarized Deed of Sale of
Unregistered Real Estate Property dated 26June 1970. The
spouses Tan tookimmediate possession of the subject property
on which they planted rubber,gemelina, and other fruit-bearing
trees. They declared the subject property for taxation
purposes in their names andpaid realty taxes thereon.

However, a certain Patermateo Casio(Casio) claimed a


portion of the subject property, prompting the spouses Tanto
file a Complaint for Quieting of Title against him before the
RTC of Cagayande Oro City. On 29 August 1989, the RTC
rendered a Decision favoring thespouses Tan and declaring
their title to the subject property thus"quieted." Casio
appealed thesaid RTC Decision to the Court of Appeals . In a
Resolution the appellate courtdismissed for lack of interest to
prosecute. Casio elevated his case to this Court via a Petition
for Review onCertiorari. In a Resolution dated 13 March 1991
the Court denied Casio's Petition for beinginsufficient in form
and substance. Thesaid Resolution became final and executory
on 3 June 1991. Refusing to give up,Casio filed an Application
for Free Patent on the subject property before theBureau of
Lands. On 8 December 1999, Casio's application was ordered
cancelledby Officer Ruth G. Sabijon of DENR-CENRO, Cagayan
de Oro City, upon the requestof herein petitioner Pedro Tan,
the declared owner of the subject propertypursuant to the 29
August 1989 Decision of the RTC. In 2000, the spouses Tanfiled
their Application for Registration of Title to the subject
propertybefore the RTC of Cagayan de Oro City. The application
of the spouses Taninvoked the provisions of Act No. 496 and/or
Section 48 of Commonwealth Act No.141,as amended. In
compliance with therequest of the Land Registration Authority
(LRA) dated 29 August 2000, thespouses Tan filed on 5 October
2000 an Amended Application for Registration ofTitle to the
subject property.

The Office of the Solicitor General(OSG) entered its appearance


on behalf of the Republic, but failed to submit awritten
opposition to the application of the spouses Tan.

When no opposition to theapplication of the spouses Tan was


filed by the time of the initial hearing theRTC issued on 23
April 2001 an order of general default, except as against
theRepublic. Thereafter, the spouses Tanwere allowed to
present their evidence ex-parte.

After the establishment of thejurisdictional facts, the RTC


heard the testimony of John B. Acero , nephew andlone witness
of the spouses Tan. After Acero's testimony, the spouses
Tanalready made a formal offer of evidence, which was
admitted by the court a quo.On 9 May 2001, the RTC rendered
a Decision granting the application of thespouses Tan. The
Republic appealed the RTC Decision to the Court of Appeals.

On 28 February 2006, the Court ofAppeals rendered a Decision


granting the appeal of the Republic, and reversingand setting
aside the 9 May 2001 Decision of the RTC on the ground that
thespouses Tan failed to comply with Section 48(b) of
Commonwealth Act No. 141,otherwise known as the Public
Land Act, as amended by Presidential Decree No.1073, which
requires possession of the subject property to start on or prior
to12 June 1945. Hence, the appellate court ordered the
spouses Tan to return thesubject property to the Republic.

The spouses Tan filed a Motion forReconsideration of the


foregoing Decision of the Court of Appeals. To refute the
finding of the appellate courtthat they and their predecessorsin-interest did not possess the subjectproperty by 12 June
1945 or earlier, the spouses Tan attached to their Motion acopy
of Tax Declaration No. 4627 covering the subject property
issued in 1948in the name of their predecessor-in-interest,
Lucio Neri. They called attention to the statement in

TaxDeclaration No. 4627 that it cancelled Tax Declaration No.


2948. Unfortunately, no copy of Tax Declaration No.2948 was
available even in the Office of the Archive of the Province of
MisamisOriental. The spouses Tan asserted that judicial notice
may be taken of thefact that land assessment is revised by the
government every four years; andsince Tax Declaration No.
4627 was issued in the year 1948, it can bepresupposed that
Tax Declaration No. 2948 was issued in the year 1944.

The Court of Appeals denied the Motion for Reconsiderationof


the spouses Tan in a Resolution dated 12 April 2007.

Theearliest evidence of possession and occupation of the


subject property can betraced back to a tax declaration issued
in the name of theirpredecessors-in-interest only in 1952.
However,the spouses Tan are now asking the kind indulgence
of this Court to take intoaccount Tax Declaration No. 4627
issued in 1948, which they had attached totheir Motion for
Reconsideration before the Court of Appeals but which
theappellate court refused to consider. Just as they had argued
before the Court of Appeals, the spouses Tanpoint out that Tax
Declaration No. 4627 was not newly issued but cancelled
TaxDeclaration No. 2948; and should the Court take judicial
notice of the factthat tax assessments are revised every four
years, then Tax Declaration No.2948 covering the subject
property was issued as early as 1944.

ISSUE
WHETHER TAXDECLARATIONS AND RECEIPTS ARE CONCLUSIVE
EVIDENCE OF OWNERSHIP

RULING

Tax declarations and receipts are not conclusive evidence


ofownership. At most, they constitute mere prima facie proofs

of ownershipof the property for which taxes have been paid. In


the absence of actual,public and adverse possession, the
declaration of the land for tax purposesdoes not prove
ownership.They
may
be
good
supporting
or
collaboratingevidence together with other acts of possession
and ownership; but bythemselves, tax declarations are
inadequate to establish possession of theproperty in the
nature and for the period required by statute for
acquiringimperfect or incomplete title to the land.

The spouses Tan purchased the subject property and came


intopossession of the same only in 1970. To justify their
application forregistration of title, they had to tack their
possession of the subjectproperty to that of their
predecessors-in-interest. While the spouses Tan undoubtedly
possessedand
occupied
the
subject
property
openly,
continuously, exclusively andnotoriously, by immediately
introducing improvements on the said property, inaddition to
declaring the same and paying realty tax thereon; in
contrast,there was a dearth of evidence that their
predecessors-in-interest possessedand occupied the subject
property in the same manner. The possession and occupation
of the subjectproperty by the predecessors-in-interest of the
spouses Tan were evidenced onlyby the tax declarations in the
names of the former, the earliest of which, TaxDeclaration No.
4627, having been issued only in 1948. No other evidence was
presented by thespouses Tan to show specific acts of
ownership exercised by theirpredecessors-in-interest over the
subject property which may date back to 12June 1945 or
earlier.

For
failure
of
the
Spouses
Tan
to
satisfy
the
requirementsprescribed by Section 48(b) of the Public Land
Act, as amended, this Court hasno other option but to deny
their application for judicial confirmation andregistration of
their title to the subject property.

Part 7

DECREE OF REGISTRATION

G.R. No.123346, December 14, 2007

MANOTOK
REALTY,
INC.
AND
MANOTOK
ESTATE
CORPORATION,PETITIONERS, VS. CLT REALTY DEVELOPMENT
CORPORATION, RESPONDENT

FACTS

On 10August 1992, CLT Realty Development


Corporation (CLT) sought to recover fromManotok Realty, Inc.
and Manotok Estate Corporation (Manotoks) the possessionof
Lot 26 of the Maysilo Estate in an action filed before the
Regional TrialCourt of Caloocan City. CLTs claim was anchored
on Transfer Certificate ofTitle (TCT) issued in its name by the
Caloocan City Register of Deeds, whichtitle in turn was derived
from Estelita Hipolito (Hipolito) by virtue of a Deedof Sale with
Real Estate Mortgage dated 10 December 1988. Hipolitos
titleemanated from Jose Dimsons (Dimson) TCT , a title issued
pursuant to an orderof the Court of First Instance (CFI) of

Caloocan City. Dimsons title appearsto have been sourced


from OCT No. 994.

For their part, the Manotoks challenged the validity of thetitle


relied on by CLT, claiming that Dimsons title, the proximate
source ofCLTs title, was irregularly issued and, hence, the
same and subsequent titlesflowing therefrom are likewise
void. The Manotoks asserted their ownership overLot 26 and
claimed that they derived it from several awardees and/or
vendees ofthe National Housing Authority. The Manotok title
likewise traced as itsprimary source OCT No. 994 which, on 9
September 1918, was transferred toAlejandro Ruiz and
Mariano Leuterio who had previously acquired the property
on21 August 1918 by virtue of an Escritura de Venta
executed by Don TomasArguelles and Don Enrique Llopis. On 3
March 1920, Ruiz and Leuterio sold theproperty to Francisco
Gonzalez who held title thereto until 22 August 1938 whenthe
property was transferred to Jose Leon Gonzalez, Consuelo
Susana Gonzalez,Juana Francisca Gonzalez, Maria Clara
Gonzalez, Francisco Felipe Gonzalez andConcepcion Maria
Gonzalez under TCT No. 35486. The lot was then, per
annotationdated 21 November 1946, subdivided into seven (7)
parcels each in the name ofeach of the Gonzalezes.

The trial court, ruling for CLT, adopted the factualfindings and
conclusions arrived at by the majority commissioners
appointed toresolve the conflict of titles. It was established
that the entire MaysiloEstate was registered under Act No. 496
by virtue of which OCT No. 994 wasissued by the Register of
Deeds of Rizal;that Lot 26 was transferred to CLT byHipolito
whose title was derived from the Dimson title and that on the
basis ofthe technical descriptions of the property appearing in
the Manotok titles, thelatters property indeed encroached on
the property described in CLTs title.

The Manotoks appealed to the Court of Appeals, whichaffirmed


the
decision
of
the
trial
court.
Their
motion
for
reconsiderationhaving been denied, they filed a petition for
review with the Supreme Court,ascribing error to the appellate
court in upholding the trial courts decisionwhich decided the

case on the basis of the majority commissioners report


andoverlooked relevant facts in the minority commissioners
report.

ISSUE

whether the titles issued in thename of CLT IS valid

RULING

With respect to G.R. No. 123346, the Court upheld thevalidity


of the trial courts adoption of the commissioners majority
report aspart of the decision. The Court pointed out that the
titles of respondents inall three cases were derived from OCT
No. 994 of the Registry of Deeds ofCaloocan City registered on
19 April 1917. The Manotoks filed their respectivemotions for
reconsideration. On 5 June 2006, the cases were elevated to
theCourt en banc. In the Manotok petition, CLT hadoriginally
filed a complaint for annulment of the titles in the name of
theManotoks, alleging that it was the registered owner of Lot
26 of the MaysiloEstate. It is evident from all three titles
CCLTs, Hipolitos andDimsonsthat the properties they
purport to cover were originally registeredon the 19th day
April 1917 in the Registration Book of the Office of theRegister
of Deeds of Rizal. As earlier established, there is no such OCT
No.994 originally registered on 19 April 1917. None of these
three titles can beaccorded recognition simply because the
original title commonly referred totherein never existed. To
conclude otherwise would constitute deliberatedisregard of
the truth. These titles could be affirmed only if it can be
proventhat OCT No. 994 registered on 19 April 1917 had
actually existed. CLT and theDimsons were given the
opportunity to submit such proof before this Court, butthey did
not. In fact, CLT has specifically manifested that the OCT No.

994they concede as true is also the one which the Office of


Solicitor Generalsubmitted as true, and that is OCT No. 994
issued on 3 May 1917.Given thisessential clarification, there is
no sense in affirming the 2005 Decision whichsustained the
complaints for annulment of title and/or recovery of
possessionfiled by CLT and the Dimson when their causes of
action are both founded on aninexistent mother title.

From these premises, the Court is able to make the


followingbinding conclusions. First, there is only one OCT No.
994. As it appears on therecord, that mother title was received
for transcription by the Register ofDeeds on 3 May 1917, and
that should be the date which should be reckoned asthe date
of registration of the title. It may also be acknowledged, as
appearson the title, that OCT No. 994 resulted from the
issuance of the decree ofregistration on 17 April 1917,
although such date cannot be considered as thedate of the
title or the date when the title took effect. Second,any title
thattraces its source to OCT No. 994 dated 17 April 1917 is
void, for such mothertitle is inexistent. This error alone is, in
fact, sufficient to invalidate theDimson and CLT claims over the
subject property if singular reliance is placedby them on the
dates appearing on their respective titles.

The land becomes a registered landonly upon the transcription


of the decree in the original registration book bythe register of
deeds, the date and time of such transcription being set
forthin the process and certified to at the foot of each entry or
certificate oftitle. The issuance of the original and owners
duplicate certificates arebasic for the valid existence of the
title. Issuance of additional copies arepermissive and their
non-existence does not affect the status of title. Acertificate of
title is deemed as regularly issued with the issuance of
theoriginal copy and owners duplicate

land of the private respondents.On July 29, 2002, the COSLAP


issued a writ of execution of its decision, ordering the DENR
Sec. to implement the August 3, 1998 decision as affirmed by
SC. DENR Sec. ordered the DENR Director of Region 12
toconduct a review and investigation of FLGLA No. 542. The
latter found violations by petitioner of the terms of the FLGLA.

On August 15, 2002, Sec. Alvarez cancelled FLGLA No. 542.


Petitioners MR was denied.On November 26, 2002, Community
Environment and Natural Resources Officer (CENRO) Andrew B.
Patricio Jr. sent a letter to petitioner, advising him to vacate
and remove all improvements in the area within 10 days from
receipt of the letter.On November 27, 2002, CENRO Patricio
issued an Installation Order, which directed the immediate
installation and occupation of the area by the private
respondents indigenous communities.Upon appeal, the CA
dismissed petitioners appeal, holding that issue was already
decided by SC in GR No. G.R. No. 145838. Hence, this instant
petition via Rule 45.Issue: w/n petitioner has residual rights
over subject property until the expiration of FLGLA 542 (Dec
PartII

1.Almirol v. Register of Deeds


22SCRA 1152

March 20, 1968

FACTS:
On June 28, 1961,TeodoroAlmirol purchased from Arcenio
Abalo a parcel of land covered byoriginalcertificate of title P-1237 in
the name of Arcenio Abalo and hisdeceased wife,Nicolasa M. Abalo.
Sometime in May 1962 Almirol went to theoffice of the Registerof
Deeds to register the deed of sale and to secure inhis name a
transfercertificate of title. Registration was refused by theRegister of
Deeds becauseaccording to the Registrar, the land was a
conjugalproperty and therefore, bothspouses must sign the Deed of
Sale. Since, as inthis case, the wife has alreadydied when the sale was
made, the survivinghusband cannot dispose of the wholeproperty.
Issue:Whether or not the Registrar has the power to determine
thevalidity of adocument sought to be registered.
Held:
No, whether adocument is validor invalid, the Register of
deeds is not granted theauthority to determine;this function belongs
properly to a court of competentjurisdiction. Althoughthe reasons relied
upon by the Registrar evince asincere desire on his part tomaintain

inviolate the law on succession andtransmission of rights over


realproperties, these do not constitute legalgrounds for his refusal to
registerthe deed. Whether the document is invalid,frivolous or
intended to harass, itis not the duty of a Register of Deeds todecide,
but a court of competentjurisdiction. (Gabriel vs. Register of Deedsof
Rizal, et al., L-17956, Sept.30, 1953).
Hence,the refusal of theregistration of document was invalid.
Balbin
vs.
Register
of
DeedsG.R.
L-20611
May
8,
1969Makalintal, J.Facts:On November 15, 1961 petitioners
presented to the register of deeds of Ilocos Sur a duplicate copy of the
registered owner's certificate of title (OCT No. 548) and an instrument
entitled "Deed of Donation inter-vivos," with the request that the same
be annotated on the title. Under the terms of the instrument sought to
be annotated one Cornelio Balbin, registered owner of the parcel of
land described in OCT No. 548, appears to have donated inter-vivos an
undivided two-thirds (/) portion thereof in favor of petitioners. The
entire area of the land is 11.2225 hectares.The register of deeds
denied the requested annotation for being "legally defective or
otherwise not sufficient in law." It appears that previously annotated in
the memorandum of encumbrances on the certificate are three
separate sales of undivided portions of the land earlier executed by
Cornelio Balbin in favor of three different buyers. The final part of the
annotations referring to the abovementioned sales contains an
additional memorandum stating that "three co-owner's duplicate
certificates of title No. 548 have been issued (by the register of deeds
of Ilocos Sur) in the name of Florentino Gabayan, Roberto Bravo and
Juana Gabayan upon verbal request of Mr. Andres Cabeldo, Notary
Public of Caoayan, I. Sur, for and in the name of the vendees, this 5th
day of January, 1956 at Vigan, I. Sur." Mainly because these three other
co-owner's copies of the certificate of title No. 548 had not been
presented by petitioners, the Register of Deeds refused to make the
requested annotation.Issue: Whether the refusal of the Register of
Deeds to make the annotation is proper.Held: Yes. Section 55 of Act
496, which provides that "the production of the owner's duplicate
certificate of title whenever any voluntary instrument is presented for
registration shall be conclusive authority from the registered owner to
the register of deeds to make a memorandum of registration in
accordance with such instrument," obviously assumes that there is
only one duplicate copy of the title in question, namely, that of the
registered owner himself, such that its production whenever a
voluntary instrument is presented constitutes sufficient authority from
him for the register of deeds to make the corresponding memorandum
of registration. There being several copies of the same title in
existence, it is easy to see how their integrity may be adversely

affected if an encumbrance, or an outright conveyance, is annotated


on one copy and not on the others. The law itself refers to every copy
authorized to be issued as a duplicate of the original, which means that
both must contain identical entries of the transactions, particularly
voluntary ones, affecting the land covered by the title. If this were not
so, if different copies were permitted to carry differing annotations, the
whole system of Torrens registration would cease to be reliable.

3(Gina)

MELITONGALLARDO and TERESA VILLANUEVA


vs.
HONORABLEINTERMEDIATE APPELLATE COURT
G.R. No.L-67742 October 29, 1987
PARAS,J.:

Facts:
Thesubject matter of this controversy involves a parcel of land situated
inCavinti, Laguna consisting of 81,300 square meters, more or less,
initiallycovered by an original Certificate of Title No. 2262, issued on
April 2, 1924owned and registered in the name of the late Pedro
Villanueva pursuant toDecree No. 150562 issued in L.R.C. Cadastral
Record No. 136, Cad. Case No. 1.

OnAugust 10, 1937, petitioner claimed that the aforestated land was
sold to themin a private document, an unnotarized deed of sale written
in Tagalog that wasallegedly signed by the late Pedro Villanueva
conveying and transfering theproperty in question in favor of the
petitioners.

Subsequently,the Original Certificate of Title was cancelled on the


basis of the privatedocument of sale and a new certificate of title was
issued in the name of thepetitioners covered by Transfer Certificate of
Title No. RT- 6293 (No. 23350)on January 4, 1944.

Duringthe Second World War, the records as well as the Office of the
Register ofDeeds of Laguna, where the original of their new transfer
certificate of titlewas kept, were completely burned. Accordingly, by
virtue of an Affidavit ofReconstitution dated December 2, 1958 and
upon presentation of the Owner'sDuplicate Certificate of Title, the title
was administratively reconstitutedand the Register of Deeds of Laguna
issued Transfer Certificate of Title No.RT-6293 (No. 23350) in the name
of the petitioners.

However,when private respondent Marta Villanueva vda. de Agana


refused to sign anAffidavit of Quit-claim, petitioners instituted court
suit against the privaterespondent and her husband, Dr. Marcelo S.
Agana, Sr. by filing a complaint forQuieting of Title and Damages with
the Court of First Instance of Laguna onFebruary 3, 1977, demanding
that their title over the questioned land befortified by a declaration of
ownership in their favor and avoiding theaf/recited Deed of
Conveyance and Release of Claim.

Accordingly,private respondents in their answer countered that the


Deed of Sale in Tagalogand petitioners' title over the land be declared
void ab initio, among other demands.

The trialcourt found that said private document was null and void and
that it was signedby somebody else not Pedro Villanueva. Such findings
of fact besides beingbased on the records, were sustained by the Court
of Appeals The IntermediateAppellate Court, on May 22, 1984, affirmed
in toto the decision of the trialcourt.

Issue:

whether or not there was a validreconstitution of Transfer Certificate of


Title No. RT-6293 (No. 23350) issuedin the names of petitioners.

Held:
No. True,as argued by appellants, a private conveyance of registered
property is validas between the parties. However, the only right the
vendee of registeredproperty in a private document is to compel
through court processes the vendorto execute a deed of conveyance
sufficient in law for purposes of registration.Plaintiffs-appellants'
reliance on Article 1356 of the Civil Code isunfortunate. The general
rule enunciated in said Art. 1356 is that contractsare obligatory, in
whatever form they may have been entered, provided all theessential
requisites for their validity are present. The next sentence providesthe
exception, requiring a contract to be in some form when the law so
requiresfor validity or enforceability. Said law is Section 127 of Act 496
whichrequires, among other things, that the conveyance be executed
"before thejudge of a court of record or clerk of a court of record or a
notary public ora justice of the peace, who shall certify such
acknowledgment substantially inform next hereinafter stated."

Such lawwas violated in this case. The action of the Register of Deeds
of Laguna inallowing the registration of the private deed of sale was
unauthorized and didnot lend a bit of validity to the defective private
document of sale.

Uponconsideration of the facts and circumstances surrounding the


execution of theassailed document, the trial court found that said
private document was nulland void and that it was signed by
somebody else not Pedro Villanueva. Suchfindings of fact besides being
based on the records, were sustained by theCourt of Appeals.

In thecase at bar, it will be noted that what transpired was an


administrativereconstitution, essentially ex-parte and without notice,
thereby lendingcredence to the claim that private respondent Marta
Agana was unaware of suchreconstitution and possession until she
discovered the same in the Office ofthe Register of Deeds in 1976. As

such it cannot be claimed that she slept onher right as from that time
on, it is
undeniablethat she filed her adverse claim on the said lot.

4(Zhon)

Cruz vs. DENR Sec., G.R. No. 135385, Dec. 6, 2000

Facts:
This is a petition assailing the constitutionality of the following
provisions of Republic Act No. 8371 (R.A. 8371), otherwise known as
the Indigenous Peoples Rights Act of 1997 (IPRA), on the ground that
they amount to an unlawful deprivation of the States ownership over
lands of the public domain as well as minerals and other natural
resources therein, in violation of the regalian doctrine embodied in
Section 2, Article XII of the Constitution:
(1) Section 3(a) which defines the extent and coverage of ancestral
domains, and Section 3(b) which, in turn, defines ancestral lands;
(2) Section 5, in relation to section 3(a), which provides that ancestral
domains including inalienable public lands, bodies of water, mineral
and other resources found within ancestral domains are private
but community property of the indigenous peoples;
(3) Section 6 in relation to section 3(a) and 3(b) which defines the
composition of ancestral domains andancestral lands;
(4) Section 7 which recognizes and enumerates the rights of the
indigenous peoples over the ancestral domains;
(5) Section 8 which recognizes and enumerates the rights of the
indigenous peoples over the ancestral lands;
(6) Section 57 which provides for priority rights of the indigenous
peoples in the harvesting, extraction,development or exploration of

minerals and other natural resources within the areas claimed to be


their ancestral domains, and the right to enter into agreements with
nonindigenous peoples for the development and utilization of natural
resources therein for a period not exceeding 25 years, renewable for
not more than 25 years; and
(7) Section 58 which gives the indigenous peoples the responsibility to
maintain, develop, protect and conserve the ancestral domains and
portions
thereof
which
are
found
to
be
necessary
for
critical watersheds, mangroves, wildlife sanctuaries, wilderness,
protected areas, forest cover or reforestation.Petitioners also content
that,
by
providing
for
an
all-encompassing
definition
of
ancestral domains and ancestral lands which might even include
private lands found within said areas, Sections 3(a) and 3(b) violate the
rights of private landowners.
In addition, petitioners question the provisions of the IPRA defining the
powers and jurisdiction of the NCIP and making customary law
applicable to the settlement of disputes involving ancestral domains
and ancestral lands on the ground that these provisions violate the due
process clause of the Constitution.
Finally, petitioners assail the validity of Rule VII, Part II, Section 1 of the
NCIP Administrative Order No. 1, series of 1998, which provides that
the administrative relationship of the NCIP to the Office of the
President is characterized as a lateral but autonomous relationship for
purposes of policy and program coordination. They contend that said
Rule infringes upon thePresidents power of control over executive
departments under Section 17,Article VII of the Constitution.

Issue: w/n the assailed provisions are constitutional

Held:
Seven (7) voted to dismiss the petition. Justice Kapunan filed
anopinion, which the Chief Justice and Justices Bellosillo, Quisumbing,
andSantiago join, sustaining the validity of the challenged provisions of
R.A.8371. Justice Puno also filed a separate opinion sustaining all
challengedprovisions of the law with the exception of Section 1, Part II,
Rule III of NCIPAdministrative Order No. 1, series of 1998, the Rules and
RegulationsImplementing the IPRA, and Section 57 of the IPRA which
he contendsshould be interpreted as dealing with the large-scale

exploitation of naturalresources and should be read in conjunction with


Section 2, Article XII of the1987 Constitution. On the other hand,
Justice Mendoza voted to dismiss thepetition solely on the ground that
it does not raise a justiciable controversyand petitioners do not have
standing to question the constitutionality of R.A.8371.
Seven (7) other members of the Court voted to grant the
petition.Justice Panganiban filed a separate opinion expressing the
view thatSections 3 (a)(b), 5, 6, 7 (a)(b), 8, and related provisions of
R.A. 8371 areunconstitutional. He reserves judgment on the
constitutionality of Sections58, 59, 65, and 66 of the law, which he
believes must await the filing ofspecific cases by those whose rights
may have been violated by the IPRA.Justice Vitug also filed a separate
opinion expressing the view that Sections3(a), 7, and 57 of R.A. 8371
are unconstitutional. Justices Melo, Pardo,Buena, Gonzaga-Reyes, and
De Leon join in the separate opinions of JusticesPanganiban and
Vitug.As the votes were equally divided (7 to 7) and the necessary
majoritywas not obtained, the case was redeliberated upon. However,
afterredeliberation, the voting remained the same.Accordingly,
pursuant to Rule56, Section 7 of the Rules of Civil Procedure, the
petition is DISMISSED.

5(May Ann)

6(Rhea B.)
CANETE V. GENUINO ICE CO. INC.,GR No. 154080 January 22,
2008Facts:Petitioners filed a complaint and an amended complaint for
cancellation of title to property covered by several TCTs for being
spurious, fictitious and issued under mysterious circumstances
considering that the holders thereof and their predecessors-in-interest
were never in actual, adverse, and physical possession of the property
rendering them ineligible to acquire title under the Friar Lands Act.
They also seek to annul OCT No. 614 from which the foregoing TCTs
originated of were derived. The amended complaint alleged that the
plaintiffs and their predecessors-in-interest are among those in actual,
adverse, peaceful, and continuous possession in the concept of owner
of unregistered parcels of land in Sitio Mabilog, Quezon City. And that

the real property in question is a portion of the friar land known as the
Piedad Estate, which is intended for distribution among the bona fide
occupants thereof pursuant to the Friar Lands Act.Respondent, sought
the dismissal of the case on the ground that it fails to state a cause of
action because petitioners are not real parties-in-interest, that no relief
may be granted as a matter of law, and that petitioners failed to
exhaust all administrative remedies. The motion to dismiss was denied.
The respondents filed a petition for certiorari to the CA which granted
the petition and dismissed the amended complain of the
petitioners.Issue:Whether the CA acted with grave abuse of discretion
in granting the certiorari and dismissing the complaintRuling:No. The
complaint and amended complaint failed to state the ultimate fact
which are essential facts constituting the plaintiffs cause of action. The
plaintiffs gave only an incomplete narration of facts unsupported by
documentary or other exhibits, and the allegations are mere
conclusions of law also, the allegations of fraud are not specific and
were not substantiated.The initial claim that OCT 164 of which all the
other subject titles are derivatives is null and void has been proven
wrong as held in previous cases (Pinlac). It has been found that OCT
614 did legally exist and was previously issued in the name of the
Philippine Government in 1910. An Ad Hoc Committee of the then
Ministry of Natural Resources specifically tasked to investigate the
historical background of the Piedad Estate, found that as early as prior
to the Second World War, all lots in the Piedad Estate had already been
disposed of. The Piedad Estate has been placed under the Torrens
system which means that all lots therein are titled. Also, as held in the
Balicudiong case one who acquired title under the Friar Land Act, as
well as his successors-in-interest, may not claim successional rights to
purchase by reason of occupation from time immemorial unless it is
shown that their predecessors-in-interest were actual settlers and
occupants at the time said land were acquired by the government.
Also, the plaintiffs did not pray to be declared owners of the subject
property-despite their alleged adverse possession-but only to be
adjudged as the bona fide occupants thereof, conceding to the States
ownership of the property. Being so, they are not real parties in interest
for the purpose of maintaining a suit for cancellation of the subject
titles. Their interest is mere expectancy based on the probability that
the government would give them preference as buyers or lessees of
the subject lands. On real-parties in interest may file for the
cancellation of title of property and not one whose interest is based on
mere expectancy.

7(Angel)

8.
G.R. No. 66807. January 26, 1989
Republic v. Alagad

Facts:
On October 11, 1951, defendants filed an application for registration of
their title over a parcel of land situated at Linga, Pila, Laguna, which
was divided into two parcels, namely, Lot 1 and Lot 2.
The Republic opposed the application on the stereo-typed ground that
applicants and their predecessors have not been in possession of the
land openly, continuously, publicly and adversely under a bona fide
claim of ownership since July 26, 1894 and the land has not ceased to
be a part of the public domain.
On January 16, 1956, by virtue of a final judgment supplemented by
orders, the Alagads were declared owners of Lot 1 and the remaining
portion, or Lot 2, was declared public land. According to the trial court,
the aforementioned parcel of land is a portion of the public domain
belonging to the Republic of the Philippines, and hence, available
disposition and registration.
The Republic filed a petition for annulment of title and reversion,
insofar as the 1.42 hectare northwestern portion on end of Lot 1 is
concerned, contending that such is foreshore land, and had since time
immemorial, been foreshore land reached and covered by the waters
of the Laguna de Bay.

Issue:
Whether properties in dispute are capable of private appropriation?

Held:

According to the trial court, the aforementioned parcel of land is a


portion of the public domain belonging to the Republic of the
Philippines, and hence, available for disposition and registration. As we
have pointed out, the Government holds otherwise, and that as
foreshore land, it is not registerable.
The law provides that property of the public dominion refers to things
held by the State by regalian right. They are things res publicae in
nature and hence, incapable of private appropriation. Thus, under the
present Constitution, with the exception of agricultural lands, all other
natural resources shall not be alienated.
Thus, assuming that the properties are foreshore in nature, the
Republic has legitimate reason to demand reconveyance.

9(Mike Jayson)

10(Lou)

RomeoMARTINEZ and Leonor Suarez v. COURT of APPEALS,


SECRETARY
andUNDERSECRETARY
OF
PUBLIC
WORKS
&
COMMUNICATIONS
G.R.No. L-31271 (April 29, 1974)

FACTS:The spouses Romeo Martinez and Leonor Suarez, now


petitioners-appellees, arethe registered owners of two (2) parcels of
land located in Lubao, Pampanga,covered by transfer certificate of title
No. 15856 of the Register of Deeds ofthe said province. Both parcels of
land are fishponds. The property involved inthe instant case is the
second parcel mentioned in the above-named transfercertificate of
title.

Toavoid any untoward incident, the disputants agreed to refer the


matter to theCommittee on Rivers and Streams. This committee
thereafter appointed aSub-Committee to investigate the case and to

conduct an ocular inspection ofthe contested property, and on March


11, 1954, said Sub-Committee submitted itsreport to the Committee on
Rivers and Streams to the effect that Parcel No. 2of transfer certificate
of title No. 15856 was not
apublic river but a private fishpond owned by the herein spouses.

Themunicipal officials of Lubao, led by Acting Mayor Mariano Zagad,


apparentlyrefused to recognize the above decision. The writ of
preliminary injunctionapplied for was issued against the respondent
municipal Mayor, who immediatelyelevated the injunction suit for
review to the Supreme Court, which dismissedMayor Zagad's petition
on September 7, 1953. With this dismissal order hereinappellee
spouses proceeded to construct the dikes in the disputed parcel ofland.

Somefour (4) years later, and while Civil Case No. 751 was still pending
theHonorable Florencio Moreno, then Secretary of Public Works and
Communications,ordered another investigation of the said parcel of
land, directing theappellees herein to remove the dikes they had
constructed, on the strength ofthe authority vested in him by Republic
Act No. 2056, approved on June 13,1958, entitled "An Act To
Prohibit, Remove and/or Demolish theConstruction of Dams.
Dikes, Or Any Other Walls In Public Navigable Waters,
OrWaterways and In Communal Fishing Grounds, To Regulate
Works in Such Waters orWaterways And In Communal Fishing
Grounds, And To Provide Penalties For ItsViolation, And For
Other Purposes.
Thesaid order which gave rise to the instant proceedings, embodied a
threat thatthe dikes would be demolished should the herein appellees
fail to complytherewith within thirty (30) days.

Thespouses Martinez replied to the order by commencing on January 2,


1959 thepresent case, which was decided in their favor by the lower
Court in a decisiondated August 10, 1959.

Asagainst this judgment respondent officials of the Department of


Public Worksand Communications took the instant appeal.

ISSUE:W/N the Court may adjudge title over non-registrable properties


such as publicnavigable river.

RULING:No.

TheLand Registration Court has no jurisdiction over non-registerable


properties,such as public navigable rivers which are parts of the public
domain, andcannot validly adjudge the registration of title in favor of a
private applicant.

InArticle 339 of the old Civil Code which reads as follows:


Propertyof public ownership is:
1.That destined to the public use, such as roads, canals, rivers,
torrents,ports, and bridges constructed by the State, and banks shores,
roadsteads, andthat of a similar character. (Par. 1)
Theabove-mentioned properties are parts of the public domain
intended for publicuse, are outside the commerce of men and,
therefore, not subject to privateappropriation. ( 3 Manresa, 6th ed.
101-104.)

In Ledesmav. Municipality of Iloilo, 49 Phil. 769, this Court held:


Asimple possession of a certificate of title under the Torrens system
does notnecessarily make the possessor a true owner of all the
property describedtherein. If a person obtains title under the Torrens
system which includes bymistake or oversight, lands which cannot be
registered under the Torrenssystem, he does not by virtue of said
certificate alone become the owner of theland illegally included.

Hence,the judgment of the Court of First Instance of Pampanga as


regards the Lot No.2 of Certificate of Title No. 15856 in the name of

petitioners-appellants maybe attacked at any time, either directly or


collaterally, by the State which isnot bound by any prescriptive period
provided for by the Statute of Limitations(Article 1108, par. 4, new Civil
Code).

Beforepurchasing a parcel of land, it cannot be contended that the


appellants whowere the vendees did not know exactly the condition of
the land that they werebuying and the obstacles or restrictions thereon
that may be put up by thegovernment in connection with their project
of converting Lot No. 2 in questioninto a fishpond. Nevertheless, they
willfully and voluntarily assumed the risksattendant to the sale of said
lot. One who buys something with knowledge ofdefect or lack of title in
his vendor cannot claim that he acquired it in goodfaith (Leung Lee v.
Strong Machinery Co., et al., 37 Phil. 664).

Theruling that a purchaser of a registered property cannot go beyond


the record tomake inquiries as to the legality of the title of the
registered owner, but mayrely on the registry to determine if there is
no lien or encumbrances over thesame, cannot be availed of as against
the law and the accepted principle thatrivers are parts of the public
domain for public use and not capable of privateappropriation or
acquisition by prescription.

Significance:

LandRegistration; Court may not adjudge title over nonregisterable land.-- The Land Registration Court has no jurisdiction
over non-registerableproperties, such as public navigable rivers which
are parts of the publicdomain, and cannot validly adjudge the
registration of title in favor of aprivate applicant. Hence, the judgment
of the Court of First Instance ofPampanga as regards the Lot No. 2 of
Certificate of Title No. 15856 in the nameof petitioners, may be
attacked at any time, either directly or collaterally,by the state which is
not bound by any prescriptive period provided for by theStatute of
Limitations.

11(Jess)

GODOFREDO NAVERA, petitioner,


ETC., ET AL., respondents.

vs. HON. PERFECTO QUICHO

Facts
On January 24, 1961, the municipality of Ligao filed with the Court of
First Instance of Albay a petition under Section 112 of Act No. 496, as
amended, for the correction of Transfer Certificate of Title No. T-9304
issued in the name of Godofredo Navera, covering Lot No. 2793-A, on
the ground that a portion of 123 sq. m. was erroneously included in
said title during the cadastral survey of Ligao.
Navera filed a motion to dismiss based on the ground that the relief
which petitioner seeks to obtain cannot be granted under Section 112
of Act 496 because the same would involve the opening of the original
decree of registration. He contends that, under said section, the court
can only authorize an alteration which may not impair the rights
recorded in the decree, or one which will not prejudice such rights, or
one which is consented to by all parties concerned, or can authorize
the correction of any error or mistake which would not involve the
reopening of the original decree of registration. Here the petition will
have such effect, for it will involve the correction of the technical
description of the land covered by the certificate of title in question,
segregating therefrom the portion alleged to have been erroneously
included, which eventually will cause the amendment of the original
decree of registration. This cannot be done at this stage after the lapse
of 23 years from the issuance of the certificate of title.
After hearing both parties, the court a quo issued an order denying the
motion to dismiss and requiring Navera to answer the petition within
the reglementary period. After his motion for reconsideration was
denied, Navera filed the present petition for certiorari disputing the
jurisdiction of the court a quo.
Issue
Whether or not it is under Section 112 of Act No 496

Held: No
The theory entertained by the court a quo that if the portion to be
segregated was really erroneously included in the title issued to
petitioner because it is part of the Natera street which belongs to the
municipality of Ligao that portion may be excluded under Section 112
of Act 496 because under the law1 any public highway, even if not
noted on a title, is deemed excluded therefrom as a legal lien or
encumbrance, is in our opinion correct. This is upon the principle that a
person who obtains a title which includes by mistake a land which
cannot legally be registered does not by virtue of such inclusion
become the owner of the land erroneously included therein.2 But this
theory only holds true if there is no dispute that the portion to be
excluded is really part of a public highway. This principle only applies if
there is unanimity as to the issue of fact involved.1wph1.t
Here said unanimity is lacking. The claim of the municipality that an
error has been committed in the survey of the lot recorded in
respondent's name by including a portion of the Natera street is not
agreed to by petitioner. In fact, he claims that is a question of fact that
needs to be proven because it is controversial. There being dissension
as to an important question of fact, the petition cannot be granted
under Section 112 of Act No. 496.
We are of the opinion that the lower court did not err in finding that it
lacks jurisdiction to entertain the present petition for the simple reason
that it involves as controversial issue which takes this case out of the
scope of Section 112 of Act No. 496. While this section, among other
things, authorizes a person in interest to ask the court for any erasure,
alteration, or amendment of a certificate of title "upon the ground that
registered interests of any description, whether vested, contingent,
expectant, or inchoate, have terminated and ceased", and apparently
the petition comes under its scope, such relief can only be granted if
there is unanimity among the parties, or there is no adverse claim or
serious objection on the part of any party in interest; otherwise the
case becomes controversial and should be threshed out in an ordinary
case or in the case where the incident properly belongs.
WHEREFORE, petition is granted. The order of respondent court dated
March 8, 1961, as well as its order dated March 25, 1961, are hereby
set aside. No costs.

12(Diane)

G.R. No. 92013 July 25, 1990

FACTS: The Roppongi property was acquired from the Japanese


government through Reparations Contract No. 300 dated June 27,
1958. A proposal was presented to President Corazon C. Aquino to
make the property the subject of a lease agreement with a Japanese
firm. On July 25, 1987, the President issued Executive Order No. 296
entitling non-Filipino citizens or entities to avail of separations' capital
goods and services in the event of sale, lease or disposition. VicePresident Laurel states that the Roppongi property is classified as one
of public dominion, and not of private ownership under Article 420 of
the Civil Code. The respondents rely upon the rule of lex situs which is
used in determining the applicable law regarding the acquisition,
transfer and devolution of the title to a property.

ISSUE: Whether or not the Roppongi property can be alienated by the


Philippine Government.

HELD: No. There can be no doubt that the property is of public


dominion. The property is classified under Art 420 of the Civil Code as
property belonging to the State and intended for some public service.
The fact that it has not been used for actual Embassy service does not
automatically convert it to patrimonial property. Such conversion
happens only if property is withdrawn from public use, through an
abandonment of the intention to use the Roppongi property for public
service and to make it patrimonial property. Abandonment must be a
certain and positive act based on correct legal premises.

The Roppongi property was acquired together with the other


properties through reparation agreements. They were assigned to
the government sector and that the Roppongi property was
specifically designated under the agreement to house the Philippine
embassy.

It is of public dominion unless it is convincingly shown that the


property has become patrimonial.

As property of public dominion, the Roppongi lot is outside the


commerce of man. It cannot be alienated.
Its ownership is a
special collective ownership for general use and payment, in
application to the satisfaction of collective needs, and resides in the
social group. The purpose is not to serve the State as the juridical
person but the citizens; it is intended for the common and public
welfare and cannot be the object of appropriation.

13(April)

14(Daniel Eblahan)

Navarav. Quicho
GRno. L-18339, June 29 1962
BautistaAneglo, J.:
Facts:
Godofredo Navera owned a lot inLigao, Albay. On January 24,
1961, the municipality of Ligao petitioned theCourt of First Instance of
Albay to correct the Transfer Certificate of Titleof Navera because a
portion of his lot was erroneously included in the title.Said portion was
a part of a street and the encroachment allegedly deprived thestreet of
an area amounting to 123 sq. meters. Navera moved to dismiss
thepetition because the correction would involve, not the correction of
the title,but the opening of the original title and return of said portion
to themunicipality. The lower court dismissed his motion and granted
the petition,ruling that he cannot be the owner of the portion of the lot

which was part ofthe street since public highways and streets cannot
be registered and isexempted as a legal lien or encumbrance.
Issue:
Whether or not the petition tocorrect the title of Navera should
be allowed.
Held:
No. The Court ruled that even thoughthe lower court was
correct in its theory that the portion of the lot wasreally a part of the
street and is excluded and cannot be registered and ownedby the
petitioner, this theory applies only if there is unanimity among
theparties that the land is in fact part of the street and there is no
adverseclaim or any serious objection from any party in interest.
Inthis case, the Court ruled that there is no unanimity since there is
anobjection coming from the petitioner, there is a controversy and it
should beresolved in an ordinary case, not through the petition to
correct petitionerstitle.

15(Melodia)
________________________________________________________________________
________________________
16(Mel)

REPUBLIC V. AYALA y SIA


14 SCRA 259

Facts:

In an amended complaint filed against Ayala Y Cia et al., the plaintiff


sought the annulment of titles allegedly obtained by the defendant
over portions of the territorial waters of the public domain. The
defendant company caused the survey and preparation of a composite
plan of Hacienda Calatagan, increasing its area from 9,652.583
hectares (as evidenced by TCT No. 722) to 12,000 hectares, by taking
or including therein lands of public dominion. Thus, plaintiff also prayed
for recovery of possession of such areas in excess of those covered by
TCT No. 722, and for which fishpond permits were already issued in
favor of bona fide applicants. Miguel Tolentino and 22 others alleged
holders of fishpond permits issued by the Bureau of Fisheries over the
areas supposedly outside the boundaries of Hacienda Calatagan, were
allowed to intervene in the case and make demand for recovery of
possession of said areas, and claim for damages for the deprivation of
possession thereof allegedly by the illegal acts of defendants.

The defendants contended that the excess was insignificant in nature


and attributable to the inaccuracy of the magnetic survey that was
used in the preparation of the plan upon which OCT No. 20 was based.

After trial, the court rendered judgment annulling TCT No. T-9550
issued to defendants Dizons covering Lots 360, 362, 363 and 182, as
well as other subdivision titles issued to Ayala y Cia. and/or Hacienda
de Calatagan over the areas outside its private property covered by
TCT No. 722. This ruling was based upon the finding that the disputed
areas form part of the navigable water, or are portions of the sea,
beach and foreshores of the bay.

Issue:

Whether the areas in dispute are territorial waters of the public


domain.

Ruling:

The decision of the lower court appealed from is hereby affirmed.

The areas in dispute (those covered by permits issued by the Bureau of


Fisheries) were found to be portions of the foreshore, beach, or of the
navigable water itself. And, it is an elementary principle of law that
said areas not being capable of registration, their inclusion in a
certificate of title does not convert the same into properties of private
ownership or confer title on the registrant.

In the present case, as the lots covered by TCT No. T-9550 issued in the
names of defendants Dizons (and which were purchased by the latter
from defendants Ayala y Cia., and/or Alfonso Zobel) were found to be
portions of the foreshore or of the territorial waters, the lower court
committed no error in rendering judgment against said defendants and
ordering the reversion of said properties to the public dominion.
___________________________

17(Rocky)

18(Maribeth)

Republicvs Sioson, 9 SCRA533


FACTS:
OnNovember 6,1951, Spouses Sioson filed with the CFI an application
forregistration of fourparcels of land situated in barrio San Roque,
Paombong,Bulacan, claiming thatthey are the owners in fee simple.
TheDirector of Landsopposed the application for registration over Lot 4
based onseveral reasons;one which is that Lot 4 is a part of the public
domain and assuch belong to theRepublic of the Philippines. After
hearing, the court adjudicated and orderedthe registration of LotNos. 1,

2 and 3 in favor of the Spouses. No adjudicationas to Lot 4 hence the


appeal with the Court of Appeals.
Onappeal, thelower courts decision was modified as the Sol. Gen., in
behalf ofthe Directorof Lands, recommended the registration of Lot 4 in
the name oftheSpouses. By virtue of the modifiedjudgment, the lower
court ordered theissuance of the decree on Lot 4.
OnApril 25, 1956,the Sol. Gen. in behalf of the Republic of the
Philippines filedwith the lowercourt a petition for review of the decree
of registration andcancellation oftitle over Lot 4 alleging actual and
extrinsic fraud by theSpouses, byintentional and deliberate
concealment of facts and connivance withthe landinspector. In the
Spouses opposition tothe petition, they allegedamong other things
that Lot 4 was and still is intruth and in fact an accretionto a titled
parcel of land. The lower court, without hearing andpresentation
ofevidence entered an order denying the petition, hence an appealby
the Republicof the Philippines.
ISSUE: Whetheror not Lot4 being a part of the bed of a navigable river
may be registered inthe name ofthe Spouses?
HELD: No.Navigable
rivers
cannot
be
appropriated
and
registeredunder the LandRegistration Act. If itis true that Lot 4 is or
forms part of the bed of anavigable steam, creek orriver, the decree
and title to it in the name of theSpouses would not givethem any right
or title to it.

19(Ailyn)

20(Jen)
SECRETARY OF DENR VS YAP, GR NO. 173775, 8 OCTOBER 2008

FACTS: On November 10, 1978, then President Marcos issued Proc.


No. 1801declaring Boracay Island, among other islands, caves and

peninsulas in the Philippines, as tourist zones and marine


reserves under the administration of the Philippine Tourism Authority
(PTA). President Marcos later approved the issuance of PTA Circular
3-82 dated September 3, 1982, to implement Proclamation No. 1801.

Claiming that Proclamation No. 1801 and PTA Circular No 3-82


precluded them from filing an application for judicial confirmation of
imperfect title or survey of land for titling purposes, respondentsclaimants Mayor Jose S. Yap, Jr., Libertad Talapian, Mila Y. Sumndad,
and Aniceto Yap filed a petition for declaratory relief with the RTC in
Kalibo, Aklan.

In their petition, respondents-claimants alleged that Proc. No. 1801 and


PTA Circular No. 3-82 raised doubts on their right to secure titles over
their occupied lands. They declared that they themselves, or through
their predecessors-in-interest, had been in open, continuous, exclusive,
and notorious possession and occupation in Boracay since June 12,
1945, or earlier since time immemorial. They declared their lands for
tax purposes and paid realty taxes on them. Respondents-claimants
posited that Proclamation No. 1801 and its implementing Circular did
not place Boracay beyond the commerce of man. Since the Island was
classified as a tourist zone, it was susceptible of private ownership.
Under Section 48(b) of the Public Land Act, they had the right to have
the lots registered in their names through judicial confirmation of
imperfect titles.

The Republic, through the OSG, opposed the petition for declaratory
relief. The OSG countered that Boracay Island was an unclassified
land of the public domain. It formed part of the mass of lands
classified as public forest, which was not available for disposition
pursuant to Section 3(a) of the Revised Forestry Code, as amended.
The OSG maintained that respondents-claimants reliance on PD No.
1801 and PTA Circular No. 3-82 was misplaced. Their right to judicial
confirmation of title was governed by Public Land Act and Revised
Forestry Code, as amended. Since Boracay Island had not been
classified as alienable and disposable, whatever possession they had
cannot ripen into ownership.

On July 14, 1999, the RTC rendered a decision in favor of respondentsclaimants, declaring that, PD 1810 and PTA Circular No. 3-82 Revised
Forestry Code, as amended.

The OSG moved for reconsideration but its motion was denied. The
Republic then appealed to the CA. On In 2004, the appellate court
affirmed in toto the RTC decision. Again, the OSG sought
reconsideration but it was similarly denied. Hence, the present petition
under Rule 45.

On May 22, 2006, during the pendency the petition in the trial court,
President
Gloria
Macapagal-Arroyo
issued
Proclamation
No.
1064 classifying Boracay Island partly reserved forest land (protection
purposes) and partly agricultural land (alienable and disposable).

On August 10, 2006, petitioners-claimants Sacay,and other


landowners in Boracay filed with this Court an original petition for
prohibition, mandamus, and nullification of Proclamation No.
1064. They allege that the Proclamation infringed on their prior
vested rights over portions of Boracay. They have been in continued
possession of their respective lots in Boracay since time immemorial.

On November 21, 2006, this Court ordered the consolidation of the two
petitions.

Hence, this case.

ISSUE: Whether private claimants have a right to secure titles over


their occupied portions in Boracay.

HELD: petitions DENIED. The CA decision is reversed.

Except for lands already covered by existing titles, Boracay was an


unclassified land of the public domain prior to Proclamation No. 1064.
Such unclassified lands are considered public forest under PD No. 705.

PD No. 705 issued by President Marcos categorized all unclassified


lands of the public domain as public forest. Section 3(a) of PD No. 705
defines a public forest as a mass of lands of the public domain
which has not been the subject of the present system of classification
for the determination of which lands are needed for forest purpose and
which are not. Applying PD No. 705, all unclassified lands, including
those in Boracay Island, are ipso factoconsidered public forests. PD No.
705, however, respects titles already existing prior to its effectivity.

The 1935 Constitution classified lands of the public domain into


agricultural, forest or timber, such classification modified by the 1973
Constitution. The 1987 Constitution reverted to the 1935 Constitution
classification
with
one
addition:
national
parks. Of
these, only agricultural lands may be alienated.Prior to Proclamation
No. 1064 of May 22, 2006, Boracay Island had never been expressly
and administratively classified under any of these grand divisions.
Boracay was an unclassified land of the public domain.
A positive act declaring land as alienable and disposable is required. In
keeping with the presumption of State ownership, the Court has time
and again emphasized that there must be a positive act of the
government, such as a presidential proclamation or an executive
order; an administrative action; investigation reports of Bureau of
Lands investigators; and a legislative act or a statute. The applicant
may also secure a certification from the government that the land
claimed to have been possessed for the required number of years is
alienable and disposable. The burden of proof in overcoming
such presumption is on the person applying for registration (or
claiming ownership), who must prove that the land subject of the
application is alienable or disposable.

In the case at bar, no such proclamation, executive order,


administrative action, report, statute, or certification was presented to
the Court. The records are bereft of evidence showing that, prior to
2006, the portions of Boracay occupied by private claimants were
subject of a government proclamation that the land is alienable and

disposable. Matters of land classification or reclassification cannot be


assumed. They call for proof.

Proc. No. 1801 cannot be deemed the positive act needed to classify
Boracay Island as alienable and disposable land. If President Marcos
intended to classify the island as alienable and disposable or forest, or
both, he would have identified the specific limits of each, as President
Arroyo did in Proclamation No. 1064. This was not done in
Proclamation No. 1801.

CENONMATEO, petitioner-appellant, vs. HON. FLORENCIO


MORENO, in his capacity asSECRETARY OF PUBLIC WORKS AND
COMMUNICATIONS, defendant-appellee. G.R. No.L-21024
July 28, 1969FACTS: Sometimein 1959 a number of residents of
Guiguinto, Bulacan, sent a letter-complaint tothe Highway District
Engineer of that province asking that the Sapang Cabay, apublic
navigable stream, which had been blocked by means of dikes and
dams andconverted into fishponds, be ordered reopened and restored
to its originalcondition. The letter was referred to the Secretary of
Public Works andCommunications, who caused an investigation to be
conducted pursuant toRepublic Act No. 2056. Acting on the report
which the investigator submitted tohim, the Secretary rendered his
decision on August 10, 1959, finding that theSapang Cabay was a
public navigable stream and ordering Cenon Mateo, the
hereinpetitioner-appellant, who had in the meantime acquired the
property insidewhich the said creek is situated, to remove the dikes
and dams thereinconstructed within thirty days from notice; otherwise
they would be removed athis expense. Mateo moved to reconsider but
was turned down, whereupon he filedthe basic petition to restrain the
respondent Secretary from enforcing hisdecision. The petition, as
already stated, was dismissed by the Court a quo.The certification of
the appeal to us was upon motion of both parties in viewof the
constitutional question involved.ISSUE: whether or not RepublicAct No.
2056 is unconstitutional because it unduly delegates judicial power
tothe Secretary and unlawfully deprives the appellant and others
similarlysituated of their property without due process of lawHELD:
Theconstitutionality of the aforesaid statute has been upheld by this
Court inLovina vs. Moreno, G.R. No. L-17821, November 29, 1963,
shortly before thepresent appeal was submitted for decision. That case
held, furthermore, thatthe absence of any mention of a navigable
stream within a property covered by acertificate of title does not
preclude a subsequent investigation anddetermination of its existence

nor make it private property of the titleholder; that the findings of fact
made by the Secretary of Public Works andCommunications should be
respected in the absence of illegality, error of law,fraud or imposition,
as long as such findings are supported by substantialevidence; and
that the ownership of a navigable stream or of the bed thereof isnot
subject to acquisitive prescription.

22(Nelson)
Republicvs CA 132 SCRA 514
Facts:
Respondents sought to register the land adjacent to their
fishpond; categorizedas lot 1, lot 2 and lot 3(later on lot 3 was
withdrawn). They are theregistered owner of parcel of lot(Transfer
Certificate of Title No. 89709)bordering on the Bocaue and Meycauyan
rivers. Lot 1 and lot 2 are accretions tothe land owned by the
respondents and so the trial court ordered theregistration of land in
favor of them. The petitioner submits that there is noaccretion to speak
of under Article 457 of the New Civil Code because whatactually
happened is that the private respondents simply transferred theirdikes
further down the river bed of the Meycauayan River, and thus, if there
isany accretion to speak of, it is man-made and artificial and not the
result ofthe gradual and imperceptible sedimentation by the waters of
the river.
Issue:
Whether or not man-made alluvial deposit are registerable.
HELD:
NO.The lower court cannot validly order the registration of Lots 1 & 2 in
thenames of the private respondents. These lots were portions of the
bed of theMeycauayan river and are therefore classified as property of
the public domainunder Article 420 paragraph 1 and Article 502,
paragraph 1 of the Civil Code ofthe Philippines. They are not open to
registration under the Land RegistrationAct. The adjudication of the
lands in question as private property in the namesof the private
respondents is null and void.
WHEREFORE,the instant petition is GRANTED. The decision appealed
from is hereby REVERSEDand SET ASIDE. The private respondents are

ordered to move back the dikes oftheir fishponds to their original


location and return the disputed property tothe river to which it
belongs.

23(JM)

MANECLANGVS. INTERMEDIATE APPELLATE COURT (IAC)


161SCRA 469

FACTS

The Municipal Council of Bugallion,Pangasinan passed


aResolutions Nos. 38 and 95 authorizing public bidding forthe lease of
allmunicipal ferries and fisheries, including the fishpond.Alfredo Maza
theawardees in the public bidding.

Adriano Maneclang et al filed a complaintfor quitting of titleover


acertain
fishpond
belonging
to
them
situated
in
Bario
Salomague,Bugallion,Pangasinan, and the annulment of Resolution No.
38 and 95 of theMunicipalCouncil of Bugallion.

The trial court dismissed the complaint upona findingthat the


body of water traversing the titled properties of petitionersis acreek
constituting a tributary of the Agno River; therefore public innatureand
not subject to private appropriation.

Due to lack of interest


commentingthe
complaint.

of respondentsAlfredo Maza in
The
parties
entered
into

amicablesettlement by Submitting to theCourt a Compromise


Agreement
praying
thatjudgment
be
rendered
recognizing
theownership of petitioners over the land thebody of water found
within theirtitled properties.

ISSUE:

Whetherthe creek or the body of watertraversing the titled properties


of Maneclang etal is susceptible to privateappropriation.

HELD:

CREEKis defined as a recess, or armextending from a river from a river


andparticipating in the ebb and flow ofthe sea, is a property belonging
to thepublic domain which is not susceptibleto private appropriation
and acquisitiveprescription, and as a public water itcannot be
registered under the TorrensSystem in the name of any individual.

24(Maris)

25(Reg)

26
G.R. No. L-30263-5

October 3, 1987

REPUBLIC OF THE PHILIPPINES, Represented by the DIRECTOR


OF LANDS, petitioners, vs. THE HON. ANDRES REYES, Presiding

Judge, Branch VI, Court of First Instance of Rizal, GODOFREDO


R. EUSEBIO, URBANO C. LARA, GIL VENEZUELA, RODOLFO
CENIDOZA, RAMON OROSA, AND JOSEFINA OROSA (Spouses),
THE PHILIPPINE COMMERCIAL AND INDUSTRIAL BANK and THE
REGISTER OF DEEDS OF RIZAL, respondents.

FACTS:
Sometime in 1965, Eusebio and Lara filed with the Bureau of Lands
their Free Patent Applications for the parcels of land designated as Free
Patent Application Nos. 7-207 and 7-208 for Lot no. 1 and Lot no. 2
respectively situated in Napindan, Taguig, Rizal. Their application was
approved and the patents were registered in the Register of Deeds. It
was later discovered by the Anti-Graft and Corruption Board of the
Bureau of Lands that the lands were under water and form part of the
Laguna de Bay no signs of cultivation or of any improvement thereon.
On March 16 and 22, 1960, both Eusebio and Lara executed separate
affidavits admitting noncompliance with the requirements of the Public
Land Act and expressly agreed to have their patents and certificates of
title cancelled.
The plaintiffs filed a case for the cancellation of the patents in the CFI
Rizal. Eusebio and Lara failed to file an answer and was declared in
default. The CFI Rizal ordered the cancellation of the patents. When
advised to surrender their owners duplicate copy of Original
Certificates, both claimed to have long surrendered the same to Atty.
Javier of the Investigation Section of the Bureau of Lands.
Five years later, on June 3, 1967, Eusebio and Lara (plaintiff here) filed
for the annulment of the decision on the same cases alleging that the
CFI Rizal, 7th District, Branch II had not acquired jurisdiction over their
persons and their decision was procured through fraud.
Despite records showing that the Director of Lands had not been
properly served, the defendants were declared in default. CFI Rizal, 7th
District, Branch IV, declared the earlier decision null and void.
The Director of Lands, claiming that the Branch IV had not acquired
jurisdiction over his person, filed a Motion to Admit Petition to Reopen
Proceedings with Additional Parties. These additional persons were
alleged to have bought the subject lands from Eusebio and Lara.
Venezuela, Cenidoza, and Orosa acquired Transfer Certificates of Title
from Eusebio and Lara for P10,000. Venezuela and Cenidoza

transferred their rights to Orosa. Spouses Orosa had executed a deed


of mortgage in favor of Philippine Commercial Industrial Bank.
The petition was then denied after a motion for reconsideration filed by
the intervenors.

ISSUE:
Whether Orosa, Venezuela and Cenidoza would be considered as
buyers in good faith.

HELD:
It is well settled that any title issued on non-disposable lots even in the
hands of an alleged innocent purchaser of value, shall be cancelled. In
the case at bar, the free patents and certificates of title issued to
Eusebio and Lara cover areas which form parts of Laguna de Bay.
These are neither agricultural nor disposable. Subject patents and titles
were erroneously issued due to misrepresentations and false reports
and must therefore be cancelled. Any false statement in an application
for public land shall ipso facto produce the cancellation of the title
granted. This rule applies even after the issuance of the certificate of
title. A certificate of title cannot be used as a shield to perpetuate
fraud, and the doctrine of indefeasibility of Torrens title does not apply
to free patent secured through fraud. Likewise, the mere possession of
land does not itself divest the land of its public character.
Void free patents and certificates of title do not divest the state of its
ownership of the land nor operate to change the public character of
the land to private.

27(Ed)

28(LJ)

29(Zax)

30(Clathem)
Republic vs Sps. Maximo, 135 SCRA 156 Facts: This case is about the
validity of the registration of 885 hectares of public forestal land
located in Mulanay, Quezon.In Land Registration Case No. 81-G of the
Court of First Instance at Gumaca, Quezon, Judge Vicente del Rosario
on March 21, 1961 rendered a decision, ordering the registration of
said land, Lot 1, allegedly located at Barrio Cambuga (Anonang),
Mulanay, in the names of the spouses Prudencio Maxino and Tarciana
Morales, less 200 hectares which should be registered in the names of
the Heirs of Lorenzo Consolacion (72, Record on Appeal). The decision
became final and executory. A decree and an original certificate of title
were issued.Issue: Whether or not the Inclusion of a forest land in a
title nullifies the title.Held: Yes. It is incontestable that Lot 1, the 885hectare area registered by the Maxinos is within the public forest, not
alienable and disposable nor susceptible of private appropriation. Thus,
Its inclusion in the public forest certified by Director of Forestry shall
annul the title of Maxino spouses.

31(Lea)

G.R. No. 181502, February 2, 2010DIAZ vs. REPUBLIC OF THE


PHILIPPINES
FACTS: Petitioners late mother, Flora Garcia (Garcia) , filed an
application for registration of a vast tract of land located in Laur, Nueva
Ecija and Palayan City. She alleged that she possessed the land as
owner and worked, developed and harvested the agricultural products
and benefits of the same continuously, publicly and adversely for more
or less 26 years.The Republic of the Philippines, represented by the
Office of the Solicitor General, opposed the application because the
land in question was within the Fort Magsaysay Military Reservation
(FMMR), established by virtue of Proclamation No. 237. Thus, it was
inalienable as it formed part of the public domain. Significantly, SC
already ruled in Director of Lands v. Reyes that the property subject of
Garcias application was inalienable as it formed part of military
reservation.The CFI ruled in favor of Garcia. The CA reversed CFIs
decision. The CA found that Reyes case was applicable to petitioners

case because it involved the same property.During the pendency of the


case in the CA, Garcia passed away and was substituted by her heirs,
one of whom was petitioner Florencia Diaz.The parties ultimately
entered into a compromise agreement with the Republic withdrawing
its claim on the more or less 4,689 hectares supposedly outside the
FMMR. For her part, petitioner withdrew her application for the portion
of the property inside the military reservation. They filed a motion for
approval of the amicable settlement in the CA and CA
approved.However, acting on a letter written by a certain Atty.
Restituto S. Lazaro, the OSG filed a motion for reconsideration and
informed the appellate court that the tract of land subject of the
amicable settlement was still within the military reservation. CA
annulled the compromise agreement.
ISSUE: Whether petitioner can acquire the subject land that is part of
military reservation.
RULING:By way of a background, SC recognized in Reyes that the
property where the military reservation is situated in forest lands are
not registrable under CA 141. Section 49 [b] of CA No. 141, as
amended, applies exclusively to public agricultural land. Forest lands or
area covered with forest are excluded. It is well-settled that forest land
is incapable of registration; and its inclusion in a title, whether such
title be one issued using the Spanish sovereignty or under the present
Torrens system of registration, nullifies the title.However, it is true that
forest lands may be registered when they have been reclassified as
alienable by the President in a clear and categorical manner (upon the
recommendation of the proper department head who has the authority
to classify the lands of the public domain into alienable or disposable,
timber and mineral lands) coupled with possession by the claimant as
well as that of her predecessors-in-interest.Unfortunately for petitioner,
she was not able to produce such evidence. Accordingly, her
occupation thereof, and that of her predecessors-in-interest, could not
have ripened into ownership of the subject land. This is because prior
to the conversion of forest land as alienable land, any occupation or
possession thereof cannot be counted in reckoning compliance with
the thirty-year possession requirement under Commonwealth Act 141
(CA 141) or the Public Land Act.Therefore, even if possession was for
more than 30 years, it could never ripen to ownership.But even
assuming that the land in question was alienable land before it was
established as a military reservation, there was nevertheless still a
dearth of evidence with respect to its occupation by petitioner and her
predecessors-in-interest for more than 30 years.

PartIII

1.DIRECTOR OF FORESTRY vs.VILLAREAL


G.R.No. L-32266 February 27, 1989
CRUZ, J.:

FACTS:
A land consistingof 178,113square meters of mangrove
swamps is located in the municipality ofSapian,Capiz. Ruperto Villareal
applied for its registration alleging that heand hispredecessors-ininterest had been in possession of the land for morethan 40years. He
was opposed by several persons, including the petitioner onbehalf
ofthe Republic of the Philippine on the ground that mangrove
swampswhich formpart of the public forests are cannot be disposed.

In the case ofMontano v.Insular Government (1909),


mangrove swamps or manglares are mudflats,alternately washed and
exposed by the tide, in which grows variouskindredplants which will not
live except when watered by the sea, extendingtheir rootsdeep into the
mud and casting their seeds, which also germinatethere.
Thus,mangrove swamps were considered agricultural lands and so
susceptibleof privateownership. However, the Philippine Legislature
subsequentlycategoricallydeclared that mangrove swamps form part of
the public forests ofthis countrywhich are inalienable.

ISSUE:Whether or not mangrove swamps are comprised within the


publicforests oragricultural lands.

HELD:

Mangrove swampsor manglares shouldbe understood as


comprised within thepublic forests of the Philippines asdefined in the
aforecited Section 1820 ofthe Administrative Code of 1917.
Thelegislature having so determined, thecourts have no authority to
ignore ormodify its decision, and in effect vetoit, in the exercise of the
courtsdiscretion.

Assuch, the Supreme Courtruled that Villareal was not able to


establish hisright to the registration ofthe subject land in his name
because mangroveswamps are inalienable.

2. Tongson vs. Director of ForestryG.R. No. L-34463 September 27,


1977Fernando, J.Facts:The testimonial evidence shows that as early as
the year 1905 the parcel of land which later became Lot 855 of the
cadastral survey of Pilar, was under the exclusive possession of
Francisco Boria who cut trees therefrom and converted them into
firewood. He also established a salt factory and that he sold the
firewood and the salt without having been disturbed by anybody. After
the death of Francisco. Boria, his son Arturo Borja took possession of
the land, up to the year 1910. On May 1, 1917 Antero Borja sold the
land to Deogracias Gayacao. On January 4, 1940, Deogracias Gayacao
sold five parcels of land to Santiago M. Bermejo and one of the parcels
known as parcel No. 4 is cadastral Lot No. 855. During his lifetime,
Santiago M. Bermejo possessed said parcel of land, cut trees for the
firewood purposes and also had a salt factory. Upon the death of
Santiago M. Bermejo in 1951, his children took possession of this
parcel of land and when Macario Bermejo was appointed judicial
administrator by the Court of First Instance of Capiz, At present Lot 855
is a completed and producing fishpond. When Macario Bermejo took
possession of the land in 1953 he converted it into a fishpond and
started to construct fishpond dikes. However, due to lack of funds, the
construction of the fishpond was not completed. On May 30, 1956,
Macario Bermejo, in his capacity as administrator of the estate of the
late Santiago M. Bermejo, leased the land to Leopoldo L. Somes with
the approval of the Court of First Instance of Capiz. At present Leopoldo
L. Somes is in actual possession of Lot 855 by virtue of said lease
contract. Issue: whether or not a parcel of land, in the possession of
the predecessors- in-interest and the oppositor Bermejo as far back as
1905, asserted to have originally been mangrove swamps, thereafter
converted into a fishpond, may still be considered as part of the timber

domain which is not disposable.Held:No. The opposition rests mainly


upon the proposition that in the land covered by the application there
are mangrove lands, but the court thinks this opposition of the Director
of Forestry is untenable, in as much as it has been definitely decided
that mangrove lands are not forests lands in the sense in which this
phrase is used in the Act of Congress. It could be said, therefore, that
even on the assumptions that the parcel of land in question could be
characterized as mangrove swamps, the conclusion reached by the
lower court is not without support in the applicable authorities.
"Mangrove swamps where only trees of mangrove species grow, where
the trees are small and sparse fit only for firewood purposes and the
trees growing are not of commercial value as lumber, do not convert
the land into public land. Such lands are not forest in character. They
do not form part of the public domain."

3(Gina)

REPUBLICOF THE PHILIPPINES vs. AMANDA LAT VDA. DE CASTILLO


G.R. No.L-69002 June 30, 1988

Facts:
Sometimein 1951, the late Modesto Castillo applied for the registration
of two parcelsof land, Lots 1 and 2, located in Banadero, Tanauan,
Batangas, described inPlan Psu-119166, with a total area of 39,755
square meters. In a decision datedAugust 31, 1951, the said Modesto
Castillo, married to Amanda Lat, was declaredthe true and absolute
owner of the land with the improvements thereon, for whichOriginal
Certificate of Title No. 0-665 was, issued to him by the Register
ofDeeds at Batangas, Batangas, on February 7, 1952.

TheRepublic of the Philippines filed Civil Case No. 2044 with the lower
court forthe annulment of the certificates of title issued to defendasors
of ModestoCastillo, and for the reversion of the lands covered thereby
(Lots 1 and 2,Psu-119166) to the State. It was alleged that said lands

had always formed partof the Taal Lake, washed and inundated by the
waters thereof, and being ofpublic ownership, it could not be the
subject of registration as privateproperty.

On theother hand, private respondents maintain that Lots 1 and 2 have


always been inthe possession of the Castillo family for more than 76
years and that their possessionwas public, peaceful, continuous, and
adverse against the whole world and thatsaid lots were not titled
during the cadastral survey of Tanauan, because theywere still under
water as a result of the eruption of Taal Volcano on May 5,1911 and
that the inundation of the land in question by the waters of Taal
Lakewas merely accidental and does not affect private respondents'
ownership andpossession thereof pursuant to Article 778 of the Law of
Waters. They finallyinsisted that this issue of facts had been squarely
raised at the hearing ofthe land registration case and, therefore, res
judicata.

Otherwisestated, it has been satisfactorily established as found by the


trial court,that the properties in question were the shorelands of Taal
Lake during thecadastral survey of 1923. The trial court decided the
case in favor of thegovernment but the decision was reversed on
appeal by the Court of Appeals.

Issue:
Whetheror not the lakeshore lands are part of the public domain.

Held:
Yes Lakeshoreland or lands adjacent to the lake, like the lands in
question must bedifferentiated from foreshore land or that part of the
land adjacent to the seawhich is alternately covered and left dry by the
ordinary flow of the tides.

Suchdistinction draws importance from the fact that accretions on the


bank of alake, like Laguna de Bay, belong to the owners of the estate

to which they havebeen added while accretion on a sea bankstill


belongs to the public domain, and is not available for private
ownershipuntil formally declared by the government to be no longer
needed for public use.

But saiddistinction will not help private respondents because there is


no accretionshown to exist in the case at bar. On the contrary, it was
established that theoccupants of the lots who were engaged in duck
raising filled up the area withshells and sand to make it habitable.

Thedefense of long possession is likewise not available in this case


because, asalready ruled by this Court, mere possession of land does
not by itselfautomatically divest the land of its public character.

4(Zhon)

Republic vs. Alagad, et al.


169 SCRA 455, G.R. No. L-66807, January 26, 1989

Facts:
On or about October 11, 1951, defendants filed an application for
registration of their title over a parcel of land situated at Linga, Pila,
Laguna, with an area of 8.1263 hectares, which was amended after the
land was divided into two parcels, namely, Lot 1 with an area of 5.2476
hectares and Lot 2 with an area of 2.8421 hectares.
The Republic opposed the application on the ground that applicants
and their predecessors have not been in possession of the land openly,
continuously, publicly and adversely under a bona fide claim of
ownership since July 26, 1894 and the land has not ceased to be a part
of the public domain.

Defendants were declared owners of Lot 1 and the remaining portion,


or Lot 2, was declared public land. Decree No. N-51479 was entered
and Original Certificate of Title No. 0- 40 1, dated October 18, 1956,
was issued in the names of defendants.
In August, 1966, Civil Case No. 52 of the Municipal Court of Pila,
Laguna, was filed by defendants to evict the barrio folk occupying
portions of Lot 1. On August 8, 1968, judgment was rendered in the
eviction case ordering the defendants therein to return possession of
the premises to herein defendants, as plaintiffs therein. The
defendants therein did not appeal.
Republic filed a petition for annulment of title and reversion, claiming
that the subject land is a foreshore land, belongs to the public domain,
and concomitantly, cannot be appropriated by respondents. On
October 6, 1970, as prayed for in the complaint, a writ of preliminary
injunction was issued enjoining the Provincial Sheriff of Laguna or his
deputies from enforcing the writ of execution issued in Civil Case No.
52, and the defendants from selling, mortgaging, disposing or
otherwise entering into any transaction affecting the area.
This case was set for pre-trial on July 6, 1971. Despite notice of the pretrial, Atty. Alejandro A. Ponferada, Special Attorney, Bureau of Lands,
representing plaintiff Republic, did not appear. On July 16, 1971, the
trial court dismissed the complaint.
In sustaining the trial court, the Court of Appeals held that under the
Rules of Court, dismissal was proper upon failure of the Republic to
appear for pre-trial. It likewise ruled that the judgment, dated January
16, 1956, in the said LRC No. 189 has long become final.

Issue:
1. w/n the Govt be bound by, or estopped from, the mistakes or
negligent acts of its official or agents2. w/n the subject land is a
foreshore land [define foreshore land]Held:1. It is well-established that
the State cannot be bound by, or estopped from, the mistakes or
negligent acts of its official or agents, 7 much more, non-suited as a
result thereof.
2. A foreshore land is a strip of land that lies between the high and low
water marks and that is alternatively wet and dry according to the flow
of the tide. If the submergence, however, of the land is due to
precipitation, it does not become foreshore, despite its proximity to the

waters.Since this issue is a question of fact, SC remanded the case to


the trial court to determine whether or not the property subject of
controversy is foreshore.

Held:
1. It is well-established that the State cannot be bound by, or estopped
from, the mistakes or negligent acts of its official or agents, 7 much
more, non-suited as a result thereof.
2. A foreshore land is a strip of land that lies between the high and low
water marks and that is alternatively wet and dry according to the flow
of the tide. If the submergence, however, of the land is due to
precipitation, it does not become foreshore, despite its proximity to the
waters.
Since this issue is a question of fact, SC remanded the case to the trial
court to determine whether or not the property subject of controversy
is foreshore.

5(May Ann)

6(Rhea B.)
DIRECTOR OF LAND MANAGEMENT VS CA205 SCRA 486Facts:Teodoro
Abistado filed a petition for original registration of his title over 648
square meters of land under Presidential Decree (P.D.) No. 1529. The
land registration court in its decision dated June 13, 1989 dismissed
the petition for want of jurisdiction, in compliance with the
mandatory provision requiring publication of the notice of initial
hearing in a newspaper of general circulation. The case was elevated
to respondent Court of Appeals which, set aside the decision of the trial
court and ordered the registration of the title in the name of Teodoro
Abistado. The Court of Appeals ruled that it was merely procedural and
that the failure to cause such publication did not deprive the trial court
of its authority to grant the application. The Director of Lands
represented by the Solicitor General thus elevated this recourse to the
Supreme Court.Issue:Whether or not the Director of Lands is correct
that newspaper publication of the notice of initial hearing in an original
land registration case is mandatory.Ruling:YES. Petition was

granted.The pertinent part of Section 23 of Presidential Decree No.


1529 requires publication of the notice of initial hearing. It should be
noted further that land registration is a proceeding in rem. Being in
rem, such proceeding requires constructive seizure of the land as
against all persons, including the state, who have rights to or interests
in the property. An in rem proceeding is validated essentially through
publication. This being so, the process must strictly be complied
with.The Supreme Court has no authority to dispense with such
mandatory requirement. The law is unambiguous and its rationale
clear. Time and again, this Court has declared that where the law
speaks in clear and categorical language, there is no room for
interpretation, vacillation or equivocation; thereis room only for
application. There is no alternative. Thus, the application for land
registration filed by private respondents must be dismissed without
prejudice to reapplication in the future, after all the legal requisites
shall have been duly complied with.
7(Angel)

8. G.R. No. L-69002 June 30, 1988


Republic vs. Vda. de Castillo, 163 SCRA 286
Facts:
In 1951, Modesto Castillo applied for the registration of two parcels of
land, Lots 1 and 2, located in Tanauan, Batangas. On August 31, 1951
Modesto, married to Amanda Lat, was declared the true and absolute
owner of the land, for which Original Certificate of Title was issued to
him by the Register of Deeds. After Modestos death, Amanda Lat Vda.
de Castillo, et al., executed a deed of partition and assumption of
mortgage in favor of Florencio L. Castillo, et al.
The Republic of the Philippines filed a case in the lower court for the
annulment of the certificates of title issued to defendantx, as
heirs/successors of Modesto Castillo, and for the reversion of the lands
covered thereby to the State. Petitioner contends that Lots 1 and 2 had
always formed part of the Taal Lake, washed and inundated by the
waters thereof. Consequently, the same were not subject to
registration, being outside the commerce of men; and that since the
lots in litigation are of public domain, the registration court (of 1951)
did not have jurisdiction to adjudicate said lands as private property.

On the other hand, private respondents maintain that Lots 1 and 2


have always been in the possession of the Castillo family for more than
76 years and that their possession was public, peaceful, continuous,
and adverse against the whole world and that said lots were not titled
during the cadastral survey of Tanauan, because they were still under
water as a result of the eruption of Taal Volcano on May 5, 1911 and
that the inundation of the land in question by the waters of Taal Lake
was merely accidental and does not affect private respondents'
ownership and possession thereof pursuant to Article 778 of the Law of
Waters.
After trial the Court of First Instance ruled in favor of the Republic of
the Philippines.
The Court of Appeals subsequently reversed the decision and
dismissed the complaint.

Issue:
Are the properties in dispute capable of being registered?

Held:
No. Shores are properties of the public domain intended for public and,
therefore, not registrable. Thus, it has long been settled that portions
of the foreshore or of the territorial waters and beaches cannot be
registered. Their inclusion in a certificate of title does not convert the
same into properties of private ownership or confer title upon the
registrant.
It has been satisfactorily established as found by the trial court, that
the properties in question were the shore lands of Taal Lake during the
cadastral survey of 1923.
Lakeshore land or lands adjacent to the lake, like the lands in question
must be differentiated from foreshore land or that part of the land
adjacent to the sea which is alternately covered and left dry by the
ordinary flow of the tides.
Such distinction draws importance from the fact that accretions on the
bank of a lake, like Laguna de Bay, belong to the owners of the estate
to which they have been added while accretion on a sea bank still

belongs to the public domain, and is not available for private


ownership until formally declared by the government to be no longer
needed for public use.
The defense of long possession is likewise not available in this case
because, as already ruled by this Court, mere possession of land does
not by itself automatically divest the land of its public character.

9
Cantoja vs Lim

Facts:
Late Roberto Cantoja Sr.filed with the office of the DENR,
GenSan City, an application for a ForshoreLease Contract over an area
situated in Makar, GenSan. Cantoja was arawded theFLA (Foreshore
Lease Agreement) on Novemver 1990. Lim protest questioning thesaid
FLA to Cantoja based on his allegation that Cantoja committed fraud
andmisrepresentation in declaring in his application that the subject
foreshorearea adjoined hi (cantojas) property. To prove, he presented
his TransferCertificate Title which adjoins the foreshore area subject of
the lease.
Regional ExecutiveDirector Momongan of DENR Davao
assigned the case to Investigator Marohomsalicfor further investigation
which found that Cantoja was in acyual possession ofthe foreshore area
which utilized as dock-board of the Cantojas FishingBusiness. On Feb
1996, Geodetic Engineer Soria, in compliance with the Oct 1995 Order
of the City and NaturalResources Office, Submitted his report stating
that there was bo overlapping ofthe lot of Cantoja and the Foreshore
area. Director Momongan dismissed theprotest.
Meanwhile, on Oct1997 DENR instituted Civil case for
annulment of Patent coth issued in the Nameof Jacinto Acharon, as well
as Lims TCT. The suit was anchored on the findingsand
recommendations of Maromhomsalic that the area in question is partly
foreshoreand partly river bed of the Makar GenSan and therefore,
inalienable.
On May 2, 2000,DENR Sec. Cerilles cancelled the FLA
previously granted to Cantoja. Cantojafiled again a motion for Recon.

Sec. Cerilles issued special order for thecreation of a team to conduct


investigatiob and ocular inspection. Withoutwaiting, however,for the
result of the investigation of said team, Sec.Cerilles set aside its 2 May
2000 in favour of Cantoja. On appeal, the Officeof the president
rendered herein assailed decision affirming the 17 Oct 2000Order of
the DENR Sec.
Lim appealed to CAwhich reinstated the May 2 2000 decision
of the DENR Sec, which cancelled andrescinded the FLC covering the
foreshore area.

Issue: Whether the CA erred in cancelling the Foreshore Lease


Contractgranted ro Cantoja covering the foreshore area.

Ruling: No. It is undisputed thatCantoja is the not the Registered owner


of the land adjacent to the foreshorearea leased to them. Cantoja
committed fraud when he misrepresented himself asthe riparian of
littoral owner in his application for the foreshore lease.Under
stipulation no. 15 of the FLA, any fraud or misrepresentation
committedby the applicant is a ground for cancellation or rescission of
the FLA.

10(Lou)

BALAIS-MABANAGv. REGISTER OF DEEDS OF QUEZON City,


CONCEPCION D. ALCARAZ, and RAMONAPATRICIA ALCARAZ
G.R.No. 153142 (29 March 2010)
BERSAMIN, J.:

FACTS: On January 19, 1985, Coronels (Romulo A. Coronel, Alarico


A.Coronel, Annette A. Coronel, Annabelle C. Gonzales, Floraida C.
Tupper, andCielito A. Coronel) executed a document entitled receipt of

down payment,stipulating that they received from respondent Ramona


Patricia Alcaraz(Ramona), through Ramonas mother, respondent
Concepcion D. Alcaraz(Concepcion), the sum of P50,000.00 as
downpayment on the total purchaseprice of P1,240,000.00 for their
"inherited house and lot, covered byTCT No. 119627 of the Registry of
Deeds of Quezon City."

OnFebruary 18, 1985, the Coronels sold the property covered by TCT
No. 327043 tothe petitioner for the higher price of P1,580,000.00 after
the latterdelivered an initial sum of P300,000.00. For this reason, the
Coronelsrescinded their contract with Ramona by depositing her
downpaymentof P50,000.00 in the bank in trust for Ramona Patricia
Alcaraz.

TheCoronels executed a deed of absolute sale in favor of the petitioner.


On June5, 1985, TCT No. 351582 was issued in the name of the
petitioner.

OnMarch 1, 1989, the RTC rendered its decision ordering defendant to


execute infavor of plaintiffs(Concepcion and Ramona) a deed of
absolute sale and nopronouncement as to costs. Upon denial of the
motion for reconsideration, theCoronels and the petitioner interposed
an appeal to the CA, which promulgated ajudgment on December 16,
1991, fully upholding the decision of the RTC. Thus,the petitioner and
the Coronels appealed the CA judgment to this Court (G.R.No. 103577),
which affirmed the CA on October 7, 1996. Thereafter, thedecision of
the RTC became final and executory.

Uponfailure of the petitioner and the Coronels to comply with the writ
ofexecution, the RTC approved the respondents motion for
appointment of suitableperson to execute deed, etc., and ordered on
April 8, 1998 the Branch Clerk ofthe RTC, Branch 83, Quezon City, to
execute the deed of absolute sale in favorof Ramona in lieu of the
defendants (i.e., the petitioner and the Coronels).

Thepetitioner and the Coronels filed in the CA a petition for certiorari


assailingthe RTCs orders of October 1, 1997 and March 10, 1998, but
the CA dismissedthe petition on July 30, 1998.

OnSeptember 2, 1998, the RTC held in abeyance the respondents


motion reiteratingprevious motion to resolve respondents motion,
whereby the respondents soughtan order to direct the petitioner to
surrender her TCT No. 331582, and theRegistrar of Deeds of Quezon
City to cancel the petitioners copy of said TCTfor her failure to comply
with the earlier order for her to surrender the TCTto the Registrar of
Deeds pending resolution by the CA of the petitioners motionfor
reconsideration.

Ultimately,on September 30, 1998, the CA denied the petitioners


motion forreconsideration.

ISSUE:W/N the petitioner lacked the personality to file the suit.

HELD:YES.

TheCourt stated that under Section 7, Batas Pambansa Blg.185, the


Solicitor
General
or
his
representative
shall
institute
escheatproceedings against its violators. Although the law does not
categoricallystate that only the Government, through the Solicitor
General, may attack thetitle of an alien transferee of land, it is
nonetheless correct to hold thatonly the Government, through the
Solicitor General, has the personality to filea case challenging the
capacity of a person to acquire or to own land based onnon-citizenship.
This limitation is based on the fact that the violation iscommitted
against the State, not against any individual; and that in the eventthat
the transferee is adjudged to be not a Filipino citizen, the
affectedproperty reverts to the State, not to the previous owner or any
otherindividual.

Herein,even assuming that Ramona was legally disqualified from


owning the subjectproperty, the decision that voids or annuls their
right of ownership over thesubject land will not inure to the benefit of
the petitioner. Instead, thesubject property will be escheated in favor
of the State in accordance withBatas Pambansa Blg. 185.
WHEREFORE,the petition for review on certiorari is denied, and the
decision datedDecember 5, 2000 promulgated in C.A.-G.R. SP No.
55576 is affirmed.

Coststo be paid by the petitioner.

SOORDERED.

Significance:Only the Government through the Solicitor General,


has the personality tofile a case challenging the capacity of a
person to acquire or to own landbased on non-citizenship.

11(Jess)

12(Diane)
OFFICE OF THE CITY MAYOR OF PARAAQUE CITY v. MARIO
D.EBIO AND HIS CHILDREN/HEIRS
G.R. No. 178411 June 23, 2010

FACTS: Respondents claim to be absolute owners of a parcel of land in


Paraaque City covered by Tax in the name of respondent Mario D.
Ebio. Said land was an accretion of Cut-cut creek. Respondents assert
that the original occupant and possessor land was their great
grandfather, Jose Vitalez, which was given to his son, Pedro Valdez, in
1930. From then on, Pedro continuously and exclusively occupied and
possessed the said lot. In 1966, after executing an affidavit
declaring possession and occupancy. He also paid taxes for the land.

Mario Ebio married Pedros daughter, Zenaida. Ebio secured building


permits from the Paraaque municipal office for the construction
of their house within the land. On March 30, 1999, the Office of the
Sangguniang Barangay of Vitalez passed a resolution seeking
assistance from the City Government of Paraaque for the construction
of an access road along Cut-cut Creek. The proposed road will run from
Urma Drive to the main road of Vitalez Compound traversing the lot
occupied by the respondents. Respondents immediately opposed and
the project was suspended.

On March 28, 2005, the City Administrator sent a letter to the


respondents ordering them to vacate the area within the next thirty
(30) days, or be physically evicted from the said property. Respondents
sent a reply, asserting their claim over the subject property and
expressing intent for further dialogue.
ISSUE: Whether or not the State may build on the land in question.

HELD: No. It is an uncontested fact that the subject land was formed
from the alluvial deposits that have gradually settled along the banks
of Cut-cut creek. This being the case, the law that governs ownership
over the accreted portion is Article 84 of the Spanish Law of Waters
of 1866, which remains in effect, in relation to Article 457 of the Civil
Code.
It is therefore explicit from the foregoing provisions that alluvial
deposits along the banks of a creek do not form part of the public
domain as the alluvial property automatically belongs to the owner of
the estate to which it may have been added. The only restriction
provided for by law is that the owner of the adjoining property must
register the same under the Torrens system; otherwise, the alluvial

property may be subject to acquisition through prescription by third


persons.
In contrast, properties of public dominion cannot be acquired
by prescription. No matter how long the possession of the properties
has been, there can be no prescription against the State regarding
property of public domain. Even a city or municipality cannot acquire
them by prescription as against the State.
Hence, while it is true that a creek is a property of public dominion, the
land which is formed by the gradual and imperceptible accumulation
of sediments along its banks does not form part of the public domain
by clear provision of law.

NOTES:
ART. 84. Accretions deposited gradually upon lands contiguous to
creeks, streams, rivers, and lakes, by accessions or sediments from the
waters thereof, belong to the owners of such lands.

Art. 457. To the owners of lands adjoining the banks of rivers belong
the accretion which they gradually receive from the effects of the
current of the waters.

13(April)

14(Daniel Eblahan)

Directorof Lands v. Rivas


GRno. L-61539, February 14, 1986
Aquino,C.J.:

Facts:
On March 14, 1873, the Alcalde Mayorand the Court of First
Instance of Tuguegarao granted Domingo Bunagan apossessory
information title called informacion posesoria, for a tract ofland
destined for grazing their cattle. On November 3, 1885, he also
obtaineda gratuitous adjustment title called composicion gratuita for
the parcel ofland in Nottab, Enrile, Cagayan. After Bunagans death,
the land was sold todifferent people. In another case the court ruled
that the whole land was soldto Cagayan Valley Agricultural
Corporation. In this case, a remainder of theland was transferred to
respondent Rivas by his brother and the land waseventually sold to the
other respondents. The trial court declared the landpublic land and
dismissed both their claims. The appellate court reversed theruling of
the lower court and granted the respondents application. TheDirector
of Lands appealed, contending that the land is part of a
forestreservation.
Issue:
Whether or not the land in dispute isinalienable land
Held:
Yes. The Court ruled to dismiss theapplications of the
respondents because the land is inalienable public grazingland, part of
a forest reserve under Presidential Proclamation 129 and cannotbe
registered. Grazing lands and timber lands are not alienable under the
1935and 1973 Constitutions and cannot be the subject of private
ownership.
In this case, the land in questionwas issued to the original
claimant in 1873 was described in the informacionpossessoria as
una estancia de ganado al terreno or grazing land. RespondentRivas
leased it in 1962 as pasture land. In the 1960 and 1968
taxdeclarations of respondent Rivas described the land as for
pastureexclusively. In the sale to respondent Pascuas father
describing the land asa parcel of pasture land. The land has always
been described and treated aspasture or grazing land.

15(Melodia)
________________________________________________________________________
_______
16(Mel)

Republic of the Philippines, petitioner


vs
Intermediate Appellate Court, Esteban Mendoza and Leon
Pasahol
155 SCRA 412 November 5, 1987

FACTS:
On December 8, 1968, a petition was filed by Esteban
Mendoza and Leon Pasahol with the then Court of First Instance of
Bataan with alleging ownership of a parcel of land which they have
purchased from its original owners and thereafter, they have actual
possession of the said land tacked on to their predecessors-in-interest
for a period exceeding 30 years.
The CFI of Bataan ruled on the case, favoring the private
respondent citing that after the sale had zealously cultivated the
property and religiously paid the taxes thereon for a good numbers of
years, the CFI of Bataan ruled that the private respondents
possession of the land was in good faith and that the private
respondent should not as a consequence, be held accountable for the
lapse of their predecessor to file a cadastral claim to the property. The
private defendant may tack their period of possession with that of their
vendors totaling to more than thirty (30) years.
When the Solicitor General appealed it to the Intermediate
Court pointing out Section 1 of Republic Act 391, as amended by
Republic Act 2061, wherein the Solicitor General reiterated that,
reopening cadastral proceedings is allowable only with respect to such
of said parcels of land as have not been alienated, reserved, leased,
granted, or otherwise provisionally or permanently disposed of by the
Government. Applying said provision to the lot in question, it is claimed

that the registration is not possible as said land is actually already


forest land and/or part of military reservation. In support of the
contention of the Solicitor General, it also cited the report of Forest
Guard Cresencio Abuzman to the District Forester which states that
the area involved is a portion of former Military Reservation turned
over to the Philippine Government, hence, not disposable under any
circumstances.
The Intermediate Appellate Court rendered its decision
affirming the decision of the CFI and disagreed with the contention of
the Solicitor General, stating that under the Proposed Land
Classification Project No. 4-C of Mariveles Bulacan, the contested lot
has already been delineated and classified by Bureau of Forestry as
alienable and disposable public land and was approved by the Director
of Lands as disposable in character.

ISSUE:
Whether or not a mere proposal to classify a Military
Reservation as alienable and disposable public land is allowed.

HELD:
No, the Supreme did not sustained the ruling of the
Intermediate Appellate Court that the land in dispute is no longer a
part of Military reservation on the basis of a mere proposal to classify it
as a alienable and disposable public land that is needed according to
the Supreme Court is a formal act declaring forest land reservation for
the purpose of classifying it as a alienable and disposable public land
need the approval of the President of the Philippines as required by
Public Land Act (C.A. No. 141) and R.A. 1275.
__________________________________________________________________

17(Rocky)

18(Maribeth)

Miller vs. DirectorofLands, 12 SCRA 292

FACTS:
OnJune 18, 1956, Miller and Espinosa applied for registration in theCFI
ofMasbate covering a parcel of land located in Tigbao, Milagros,
Masbate.Afternotice and publication, initial hearing was held. The
Director of Landsand Bureauof Public Highways filed written
oppositions, while 35 individualsappeared andexpressed verbal
oppositions.
Applicantspresentation of evidence ensued; private oppositors
weregiven 5 days to filewritten opposition for which 28 filed written
butunverified opposition. OnAugust 20, 1958 applicants finished
adducing evidenceand rested their case.
OnAugust 27, 1958 the private oppositors presented their firstwitness.
After hiscross-examination, counsel for applicants called the
Court'sattention to thelack of verification in the opposition filed by the
privateoppositors and movedto dismiss the same.
Thus,the private oppositors offered to verify their opposition.After
parties hadfiled memoranda, the court issued an order dismissing
theunverified opposition.Motion for reconsideration was denied. The
privateoppositors appealed from bothorders alleging among others
that lack of, ordefect in the verification of apleading may be waived by
the adverse party'sfailure to make a proper andtimely objection
thereto. Where a party proceedswith the case as though hisadversary's
pleading were verified, he waives thelack of verification of
suchpleading.

ISSUE: Whetheror not the belatedobjection to the unverified


opposition to the applicationshall be considered awaiver to such
defect?
HELD: Yes.The court heldthat the written appearance with opposition
presentedbypetitioners was a valid one, and sufficient to give them
legal standingincourt and would entitle them to notice, as a matter of
right. The lowercourterred in choosing to ignore the written
appearance with opposition, whichwas asubstantial compliance with

the law, that requires a formal answer. Theprivate oppositors should be


allowed, asthey had requested, to verify theiropposition because, in
any event, thesupposed defect is deemed waived.Theorders appealed
from are set aside and the case is remanded to the court aquo for
furtherproceedings.

19(Ailyn)

20(Jen)

REP. VS. CA, 73 SCRA 146


FACTS: On January 22, 1921, Eugenio de Jesus, the father of
respondent Alejandro de Jesus, applied with the Bureau of Lands for
Sales Patent (Sales Application No. 5436) of a 33-hectare situated in
barrio Libaron, Municipality of Davao (now Davao City). The property
applied for was a portion of what was then known as Lot 522 of the
Davao Cadastre.

On January 23, 1934, the Bureau of Lands, through its Davao District
Land Officer, accepted sealed bids for the purchase of the subject land.
One Irineo Jose bidded for P20.00 per hectare, while a certain Dr. Josc
Ebro submitted a bid of P100.50 per hectare The Director of Lands,
however, annulled the auction sale for the reason that the sales
applicant, Eugenio de Jesus, failed to participate in the bidding for nonservice of notice on him of the scheduled bidding.

In lieu of that sale, another bidding was held on October 4, 1934. Sales
applicant Eugenio de Jesus was the lone bidder. He equalled the bid
previously submitted by Dr. Jose Ebro and made a deposit of P221.00
representing 10% of the price of the land at P100.50 per hectare.

On November 23, 1934, the Director of Lands issued to Eugenio de


Jesus an Order of Award.

On August 28, 1936, the Director of Lands ordered an amendment of


the Sales Application of Eugenio de Jesus stating that "a portion of the
land covered by Sales Application No. 5436 (E-3231) of Eugenio de
Jesus is needed by the Philippine Army for military camp site purposes,
the said application is amended so as to exclude therefrom portion "A"
as shown in the sketch on the back thereof, and as thus amended, it
will continue to be given due course." The area excluded was Identified
as Lot 1176-B-2, the very land in question, consisting of 12.8081
hectares.

On September 7, 1936, President Manuel L. Quezon issued


Proclaimation No. 85 withdrawing Lot No. 1176-B-2 from sale and
settlement and reserving the same for military purposes, under the
administration of the Chief of Staff, Philippine Army.

On November 29, 1939, Eugenio de Jesus paid P660.45 covering the


8th and 10th installment for 20.6400 hectares, the remaining area
after his Sales Application was amended. This payment did not include
the military camp site (Lot No. 1176-B-2) as the same had already been
excluded from the Sales Application at the time the payment was
made.

On August 11, 1956, President Ramon Magsaysay revoked


Proclamation No. 85 and declared the disputed Lot 1176-B-2 open to
disposition under the provisions of the Public land Act for resettlement
of the squatters in the Piapi Beach, Davao City. In the following
October 9, President Magsaysay revoked this Proclamation No. 328 and
reserved the same Lot No. 1176-B-2 for medical center site purposes
under the administration of the Director of Hospital.

Whereupon, on December 6, 1969, petitioner Mindanao Medical Center


applied for the Torrens registration of the 12.8081-hectare Lot 1176-B-2
with the Court of First Instance of Davao. The Medical Center claimed

"fee simple" title to the land on the strength of proclamation No. 350
reserving the area for medical center site purposes.

Respondent Alejandro de Jesus, the son and successor-in-interest of


sale applicant Eugenio de Jesus, opposed the registration oil the
ground that his father, Eugenio de Jesus, had aquired a vested right on
the subject lot by virtue of the Order of Award issued to him by the
Director of Lands.

A certain Arsenio Suazo likewise filed his opposition to the registration


on the claim that the 2-hectare portion on the northeastern part of Lot
1176-B-2 belongs to him.

On September 2, 1966 the CFI of Davao awarded the lot to Mindanao


Medical Center, Bureau of Medical Services, Department of Health.

The two oppositors, Alejandro de Jesus and Arsenio Suazo appealed the
case to the respondent Court of Appeals.

CA modified the judgment denying the claim of Arsenio Suazo and


affirmed the appeal of Alejandre Y. de Jesus.
On July 5, 1974, petitioner Mindanao Medical Center moved for
reconsideration, maintaining ownership over the entire area of 12.8081
hectares, but the Appellate Court in a Special Division of Five denied
the motion on June 17, 19750.

Hence, this case.

ISSUE: Whether or not petitioner Mindanao Medical Center has


registerable title over a full 12.8081-hectare land by virtue of an

executive proclamation in 1956 reserving the area for medical center


site purposes.

HELD: Petitioner Mindanao Medical Center has registerable title over


the whole contested area of 12.8081 hectares, designated Lot No.
1176-B-2, and not only on a portion thereof occupied by the Medical
Center, its nervous disease pavilion and their reasonable
appurtenances. Proclamation No. 350, dated October 9, 1956, of
President Magsaysay legally effected a land grant to the Mindanao
Medical Center, Bureau of Medical Services, Department of Health, of
the whole lot, validity sufficient for initial registration under the Land
Registration Act. Such land grant is constitutive of a "fee simple" tile or
absolute title in favor of petitioner Mindanao Medical Center. Thus,
Section 122 of the Act, which governs the registration of grants or
patents involving public lands, provides that "Whenever public lands in
the Philippine Islands belonging to the Government of the Philippines
are alienated, granted, or conveyed to persons or to public or private
corporations, the same shall be brought forthwith under the operation
of this Act [Land Registration Act, Act 496] and shall become registered
lands." 9 It would be completely absurd to rule that, on the basis of
Proclamation No. 350, the Medical Center has registerable title on the
portion occupied by it, its nervous disease pavilion and the reasonable
appurtenances, and not on the full extent of the reservation, when the
proclamation explicitly reserved the entire Lot 1176-B-2 of 12.8081
hectares to the Center.

REPUBLICOF THE PHILIPPINES, THE SUPERINTENDENT OF THE


PHILIPPINE MILITARY ACADEMY,petitioners, vsHON. PIO R.
MARCOS, JUDGE, Court of First Instance of BaguioCity, KOSEN
PIRASO, SAMAY PIRASO, COTILENG PIRASO, PETER PARAN and
MARTINAPIRASO, DAISY PACNOS, SPOUSES ALBINO REYES and
ISABEL SANTAMARIA, and ARTUROTONGSON, respondents.
FACTS: It is by statute provided thatall persons "claiming title to
parcels of land that have been the objectof cadastral proceedings" in
actual possession of the same at the time ofthe survey but unable for
some justifiable reason to file their claim in theproper court during the
time, limit established by law, "in case suchparcels of land on account
of their failure to file such claims, have been, orare about to be
declared land of the public domain by virtue of judicialproceedings"
instituted within the forty-year period next preceding June20, 1953, the
time of the approval of this particular enactment, are granted"the right

within five years" from said date to petition for areopening of the
judicial proceedings but "only with respect to such ofsaid parcels of
land as have not been alienated, reserved, leased, granted,
orotherwise provisionally or permanently disposed of by the
Government, ...."The jurisdiction of respondent Judge Pio R. Marcos to
act in accordancewith Republic Act No. 931 in connection with the
petition for a reopening filedby respondent Kosen Piraso, joined by his
kinsmen, likewise respondents, allsurnamed Piraso, is assailed in this
certiorari and prohibition proceeding,included in which are the other
respondents, Daisy Pacnos and the spousesAlbino Reyes and Isabel
Santamaria, petitioners being the Republic of thePhilippines and the
Superintendent of the Philippine Military Academy.
ISSUE:Whether or not , respondent Judge is devoid of jurisdiction to
pass uponthe claim of private respondents invoking the benefits of
Republic Act No. 931.
HELD:Republic Act No. 931 speaks in a manner far from ambiguous. It
is quiteexplicit and categorical. Only persons "claiming title to parcels
of landthat have been the object of cadastral proceedings" are granted
the rightto petition for a reopening thereof if the other conditions
named therein aresuccessfully met. It cannot admit of doubt, therefore,
that if the parcels ofland were not the object of cadastral proceedings,
then this statute finds noapplication. Considering that as far back as
October 10, 1910, the thenPresident of the United States, William H.
Taft, issued an executive orderreserving for naval purposes the lots
now disputed, they could not have beenthe object of the cadastral
proceeding involving the Baguio townsitereservation, decided only on
November 13, 1922.What is even more conclusive asto the absence of
any right on the part of the private respondents to seek areopening
under Republic Act No. 931 is our ruling in Government v. Court ofFirst
Instance of Pampanga, a 1926 decision. We there explicitly held:"The
defendant's contention that the respondent court, in a cadastralcase,
has jurisdiction to order the registration of portions of a
legallyestablished military reservation cannot be sustained. The
establishment ofmilitary reservations is governed by Act No. 627 of the
Philippine Commissionand Section 1 of that Act provides that 'All lands
or buildings, or anyinterest therein, within the Philippine Islands lying
within the boundaries ofthe areas now or hereafter set apart and
declared to be military reservationsshall be forthwith brought under
the operations of the Land Registration Act,... .' "The conclusion is
therefore inescapable that, as contended bypetitioners, respondent
Judge is devoid of jurisdiction to pass upon the claimof private
respondents invoking the benefits of Republic Act No. 931.

22(Nelson) hindi ko makita ung case ni marcos

23(JM)

REPUBLICVS. JUDGE SINFOROSOFAGONIL


133SCRA 517

FACTS:After more than half a century fromthe 1922 decision declaring


BaguioTownsiteReservation as public domain, Modesta Paris, Lagya
Paris, SamuelBaliwan, PabloRamos, Jr., Josephine Abanag, Menita T.
Victor, EmilianoBautista and OdiDianson filed with the Court of First
Instance of Baguioapplications for theregistration of lots (with
considerable
areas)
inside
theBaguio
TownsiteReservation.
Alternatively, they allege that in case the lotsare notregisterable under
Act No. 496, then section 48(b) and (c) of thePublic LandLaw should be
applied because they and their predecessors have beeninpossession of
the lots for more than thirty years. The Director of lands ontheother
hand contented that thedisposition of the said lots should be made by
theDirector of Lands underChapter 11 of the Public Land Law regarding
TownsiteReservations.

ISSUE:Whetheror not the applicants may file for the registration of


lands covered bytheBaguio Townsite Registration.

HELD:No. Theapplicants do not base their applications under Act No.


496 on anypurchase orgrant from the State nor on possession since
time immemorial. Thatis why ActNo. 496 cannot apply to them. The
period of more than fifty yearscompletelybars the applicants from
securing relief due to the alleged lack ofpersonalnotice to their
predecessors. The law helps the vigilant but not thosewhosleep on
their rights. For time is a means of destroying obligations and
actions,becausetime runs against the slothful and contemners of their
own rights.

24(Maris)

25(Reg)

26
G.R. No. 199310

February 19, 2014

REPUBLIC OF THE PHILIPPINES, Petitioner, vs. REMMAN


ENTERPRISES, INC., represented by RONNIE P. INOCENCIO,
Respondent.

FACTS:

December 3, 2001, Remman Enterprises, Inc. filed an application with


the RTC for judicial confirmation of title over two parcels of land
situated in Barangay Napindan, Taguig, MM. 10 days later, the RTC
issued the Order finding REIs application for registration sufficient in
form and substance and set it for initial hearing on Feb. 21, 2002
reset to May 30. Notice of Initial Hearing was published in the Official
Gazette, Peoples Balita, and likewise posted in a conspicuous place on
the subject lots and bulletin board of the City hall of Taguig. Only
Laguna Lake Development Authority appeared as oppositor, thus an
order of general default was issued.
REI presented witness testimonies that they and its predecessors-in
interest have been in open, continuous, exclusive and notorious
possession of the said parcels of land long before June 12, 1945. The
original possessor of the land, Veronica Jaime, cultivated and planted
different kinds of crops in the said lots, through her caretakers and
hired farmers.

The following documents were presented by REI: Deed of Absollute


Sale dated August 28, 1989; survey plans of subject properties;
technical description of subject properties; Geodetic Engineers
Certificate; tax declarations for 2002; certification issued by Corazon
Calamno, Senior Forest Management Specialist of the DENR attesting
the lots form part of the alienable and disposable lands of the public
domain.
RTC ruled on May 16, 2007 granting REIs application for registration of
title. This was affirmed by the CA.

ISSUE:
Whether the CA erred in affirming the RTC Decision dated May 16,
2007, which granted the application for registration filed by the
respondent.

HELD:
Petition is meritorious.
Section 14(1) of PD 1529 provides that: Sec. 14. Who may apply. The
following persons may file in the proper CFI an application for
registration of title to land, whether personally or through their duly
authorized representatives: (1) Those who by themselves or through
their predecessors-in interest have been in open, continuous, exclusive
and notorious possession and occupation of alienable and disposable
lands of the public domain under a bona fide claim of ownership since
June 12, 1945, or earlier.
Section 14 of PD 1529 refers to the judicial confirmation of imperfect or
incomplete titles to public land acquired under Section 48(b) of
Commonwealth Act no. 141, or the Public Land Act as amended. Under
Section 14(1), applicants for registration of title must sufficiently
establish:
First, that the subject land forms part of the disposable and alienable
lands of the public domain;
Second, that the applicant and his predecessors-in interest have been
in open, continuous, exclusive and notorious (OCEN) possession and
occupation of the same; and

Third, that it is under a bona fide claim of ownership since June 12,
1945, or earlier.
The first requirement was not satisfied in this case. To prove that the
subject property forms part of the alienable and disposable lands of the
public domain, the respondent presented two certifications, issued by
Calamno attesting that the subject lots form part of the alienable and
disposable lands of the public domain. This is insufficient. The Court
clarified in Republic v. T.A.N. Properties, Inc. that in addition to the
certification issued by the proper government agency that a parcel of
land is alienable and disposable, applicants for land registration must
prove that the DENR Secretary had approved the land classification
and released the land of public domain as alienable and disposable.
They must present a copy of the original classification approved by the
DENR Secretary and certified as true copy by the legal custodian of the
records. PENRO or CENRO certifications are not enough. Roche only
presented the survey map and technical description of the land,
bearing no info on the lands classification.
For the second requirement, proof of specific acts of ownership must
be presented to substantiate the claim of OCEN possession and
occupation of the subject land. It consists of manifestation of acts of
dominion over it of such a nature as a party would actually exercise
over his own property. Mere casual cultivation of portions of the land
does not constitute possession under claim of ownership.

27(Ed)

28(LJ)

29(Zax)

30(Clathem)
Del Rosario-Igbiten V. Republic, GR No. 158449, 22 October 2004Facts:
On 08 January 1998, petitioners filed with the trial court an application

for registration of land under Presidential Decree (PD) No. 1529,


otherwise known as the Property Registration Decree. The application
covered a parcel of land in Silang, Cavite. Petitioners alleged that they
acquired the Subject Property by purchase, and that they, by
themselves and through their predecessors-in-interest, had been in
actual, continuous, uninterrupted, open, public, and adverse
possession of the Subject Property in the concept of owner for more
than 30 years. Teodoro Calanog came into possession of the Subject
Property in 1968. In the same year, the Subject Property was
transferred to spouses Alfredo Tonido and Agatona Calanog. On 21
November 1995, the Tonido family sold the Subject Property to
petitioners, as evidenced by a Deed of Absolute Sale. On 15 August
2000, the trial court rendered a decision approving petitioners
application for registration of the Subject Property.Issue: whether or not
the the decision approving petitioners application for registration of the
subject property is valid.Held: No. petitioners have failed to comply
with the period of possession and occupation of the Subject Property,
as required by both the Property Registration Decree and the Public
Land Act. It bears noting that, proceedings under the Property
Registration Decree and the Public Land Act are the same in that both
are against the whole world, both take the nature of judicial
proceedings, and the decree of registration issued for both is
conclusive and final. They differ mainly in that under the Property
Registration Decree, there already exists a title which the court only
needs to confirm. On the other hand, under the Public Land Act, there
exists a presumption that the land applied for still pertains to the State,
and that the occupants and possessors can only claim an interest in
the land by virtue of their imperfect title or continuous, open, and
notorious possession thereof. Nonetheless, in the end, the two laws
arrive at the same goal, namely, a Torrens title, which aims at
complete extinguishment, once and for all, of rights adverse to the
record title.

31(Lea)

G.R. No. L-57461 September 11, 1987DIRECTOR OF LANDS vs.


MERALCO FACTS:Manila Electric Company filed an amended
application for registration of a parcel of land located in Taguig, Metro
Manila on December 4, 1979. On August 17, 1976, applicant acquired
the land applied for registration by purchase from Ricardo Natividad

who in turn acquired the same from his father Gregorio Natividad as
evidenced by a Deed of Original Absolute Sale executed on December
28, 1970 Applicant's predecessors-in-interest have possessed the
property under the concept of an owner for more than 30 years. The
property was declared for taxation purposes under the name of the
applicant and the taxes due thereon have been paid.On May 29, 1981
respondent Judge rendered a decision ordering the registration of the
property in the name of the private respondent. The Director of Lands
interposed this petition.ISSUE: Whether a corporation may apply for
registration of title to land. RULING: Yes.In the Acme decision (The
Director of Lands v. Intermediate Appellate Court and Acme Plywood &
Veneer Co., Inc., etc., No. L-73002 (December 29, 1986), 146 SCRA
509), this Court upheld the doctrine that open, exclusive and
undisputed possession of alienable public land for the period
prescribed by law creates the legal fiction whereby the land, upon
completion of the requisite period ipso jure and without the need of
judicial or other sanction, ceases to be public land and becomes
private property.In the case at bar, if the land was already private at
the time Meralco bought it from Natividad, then the prohibition in the
1973 Constitution against corporations holding alienable lands of the
public domain except by lease (1973 Const., Art. XIV, See. 11) does not
apply.Petitioner, however, contends that a corporation is not among
those that may apply for confirmation of title under Section 48 of
Commonwealth Act No. 141, the Public Land Act.As ruled in the Acme
case, the fact that the confirmation proceedings were instituted by a
corporation is simply another accidental circumstance, "productive of a
defect hardly more than procedural and in nowise affecting the
substance and merits of the right of ownership sought to be confirmed
in said proceedings." Considering that it is not disputed that the
Natividads could have had their title confirmed, only a rigid
subservience to the letter of the law would deny private respondent
the right to register its property which was validly acquired.

PartIV

1.SUSIv. RAZON
G.R.No.L-24066

December 9, 1925

VILLA-REAL,J.:

FACTS:
OnDecember 18,1880, Pinlacsold the land in question, then a
fish pond, to Garciaand Mendoza. After havingbeen in possession
thereof for about eight years,Garcia and Mendoza sold theland to
Valentin Susi for the sum of P12. Thepossession and occupation of
theland in question by different owners has beenopen, continuous,
adverse andpublic, without any interruption, except duringthe
revolution, or disturbance.However, on September 13, 1913, Angela
Razoncommenced an action in the Court ofFirst Instance of Pampanga
to obtain thesaid land but the court dismissed thecomplaint. Having
failed in her attemptto obtain possession of the land inquestion
through the court, Angela Razonapplied to the Director of Lands forthe
purchase thereof. After making theproper administrative investigation,
theDirector of Lands sold the land toAngela Razon. By virtue of said
grant, Razonapplied for the proper certificateof title and the register of
deeds ofPampanga issued thereafter issued it.
ISSUE:Whether or not Angela Razon mayapply for the registration of
said land in herfavor.
HELD:
No because she Susi isthelawful owner of the land. It clearly
appears from the evidence thatValentinSusi has been in possession of
the land in question openly, continuously,adversely,publicly,
personally, and through his predecessors, since the year1880, thatis,
for about forty-five years. Angela Razon is not allowed to applyfor
thegrant in her favor since Valentin Susi had already acquired,
byoperation oflaw, not only a right to a grant, but a grant of the
Government.Consequently,the act of the Director of Deeds in selling
the land over whichhe had no anytitle or control to Angela Razon was
void and of no effect. Thus,Angela Razondid not thereby acquire any
right.

2(Charleen)

3(Gina)

Priolovs Priolo, GR No. 37698, 71 Of. Gaz. 48, December 1, 1975


*casecannot be found. If you find it, please tell me so I can digest it for
theclass. Thank you very much. :)

4(Zhon)

Republic vs. CA and Alejandro de Jesus


G.R. No. L-40912, September 30, 1976

Facts:
On January 22, 1921, Eugenio de Jesus, the father of respondent
Alejandro de Jesus, applied with the Bureau of Lands for Sales Patent
(Sales Application No. 5436) of a 33-hectare situated in barrio Libaron,
Municipality of Davao (now Davao City). On November 23, 1934, the
Director of Lands issued to Eugenio de Jesus an Order of Award.
However, after the conduct of survey by the Bureau of Lands, the area
was reduced to 20.6400 hectares.
On August 28, 1936, the Director of Lands ordered an amendment of
the Sales Application of Eugenio de Jesus stating that "a portion of the
land covered by Sales Application No. 5436 of Eugenio de Jesus is
needed by the Philippine Army for military camp site purposes. The
area excluded was identified as Lot 1176-B-2, the very land in
question, consisting of 12.8081 hectares.
On September 7, 1936, President Manuel L. Quezon issued
Proclaimation No. 85 withdrawing Lot No. 1176-B-2 from sale and
settlement and reserving the same for military purposes, under the
administration of the Chief of Staff, Philippine Army.
On May 15, 1948, then Director of Lands Jose P. Dans ordered the
issuance of patent to Eugenio de Jesus. On the same date, then

Secretary of Agriculture and Natural Resources Mariano Garchitorena


granted the Sales Patent to Eugenio de Jesus.
On August 11, 1956, President Ramon Magsaysay issued Proclamation
No, 328, revoking Proclamation No. 85 and declaring the disputed Lot
1176-B-2 open to disposition under the provisions of the Public land Act
for resettlement of the squatters in the Piapi Beach, Davao City.
On October 9, 1956, President Magsaysay issued Proclamation No. 350,
revoking this Proclamation No. 328 and reserving the same Lot No.
1176-B-2 for medical center site purposes under the administration of
the Director of Hospital.
On December 6, 1969, petitioner Mindanao Medical Center applied for
the Torrens registration of Lot 1176-B-2 with the CFI of Davao.
Respondent Alejandro de Jesus, the son and successor-in-interest of
sale applicant Eugenio de Jesus, opposed the registration.
On September 2, 1966, the CFI of Davao rendered judgment directing
the registration of the title to Lot No. 1176-B-2 of in the name of the.
Upon appeal, CA held that Lot 1176-B-2 must be registered in the
name of Alejandro de Jesus, but he is hereby ordered to relinquish to
the Mindanao Medical Center that portion of Lot 1176-B-2 which is
occupied by the medical center and nervous disease pavilion and their
reasonable appurtenances.
Hence, this petition.

Issue: whether or not petitioner Mindanao Medical Center has


registerabletitle over a full 12.8081-hectare land by virtue of an
executiveproclamation in 1956 reserving the area for medical center
sitepurposes

Held:
Petitioner has registerable title over the whole contested area
of12.8081 hectares, designated Lot No. 1176-B-2, and not only on a
portionthereof occupied by the Medical Center, its nervous disease
pavilion andtheir reasonable appurtenances. Proclamation No. 350
issued by PresidentMagsaysay, which legally effected a land grant of
the whole lot to theMindanao Medical Center, is sufficient for initial

registration under the LandRegistration Act [Act 496]. Section 122 of


the Act, provides that "Wheneverpublic lands in the Philippine Islands
belonging to the Government of thePhilippines are alienated, granted,
or conveyed to persons or to public orprivate corporations, the same
shall be brought forthwith under theoperation of this Act and shall
become registered lands.
Further, section 64 (e) of the Revised Administrative Code
empowersthe president "(t)o reserve from sale or other disposition and
for specificpublic uses for service, any land belonging to the private
domain of theGovernment of the Philippines, the use of which is not
otherwise directed bylaw. The land reserved "shall be used for the
specific purposes directed bysuch executive order until otherwise
provided by law."
Similarly, Section 83 of the Public Land Act (CA 141) authorizes
thePresident to "designate by proclamation any tract or tracts of land
of thepublic domain as reservations for the use of the commonwealth
of thePhilippines or of any of its branches, or of the inhabitants thereof,
... or forquasi-public uses or purposes when the public interest requires
it, includingreservations for ... other improvements for the public
benefit.

5(May Ann)

6(Rhea B.)
LARAGAN
G.R.

VS
No.

L-47644

August

CA
21,

1987

Facts:
On 14 October 1968, the herein petitioners filed an application with the
CFI of Isabela for the registration of their title over a parcel of land with
an area of 221,667 sq. m., more or less, situated in the Barrio of Sto.
Tomas, Ilagan, Isabela. The applicants alleged that they acquired said
parcel of land by way of an absolute deed of sale from the spouses
Anastacio and Lucrecia Sibbaluca and that they have been in
possession thereof for more than 34 years. The Land Registration
Commission issued a notice of initial hearing. On 7 July 1969, the
Solicitor General filed a written opposition, on behalf of the Director of

Lands, alleging that the applicants and their predecessor-in-interest do


not have sufficient title to the parcel of land sought to be registered.
He prayed that the land be declared public land. On 2 August 1969,
Teodoro Leano, Tomas Leano, Vicente Leano, Francisco Leano, and
Consolacion Leano filed their opposition to the application claiming
that they are the owners, pro indiviso, of the southern part of the land
applied for, with an area of 16 hectares of their deceased parents and
which has been in their possession for more than 30 years.
The trial court rendered judgment confirming the title of the applicants
over the parcel of land applied for and ordering its registration in the
names of the applicants. The oppositors appealed to the CA. On 9
November 1977, the appellate court affirmed the judgment of the trial
court, but excluded the southern portion of the land applied for, the
appellate court declaring such excluded portion to be public land, and
part
of
the
public
domain,
in
view
of
the failure of the applicants and oppositors to prove registrable title
over the same. The petitioners filed a motion for reconsideration of the
decision
but
their
motion
was
denied.
Issue:
Whether the appellate court acted without or in excess of jurisdiction in
declaring the parcel of land in question as public land.
.
Held:
The argument is untenable. While it may be true that the Director of
Lands did not appeal from the decision of the trial court, his failure to
so appeal did not make the decision of the trial court final and
executory, in view of the appeal interposed by the other oppositors,
Teodoro Leano, Tomas Leano, Francisco Leano, and Consolacion Leano,
who also seek the confirmation of their imperfect title over the land in
question. Neither did such failure of the Director of Lands to appeal
foreclose the appellate court from declaring the land in question to be
public land, since the oppositors and the herein petitioners are both
seeking the registration of their title pursuant to the provisions of
Section 48 (b) of the Public Land Law where the presumption always is
that the land pertains to the state, and the occupants and possessors
claim an interest in the same, by virtue of their imperfect title or
continuous, open, exclusive and notorious possession and occupation
under a bona fide claim of ownership for the required number of years.

Besides, it is an established rule that an applicant for registration is not


necessarily entitled to have the land registered in his name simply
because no one appears to oppose his title and to oppose the
registration of the land. He must show, even in the absence of
opposition, to the satisfaction of the court, that he is the absolute
owner, in fee simple. Courts are not justified in registering property
under the Torrens system, simply because there is no opposition
offered. Courts may, even in the absence of any opposition, deny
registration of the land under the Torrens system, if the facts presented
do not show that the petitioner is the owner, in fee simple, of the land
which
he
seeks
to
register.
The petition is denied for lack of merit.

7(Angel)

8(Janine)

9 Kidpalos vs Baguio Gold MiningCompany 14 SCRA 913

Facts:
Petitioners sued the Baguio Gold Mining Companyand the
director of Mines in the court of First Instance in Baguio,
seekingjudgement declared said plaintiffs to be the owners of certain
parcels of landsituated in Sitio Binanga, Barrio of Tuding, Itogon,
Benguet, Mt. Province, to annulthe declarations of location of certain
mineral claims of the Baguio GoldMining Company, overlapping the
parcels claimed by plaintiffs, and to recoverdamages from the
company. The complaint also sought to enjoin the director ofmines
from proceeding with the lode patent applications of the Mining
Companyand to have the mine buildings erected on the land in
question demolished atthe latters expense. The defendant BGMC,
claiming the virtue of valid locationsof the claims since 1925 to 1930.

Issue:
WoN an applicant who was previously deniedclaims of
ownership in revindicatory action cannot file for registration ofsame
land involved.

Held: If the record of theformer trial shows that the judgement could
not have been rendered withoutdeciding the particular matter, it will
be considered as having settled thatmatters as to all future actions
between the parties, and if a judgement necessarilypresupposes
certain premises, they are as conclusive as the judgement itself.
Since there can beno registration of land without applicant
being its owner, the final judgementof CA in the previous
litigationdeclaring that the mining companys title is superior to that of
appellantsshould be conclusive on the question in the present case.

10(Lou)

SPOUSESFRANCISCO LAHORA and TORIBIA MORALIZON, vs.


EMILIODAYANGHIRANG, JR., and THE DIRECTOR OF LANDS,
(37SCRA 346)
G.R.No. L-28565 (January 30, 1971)

FACTS:On 26 November 1965, appellants petitioned the Court of First


Instance of Davaofor registration of nine (9) parcels of land located in
barrio Zaragosa,municipality of Manay, province of Davao. Appellant
(Toribia Moralizon) alleged one-half of the parcels of land having been
acquired by inheritance,and the other half by purchase and by
continuous, open, public and adversepossession in the concept of
owner. One of the said parcels of land isidentified as lot No. 2228, plan
SWO-36856, Manay Cadastre.

Oppositorsalleged that lands belonging to him and his wife were


included in theapplication for registration, mentioning specifically Lot
No. 2228 which wassaid to be already covered by Original Certificate of
Title no.P-6055 in thename of oppositor's wife on 21 June 1956. Thus,
Oppositors filed a motion forcorrection of the number of the certificate

of title covering Lot No. 2228,erroneously referred to as OCT No. P6055, when it should properly be OCT No.P-6053 and also prayed in the
same motion that the petition be dismissed.

TheDirector of Lands also filed an opposition to the petition,


contending that theapplicants or their predecessors-in-interest never
had sufficient title overthe parcels of land sought to be registered, nor
have they been in open,continuous, and notorious possession thereof
for at least 30 years.

TheCourt of First Instance of Davao (in Land Reg. Case No. N-86),
dismissing theirpetition with respect to Lot No. 2228 on the ground of
previous registration,said appellants claiming that the question of the
validity of a certificate oftitle based on a patent allegedly obtained by
fraud can be raised by them in aland registration proceeding, contrary
to the ruling of the court a quo.

ISSUE:W/N Lot No. 2228 can be the subject of two registration


proceedings.

RULING:No, Lot No. 2228 cannot be the subject of two registration


proceedings.

Inthe present case, Lot No. 2228 was registered and titled in the name
ofoppositors' wife as of 21 June 1956, nine (9) years earlier. On 26
November1965, the appellants' petition for registration of the same
parcel of landbased on the ground that the first certificate of title (OCT
No. P-6053)covering the said property is a nullity, can no longer
prosper.

Orderlyadministration of justice precludes that Lot 2228, of the Manay


Cadastre,should be the subject of two registration proceedings. Having
become registeredland under Act 496, for all legal purposes, by the
issuance of the public landpatent and the recording thereof, further

registration of the same would leadto the obviously undesirable result


of two certificates of title being issuedfor the same piece of land, even
if both certificates should be in the name ofthe same person. And if
they were to be issued to different persons, theindefeasibility of the
first title, which is the most valued characteristic ofTorrens titles, would
be torn away.

Forthis reason, this Court has ruled in Pamintuan vs. San Agustin,
43Phil. 558, that in a cadastral case the court has no jurisdiction to
decreeagain the registration of land already decreed in an earlier case;
and that asecond decree for the same land would be null and void. Of
course, if thepatent had been issued during the pendency of the
registration proceedings, thesituation would be different.

WHEREFORE,finding no error in the order appealed from, the same is


hereby affirmed, withcosts against the appellants.

Significance:Land Registration Act; Character of certificate of


title for public landpatents. -- The rule in this jurisdiction,regarding
public land patents and the character of the certificate of titlethat may
be issued by virtue thereof, is that where land is granted by
thegovernment to a private individual, the corresponding patent
therefor isrecorded, and the certificate of title is issued to the grantee
becomingentitled to all safeguards provided to the grantee becoming
entitled to all thesafeguards provided in Section 38 of the land
Registration Act, the titleissued to the grantee becoming entitled to all
the safeguards provided inSection 38 of the said Act. In other words,
upon expiration of one year fromits issuance, the certificate of title
shall become irrevocable andindefeasible like a certificate issued in a
registration proceeding.

Periodfor review of decree; Proper party and relief after


expiration of period.-- Even assuming arguendo that there indeed
exists a proper case forcancellation of the patent for intrinsic fraud, the
action for review of thedecree should have been filed before the one
year period had elapsed.Thereafter, the proper party to bring the
action would only be the personprejudiced by the alleged fraudulent
act -- the owner and grantor -- and notanother fraudulent act -- the

owner and grantor -- and not another applicant orclaimant.


Furthermore, the relief provided by the law in such instance may
besecured by the aggrieved party, not in another registration, for land
alreadyregistered in the name of a person cannot the subject of
another registration,but in an appropriate action such as one for
reconveyance or reversion, or fordamages in case the property has
passed into the hands of an innocent purchaserfor value.

Correctionof typographical mistake in title number in motion.-As regards the complaint against the alleged correction of the number
of thecertificate of title covering Lot No. P-6053, it appearing that the
motion wasintended to rectify a clearly typographical mistake, there is
nothing irregularin the lower court's order granting the same.

NOTES.-- Effect of registration of administrative land patent. -Aftera free patent application is granted and the corresponding
certificate of titleis issued, the land covered thereby ceases to be a
part of the public domainand becomes private property over which the
Director of Lands has neithercontrol nor jurisdiction. (Sumail vs.
Judge of First Instance of Cotobato,L-8278, April 30, 1955, 51
O.G. 2413)

A certificate of title issued on registration of a homestead patent


partakes ofthe nature of a certificate in judicial proceedings as long as
the land isreally part of the disposable land of the public domain, and
the certificatebecomes indefeasible and incontrovertible upon the
expiration of one year fromits date of issuance ( Lucas vs. Durian, L7886, Sept. 23, 1957). If theland was private, the grant is a nullity
(Vital vs. Anore, L-4136, Feb. 29,1952, 53 O.G. 3739).

As stated in Acierto vs. De los Santos, L-5828, Sept. 29,


1954,once a homestead patent is registered, as required by, the land
become"registered land" within the meaning of Act no. 496 as to
whichadverse title cannot be acquired by prescription.

A homestead acquired in public land become registered land as


soon as thepatent issued to the homesteader is recorded in the
registry of deeds and thecertificate of title issued, thereafter being
subject to the same rights andprivileges as other registered lands
(Eugenio vs. Perdido, L-7083, May 19,1955)

11(Jess)

12(Diane)
ATOK BIG-WEDGE MINING COMPANY, PETITIONER, VS. HON.
INTERMEDIATE APPELLATE COURT and TUKTUKAN SAINGAN
[G.R. No. 63528. September 9, 1996]

FACTS: Atok Big Wedge Company and Tuktuktan Saingan are both
claiming ownership over subject land situated in the barrio of Lucnab,
Itogon, Benguet. Atok Big Wedge Mining Company contended that the
said parcel of land was being registered in the office of Mining Recorder
in 1921 and 1931 pursuant to Philippine Bill of 1902. It is about sixteen
years before TUKTUKAN declared the land in question for taxation
purposes and thirty four (34) years before private respondent filed the
land registration proceedings in 1965. They also showed the payment
of annual assessment fees for the said land since 1931. They also
claim that it is a mineral land. Tuktukan who was 70 years old at the
time he testified shows that he acquired the land from his father-in-law,
Dongail, when he married his daughter; that he was then 18 years old;
that at the time of his acquisition, it was planted with camotes, casava,
langka, gabi, coffee and avocados; that he lived on the land since his
marriage up to the present; that he has been paying the taxes during
the Japanese occupation and even before it; that he was never
disturbed in his possession. Supporting his oral testimony, he
submitted tax declarations both dated March 20, 1948, the former for a
rural land and the latter for urban land and improvement therein.

ISSUE: Whether or not Atok Wedge Mining Company has a better right
on the subject land.

HELD: No. The SC ruled in favor of the Tuktukan because he has


proven that he had in possession of the said land in a concept of an
owner, continuously, open and uninterrupted for a period of more than
30 years. He had improved also almost 90% of the said parcel of land.
He also had paid tax declaration of the said land since 1948 up to
present. SC also said that payment of annual assessment fee is not
enough proof. There must be an annual performance of labor or
undertaking of improvements in the mine. When an ocular survey was
made, it was evident that there was no improvements being made in
the said land and there is any sign of mining had happened in the land.

13(April)

14(Daniel Eblahan)

Ignaciov. Director of Lands


GRno. L-12958, May 30, 1960
Montemayor,J.:
Facts:
On January 25, 1950, FaustinoIgnacio applied to register a
parcel of mangrove land in Gasac, Navotas, Rizal.This parcel of land,
formed by accretion and alluvial deposits caused by theaction of the
Manila Bay, adjoins a parcel of land acquired by Ignacio from
thegovernment in 1936 and has occupied the land since 1935. The
Director of Landsopposed this application because the land is public
domain, a foreshore land.The trial court dismissed the application,
ruling that the parcel of land isforeshore land.

Issue:
Whether or not the parcel ofland in question is public domain
Held:
Yes. The Court affirmed thetrial court in ruling that the land is
public domain. The Court ruled that theland formed by the actions of
the sea is property of the State and that the lawon accretion only
applies to lands that are situated on the banks of rivers.The court
disagreed with the petitioner that Manila bay is not a sea. It is apart of
the sea, a mere indentation of the same.
In this case, the petitionercannot claim private ownership of
the land because the land in question is notsituated in a river bank, it
is situated in Manila Bay. The alluvial depositsand accretion was caused
by the actions of the Manila Bay, which is a part ofthe sea. The land
formed is thus, part of the public domain.

15(Melodia)
________________________________________________________________________
_________________________
16(Mel)

REPUBLIC VS. DE PORKAN


G.R. No. L-66866
June 18, 1987, 151 SCRA 88

- Possession resulting in presumption of right to grant application

FACTS
Minda de Porkan and Lolita Macatindog acquired Lots Nos. 1099 and
1546 from their predecessors-interests, who in turn acquired said lots
though a grant by the government by virtue of their proven, open,
exclusive and undisputed possession for more than 30 years. An issue
over said lots arose when a certain Viola Azurin obtained from the then
Philippine Fisheries Commission an Ordinary Fishpond Permit covering
portions of Lots Nos. 1099 and 1546. Azurin filed with the Bureau of
Lands a complaint for correction, amendment or cancellation of the
Homestead Patent of De Porkan over Lot no. 1546 and the Free Patent
of Macatindog over Lot No. 1099 alleging among others that the
patentees secured their patents and titles through fraud,
misrepresentation and illegal machinations. The Solicitor General sided
with Azurin; when the case was brought to the Court of First Instance,
the SG stated that the disputed portions of land were actually claimed
by Azurin and that such lands could not be disposed by the Director of
Lands under the Public Land Act. Hence, the patents and titles issued
to de Porkan and Macatindog were void in so far as the portion
occupied and covered by the fishpond permit of Azurin. After hearing
however, the CFI dismissed the complaints and upheld the validity of
the titles/patents of de Porkan & Macatindog over the lands in dispute.
The SG in the present petition avers among others that the lots in
dispute could not be the subject of disposition under the Homestead
and Free Patent provisions of the Public Act since they are marshy and
swampy, certified as such as more suitable for fishpond development,
disposable only thru lease under the Public Land Act.

ISSUE:
Whether or not possession and cultivation of a land for more than 30
years will entitle the possessor thereof of a government grant and a
certificate of title.

HELD:
Yes. As early as 1953, the respondents had already acquired by
operation of law not only a right to a grant over Lot No. 1099, but a
grant of the Government over the same alienable land by virtue of
their proven, open, exclusive and undisputed possession for more than
30 years, since the Spanish colonial period.

The possession of a public land identified as Lot No. 1099 dates back to
the time of the Spanish colonial period. Such possessions of the said
public land has attained the character and duration prescribed by law
as the equivalent of an express grant from the Government. The
mandate of the law itself provides that possessors shall be
conclusively presumed to have performed all the conditions essential
to a government grant and shall be entitled to a certificate of title. By
legal fiction, the land ceases to be public and thus becomes a private
land.
________________________________________________________________________
__________________________

17(Rocky)

18(Maribeth)

REPUBLICOF THE PHILIPPINES vs CAet. al., 99 SCRA 742


FACTS:
OnMay 16, 1966, PerpetuoAlpuerto, filed with the CFI of Quezon
(Branch II), amotion to reopenCadastral Case No. 97, LCR Cad. Rec. No.
1555, and to admit hisanswer over LotNo. 7718 of the Mauban
(Quezon) cadastre. After trial, the lowercourt renderedits decision
adjudicating to said Alpuerto Lot No. 7718 togetherwith
itsimprovements, and ordered the issuance in his favor of
thecorresponding decreeof registration, thus an order for the issuance
of
a
decreeof
registrationover
the
said
lot.
The
Land
RegistrationCommission then issuedDecree No. 127177 as the basis of
the issuance of OCTNo. 0-13541 in his favor.Portions of the lot were
subsequently transferred tovarious persons who wereissued their
respective
transfer
certificates
oftitle,
among
whom
are
privaterespondents.
OnApril 6, 1971, the Sol.Gen. filed for the government a complaint for
annulment,cancellation of titlesand for reversion of Lot No. 7718 of
Cadastral Survey ofMauban, Quezon to theState on the ground that the
decision of the court adjudicatingLot No. 7718 toAlpuerto, its order for
the issuance of the decree ofregistration as well asOCT No. 013541 and

all its derivative titles are allnull and void and withoutlegal effect
because the court had no jurisdiction toallocate the subjectland, which
is inalienable.
HereinRespondents IndustrialMarketing and Investments Corporation,
Henry O. AntonioAngeles and AureaAngeles filed a motion to dismiss
the complaint alleging thatthe action isbarred by a prior judgment and
that the court lacks jurisdictionover thenature of the action or suit.
Defendant, Andres Laredo, likewise fileda motionfor dismissal of the
complaint, based on the grounds that the complaintstatesno cause of
action and that venue is improperly laid.
Afterhearing the motionsfor dismissal and the opposition thereto, the
lower courtissued an orderdenying the motion to dismiss filed by
Respondent Laredo, butgranting themotion to dismiss filed by
Respondents Industrial Marketing andothers anddismissing the
complaint filed by petitioner. The motion forreconsideration was further
denied. Consequently,petitioner appealed the caseto the CA which
affirmed the order of dismissal bythe lower court. The motionfor
reconsideration was likewise denied, hence,this petition.
ISSUE: Whetheror not thecourt had jurisdiction to allocatethe subject
land, which isinalienable?
HELD: No.It was held that where the land covered by the homestead
applicationofpetitioner was still within the forest zone or under the
jurisdiction of theBureauof Forestry, the Director of Lands had no
jurisdiction to dispose of saidlandunder the provisions of the Public
Land Law, and the petitioner acquirednoright to the land. It follows that
"if a person obtains a title underthePublic Land Act which includes, by
oversight, lands which cannot beregisteredunder the Torrens system,
or when the Director of Lands did nothavejurisdiction over the same
because it is a public forest, the grantee doesnot,by virtue of the said
certificate of title alone, become the owner of thelandillegally
included". The title thus issued is void at law, sincetheofficer who
issued it had no authority to do so.

19(Ailyn)

20(Jen)

G.R.No.
L-26127
June
28,
1974
Benin
vs.
Tuason,
SupraFACTS:The plaintiffs in these three civil cases uniformly alleged,
in theirrespective complaint, that sometime in the year 1951 while
they were enjoyingthe peaceful possession of their lands, the
defendants, particularly thedefendant J.M. Tuason and Co. Inc., through
their agents and representatives,with the aid of armed men, by force
and intimidation, using bulldozers andother demolishing equipment,
illegally entered and started defacing,demolishing and destroying the
dwellings and constructions of plaintiffs'lessees, as well as the
improvements. They made inquiries regarding theprobable claim of
defendants, and in 1953 they discovered for the first timethat their
lands, as described in their respective complaint, had either
beenfraudulently or erroneously included, by direct or constructive
fraud, in whatappears as Parcel No. 1 (known as Santa Mesa Estate) in
Original Certificate ofTitle No. 735 of the Land Records of the province
of Rizal in the names of theoriginal applicants for registration, now
defendants, Mariano Severo Tuason yde la Paz, Teresa Eriberta Tuason
y de la Paz, Juan Jose Tuason y de la Paz,Demetrio Asuncion Tuason y
de la Paz, and Augusto Huberto Tuason y de la Paz.The plaintiffs in
each of the three complaints also alleged that the registeredowners
had applied for the registration of two parcels of land (known as
theSanta Mesa Estate and the Diliman Estate; that the registration
proceedingswere docketed as LRC No. 7681 of the Court of Land
Registration; They allegethat the application for registration in LRC No.
7681, containing theboundaries, technical descriptions and areas of
parcel No. 1 (Santa MesaEstate) and parcel No. 2 (Diliman Estate) was
published in the OfficialGazette; that before the decision was handed
down in LRC No. 7681, the area,boundaries and technical descriptions
of parcel No. 1 were altered and amended;that the area of parcel No. 1
as mentioned in Decree No. 17431 is bigger thanthe area of parcel No.
1 appearing in the application for registration aspublished in the
Official Gazette; that the amendments and alterations, whichwere
made after the publication of the original application, were
neverpublished; that on March 7, 1914 a decision was rendered in LRC
No. 7681 basedon the amended plan; that pursuant to the decision of
March 7, 1914 a decree ofregistration was issued on July 6, 1914,
known as Decree No. 17431, decreeingthe registration in the names of
the applicants of the two parcels of land(Santa Mesa Estate and
Diliman Estate).Issue: W/N the LRC hadjurisdiction to render the
decision for the reason that the amendment to theoriginal plan was not
published. HELD: We believe that the lower courterred when it held
that the Land Registration Court was without jurisdiction torender the

decision in LRC No. 7681. Under Section 23 of Act 496, theregistration


court may allow, or order, an amendment of the application
forregistration when it appears to the court that the amendment is
necessary andproper. Under Section 24 of the same act the court may
at any time order an applicationto be amended by striking out one or
more parcels or by severance of theapplication. The amendment may
be made in the application or in the surveyplan, or in both, since the
application and the survey plan go together. If theamendment consists
in the inclusion in the application for registration of anarea or parcel of
land not previously included in the original application, aspublished, a
new publication of the amended application must be made.
Thepurpose of the new publication is to give notice to all persons
concernedregarding the amended application. Without a new
publication the registrationcourt can not acquire jurisdiction over the
area or parcel of land that isadded to the area covered by the original
application, and the decision of theregistration court would be a nullity
insofar as the decision concerns thenewly included land. The reason is
because without a new publication, the lawis infringed with respect to
the publicity that is required in registrationproceedings, and third
parties who have not had the opportunity to presenttheir claim might
be prejudiced in their rights because of failure of notice.But if the
amendment consists in the exclusion of a portion of the area
coveredby the original application and the original plan as previously
published, anew publication is not necessary. In the latter case, the
jurisdiction of thecourt over the remaining area is not affected by the
failure of a newpublication.

22(Nelson)
ROXASVS. CA
G.R.No. 118436
March21, 1997

FACTS:

OnJuly 1990, herein private respondent Maguesun Management and


DevelopmentCorporation (Maguesun Corporation) filed an Application
for Registration of twoparcels of unregistered land located in Tagaytay

City. In support of itsapplication for registration, Maguesun Corporation


presented a Deed of AbsoluteSale dated June 10, 1990, executed by
Zenaida Melliza as vendor and indicatingthe purchase price to be
P170,000.00. Zenaida Melliza in turn, bought theproperty from the
original petitioner herein (because she was substituted byher heirs in
the proceedings upon her death), Trinidad de Leon vda. de Roxasfor
P200,000.00 two and a half months earlier, as evidenced by a Deed of
Saleand an Affidavit of Self-Adjudication.
Noticesof the initial hearing were sent by the Land Registration
Authority (LRA) onthe basis of Maguesun Corporations application for
registration enumeratingadjoining owners, occupants or adverse
claimants; Since Trinidad de Leon vda.de Roxas was not named therein,
she was not sent a notice of the proceedings.After an Order of general
default was issued, the trial court proceeded to hearthe land
registration case. Eventually, on February 1991 the RTCgranted
Maguesun Corporations application for registration.
Itwas only when the caretaker of the property was being asked to
vacate the landthat petitioner Trinidad de Leon Vda. de Roxas learned
of its sale and theregistration of the lots in Maguesun Corporations
name.
Hence,on April 1991, petitioner filed a petition for review before the
RTC to setaside the decree of registration on the ground that Maguesun
Corporationcommitted actual fraud. She alleged that the lots were
among the properties sheinherited from her husband, former President
Manuel A. Roxas and that her familyhad been in open, continuous,
adverse and uninterrupted possession of thesubject property in the
concept of owner for more than thirty years before theyapplied for its
registration under the Torrens System of land titling (in whichno
decision has been rendered thereon). Petitioner further denied that she
soldthe lots to Zenaida Melliza whom she had never met before and
that hersignature was forged in both the Deed of Sale and the Affidavit
ofSelf-Adjudication. She also claimed that Maguesun Corporation
intentionallyomitted her name as an adverse claimant, occupant or
adjoining owner in theapplication for registration submitted to the LRA
such that the latter couldnot send her a Notice of Initial Hearing.
Adocument examiner from the PNP concluded that there was no
forgery.Uponpetitioners motion, the signatures were re-examined by
another expert fromNBI. The latter testified that the signatures on the
questioned and sampledocuments were, however, not written by the
same person.

Despitethe foregoing testimonies and pronouncements, the trial


courtdismissed thepetition for review of decree of registration. Placing
greater weight on thefindings and testimony of the PNP document
examiner, it concluded that thequestioned documents were not forged
and if they were, it was Zenaida Melliza,and not Maguesun
Corporation, who was responsible. Accordingly, MaguesunCorporation
did not commit actual fraud.
Ina decision dated December 8, 1994, respondent court deniedthe
petition for review and affirmed the findings of the trial court.
TheCA held that petitioner failed to and demonstrate that there
was actualor extrinsic fraud, not merely constructive or
intrinsic fraud, a prerequisitefor purposes of annuling a
judgment or reviewing a decree of registration.
Hence,the instant petition for review where it is alleged that the CA
erred in rulingthat Maguesun Corporation did not commit actual fraud
warranting the settingaside of the registration decree and in resolving
the appeal on the basis ofMaguesun Corporations good faith.
Petitioners pray that the registration ofthe subject lots in the name of
Maguesun Corporation be cancelled, that saidproperty be adjudicated
in favor of petitioners and that respondent corporationpay for
damages.
ISSUE: WONprivate respondent Maguesun Corporation committed
actual fraud (signatureforgery) in obtaining a decree of registration
over the two parcels ofland, actual fraud being the only ground to
reopen or review a decreeof registration.

HELD: WHEREFORE,the instant petition is hereby GRANTED. The


Decision of the CA is herebyREVERSED AND SET AS
1.The Court here finds that respondent Maguesun Corporation
committed actualfraud in obtaining the decree of registration sought to
be reviewed bypetitioner. A close scrutiny of the evidence on record
leads the Court to theirresistible conclusion that forgery was indeed
attendant in the case at bar.Although there is no proof of respondent
Maguesun Corporations directparticipation in the execution and
preparation of the forged instruments, thereare sufficient indicia which
proves that Maguesun Corporation is not theinnocent purchaser for
value who merits the protection of the law. Even to alaymans eye,
the documents, as well as the enlarged photographic exhibit of
thesignatures, reveal forgery. Additionally, Zenaida Mellizas nonappearanceraises doubt as to her existence

Petitionerand her family also own several other pieces of property,


some of which areleased out as restaurants. This is an indication that
petitioner is not unawareof the value of her properties. Hence, it is
unlikely that indication that shewould sell over 13,000 sqm of prime
property in Tagaytay City to a stranger fora measly P200,000.00. Would
an ordinary person sell more than 13,000 sqm of primeproperty for
P170,000.00 when it was earlier purchased for P200,000.00?
3.Petitioner Vda. de Roxas contended that Maguesun Corporation
intentionallyomitted their name, or that of the Roxas family, as having
a claim to or as anoccupant of the subject property.
Thenames in full and addresses, as far as known to the undersigned, of
the ownersof all adjoining properties; of the persons mentioned in
paragraphs 3 and 5(mortgagors, encumbrancers, and occupants) and
of the person shown onthe plan (original application submitted in
LRC No) asclaimants are as follows:
HilarioLuna, Jose Gil, Leon Luna, Provincial Road
allat Tagaytay City (no house No.) 30
Thehighlighted words are typed in with a different typewriter, with the
first fiveletters of the word provincial typed over correction fluid.
MaguesunCorporation, however, annexed a differently-worded
application for thepetitionto review case. In the copy submitted to
the trial court, the answer to thesame number is as follows:
HilarioLuna, Jose Gil, Leon Luna, Roxas.
Thediscrepancy which is unexplained appears intentional. If the word
Roxas wereindeed erased and replaced with Provincial Road all at
Tagaytay City (no houseNo.) in the original application submitted in
LRC No. TG-373 BUT thecopy with the word Roxas was submitted to
the trial court, it is reasonableto assume that the reason is to mislead
the court into thinking that Roxaswas placed in the original
application as an adjoining owner, encumbrancer,occupant or
claimant, the same application which formed the basis for the
LRAAuthority in sending out notices of initial hearing. (Section 15 of PD
No. 1529actually requires the applicant for registration to state the full
names andaddresses of all occupants of the land and those of
adjoining owners, if knownand if not known, the extent of the search
made to find them. Respondentcorporation likewise failed to comply
with this requirement of law.)

Respondentcorporations intentional concealment and representation


of petitionersinterest in the subject lots as possessor, occupant and
claimantconstitutesactual fraud justifying the reopening and
review of the decree of registration.Through such misfeasance,
the Roxas family was kept ignorant of theregistration proceedings
involving their property, thus effectively deprivingthem of their day in
court

Thetruth is that the Roxas family had been in possession of the


property uninterruptedlythrough their caretaker, Jose Ramirez.
Respondent Maguesun Corporationalso declared in number 5 of the
same application that the subject land wasunoccupied when in truth
and in fact, the Roxas family caretaker resided in thesubject property.
Toconclude, it is quite clear that respondent corporation cannot tack
itspossession to that of petitioner as predecessor-in-interest. Zenaida
Mellizaconveyed not title over the subject parcels of land to Maguesun
Corporation asshe was not the owner thereof.
Maguesun
Corporation is thus notentitled to the registration decree
which the trial court granted in itsdecision.
Petitionerhas not been interrupted in her more than thirty years of
open, uninterrupted,exclusive and notorious possession in the concept
of an owner over the subjectlots by the irregular transaction to Zenaida
Melliza.She therefore retainstitle proper and sufficient for original
registration over the twoparcels of land in question pursuant to Section
14 of PD No. 1529.

23(JM)

Beninv. Tuason
G.R.No. L-26127
Facts:

June 28, 1974

Theplaintiffsalleged that they were the owners and possessors of the


threeparcels ofagricultural lands, described in paragraph V of the
complaint,located in thebarrio of LaLoma (now barrio of San Jose) in
the municipality(now city) ofCaloocan, province of Rizal, that they
inherited said parcels ofland from theirancestor Sixto Benin, who in
turn inherited the same from hisfather, EugenioBenin; that they and
their predecessors in interest hadpossessed these threeparcels of land
openly, adversely, and peacefully,cultivated the same andexclusively
enjoyed the fruits harvested therefrom;that Eugenio Benin,plaintiffs
grandfather, had said parcels of land surveyedon March 4 and 6,1894,
that during the cadastral survey by the Bureau of Landsof the lands
inBarrio San Jose in 1933 Sixto Benin and herein plaintiffs claimthe
ownershipover said parcels of land; that they declared said lands
fortaxation purposesin 1940 under Tax Declaration No. 2429; that after
theoutbreak of the lastWorld War, or sometime in 1942 and
subsequentlythereafter, evacuees from Manilaand other places, after
having secured thepermission of the plaintiffs,constructed their houses
thereon and paid monthlyrentals to plaintiffs. Onlydefendant J.M.
Tuason & Co., Inc. was actuallyserved with summons. Theother
defendants were ordered summoned by publicationin accordance
withSections 16 and 17of the Rules of Court. Only defendant J.M.Tuason
& Co.,Inc. appeared. The other defendants were all declared indefault.
Issue:
Doesan amendment due to exclusion does not requirea publication?
HELD:
Yes,it is presumed, therefore, that as occupantsproper notices thereof
were servedon them and that they were aware of saidproceedings. .
Therefore, thatthedecision of this Court, which affirmed the order of
the Court of FirstInstanceof Rizal dismissing the complaint of Jose
Alcantara, Elias Benin andPascualPili(along with four other plaintiffs)
should apply not only against theheirs,of Elias Benin, against Jose
Alcantara, and against Pascual Pili, asplaintiffsin Civil Cases Nos. 3621,
3622and 3623, respectively, but alsoagainst all theother plaintiffs in
those cases. We find that the plaintiffs donot claim aright which is
different from that claimed by Elias Benin.

24(Maris)

25(Reg)

26
G.R. No. L-32398

January 27, 1992

IN THE MATTER OF THE PETITION OF PO YO BI TO BE ADMITTED


AS CITIZEN OF THE PHILIPPINES: PO YO BI, petitioner-appellee,
vs. REPUBLIC OF THE PHILIPPINES, respondent-appellant.

FACTS:
On February 9, 1957, Po Yo Bi filed a petition for naturalization. On
March 5, 1959, the trial court, through the Deputy Clerk of Court,
issued a Notice of Petition for Philippine Citizenship setting the hearing
of the petition to January 18, 1960 and ordered the publication and
posting of the notice. Po Yo Bi filed for an amendment of information.
An Amended Notice of Petition was issued on January 18, 1960 setting
the hearing of the petition to October12, 1960 and ordering the
publication of the said notice once a week for three consecutive weeks
in the Official Gazette and in the YUHUM, newspaper of general
circulation in the prov/city of Iloilo. Another amendment was filed
(without claiming good moral character) and the Deputy Clerk of Court
issued another Amended Notice of Petition for Philippine Citizenship
setting the hearing to February 26, 1962 and directing publication of
the order in the Official Gazette and in the Guardian, newspaper of
gen. circulation in the prov/city of Iloilo. HOWEVER, the second
amended petition ITSELF was not published or posted in a public and
conspicuous place in the Office of the Clerk of Court or in the building
where such office is located. The petition was granted after trial on
October 15, 1963.
The Assistant City Fiscal of Iloilo, Vicente Gengos, on behalf of the
SolGen, filed a motion to reconsider as Po Yo Bi had not complied with
Sec.4 of the Revised Naturalization Law on filing his declaration of
intention.

ISSUE:

Whether the court erred in not finding that the amended petition for
naturalization was not published in accordance to the requirements of
section 9 of CA 473, as amended.

HELD:
The second amended petition was not published. Neither were the
original and the amended petitions. What the Office of the Clerk of
Court did was to prepare and issue NOTICES of petition. IT was said
notices alone which were ordered to be published and posted.
Section 9 of the Revised Naturalization Law requires that the petition
itself must be published. This provision demands the compliance of the
following requirements, namely: (1) the publication must be weekly; (2)
it must be made three times; (3) and these must be consecutive. The
Court further ruled that the publication is a jurisdictional requirement.
Thus: In short, non-compliance with the requirements thereof, relative
to the publication of the petition, affects the jurisdiction of the court. It
constitutes a FATAL defect, for it impairs the very root or foundation of
the authority to decide the case, regardless of whether the one to
blame therefor is the clerk of court or the petitioner or his counsel.
Failure to raise this question in the lower court would not cure such
defect.
The Court held that the requirement that a copy of the petition to be
posted and published should be textual or verbatim restatement of the
petition as filed is jurisdictional. Non-compliance therewith nullifies the
proceedings in the case, including the decision rendered in favor of the
applicant.

27(Ed)

28(LJ)

29(Zax)

30(Clathem)

LEYVA V. JANDOC, 4 SCRA 595

FACTS: On September 10, 1958, ManuelaJ andoc applied, in the Court


of First Instance of Cotabato, for theregistration of three (3) parcels of
land situated in Cotabato. Eligio T. Leyvaobjected thereto on December
10, 1958, with respect to a portion of said landof about one (1) hectare
which he claimed to have adversely possessed in goodfaith and under
legal title since 1937. On or about December 17, 1958 his wife Eufemia
L. Leyva filed another opposition alleging that she and her husband
hadoccupied a portion of the land in question, which they had acquired
from thedefunct "NARRA", and that both had similarly occupied as
owners since 1937, another portion of said land of about 88 meters by
6.66 meters,"with their improvements ... made in good faith." Later,
otheroppositions were filed, also, including one by the Bureau of Lands,
whichclaimed the land applied for as part of the public domain.
The Court of Appeals denied the opposition of Leyvas declaring that
the claim oft he Leyvas may be recognized only if the land in question
is public land, and sincethe Director of Lands has already filed an
opposition based on the ground that the land sought to be registered is
public land, the court ruled that theLeyvas have no right to appear as
independent oppositors, but could collaborate with the provincial fiscal.
Leyva's counsel was allowed to stay in court to collaborate, as in fact
he did collaborate, with the provincial fiscal.

Issue: whether or not the opposition of Leyvas will prosper.

Held: No.
The law provides that any person claiming any kind of interest to file
an opposition to an application for registration must be based on a

right of dominion or some other real right independent of, and not at
all subordinate to, the rights of the Government.
However,in the case, since the petitioners are foreshore lessees of
public land, their right is completely subordinate to the interests of the
Government, and must necessarily be predicated upon the property in
question being part of the public domain.

Invariably,it is incumbent upon the duly authorized representatives of


the Government torepresent its interests as well as private claims
intrinsically dependent uponit. It is well-settled that the interests of the
Government cannot berepresented by private persons.

31(Lea)

LEYVA vs. JANDOC4 SCRA 595


FACTS: Respondent Laiz filed before the Court of First Instance of
Cotabato a petition for cancellation of adverse claim annotated on the
back of his transfer certificate of title covering a parcel of land
registered in the name of Manuela Jandoc.Petitioner Leyva claimed that
he executed a Compromise Agreement in 1963 (notarized in 1972)
with Manuela Jandoc where the former will withdraw his opposition in
the Land Registration case in the condition that certain properties
automatically belong to him in case of favorable judgment for Jandoc.
Respondent Laiz reasoned that a private agreement of sale was
executed in 1959 between him and Respondent Jandoc.The Court of
First Instance of South Cotabato ruled against petitioner and confirmed
the sale between Laiz and Jandoc pertaining to the parcel of land. The
CFI also denied petitioners Motion for Reconsideration. The
Intermediate Appellate Court affirmed in toto the lower courts
decision.Hence a petition for review on certiorari was filed by petitioner

contending that the IAC committed grave error in ruling that Laizs
deed of absolute sale prevailed over Levyas compromise agreement.
ISSUE:Whether petitioner Levya, basing his claim on a Compromise
Agreement, has a better right over the parcel of land than respondent
Laiz, who executed a Deed of Sale with respondent Jandoc.
RULING:The instant petition is without merit.Upon consideration of the
foregoing, the Court of appeals concluded that the Compromise
Agreement whether executed in 1963 or 1972 between Leyva and
Jandoc cannot prevail over the Agreement of Sale between Laiz and
Jandoc and that Laiz has a better right over the property in question
than Leyva.But Leyva lays much stree on the findings of Francisco
Cruz, Jr., a handwriting expert of the PC Crime Laboratory, Camp
Crame who testified that the purported signature of Jandoc appeared
to be of another person and of Col. Crispin B. Garcia, formerly Chief of
the Chemistry Branch in the PC Crime Laboratory who testified that the
Agreement could have been written within the years 1964 to
1967.Both the trial court and the Court of Appeals gave more weight to
the testimony of Fiscal Sarinas as corroborated by Catolico, one of the
signatories in the Deed of Transfer, and Versoza, a notary public,
upholding the validity of the assailed Sale Agreement.This Court has
ruled in the case of Vda. De Roxas v. Roxas, that the positive
testimony of the three attesting witnesses ought to prevail over the
expert opinions which cannot be mathematically precise but which on
the contrary, are subject to inherent infirmities. In any event, it is well
established that the appellate court will not disturb the factual findings
of the lower court for the latter is in a better position to gauge
credibility of witnesses.
PartV

1.De Castro v. Marcos


January27, 1969 G.R. No. L-26093
SANCHEZ,J.:
FACTS:

Privaterespondent Rufino Akiacommenced a petition before


the Court of First Instanceof Baguio City, actingas a cadastral court, to
register in his name 15, 922square meters of landsituated in the City of
Baguio. However, petitionerVirginia L. de Castro movedto intervene.
Her interest is in the 1,000 squaremeters is allegedly includedin the
15,922 square meters of land specified inAkia's petition.
Itappears that De Castro wasthe highest bidder in the public
sale of a 1,000square meter-parcel of land inQuezon Hill, Baguio City
and such land was thereafterawarded in her favor. DeCastro then filed
with the Bureau of Lands TownshipSales Application coveringthe parcel
of land. Petitioner, as claimed, had beenpaying taxes on the lot.
However,the Respondent Judgedismissed the opposition of de
Castro on the ground a mereapplicant has nopersonality to sue but de
Castro moved to reconsider. Shestressed the fact thatshe was not a
mere applicant of public land but anequitable owner thereofbecause
she paid to the government in full the value ofthe land she appliedfor.

ISSUE:Does petitioner Virginia L. de Castro have legalstanding in the


proceedingsbelow?

HELD:
YES.Mere citizens could haveno interest in public land and to
give a partystanding in a court of landregistration, he must make some
claim to theproperty. Homesteader, purchaser offriar land, and all
persons who claimto be in possession prior the issuance oftheir titles or
awards to public landare can be considered as equitable owners.To
claim ownership, a homesteaderwho had not yet been issued his title
but whohad fulfilled all the conditionsrequired by law, should be
regarded as an equitableowner of theland. Similarly, a purchaser of
friar land must have anequitable title tothe land before the issuance of
the patent.
Recently,we declared thatpersons who claim to be in
possession of a tract of publicland and have appliedwith the Bureau of
Lands for its purchase havethe necessary personality tooppose
registratior. Since Petitioner Virginiade Castro is an awardee inthe
public sale held upon her own township salesapplication. The award
itselfis enough to prevent the reopening by respondentAkia as to the
land disputed.Of course,the award up to now has not been fully

implemented because she hasnot yetcomplied with one condition


imposed on her. But, if the award is notapermanent disposition, it is at
least a provisional one, enough topreventreopening by respondent Akia
as to the land disputed.
TheSupreme Court, accordingly,ruled that petitioner has legal
standing before thecadastral court.

2(Charleen)

3(Gina)

ANA P. FERNANDEZvs. FELIZA ABORATIGUE


G.R. No.L-25313

December 28, 1970

Makalintal,J.:

Facts:
Theplaintiff, claiming ownership of a parcel of land situated in sitio
Dipulao,barrio Central, township of Coron, Palawan, filed suit 1 in the
Palawan Courtof First Instance to have the defendants vacate a portion
of said land andcease exercising acts of ownership therein.

It wasalleged in the complaint that Vicente Aboratigue, father of


defendant FelizaAboratigue, was during his lifetime employed as guard
at one of the gates tothe property, and as such was allowed to plant

fruit trees in a small portionin the immediate vicinity; and that after his
death the said defendant,together with her husband Restituto Bacnan,
were allowed to stay therein, butthat later on they claimed to be the
owners of the said portion. In theiranswer to the complaint the
defendants alleged, as basis of their claim ofownership, continuous
possession and cultivation of the disputed area, bythemselves and
through their predecessors-in-interest since the year 1901.

Issue:
Whetheror not the improvements on the disputed land belong to the
Aboratigue.

Held:
Yes. Therule is that the owner of buildings and improvements should
claim them duringthe proceedings for registration and the fact of
ownership, if upheld by thecourt, must be noted on the face of the
certificate. There is no such notationhere in favor of the appellants.
However, inasmuch as the improvements in thedisputed area have
been acknowledgment by plaintiff in her complaint asbelonging to the
appellants' predecessor-in-interest, and the lower court'sdecision
allowing them to recover the value of the improvements is not now
inquestion, this right of the appellants must be upheld. The trial court
has leftthe determination of such value to mutual agreement between
the parties. Thisdisposition should be modified in the sense that if they
fail to agree, thematter should be submitted to the said court for
hearing and adjudication.

4(Zhon)

Republic vs. Antonio Bacas


G.R. No. 182913, November 20, 2013

Facts:
In 1938, Commonwealth President Manuel Luis Quezon (Pres. Quezon)
issued Presidential Proclamation No. 265, which took effect on March
31, 1938, reserving for the use of the Philippine Army 3 parcels of the
public domain situated in the barrios of Bulua and Carmen, then
Municipality of Cagayan, Misamis Oriental. The parcels of land were
withdrawn from sale or settlement and reserved for military purposes,
"subject to private rights, if any there be."
The Bacases filed their Application for Registration on November 12,
1964 covering a parcel of land [Lot No. 4354] which is a portion of
those reserved in PP No. 265. They alleged, inter alia, the names of
adjoining owners, as well as the occupancy of the PH army by mere
tolerance. The LRC granted the application. The Republic failed to
appeal said decision.
On May 8, 1974, the Chabons filed their application for Registration of
Lot No. 4357, also reserved in PP No. 265. They indicated therein the
names and addresses of the adjoining owners, but no mention was
made with respect to the occupation, if any, by the Philippine Army.
LRC granted the application. The Republic failed to appeal said
decision.
Republic filed with RTC a civil case for annulment, cancellation of
original certificate of title, reconveyance of lot or damages against
Bacases [Civil Case No. 3494] and another against Chabons [Civil Case
No. 5918].
The RTC dismissed the petition, holding the following: (1) the stated
fact of occupancy by Camp Evangelista over certain portions of the
subject lands in the applications for registration by the respondents
was a substantial compliance with the requirements of the law and the
respondents did not commit fraud in filing their applications for
registration and that the Republic was then given all the opportunity to
be heard as it filed its opposition to the applications, appeared and
participated in the proceedings so the Republic is estopped from
questioning the proceeding; (2) assuming arguendo that respondents
were guilty of fraud, the Republic lost its right to a relief for its failure
to file a petition for review on the ground of fraud within one (1) year
after the date of entry of the decree of registration; and (3) the subject
parcels of land were exempted from the operation and effect of the
Presidential Proclamation No. 265 pursuant to a proviso therein that
the same would not apply to lands with existing"private rights." The
presidential proclamation did not, and should not, apply to the
respondents because they did not apply to acquire the parcels of land

in question from the government, but simply for confirmation and


affirmation of their rights to the properties so that the titles over them
could be issued in their favor.
Upon appeal, CA affirmed the RTC decision.

Issue:
1. whether or not the decisions of the LRC over the subject lands can
still bequestioned
2. whether or not the applications for registration of the subject parcels
ofland should be allowed

Held:
1. The Republic can question even final and executory judgment
whenthere was fraud. Pursuant to , the application for registration shall
state thename in full and the address of the applicant, and also the
names andaddresses of all adjoining owners and occupants, if known;
and, if notknown, it shall state what search has been made to find
them. The purposeof this is to serve as notice and publication of the
hearing that would enableall persons concerned, who may have any
rights or interests in the property,to come forward and show to the
court why the application for registrationthereof is not to be granted.
The Chabons did not make any mention of the ownership or
occupancyby the Philippine Army. They also did not indicate any efforts
or searchesthey had exerted in determining other occupants of the
land. Such omissionconstituted fraud and deprived the Republic of its
day in court. Not beingnotified, the Republic was not able to file its
opposition to the applicationand, naturally, it was not able to file an
appeal either.
2. Even if the Bacases successfully allege the names and addresses of
allpersons concerned who may have any rights or interests in the
property, theGovernment can still question the registration of Bacas
land on the groundthat the same is still under the public domain
pursuant to Pres. Proc. No.265. As such the subject land is inalienable
and cannot be registered.

Further,
prescription
or
estoppel
cannot
lie
against
the
governmentbecause it is a well-settled rule in our jurisdiction that the
Republic or itsgovernment is usually not estopped by mistake or error
on the part of itsofficials or agents.

5(May Ann)

6(Rhea B.)
G.R.

Nos.

85991-94

July

3,

1991

REPUBLIC CEMENT CORPORATION, petitioner, vs. COURT OF APPEALS,


MOISES CORREA and REGISTER OF DEEDS OF BULACAN, respondents.
FACTS:
Republic Cement Corp. filed a petition in the CFI of Bulacan for the
registration in its name a parcel of land which they bought form
persons who had occupied and cultivated it continuously since the
Spanish regime. It was opposed by Rayo, Mangahas and Legaspi
stating that they are the owners of the eastern and east central portion
which is sought to be registered by petitioner, having been in actual,
open, public, adverse, peaceful and uninterrupted possession and
occupation thereof in the concept of owner for a period of over 60
years and having acquired ownership thereof by donation on the
occasion of their marriage from the parents of Jose Rayo. They were
later substituted by Moises Correa as subsequent purchaser of said
parcel of land. The court ordered the registration of the 3 parcels of
land in the name of Correa. Republic Cement Corp. contends that
respondents failed to prove the genuineness of their title.
ISSUE: whether or not respondent established the identity of and title
to
the
land
sought
to
be
registered
in
his
name?
HELD:
Contrary to the claim of petitioner, private respondent, through his
predecessors in interest, was able to establish the identity of and title
to the land sought to be registered in his name. The technical
description and the survey plan duly approved by the Director of Lands

submitted

in

evidence

by

private respondent fully describes the metes and bounds of the parcels
of
land
involved.
Survey plan of the property showing its boundaries and total are
clearly identifies and delineates the extent of the land.
Based on said transfers, petitioner is now seeking the registration of
the whole of Lot No. 2880 in its name. This we cannot allow. The deeds
of sale relied upon by petitioner do not constitute sufficient legal
justification for petitioner's claim over all of Lot No. 2880. Petitioner's
title over said lot, as the successor in interest of said heirs, is limited
only to whatever rights the latter may have had therein. It is
elementary that a grantor can convey no greater estate than what he
has or in which he has an alienable title or interest.

7(Angel)

8(Janine)

9(Mike Jayson)

10(Lou)

11(Jess)

12(Diane)
FELIPA S. LARAGAN, INDEPENDENCIO SIBBALUCA, AURORA C.
SIBBALUCA, and ZENAIDA S. VALDEZ vs. HONORABLE COURT OF
APPEALS, TEODORO LEAO, TOMAS LEAO, FRANCISCO LEAO,
and CONSOLACION LEAO

G.R. No. L-47644 August 21, 1987

FACTS: Felipa Laragan, Independencio Sibbaluca, Aurora Sibbaluca,


and Zenaida Valdez filed an application with the CFI of Isabela for the
registration of their title over a parcel of land situated in the Barrio of
Sto. Tomas, Ilagan, Isabela. The applicants alleged that they acquired
said parcel of land by way of an absolute deed of sale from the spouses
Anastacio and Lucrecia Sibbaluca and that they have been in
possession thereof for more than 34 years. Solicitor General filed a
written opposition alleging that the applicants and their predecessor-ininterest do not have sufficient title to the parcel of land sought to be
registered. He prayed that the land be declared public land. On 2
August 1969, Teodoro Leano, Tomas Leano, Vicente Leano, Francisco
Leano, and Consolacion Leano filed their opposition to the application
for registration. They claimed that they are the owners of the southern
part of the land and that they have been in possession for more than
30 years.
The trial court rendered judgment in favor of Laragan. CA affirmed the
decision but declared the southern part as public land.

ISSUE: Whether petitioners can register the subject land.

HELD: No. The application for registration, filed with the lower court,
was for the confirmation of an imperfect title. The law applicable is
Section 48 (b) of the Public Land Act.
It is an established rule that an applicant for registration is not
necessarily entitled to have the land registered in his name simply
because no one appears to oppose his title and to oppose the
registration of the land. He must show, even in the absence of
opposition, to the satisfaction of the court, that he is the absolute
owner, in fee simple.
It would appear, however, that the possession and occupation of the
land by the spouses Anastacio and Lucresia Sibbaluca are tainted with
bad faith so that the petitioners are not entitled to the benefits of the
provisions of Section 48 (b) of the Public Land Law.

NOTES:
Sec. 48. The following described citizens of the Philippines, occupying
lands of the public domain or claiming to own any such lands or an
interest therein, but whose titles have not been perfected or
completed, may apply to the Court of First Instance of the province
where the land is located for confirmation of their claims and the
issuance of a certificate of title therefor, under the Land Registration
Act, to wit:
(b) Those who by themselves or through their predecessors-ininterest have been in open, continuous, exclusive, and notorious
possession and occupation of agricultural lands of the public domain,
under a bona fide claim of acquisition of ownership, for at least thirty
years immediately preceding the filing of the application for
confirmation of title except when prevented by way or force majeure.
These shall be conclusively presumed to have performed all the
conditions essential to a Government grant and shall be entitled to a
certificate of title under the provisions of this chapter.

13(April)

14(Daniel Eblahan)
Director of Lands v. CAGR no. L-50260, July 29, 1992Romero, J.:Facts:
In 1914, Estanislao Patricio was a known to possess a 256 hectare tract
of land within Timberland Block A in Masbate. He used it as pasture
land. This land was then sold to Vicente Montenegro, along with 100
cattle and their offspring, corrals and enclosures. Montenegro used the
land the same way Patricio did. He sold the land to Maria Calanog in
1938 and used it the same way, rearing up to 500 animals by 1941. On
January 20, 1956, Calanog then applied to register the land but she
was opposed by the Director of Lands and the Director of Forestry. The
Directors alleged that neither Calanog nor her predecessors-in-interest
had sufficient title over the land and that it is public domain. The court
issued a general default. Thirty-seven other people petitioned to lift the
general default, claiming two-thirds of the land, among them are two
people issued with certificates of title. They also opposed the
application of title by Calanog. The trial court granted the application,
excluding the two other lots with certificates of title. The Court of
Appeals affirmed the decision. Issue: Whether or not there the
respondent has a valid claim to the landHeld: No. The Court ruled that

respondent can claim neither the 256 hectare land disputed by the
public petitioners nor the remaining land claimed by the thirty-seven
oppositors. In order to register a land of public domain under the Public
Land Act before its amendment, it required adverse possession from
July 26, 1894 and must have open, notorious, and uninterrupted
possession of the land and it must be proven by the applicant. In this
case, the Court found no adequate proof for the respondents claim. It
found out that in the deed of sale in 1928, only 200 hectares of the
land was sold, very different to the 598 hectare land sought to be
registered. No convincing proof was given by the respondent on the
matter. The respondent also did not have proof of the required
possession under the Public Land Act, the earliest evidence of
possession by her predecessor-in-interest was only in 1914. On the
other hand, the Court found that both the public respondents and the
oppositors have evidence to support their claims on the land. The
public respondents had satisfactory proof that the 256 hectares of the
land are timberland as evidenced by documents such as reports and
sketch plans. The oppositors also had proof that they had been
occupying and possessing the 342 hectares of the land as early as
1954 as evidenced by homestead applications, free patents, and
transfer certificates of title.
15(Melodia)
________________________________________________________________________
________________________
16(Mel)

Bureau of Forestry, Bureau of Lands and Philippine Fisheries


Commission, petitioners
vs.
Court of Appeals and Filomeno Gallo, respondents
G.R. # L-37995 August 1987

FACTS:
On July 11, 196,1 four (4) parcels of lands situated in
Buenavista, Iloilo described in Plan PSU-150727 containing an

approximate area of 30.5943 hectares were the subject


application for registration by Mercesdes Diago who alleged
others that she herself occupied said parcel of lands having
them from the estate of late Jose Ma. Nava who in his lifetime
the lands in turn from Canuto Gustillo on June 21, 1934.

of an
among
bought
bought

On June 30, 1965, respondent Filomeno Gallo having


purchased the subject parcels of land from Mercedes Diago on April 27,
1965, move to be substituted in place of the latter attaching to his
motion an amended application for registration of title. Also on August
30, 1965, petitioner Philippine Fisheries Commission also moved to be
substituted in place of Bureau of Forestry as oppositor over portion of
the land sought to be registered.
On April 6, 1966, the trial court rendered its decision
ordering the registration of the four (4) parcels of land in the name of
respondent Filomeno Gallo. Petitioners appealed from said decision to
the respondent court of appeals which in turn affirmed the Regional
court decision.

ISSUE :
Whether or not the court has the power to classify or reclassify public
lands into alienable and disposable lands.

HELD :
No, The supreme court ruled that the classification or reclassification of
public lands into alienable or disposable, mineral or forest land is now
a prerogative of the executive department of the government and not
of the courts. With these rules, there should be no more room for doubt
that is not the court which determines the classification of lands of
public domain into agricultural, forest or mineral but the executive
branch of government through the office of the president.
________________________________________________________________________
__________________________

17(Rocky)

18(Maribeth)

REPUBLIC OF THE PHILIPPINES vs CAet. al., 99 SCRA 742


FACTS:
On May 16, 1966, PerpetuoAlpuerto, filed with the CFI of Quezon
(Branch II), a motion to reopenCadastral Case No. 97, LCR Cad. Rec.
No. 1555, and to admit his answer over LotNo. 7718 of the Mauban
(Quezon) cadastre. After trial, the lower court renderedits decision
adjudicating to said Alpuerto Lot No. 7718 together with
itsimprovements, and ordered the issuance in his favor of the
corresponding decreeof registration, thus an order for the issuance of a
decree
of
registrationover
the
said
lot.
The
Land
RegistrationCommission then issued Decree No. 127177 as the basis of
the issuance of OCTNo. 0-13541 in his favor. Portions of the lot were
subsequently transferred tovarious persons who were issued their
respective transfer certificates oftitle, among whom are private
respondents.
On April 6, 1971, the Sol.Gen. filed for the government a complaint for
annulment, cancellation of titlesand for reversion of Lot No. 7718 of
Cadastral Survey of Mauban, Quezon to theState on the ground that
the decision of the court adjudicating Lot No. 7718 toAlpuerto, its order
for the issuance of the decree of registration as well asOCT No. 013541
and all its derivative titles are all null and void and withoutlegal effect
because the court had no jurisdiction to allocate the subjectland, which
is inalienable.
Herein Respondents IndustrialMarketing and Investments Corporation,
Henry O. Antonio Angeles and AureaAngeles filed a motion to dismiss
the complaint alleging that the action isbarred by a prior judgment and
that the court lacks jurisdiction over thenature of the action or suit.
Defendant, Andres Laredo, likewise filed a motionfor dismissal of the
complaint, based on the grounds that the complaint statesno cause of
action and that venue is improperly laid.

After hearing the motionsfor dismissal and the opposition thereto, the
lower court issued an orderdenying the motion to dismiss filed by
Respondent Laredo, but granting themotion to dismiss filed by
Respondents Industrial Marketing and others anddismissing the
complaint filed by petitioner. The motion for reconsideration was
further denied. Consequently,petitioner appealed the case to the CA
which affirmed the order of dismissal bythe lower court. The motion for
reconsideration was likewise denied, hence,this petition.
ISSUE: Whether or not thecourt had jurisdiction to allocatethe subject
land, which is inalienable?
HELD: No. It was held that where the land covered by the homestead
application ofpetitioner was still within the forest zone or under the
jurisdiction of the Bureauof Forestry, the Director of Lands had no
jurisdiction to dispose of said landunder the provisions of the Public
Land Law, and the petitioner acquired noright to the land. It follows
that "if a person obtains a title under thePublic Land Act which
includes, by oversight, lands which cannot be registeredunder the
Torrens system, or when the Director of Lands did not havejurisdiction
over the same because it is a public forest, the grantee does not,by
virtue of the said certificate of title alone, become the owner of the
landillegally included". The title thus issued is void at law, since
theofficer who issued it had no authority to do so.

19(Ailyn)

20(Jen)

IHVCP vs. UP 200 SCRA 554, International Hardwood and


Veneer Company of the Philippines, vs. University of the
Philippines, and Jose Campos, Jr. G.R. No. L-52518 August 13,
1991Facts: IHVCP is a company engaged in the manufacture,
processing and exportation of plywood. It renewed its timber license,
which was granted by the government and shall be valid for 25 years,
in early 1960. Said license authorizes the company to cut, collect and

remove timber from the portion of timber land located in certain


Municipalities of Laguna, including Paete.In 1964, the Congress
enacted R.A. 3990, an act establishing an experiment station for UP.
The said experiment station covers a portion of timberland in Paete,
occupied by IHVCP. So UP, who claims ownership of said portion of
timberland, demanded the latter to pay the forest charges to it, instead
of the BIR. IHVCP rejected the demand and it filed a suit against UP,
claiming that R.A. 3990 does not empower UP to scale, measure and
seal the timber cut by it within the tract of land referred to in said Act,
and collect the corresponding forest charges prescribed by the
BIR.Issue: Whether or not UP is the owner of the portion of timberland
in Paete.Held: Yes. The court ruled that R.A. 3990 ceded and
transferred in full ownership to UP the area, which means that the
Republic of the Philippines completely recovered it from the public
domain. In respect o the areas covered by the timber license of IHVCP,
the said Act removed and segregated it from being a public forest.The
court further cited Sec. 3 of R.A. 3990, which provides that, any
incidental receipts or income therefrom shall pertain to the general
fund of the University of the Philippines. The provision of the Act is
clear that UP, being the owner of the said land, has the right to collect
forest charges and to supervise the operations of IHVCP insofar as the
property of the UP within it is concerned.

22(Nelson)

G.R. No. 83609 October 26, 1989


petitioner, vs. COURT OF APPEALS, IBARRA BISNAR and AMELIA
BISNAR, respondents.
Facts:
Ibarra Bisnar and Amelia Bisnar applied for the registration of
their two titles to two parcels of land, respectively containing 24
hectares and 34 hectares of land in barrio Gen. Hizon, Mun. of
President Roxas, province of Capiz. The CA granted their application for
confirmation and registration of the titles. The parcels of land in
question here are forest lands but their predecessors-in-interest
possessed the land for more than 50 years and in an open, continuous,
exclusive and notorious possession over the subject land.
Issue:

Whether or not the CA erred in granting the application of the


claimant.
Held:
Yes. It bears emphasizing that a positive act of the government
is needed to declassify land which is classified as forest and to convert
it into alienable or disposable land for agricultural or other purposes.
Unless and until the land classified as forest is released in an official
proclamation to that effect so that it may form part of the disposable
agricultural lands of the public domain, the rules on confirmation of
imperfect title do not apply.
Thus, possession of forest lands, however long, cannot ripen into
private ownership. A parcel of forest land is within the exclusive
jurisdiction of the Bureau of Forestry and beyond the power and
jurisdiction of the cadastral court to register under the Torrens System.

23(JM)

HEIRS OF JOSE AMUNATEGUI, PETITIONERS, VS. DIRECTOROF


FORESTRY, RESPONDENT.
G.R. No.L-27873,
November 29, 1983
FACTS:

Roque Borre and Melquiades Borre filedan application for a


registration of a certain land located in Pilar, Capiz. Indue time, the
heirs of Amunategui filed an opposition to the application of theBorres
and prayed that a portion of the lot be conveyed to them.
Anotheroppositor, Emeterio Bereber, filed his claim over another
portion of the lot.Subsequently, the director of Forestry also filed an
opposition regarding allthe claims of the land claiming that the land

was a forest land. During thetrial Roque borre sold his rights to Angel
alpasan who later on filed his claimon the said land. The CFI of Capiz
ruled on the issue and favored Bereber,Alpasan and Melquiades Borre
of their claims. Thereafter, the heirs ofamunategui and the Director of
Forestry appealed the case to the CA. the CAissued a ruling denying all
claims on the land and seconded the stand of theDirector of Forestry
that the land is a forest land therefore it can not beowned privately.
The heirs of Amunategui appealed the case to the SupremeCourt. They
claim that the land is not a forest land since it is already swampyand
mangroves have grown on the area. Further, there are no trees
classified insection 1820 of the Revised Administrative Code which
would make a land aforest land. In addition they claim that the land is
more productive as anagricultural land than as a forest land. They
further contend that they havebeen in possession of said land for many
years however such claim was belliedby the Director of Forestry
through his evidence that even Jose Amunategui evenapplied for
permit to cut timber on said land.

ISSUE:

Can a land classified as forest land,be subject to private


ownership by the fact that it was used as an agriculturalland and that
its actual state is no longer a forested area?

HELD:

NO, a land classified as a forest landcan not be owned by private


individuals by virtue of its actual state. A forestland is defined by the
law even if it is stripped of its forest cover. It cannot be said that by
virtue of the cultivation of private persons using suchland as an
agricultural land changes its classification as a forest land.
Classificationof lands is left to the executive department of the
government. Devoid of anygovernment act, the land remains as a
forest land thereby not subject toprivate ownership. The contention of
the petitioners with regards to theprovision of Section 48,
Commonwealth Act No. 141, as amended by Republic ActNo. 1942 is
untenable since such act only apply to agricultural lands and notto

forest lands. In the case at bar the land in question had never ceased
to bea forest land.

24(Maris)

25(Reg)

26
G.R. No. L-50464

January 29, 1990

SUNBEAM
CONVENIENCE
FOODS
INC.,
CORAL
BEACH
DEVELOPMENT CORP., and the REGISTER OF DEEDS OF BATAAN,
petitioners, vs. HON. COURT OF APPEALS and THE REPUBLIC OF
THE PHILIPPINES, respondents.

FACTS:
On April 29, 1963, a Sales Patent issued by the Director of Lands in
favor of Sunbeam for parcels of land situated in Mariveles, Bataan. It
was registered to the Register of Deeds who in turn issued Original
Certificate of Title to Sunbeam and Transfer Certificate of Title in favor
of Coral Beach. The SolGen instituted an action for the reversal of the
Civil Case No. 4062 favoring Sunbeam and Coral Beach claims with the
CFI of Bataan.
Sunbeam and Coral Beach moved for dismissal and was approved. CA
reversed the CFI decision and decides to give due course to the
petition of the SolGen.

ISSUE:
Whether the classification of the subject lots are forest lands.
Whether the title conferred by the Director of Lands is valid for forest
lands.

HELD:
Our adherence to the Regalian doctrine subjects all agricultural,
timber, and mineral lands to the dominion of the State. Thus, before
any land may be declassified from the forest group and converted into
alienable or disposable land for agricultural or other purposes, there
must be a positive act from the government. Even rules on the
confirmation of imperfect titles do not apply unless and until the land
classified as forest land is released in an official proclamation to that
effect so that it may form part of the disposable agricultural lands of
the public domain.
The mere fact that a title was issued by the Director of Lands does not
confer any validity on such title if the property covered by the title or
patent is part of the public forest.
Petition denied.

27(Ed)

28(LJ)

29(Zax)

30(Clathem)

REPUBLIC OF THE PHILIPPINES, DIRECTOR OF LANDS


andDIRECTOR
OF
FORESTRY,
petitioners
vs.
HON. COURT OF APPEALS, PAULINA PARAN, ELISAPARAN
MAITIM and SINA PARAN, respondents. G.R. No. 48327 August
21, 1991

Facts:Private respondents are applicants for registration ofa parcel of


land situated in Beckel La Trinidad,Benguet, covered by Survey Plan
Psu-105218. In their application dated 13 February1970, private
respondents claim to have acquired the land from their fatherDayotao
Paran and by actual, physical, exclusive and open possession
thereofsince time immemorial. Private respondents do notdispute that
the land was formerly a part of the Central Cordillera ForestReserve,
they contend that it had already been released therefrom.
Privaterespondents submitted a certification 17 signed by Leopoldo
Palacay,Chief of Land Classification Party No. 57 of the Bureau of Forest
Development,Department of Agriculture and Natural Resources,
showing the legal nature orstatus of the land as alienable or
disposable.
On 18 November 1970, the Office of theSolicitor General filed on behalf
of the Director of Lands anOpposition contending that: (1) private
respondents have no registrabletitle; (2) the parcel of land sought to
be registered is part of the publicdomain belonging to the Republic of
the Philippines; and (3) the applicationfor registration was filed after
expiration of the period provided for in R.A.No. 2061, hence the land
registration court did not acquire jurisdiction overthe case.
The Office of the Provincial Fiscal ofBaguio and Benguet, on the other
hand, filed a Motion to Dismiss. On 12November 1970, the land
registration court issued anOrder 5 declaring a general default against
the whole world exceptthe Bureau of Lands, the Reforestration
Administration, and the Bureau ofForestry. Another Order 6 was then
issued denying the motion todismiss filed by the Provincial Fiscal.

Issue: Whether or not the certificationsigned by the chief of Bureau of


Forest Development is an adequate proof toreclassify the land.

Held: No. while the Court considered the reports of the District
Forester and theDistrict Land Officer as adequate proof that the land
applied for was no longerclassified as forestal. The difficulty in the
instant case is that while thecertification of Leopoldo Palacay on which
private respondents rely may,standing alone, be evidence that a
reclassification had occurred, it iscontradicted by an official report of
Luis Baker. Moreover, the privaterespondents' application for
registration was in fact opposed by the Directorof Lands as well as the
Director of Forestry.
Furthermore, the evidence of record thusappears unsatisfactory and
insufficient to show clearly and positively that theland here involved
had been officially released from the Central CordilleraForest Reserve
to form part of the alienable and disposable lands of the publicdomain.
Once a parcel of land is shown to have been included within a
ForestReservation duly established by Executive Proclamation, as in
the instant case,a presumption arises that the parcel of land continues
to be part of suchReservation until clear and convincing evidence of
subsequent withdrawaltherefrom or de-classification is shown. A
simple, unsworn statement of a minorfunctionary of the Bureau of
Forest Development is not, by itself, suchevidence.

31(Lea)

G.R. No. L-46048 November 29, 1988


REPUBLIC vs. CA

FACTS:
Spouses Jose Zurbito and Soledad Zurbito purchasedsmall parcels of
land from various owners adjoining the 30 hectares of land located in
Calulod Pauwa, Masbate inherited by Jose Zurbito from his parents. The
land thus acquired by the spouses comprises the two parcels of land).
The spouses occupied and cultivated these properties.Upon the death
of Jose Zurbito, his wife Soledad Buencamino Zurbito and his children
succeeded to his estate and continued his possession. The estate of
Jose Zurbito was placed under administratix thereof.

Soledad vda. de Zurbito eventually sold her rights, title, interest and
participation in the parcels of land subject of this registration
proceedings in favor of the herein applicant and her son-in-law, Dr.
Miguel Marcelo under a deed of absolute sale for and in consideration
of the sum of P10,000.00. Then Soledad vda. de Zurbito, as
administratix of the estate of the deceased Jose Zurbito and in behalf
of her children co-signed with her other children a deed of sale with a
right to repurchase wherein they sold to Miguel Marcelo for and in
consideration of the sum of P12,000.00 the remaining one-half
undivided portion of the properties mentioned in the previous deed of
sale.

Under the provisions of the aforementioned deed, the vendors were


given the option to repurchase the properties within five years from the
date thereof (October 24, 1944) extendible for another period of one
year at the option of vendors and that upon failure of the vendors to
redeem the properties, within the alloted time, title shall automatically
vest in the vendee, Dr. Miguel Marcelo. When the vendors-a-retro failed
to repurchase the properties within the period agreed upon, Dr.
Marcelo consolidated ownership of the properties by executing and
registering in 1954 an affidavit of consolidation dated December
2,1953 in the office of the Register of Deeds of Masbate. However, he
allowed his mother-in-law, Soledad vda. de Zurbito, to continue
managing the properties. This arrangement was made in consideration
of the personal relationship between the applicant and Soledad vda. de
Zurbito.

The Opposition of the Director of Forestry is centered on the claim that


approximately 22 hectares of the land is within the forest zone. The
authentic document evidencing the classification of the land applied
for registration as a forest zone has been presented. CA confirmed and
ordered the registration of the title over the property in the name of
Miguel Marcelo, married to Celia Zurbito, Filipino, of legal age and
residing at 687 General Geronimo, Sampaloc, Manila.The Director of
Forestry (now Director of Forest Development) and the Heirs of Jose
Zurbito, as oppositors, appealed the aforequoted decision to
respondent Court of Appeals.CA rendered judgment declaring the
applicant and the private oppositors, now the private respondents
herein, as co-owners, in stated shares, of the entire property involved

while denying a motion for reconsideration filed by oppositor-appellant


Director of Forestry.

ISSUE:Whether the 22 hectares area which forms part of the land


applied for registration by and decreed in favor of herein private
respondents is disposable agricultural land.

RULING:
Yes.

Forest reserves of public land can be established as provided by law.


When the claim of the citizen and the claim of the Government as to a
particular piece of property collide, if the Government desires to
demonstrate that the land is in reality a forest, the Director of Forestry
should submit to the court convincing proof that the land is not more
valuable for agricultural than for forest purposes. Great consideration,
it may be stated, should, and undoubtedly will be, paid by the courts to
the opinion of the technical expert who speaks with authority on
forestry matters. But a mere formal opposition on the part of the
Attorney-General for the Director of Forestry, unsupported by
satisfactory evidence, will not stop the courts from giving title to the
claimant.

It is not disputed that the aforesaid Land Classification Project No. 3,


classifying the 22-hectare area as timberland, was certified by the
Director of Lands only on December 22, 1924, whereas the possession
thereof by private respondents and their predecessor-in-interest
commenced as early as 1909. While the Government has the right to
classify portions of Public land, the primary right of a private individual
who Possessed and cultivated the land in good faith much prior to such
classification must be recognized and should not be prejudiced by
after-events which could not have been anticipated.

Thus, We have held that the Government, in the first instance may, by
reservation, decide for itself what portions of public land shall be

considered forestry land, unless private interests have intervened


before such reservation is made.
Part VI

1. REPUBLIC v. CA (135 SCRA 156)


135 SCRA 156 January 31, 1985

FACTS:
Spouses Maxino andTarciana Morales registered their parcel
of land located at Barrio Cambuga(Anonang), Mulanay with the Court
of First Instance at Gumaca, Quezon. On March21, 1961, Judge Vicente
del Rosario rendered a decision, ordering theregistration of said land.
The decision became final and executory. A decreeand an original
certificate of title were issued.
More than eightyears later, the Republic of the Philippines
filed an amended petition tonullify the decision, decree, and title on
the ground that the decision wasvoid because the land in question was
still a part of the unclassified publicforest. It is axiomatic that public
forestal land is not registerable. Itsinclusion in the public forest was
certified by Director of Forestry FlorencioTamesis . The certification was
reiterated by the Director of Forestry on May20, 1948.
On the other hand, the spouses contended thatthe former
declaration that the land is alienable estopped the Government
toannul the decision on the ground that the said land was discovered
to be apublic forest and thereafter cannot be registered.

ISSUE: Whether or not the mistake of the lower court in granting


theland title in favor of the spouses estopped the Government from

declaring theland not alienable and not susceptible of private


appropriation.

HELD:
NO, the Governmentshould not be estopped by the mistakes
or error of its agents.It isincontestable that the area registered by the
Maxinos, is within the publicforest and therefore not alienable and
disposable nor susceptible of privateappropriation. Possession of public
forestal lands, however long, cannot ripeninto private ownership

2(Charleen)

3(Gina)

Directorof Lands vs. Court of Appeals


G.R. No.

May 31,1984

Facts:
The landin question, Identified as Lot 2347, Cad-302 is situated in
Obando, Bulacan,and has an area of approximately 9.3 hectares. It
adjoins the Kailogan Riverand private respondents have converted it
into a fishpond.

In theirapplication for registration filed on May 10, 1976, private


respondents(Applicants, for brevity) claimed that they are the co-

owners in fee simple ofthe land applied for partly through inheritance
in 1918 and partly by purchaseon May 2, 1958; that it is not within any
forest zone or military reservation;and that the same is assessed for
taxation purposes in their names.

TheRepublic of the Philippines, represented by the Director of the


Bureau ofForest Development opposed the application on the principal
ground that theland applied for is within the unclassified region of
Obando, Bulacan, per BFMap LC No. 637 dated March 1, 1927; and that
areas within the unclassified regionare denominated as forest lands
and do not form part of the disposable andalienable portion of the
public domain.

Afterhearing, the Trial Court ordered registration of the subject land in


favor ofthe Applicants. This was affirmed on appeal by respondent
Appellate Court,which found that "through indubitable evidence
(Applicants) and theirpredecessors-in-interest have been in open,
public, continuous, peaceful andadverse possession of the subject
parcel of land under a bona fide claim ofownership for more than 30
years prior to the filing of the application"and are, therefore, entitled to
registration. It further opined that"since the subject property is entirely
devoted to fishpond purposes, itcannot be categorized as part of forest
lands. "

Issue:
Whetheror not applicants are entitled to judicial confirmation of title.

Held:
No. Ineffect, what the Courts a quo have done is to release the subject
property fromthe unclassified category, which is beyond their
competence and jurisdiction.The classification of public lands is an
exclusive prerogative of the ExecutiveDepartment of the Government
and not of the Courts. In the absence of suchclassification, the land
remains as unclassified land until it is releasedtherefrom and rendered
open to disposition. This should be so undertime-honored
Constitutional precepts. This is also in consonance with theRegalian

doctrine that all lands of the public domain belong to the State,
andthat the State is the source of any asserted right to ownership in
land andcharged with the conservation of such patrimony.

Sincethe subject property is still unclassified, whatever possession


Applicants mayhave had, and, however long, cannot ripen into private
ownership.

4(Zhon)

Republic vs. Francisco Bacus


G.R. No. 73261, August 11, 1989

Facts:
Respondent Bacus claims to be the owner of a parcel of land consisting
of 496 square meters and situated in Manga, Tinago, Ozamis City.
Although classified as forest land, the subject land has become highly
developed residential-commercial land.On September 14, 1981, Bacus
filed an application for its registration in his name with the Court of
First Instance of Misamis Occidental. The application was opposed by
the Republic of the Philippines through the Director of Lands on the
grounds that the applicant did not have title to the property nor was it
available for private appropriation since it was still part of the public
domain.
The registration court ruled in favor of the applicant [Bacus]. The
Republic appealed but CA affirmed in toto the appealed decision,
prompting the instant petition.

Issue: w/n the subject land has changed its status as forest land due to
its development into residential and commercial land

Held:
The subject land is still a forest land despite it being used as residential
and commercial lands. It is for the claimant to show that it has been
released for private appropriation, but this he has failed to do. No
evidence has been offered to prove that the lot had earlier been
declassified as forest land by the proper authority, to wit, the President
of the Philippines.

Nota bene:
1. Forest lands are not alienable as such and can be the subject of
private appropriation only when they are declassified and declared as
alienable. As long as they remain forest lands, no court has jurisdiction
to order their registration in the name of a private person.
2. Sec. 6 of Act No. 141 provides,The President, upon the
recommendation of the Secretary of Agriculture and Natural Resources,
shall from time to time classify the lands of the public domain into:(a)
Alienable or disposable, (b) Timber, and (c) Mineral lands,and may at
any time and in a like manner transfer such lands from one class to
another, for the purposes of their administration and disposition.
3. Sec. 1827 of Revised Administrative Code provides,Lands in public
forests, not including forest reserves, upon the certification of the
Director of Forestry that said lands are better adapted and more
valuable for agricultural than for forest purposes and not required by
the public interests to be kept under forest, shall be declared by the
Department Head to be agricultural lands.

5(May Ann)

6(Rhea B.)

REPUBLIC

vs

SAYO

Facts:
The respondent spouses filed an original application for registration of
a tract of land having an area of 33,950 hectares. Oppositions were
filed by the Government, through the Director of Lands and the
Director of Forestry, and some others. The case dragged on for about
twenty (20) years. The remaining area of 5,500 hectares was, under
the compromise agreement, adjudicated to and acknowledged as
owned by the Heirs of Casiano Sandoval, but out of this area, 1,500
hectares were assigned by the Casiano Heirs to their counsel, Jose C.
Reyes, in payment of his attorney's fees. In a decision rendered on
1981, the respondent Judge approved the compromise agreement and
confirmed the title and ownership of the parties in accordance with its
terms.
The Solicitor General contends that no evidence whatever was
adduced by the parties in support of their petitions for registration;
neither the Director of Lands nor the Director of Forest Development
had legal authority to enter into the compromise agreement; as
counsel of the Republic, he should have been but was not given notice
of the compromise agreement or otherwise accorded an
opportunity to take part therein; that he was not even served with
notice of the decision approving the compromise; it was the
Sangguniang Panlalawigan of Quirino Province that drew his attention
to the "patently erroneous decision" and requested him to take
immediate remedial measures to bring about its annulment.
The respondents contended that the Solicitor General's arguments are
premised on the proposition that the disputed land is public land, but it
is
not.
Issue:
Whether there was no evidence adduced by the parties in support of
their
petitions
for
registration
Held:
Yes. There was no competent evidence adduced by the parties in
support
of
their
petitions
for
registration.
The assent of the Directors of Lands and Forest Development to the
compromise agreement did not and could not supply the absence of

evidence

of

title

required

of

the

private

respondents

It thus appears that the decision of the Registration Court a quo is


based solely on the compromise agreement of the parties. But that
compromise agreement included private persons who had not adduced
any competent evidence of their ownership over the land subject of
the registration proceeding. Portions of the land in controversy were
assigned to persons or entities who had presented nothing whatever to
prove their ownership of any part of the land. What was done was to
consider the compromise agreement as proof of title of the parties
taking part therein, a totally unacceptable proposition. The result has
been the adjudication of lands of no little extension to persons who had
not submitted any substantiation at all of their pretensions to
ownership, founded on nothing but the agreement among themselves
that
they
had
rights
and
interests
over
the
land.
In the proceeding at bar, it appears that the principal document relied
upon and presented by the applicants for registration, to prove the
private character of the large tract of land subject of their application,
was a photocopy of a certification of the National Library. But, as this
Court has already had occasion to rule, that Spanish document cannot
be considered a title to property, it not being one of the grants made
during the Spanish regime, and obviously not constituting primary
evidence of ownership. It is an inefficacious document on which to
base any finding of the private character of the land in question.
It thus appears that the compromise agreement and the judgment
approving it must be, as they are hereby, declared null and void, and
set aside. Considerations of fairness however indicate the remand of
the case to the Registration Court so that the private parties may be
afforded an opportunity to establish by competent evidence their
respective
claims
to
the
property.
The decision of the respondent Judge complained of is annulled and set
aside.
7(Angel)

8(Janine)

9(Mike Jayson)

10(Lou)

11(Jess)

12(Diane)
BERNARDO CARABOT, JUAN BANQUILES, LEONIDA V. ENDIAPE,
LEON VILLANUEVA, OLIMPIA L. BANQUILES, FELISA BANQUILES,
SEBASTIAN VILLANUEVA AND JUAN BANDAYRIL vs. THE Hon.
COURT OF APPEALS, (FIRST DIVISION), SAMUEL PIMENTEL in
his own behalf and in behalf of the minors, ALEXANDER
MANUEL, JR., ALMABELLA and CLARIBEL all surnamed,
PIMENTEL, NATIVIDAD RIOFLORIDO, assisted by her husband
GREGORIO DINGLASAN
G.R. No. L-50622-23 November 10, 1986

FACTS: Samuel Pimentel and his children, as plaintiffs, alleged that


they are the pro-indiviso owners of a parcel of land in Bo. Tala, San
Narciso, Quezon. The Pimentels claimed that during the lifetime of
Estrella Ribargoso, she placed as tenants on portions of the land the
herein petitioners and that, after the death of Estrella, the Carabots
and the Villanuevas asserted interest adverse to the Pimentels. The
Pimentels prayed that the Carabots and the Villanuevas "be compelled
to disclose the facts on which they base their claims" and be declared
to have no title to or interest of any kind in the property.
Natividad Rioflorido assisted by her husband Gregorio Dinglasan,
claimed ownership of a "portion of the land in Bo. Tala, San Narciso,
Quezon. She alleged that the herein petitioners, "who have been hired
as tenants on the land have been illegally occupying for more than one
year now several portions of the said property, pretending or assuming
to be public land applicants." Ribargoso prayed that she be declared
the lawful owner of the property and that her possession over the
occupied portions be restored.
In both cases, the herein petitioners, as defendants, asserted that they
have never been tenants of the private respondents, that they

occupied the land as their homesteads and that they have already
applied for patents under the Public Land Law.
The trial court ruled that the homestead and free patent titles of the
petitioners are null and void. CA affirmed.

ISSUE: Whether or not the parcels of land occupied by the petitioners


and titled in their names as a result of homestead and/or free patents
were already private property.

HELD: No. The parcels of land are not private property. It appears,
then, that the titles of Dona Agripina Paguia were secured pursuant to
the provisions of the Royal Decree of December 26, 1884. There was
no proof regarding the validity of the Titulo de Composition con El
Estado, granting that the existence of such documents has been
proven by the certification of the Registrador de Titulos. SC has already
ruled that "In order that natural boundaries of land may be accepted
for the purpose of varying the extent of the land included in a deed of
conveyance the evidence as to such natural boundaries must be clear
and convincing. Petitioners failed to do so. In this regard the Supreme
Court has ruled that:
All lands that were not acquired from the Government, either by
purchase or by grant, belong to the public domain. An exception to the
rule would be any land that should have been in the possession of an
occupant and of his predecessors-in-interest since time immemorial;
for such possession would justify the presumption that the land had
never been part of the public domain or that it had been a private
property even before the Spanish conquest.
Furthermore, as previously stated, there is no sufficient proof to show
that what was described in the Spanish titles was precisely the same
land in dispute.

13(April)

14(Daniel Eblahan)
Tan v. RepublicGR no. 177797, December 4, 2008Chico-Nazario
J.:Facts: Spouses Tan acquired an unregistered parcel of land, a land
forming part of the public domain that became alienable on December
31, 1925, in Misamis Oriental from the Neris through a sale dated on
June 26, 1970. The spouses immediately possessed the property and
planted trees and fruit bearing plants. The spouses sought to register
the land and was allowed by the RTC. The Republic assailed the
decision in its appeal to the Court of Appeals and contended that the
Tans did not have an open, continuous and notorious possession of the
land because their predecessors-in-interest did not possess it before
June 12 1945, making the property public domain. The Court of
Appeals ordered the Tans to return the property to the Republic.Issue:
Whether or not the Tans have an open, continuous, exclusive, and
notorious possession of the subject property.Held: No, the Court
denied the petition because the spouses failed to satisfy the
requirements of the Public Land Act. The Court ruled that in the Public
Land Act, persons occupying or claiming to own public domain should
prove that they or their predecessors-in-interest have been in open,
continuous, exclusive, and notorious possession and occupation of
agricultural lands of the public domain since June 12, 1945. To prove
this, the possession and occupation of the land must have started from
June 12, 1945 or earlier. In this case, the spouses failed to prove said
requirement. Even if their predecessors-in-interest have possessed the
land for thirty years or more, the earliest evidence of possession was a
tax declaration from 1952, and there is no other evidence showing acts
of ownership exercised by their predecessors-in-interest. The mere
showing of possession for thirty years or more is not sufficient.
Moreover, tax declarations are not conclusive evidence of ownership in
the
absence
of
actual,
public,
and
adverse
possession.Significance:Proof of ownership: tax declarations are not
conclusive evidence of ownership.
15(Melodia)

16(Mel)

HEIRS OF MARIO
PHILIPPINES

MALABANAN

GR No. 179987April 29, 2009

vs.

REPUBLIC

OF

THE

Facts:

On February 20 1998, Mario Malabanan filed an application for land


registration before the RTC of Cavite, covering a parcel of land situated
in Silang Cavite, consisting of 71,324 sq. meters. Malabanan claimed
that he had purchased the property from Eduardo Velazco, and that he
and his predecessors-in-interest had been in open, notorious, and
continuous adverse and peaceful possession of the land for more than
30 years. Velazco testified that the property was originally belonged to
a twenty-two hectare property owned by his great-grandfather, Lino
Velazco. Lino had four sons Benedicto, Gregorio, Eduardo and
Estebanthe fourth being Aristedess grandfather. Upon Linos death,
his four sons inherited the property and divided it among themselves.
But by 1966, Estebans wife, Magdalena, had become the
administrator of all the properties inherited by the Velazco sons from
their father, Lino. After the death of Esteban and Magdalena, their son
Virgilio succeeded them in administering the properties, including Lot
9864-A, which originally belonged to his uncle, Eduardo Velazco. It was
this property that was sold by Eduardo Velazco to Malabanan. Among
the evidence presented by Malabanan during trial was a Certification
dated 11 June 2001, issued by the Community Environment & Natural
Resources Office, Department of Environment and Natural Resources
(CENRO-DENR), which stated that the subject property was verified to
be within the Alienable or Disposable land per Land Classification Map
No.3013 established under project no. 20-A and approved as such
under FAO 4-1656 on March 15, 1982. On December 3, 2002, the RTC
approved the application for registration.

The Republic interposed an appeal to the Court of Appeals, arguing


that Malabanan had failed to prove that the property belonged to the
alienable and disposable land of the public domain, and that the RTC
had erred in finding that he had been in possession of the property in
the manner and for the length of time required by law for confirmation
of imperfect title. On February 23, 2007, the Court of Appeals
reversed the RTC ruling and dismissed the application of Malabanan.

Issues:

1. In order that an alienable and disposable land of the public domain


may be registered under Section 14(1) of Presidential Decree No. 1529,
otherwise known as the Property Registration Decree, should the land
be classified as alienable and disposable as of June 12, 1945 or is it
sufficient that such classification occur at any time prior to the filing
of the applicant for registration provided that it is established that the
applicant has been in open, continuous,exclusive and notorious
possession of the land under a bona fide claim of ownership since
June12, 1945 or earlier?

2. For purposes of Section 14(2) of the Property Registration Decree,


may a parcel of land classified as alienable and disposable be
deemed private land and therefore susceptible to acquisition by
prescription in accordance with the Civil Code?

3. May a parcel of land established as agricultural in character either


because of its use or because its slope is below that of forest lands be
registrable under Section 14(2) of the Property Registration Decree in
relation to the provisions of the Civil Code on acquisitive prescription?

4. Are petitioners entitled to the registration of the subject land in their


names under Section14 (1) or Section 14(2) of the Property
Registration Decree or both?

Ruling:

The Petition is denied.

In connection with Section 14(1) of the Property Registration Decree,


Section 48 (b) of the Public Land Act recognizes and confirms that
those who by themselves or through their predecessors in interest
have been in open, continuous, exclusive, and notorious possession

and occupation of alienable and disposable lands of the public domain,


under a bona fide claim of acquisition of ownership, since June 12,
1945 have acquired ownership of, and registrable title to, such lands
based on the length and quality of their possession. (a) Since Section
48(b) merely requires possession since 12 June 1945 and does not
require that the lands should have been alienable and disposable
during the entire period of possession, the possessor is entitled to
secure judicial confirmation of his title thereto as soon as it is declared
alienable and disposable, subject to the timeframe imposed by Section
47 of the Public Land Act. (b) The right to register granted under
Section 48 (b) of the Public Land Act is further confirmed by Section 14
(1) of the Property Registration Decree.

In complying with Section 14 (2) of the Property Registration Decree,


consider that under the Civil Code, prescription is recognized as
a mode of acquiring ownership of patrimonial property. However, public
domain lands become only patrimonial property not only with a
declaration that these are alienable or disposable. There must also be
an express government manifestation that the property is already
patrimonial or no longer retained for public service or the development
of national wealth, under Article 422 of the Civil Code. And only
when the property has become patrimonial can the prescriptive period
for the acquisition of property of the public dominion begin to run.
Patrimonial property is private property of the government. The person
acquires ownership of patrimonial property by prescription under the
Civil Code is entitled to secure registration thereof under Section 14 (2)
of the Property Registration Decree.

17(Rocky)

18(Maribeth)

MUNICIPALITY OF SANTIAGO vs. CA, 120 SCRA 734

FACTS:
Lots Nos. 4976-A and 8000-A locatedin the Municipality of Santiago are
being claimed by respondent Maria Cauian,the surviving spouse of
Eulalio Bayaua, son of Liberato Bayaua, the allegedoriginal occupant
thereof. They claimthat they possessed said property in the concept of
owners since timeimmemorial through their predecessors-in-interest.
They base their claim ofownership on (1) a Spanish document entitled,
Estadisticade los Terrenos de propiedad particular existantes en este
pueblo de Carig,Provincia de la Isabela de Luzon Num. 6 del ano 1896"
on file with theDivision of Archives, Bureau of Public Libraries, wherein
the name of "DonLiberato Bayaua" was listed as "proprietario" over a
3hectare, 61 centare lot; (2) on Tax Declarations for the years 1921,
1938-1939,1945, 1957-1959 in the name of Eulalio Bayaua; and (3) on
tax receipts. 5
During the cadastral surveyof the Municipality of Santiago from
October 30, 1927 to March 23, 1932, andthe cadastral proceedings
that followed in 1939, private respondents and theirpredecessors-ininterest did not file any claim to Lot 8000-A, which includedthe bigger
portion of the market site.
Eulalio Bayaua filed a claimwith respect to Lot 4976 during the
cadastral survey, but did not file anAnswer during the cadastral
proceedings. The only ones who filed theircadastral Answers with
regards to Lot 4976 were the Municipality of Santiagoand one Antero
Catabas.
After the suspension of thehearing due to war, respondent heirs of
Eulalio Bayaua filed a Petition toAdmit their Answers to Lot No. 4976
and Lot No. 1-D, alleging that previousanswers had been filed by
Eulalio within the statutory period, but due toaccident, mistake or
excusable negligence, the same could not be found in theCourt
records. Petitioner opposed admission. The Court denied admission of
theAnswer. Private respondents did not move for the reconsideration of
said Ordernor did they take other remedial steps to have the Answer
admitted. The lower courtthen issued another Order declaring Lot No.
4976 public land subject, however, to whatever rightsthe Municipality
of Santiago, Province of Isabela, may have by virtue andpursuant to
Presidential Proclamation No. 131.
Private respondents did notmove for the reconsideration of the
foregoing Order, nor did they appealtherefrom.
Apparently, privaterespondents had reiterated their petition to admit
answer, for, on July 19,1968, another Order was issued in the cadastral

case denying admission on theground that Lot 4976 had been declared
public land in the Order of September17, 1963, which Order had
become final (Exhibit "6").
Private respondents appealedto the Court of Appeals which reversed
the judgment of the Trial Court anddeclared private respondents the
lawful owners and possessors of the disputedland through acquisitive
prescription. Hence, the present appeal by certiorariby the Municipality.
ISSUE: Whether or not theproof of possession adduced by private
respondents constitutes exclusive andnotorious possession under claim
of ownership?
HELD: No. Private respondents failed to submit convincingevidence of
actual, peaceful and adverse possession in the concept of owner ofthe
entire area in question until the time they were allegedly dispossessed
bythe Municipality sometime in 1951. As pointed out by the Trial Court,
privaterespondents merely showed a "sporadic feeble cultivation of
portionsthereof which does not amount to possession as owner". A
mere casual cultivation of portionsof the land by the claimant does not
constitute possession under claim ofownership. In that sense,
possession is not exclusive and notorious so as togive rise to a
presumptive grant from the State.

19(Ailyn)

20(Jen)

Municipality of Antipolo
December 26, 1984

vs.

Zapanta

G.R.

No.

L-65334

FACTS: On August 8, 1977, a single application for the registration of


two distinct parcels of land was filed by two distinct applicants before
the then Court of First Instance of Rizal, Branch XV, Makati (the
Registration Court, for short). One of the two applicants was Conrado
Eniceo. He had applied for registration under the Torrens system of a
parcel of land containing 258 square meters. The other applicant was
"Heirs of Joaquin Avendao", and the land they were applying for

registration was a parcel (hereinafter called the DISPUTED PROPERTY)


containing 9,826 square meters surveyed in the name of the
Municipality of Antipolo (ANTIPOLO, for short). Both parcels were
situated in the Municipality of Antipolo. The applications were approved
by the Registration Court on February 26, 1980. ANTIPOLO took steps
to interpose an appeal but because it failed to amend the Record on
Appeal, its appeal was disallowed.On May 22, 1981, ANTIPOLO filed a
complaint in Civil Case No. 41353, also of the Court of First Instance of
Rizal, Branch XIII, Pasig (the CASE BELOW, for short) against named
"Heirs of Joaquin Avendao", and their assignees (hereinafter called the
AVENDAO HEIRS) praying for nullification of the judgment rendered
by the Registration Court. The defendants, in their Answer, pleaded a
special defense of res judicata, After a preliminary hearing on the
mentioned special defense, the CASE BELOW was dismissed. ANTIPOLO
perfected an appeal to the then Court of Appeals.A notice to file Brief
was issued by the Appellate Court, which ANTIPOLO claimed it had not
received. Upon motion of the defendants-appellees to dismiss on the
ground that ANTIPOLO had not filed its Brief within the reglementary
period, the appeal was dismissed despite the fact that before the
dismissal, ANTIPOLO had submitted its Appellant's Brief.Issue:
Whether or not the registration court did not have jurisdiction over the
land subject of registration, it being property of the Municipality of
Antipolo, used long before the war as a public market and other public
purposes, and hence actually devoted to public use and service.Held:
The claim of the AVENDAO HEIRS that they merely tolerated
occupancy by ANTIPOLO which had borrowed the DISPUTED PROPERTY
from them, since they had been in possession, since as far back as
1916, erroneously presupposes ownership thereof since that time.
They forget that all lands are presumed to be public lands until the
contrary is established. The fact that the DISPUTED PROPERTY may
have been declared for taxation purposes in their names or of their
predecessors-in-interest as early as 1918 does not necessarily prove
ownership. They are merely indicia of a claim of ownership. ANTIPOLO
had also declared the DISPUTED PROPERTY as its own in Tax
Declarations Nos. 909, 993 and 454.Since the Land Registration Court
had no jurisdiction to entertain the application for registration of public
property of ANTIPOLO, its Decision adjudicating the DISPUTED
PROPERTY as of private ownership is null and void. It never attained
finality, and can be attacked at any time. It was not a bar to the action
brought by ANTIPOLO for its annulment by reason of res judicata. It
follows that the titles issued in favor of the AVENDAO HEIRS must also
be held to be null and void. They were issued by a Court with no
jurisdiction over the subject matter. Perforce, they must be ordered
cancelled.

22(Nelson) HINDI KO PO SURE KUNG TAMA UNG CASE


G.R. No. L-61462 July 31, 1984
REPUBLIC OF THE PHILIPPINES (Director of Lands), petitioner,
vs. COURT OF APPEALS and TEDITA INFANTE TAYAG, respondents.
Facts:
This is an application for registration of land with an area of more than
11 hectares located at Barrio Tambo, Buhi, Camarines Sur which was
opposed by the Director of Lands.
Tedita, who claimed the land on 1976, contended that the subject land
was possessed by her father Froilan Infante, who died in 1937. Tedita
knew the land but not aware of its history. The only witness of Tedita
was Abraham Morandarte, who testified that he was the overseer of
Teditas father and he supported the claim of Tedita. Furthermore, The
realty taxes of the subject land from 1946 up to 1976 were only paid in
1976 by Teditas brother-in-law.
Issue:
Whether or not the evidence presented by Tedita is enough to
grant the application for registration.
Held:
No. Under that law, the applicant must prove that he, by
himself and through his predecessors in interest, have been in the
open, continuous, exclusive and notorious possession and occupation
of public agricultural land, under abona fide claim of acquisition of
ownership, for at least thirty years immediately preceding the filing of
the application for confirmation of title except when prevented
by force majeure.
After a study of the records, we find that contention to be meritorious.
The testimonies of Mrs. Tayag (who does not know the boundary
owners and the area of the land) and Morandarte, her overseer since
1973, are not sufficient to prove the alleged thirty years' possession in
the concept of owner by the applicant, her sister, mother and father.
The taxes for 31 years, 1946 to 1976, were paid only in 1976, a few
months prior to the filing of the application.

The applicant failed to satisfy the requirements for judicial confirmation


of her alleged title (Maloles vs. Director of Lands, 25 Phil. 548). The
said land must be presumed to be still a part of the public domain (Oh
Cho vs. Director of Lands, 75 Phil. 890).

23(JM)

VICENTE C. REYES vs.FRANCISCO SIERRA et. al., 93 SCRA 472


(G.R.No. L-28658 October 18, 1979)

Facts:

Vicente Reyes filed an application for registration ofhis title to a parcel


of land in Rizal. He declared that he inherited the landfrom his father
Vicente Reyes Sr. who died in 1944. An opposition was filed
byFrancisco Sierra and Emilio Sierra. The land was originally owned by
BasiliaBeltrans parents. Basilia borrowed from Vicentes father and
secured the loanof P 100 with the piece of land in question. Since the
execution of thedocument, the father of Vicente began paying taxes up
to his death. Vicenteclaimed it was a contract of sale.
However, the court is of opinion that it is mortgagecontract. The
intention of the parties at the time of execution of the contractmust
prevail. Registering the property in his own name because of the
failureof the mortgagor to redeem the property would constitute
pactum commissorium.
Although the document is a contract of mortgage and not acontract of
sale, the trial court ruled that the petitioner acquired ownershipover
the property by prescription having been in constructive possession of
theland since 1926.

The trial court also ruled that applicant and his fatherscontinuous
payment of the realty taxes constitutes strong corroboratingevidence
of applicant's adverse possession.

Issue:
Whether payment of taxes is a sufficient evidence toprove entitlement
to register the property

Held:

No. In the case at bar, thepossession of applicant was not


adverse, nor continuous.
When possession is asserted to convert itself intoownership, a new
right is sought to be created, and the law becomes moreexacting and
requires positive proof of title. Applicant failed to presentsufficient
evidence to prove that he is entitled to register the property. Thetrial
court's finding that since applicant and his father had been
continuouslypaying the realty taxes, that fact "constitutes strong
corroboratingevidence of applicant's adverse possession," does not
carry much weight.Mere failure of the owner to pay the taxes does not
warrant a conclusion thatthere was abandonment of a right to the
property. The payment of taxes onproperty does not alone constitute
sufficient evidence of title.

24(Maris)

25(Reg)

26
G.R. Nos. 85991-94

July 3, 1991

REPUBLIC CEMENT CORPORATION, petitioner, vs. COURT OF


APPEALS, MOISES CORREA and REGISTER OF DEEDS OF
BULACAN, respondents.

FACTS:
Republic Cement Corporation filed a petition in the CFI Bulacan, Branch
V for the registration in its name of a parcel of land located in Bario
Minuyan, Norzagaray, Bulacan. Owners of the east central portion of
the subject land opposed the application. A counter application was
filed for the two oppositors. The three applications were tried jointly.
The CFI rendered in Land Registration Cases in favor of the oppositors,
ordering the registration of the claimed lots for Legaspi, spouses Rayo
and Mangahas. Mangahas was later substituted by Moises Correa. RCC
appealed the CFI decision. The CA upheld the decision of the trial
court.

ISSUES:
Whether Legaspi, spouses Rayo and Mangahas were able to prove the
genuineness of his title and the identity of the lands he claims for his
own in the manner and with the degree of evidence required by law.

HELD:
Petition lacks merit.
Contrary to the belief of RCC, Correa, through his predecessors-in
interest, was able to establish the identity of and title of the land
sought to be registered in his name. The technical description and the
survey plan duly approved by the Director of Lands submitted in
evidence by private respondent fully describes the metes and bounds
of the parcels of land involved. Survey plan of the property, showing its
boundaries and total area, clearly identifies and delineates the extent
of the land.

An applicant for registration of land, if he relies on a document


evidencing his title thereto, must prove not only the genuineness of
said title but also the identity of the land therein referred to. If he only
claims a portion of what is included in his title, he must clearly prove
that the property sought to be registered is included in that title.

27(Ed)

28(LJ)

29(Zax)

30
REPUBLIC OF THE PHILIPPINES, represented bythe MINDANAO
MEDICAL
CENTER,
petioner,
vs.
HON. COURT OF APPEALS and ALEJANDRO Y DE JESUS,
respondents. G.R. No.L-40912 September 30, 1976
Facts: On January 22, 1921, Eugenio de Jesus,the father of respondent
Alejandro de Jesus, applied with the Bureau of Landsfor Sales Patent
situated Davao City.
On January 23, 1934, the Bureau of Lands,through its Davao District
Land Officer, accepted sealed bids for the purchaseof the subject land.
The Director of Lands, however, annulled the auction salefor the
reason that the sales applicant, Eugenio de Jesus, failed toparticipate
in the bidding for non-service of notice on him of the scheduledbidding.
In lieu of that sale, another bidding was held on October 4, 1934.Sales
applicant Eugenio de Jesus was the lone bidder.

On September 7, 1936, President Manuel L.Quezon issued


Proclaimation No. 85 withdrawing Lot No. 1176-B-2 from sale
andsettlement and reserving the same for military purposes, under
theadministration of the Chief of Staff, Philippine Army.
On October 9 1956, President Magsaysayrevoked this Proclamation No.
328 and reserved the same Lot No. 1176-B-2 formedical center site
purposes under the administration of the Director ofHospital.
Whereupon, on December 6, 1969, petitionerMindanao Medical Center
applied for the Torrens registration of the12.8081-hectare Lot 1176-B-2
with the Court of First Instance of Davao. TheMedical Center claimed
"fee simple" title to the land on the strengthof proclamation No. 350
reserving the area for medical center site purposes.
The Court of appeals denied the petition ofMindanao Medical Center.

Issue: whether or not petitioner MindanaoMedical Center has


registerable title over a full 12.8081-hectare land byvirtue of an
executive proclamation in 1956 reserving the area for medicalcenter
site purposes.

Held: Yes.
Proclamation no. 350 is free of any legalinfirmity. It proceeds from the
recognized competence of the president toreserve by executive
proclamation alienable lands of the public domain for aspecific public
use or service.
Petitioner Mindanao Medical Center hasregisterable title over the whole
contested area of 12.8081 hectares,designated Lot No. 1176-B-2, and
not only on a portion thereof occupied by theMedical Center, its
nervous disease pavilion and their reasonableappurtenances. Such
land grant is constitutive of a "fee simple" tileor absolute title in favor
of petitioner Mindanao Medical Center. Thus, Section122 of the Act,
which governs the registration of grants or patents involvingpublic
lands, provides that "Whenever public lands in the PhilippineIslands
belonging to the Government of the Philippines arealienated,
granted,or conveyed to persons or to public or private corporations,

the same shall bebrought forthwith under the operation of this Act
[Land Registration Act, Act496] and shall become registered lands."

31(Lea)

DIRECTOR OF LANDS vs. IAC195 SCRA 38FACTS: The application


for the registration of the subject lot, which allegedly contained an
area of 1,036,172 square meters, was filed by Espartinez on May 17,
1972. He alleged therein that he acquired lot by purchase from Sotera
Llacer. He invoked Section 48 of Commonwealth Act No. 141, as
amended by Republic Act No. 1942, should the Land Registration Act
be not applicable.The jurisdictional requirements of publication of
notice of initial hearing and posting of such notices in conspicuous
places in the parcel of land involved and in the municipal building
having been complied with, and considering that only the Bureau of
Lands and the Bureau of Forestry represented by the fiscal had
appeared, the lower court issued an order of general default with the
exception of said government agencies. Thereafter, one Perpetua
Llarena appeared and, together with the fiscal, she was required to file
an opposition to the application.Inasmuch as both the fiscal and
Llarena failed to file their respective oppositions within the period set
by the court, on December 12, 1972, it commissioned the clerk of court
to receive evidence. On the same day, however, the Solicitor General
entered his appearance for the government and at the same time, filed
an opposition to the application for registration. He alleged therein that
neither Espartinez nor his predecessors-in-interest had sufficient title
to acquire ownership of the land the same not having been acquired by
means of any of the various types of title issued by the Spanish
government or any other recognized mode of acquisition of title over
realty under pertinent laws; that neither Espartinez nor his
predecessors-in-interest were in open, continuous, exclusive and
notorious possession of the land for at least thirty (30) years prior to
the filing of the application; that Espartinez may not avail of the
provisions of Section 48 of the Public Land Act for failure to fulfill the
requisites prescribed therein; and that the parcel of land involved is
part of the public domain and therefore, not subject to private
appropriation.ISSUE:Whether the subject parcel of land is alienable
and disposable. RULING:No.Espartinez' reliance on Section 48(b) of
Commonwealth Act No. 141 is misplaced. That law is premised on the
prior classification of the land involved as a disposable agricultural

land.Anyone who applies for confirmation of imperfect title under this


provision has, under the ruling in Heirs of Amunategui vs. Director of
Forestry, the burden of overcoming the presumption that the land
sought to be registered forms part of the public domain. Espartinez
having failed to present any proof that the land in question has been
classified as and forms part of the disposable public domain, whatever
possession he might have had, and however long, cannot ripen into
private and his failure to adduce clear and convincing evidence of his
claim over the land has given rise to the presumption that Lot 6783 is
still part of the public domain (Director of Lands vs. Heirs of Juana
Carolino, supra).PREMISES CONSIDERED, the land subject of the
application for registration and confirmation of imperfect title is hereby
DECLARED as part of the public domain.
Part VII

1. REPUBLIC V. CA
154 SCRA 476

Facts:

In 1968,Martina Carantes for and in behalf of the heirs of Salming


Piraso filed in theCFI of Baguio an application for the registration of the
land, which the latterclaimed to be in their possession and occupation
openly, continuously,exclusively, notoriously since 1915. The Director
of lands, through theSolicitor General and the Director of Forestry,
opposed the application on theground that the said portion land is
within the Central Cordillera ForestReserve as shown in the reports and
testimonies of the district foresters. TheCFI granted the application,
which was also affirmed by the CA. Thegovernments failure to show
that the disputed land is more valuable for forestpurposes is one of the
reasons for the CAs ruling. It also noted the failureto prove that trees
are thriving in the land.

Issue:

Whetherthe land in dispute is alienable and disposable.

Ruling:

No. TheCourt ruled that the petitioner clearly proved thru the reports
and testimoniesof the district foresters that the land applied for
registration is a part of aforestland. As to the claim of the applicants
that they have been in possessionof the land since 1915, the court
cited its decision in Director of Forestry v.Munoz (23 SCRA 1184),where
it stated that possession of forest lands, no matterhow long, cannot
ripen into private ownership.

In itsdecision, the Court also addressed the CAs ruling by citing its
decision inHeirs of Amunatequi v. Director of Forestry (126 SCRA 69,
75), where it ruled,A forested area classified as forest land of the
public domain does not losesuch classification simply because loggers
or settlers may have stripped it ofits forest cover.

Forestlandsdo not have to be on mountains or in out of the way places.


The classificationis descriptive of its legal nature or status and does not
have to bedescriptive of what the land actually looks like.

The Courtagain reiterated that there must first be a formal Government


declaration thatthe forestland has been re-classified into alienable and
disposableagricultural land, before private persons in accordance with
the various modesof acquiring public agricultural lands can acquire it.

2. GOMEZ VSCOURT OF APPEALS


168 SCRA503

FACTS:

Acourt ruling (Philippine Islands vs Abran) settled that 12 parcels of


landbelonged to one Consolacion Gomez. Consolacion later died and
the 12 parcels ofland were inherited by Gomez et al her heirs. The
heirs agreed to divide theproperty among them. After notice and
publication, and there being noopposition to the application, the trial
court issued an order of generaldefault. On 5 August 1981, the court
rendered its decision adjudicating thesubject lots in Gomez et als
favor. The decision became final and executoryhence the court
directed the Chief of the General Land Registration Office toissue the
corresponding decrees of registration over the lots adjudicated.
GLROChief Silverio Perez opposed the adjudication and petitioned for
its settingaside. He discovered that the 12 parcels of land were
formerly part of a titledland which was already granted by homestead
patent in 1929. Under the law, land already granted byhomestead
patent can no longer be the subject of another registration. Thelower
court granted Silverios recommendation. Gomez et al invoked Sec. 30
and32 of PD 1529 (Land Registration Act) which provides that after
judgment has become final andexecutory, the court shall forthwith
issue an order to the Commissioner of LandRegistration for the
issuance of the decree of registration and certificate oftitle. That once
the judgment becomes final and executory under Sec 30, thedecree of
registration must issue as a matter of course.

ISSUE:
Whetheror not to set aside the lower courts initial ruling on approving
theadjudication even after it had become final and executory.

HELD:
Yes.Unlike ordinary civil actions, the adjudication of land in a cadastral
or landregistration proceeding does not become final, in the sense
ofincontrovertibility until after the expiration of one (1) year after the
entryof the final decree of registration. TheSupreme Court has held
that as long as a final decree has not been entered bythe Land
Registration Commission (now NLTDRA) and the period of one (1)
yearhas not elapsed from date of entry of such decree, the title is not
finally adjudicatedand the decision in the registration proceeding
continues to be under thecontrol and sound discretion of the court
rendering it.

HENCE,the case may still be reopened and the decision set


aside when granted.

3. MENDOZA VS. CA
84 scra 76
Facts:
In 1964, it was proven that aparcel of land located in Sta. Maria,
Bulacan, is owned by Mendoza. Mendozaapplied for a title. During
pendency of the application before the landregistration court, Mendoza
sold the land to Daniel Cruz. The contract of salewas admitted in court
in lieu of the pending application for land title. Theregistration court
rendered a decision in July 1965, ordering the registrationof the two
parcels of land in the name of Cruz subject to the usufructuaryrights of
Mendoza.
The decision became final andexecutory. In 1968, however, uponfailure
of Cruz to pay Mendoza, Mendoza petitioned that the title issued in
thename of Cruz be cancelled. The land registration court ruled in favor
ofMendoza on the ground that the court erred in its earlier decision in
issuingthe land title to Cruz who was not a party to the application of
titleinitiated by Mendoza. Cruz appealed. The Court of Appeals ruled in
favor ofCruz.
Issue:

Whether the title can be dealt with in the name of athird party.

Ruling:

Yes. The Court ofAppeals ruling must be sustained. First of all, it was
proven that Mendozacaused the registration in the name of Cruz

pursuant to their contract of sale.Second, Mendoza overlooks Section


29 of the Land Registration Act whichexpressly authorizes the
registration of the land subject matter of aregistration proceeding in
the name of the buyer (Cruz) or of the person towhom the land has
been conveyed by an instrument executed during the intervalof time
between the filing of the application for registration and the issuanceof
the decree of title.
SEC. 29. After the filing of theapplication and before the issuance of
the decree of title by the Chief of theGeneral Land Registration Office,
the land therein described may be dealt withand instruments relating
thereto shall be recorded in the office of theregister of deeds at any
time before issuance of the decree of title, in thesame manner as if no
application had been made. The interested party may,however,
present such instruments to the Court of First Instance instead
ofpresenting them to the office of the register of deeds, together with a
motionthat the same be considered in relation with the application,
and the courtafter notice to the parties, shall order such land
registered subject to theencumbrance created by a said instruments,
or order the decree of registrationissued in the name of the buyer or of
the person to whom the property has beenconveyed by said
instruments. . . .
A stranger or a third party maybe dealt with in the land registration
proceedings. The only requirements ofthe law are: (1) that the
instrument be presented to the court by theinterested party together
with a motion that the same be considered in relationwith the
application; and (2) that prior notice be given to the parties to thecase.
And the peculiar facts and circumstances obtaining in this case show
thatthese requirements have been complied with in this case.

4. JULIA CARAGAY-LAYNO VS CA
26 DECEMBER 1984, 133 SCRA 718

Facts:

Mariano De Vera died in 1951. Hiswidow administered his property until


her death in 1966. De Veras nephew(Salvador Estrada) took over as
administrator of De Veras estate. Prior to thewidows death, she made
an inventory showing that De Veras property (locatedin Calasiao,
Pangasinan) measures 5417 sq. m (more or less). Estrada
howevernoticed that the Torrens title under De Vera indicated that his
propertymeasures 8752 sq. m. He learned that the discrepancy is the
3732 sq. m. beingoccupied by Juliana. Estrada sued to evict Juliana.

Juliana averred that she and herfather have been in open, continuous,
exclusive and notorious possession and inthe concept of an owner of
the land since 1921; that theyve been paying taxes;that the title held
by Estrada was registered in 1947 but it only took them toinitiate an
action in 1967 therefore laches has set in.

Issue:

Whether the disputed portionshould be adjudged in favor of De Veras


estate.

Ruling:

No. The inclusion ofJulianas land in De Veras title was erroneously


done. It was shown thatJuliana, an unlettered woman, agreed to have
Mariano de Vera borrow her titlefor the purposes of Mariano obtaining a
loan during de Veras lifetime; thatwhen de Vera registered his portion
of land adjoined to that of Juliana, thelatters land was erroneously
included.
The error ishighlighted by the fact that de Veras widow, in her
inventory before she died,attested that de Veras portion of land is only
5417 sq. m. more or less. Thediscrepancy approximates the portion of
land actually being occupied byJuliana. By that, the only portion that
can be adjudged in favor of de Verasestate is that which was being
claimed by the widow (in her inventory). Arecalculation must however
be made to specify the exact measure of landbelonging to each: 3732

sq m should be retained by Juliana (portion which sheactually occupies)


and 5020 sq. m. should go to de Veras estate.

5. DIRECTOR OFLANDS VS THE COURT OF APPEALS


G.R. No. L-17696
Facts:
On November 19, 1926 a salesapplication was filed with the Bureau of
Lands by Benito Tolentino for a tractof public agricultural land with an
area of 5 hectares, situated in BarrioCallang Municipality of Gamu,
Province of Isabela. In accordance with theapplication, which was given
number 8706, the land was advertised for sale tothe highest bidder
and on February 15, 1928 was duly awarded to the applicantpursuant
to the provisions of Chapter V of the Public Land Act On January
19,1950, Tolentino having complied with the legal requirements as to
actualoccupancy, cultivation and improvement of the area applied for
as well as thepayment of the purchase price, the Director of Lands
signed the correspondingorder for the issuance of a patent in his favor.
Upon investigation, however,the applicant discovered that a portion of
the land covered by his applicationwith an area of 2.3506 hectares,
specifically that portion identified as LotNo. 8091 Pls-62, had been
applied for as a homestead by the herein defendantBraulio Cosme on
March 22, 1949 and that Homestead Patent No. V-19 had beenissued
to him by the Bureau of Lands on the following August 19, pursuant
towhich he obtained Original Certificate of Title No. P-880 from the
Office ofthe Register of Deeds for the Province of Isabela on November
10, 1949. Thehomestead settlement application had been filed with
the now defunct NationalLand Settlement administration pursuant to
Executive Proclamation No. 610promulgated in 1940, under which
certain areas of public agricultural land inIsabela were reserved for
settlement purposes, and it was upon recommendationof that office
that the patent was issued by the Director of Lands.
Upon protest by Benito Tolentinofiled with the Bureau of Lands, an
investigation was conducted by the DistrictLand Officer for Isabela, and
when it was verified that the land covered by thehomestead patent
was embraced within the area awarded to Tolentino in 1928,
theDirector of Lands filed the present action on November 27, 1953,
for thecancellation of the homestead patent and the original certificate
of titleissued to the defendant Braulio Cosme.

Their principal contention isthat after the certificate of title was issued
on November 10, 1949 by virtueof Homestead Patent No V-19 the land
in question came under the operation ofthe Land Registration Act as
provided in Section 122 thereof, and that upon theexpiration of one
year
from
the
date
of
its
issuance,
the
said
title
becameincontrovertible.
Issue:
Whether the court can invalidatethe lands covered by titles based on
homestead,free or sales patent.
Ruling:
No. Where a portion of a landsubject of a land registration case are
covered by titles based on homestead,free or sales patent, the court
cannot simply invalidate them.
A certificate of title based on a patent, even after the expiration ofone
year from the issuance thereof, isstill subject to certain conditions and
restriction.As a matter of fact, in appropriate cases and after
prioradministrative investigations by the Director of Lands,
properactions may be instituted by said official which may lead to the
cancellationof the patent and the title, and the consequent reversion of
the land to theGovernment.On the other hand certificate of title issued
pursuant to Act2259, after the lapse of one (1) year, becomes
incontrovertible.The inescapableconclusion, therefore, is that, while
with the due registration and issuance ofa certificate of title over a
land acquired pursuant to the Public Land Law,said property becomes
registered in contemplation of Act 496,in view of itsnature and manner
of acquisition, such certificate of title, when in conflictwith one
obtained on the same date through judicial proceedings, must give
wayto the latter.7
A certificate of title issuedpursuant to a homestead patent partakes of
the patent to a certificate issuedin a judicial proceeding, as long as the
land disposed of is really a partof the disposable land of the public
domain (El Hogar Filipino vs. Olviga, 60Phil. 22; Ramoso vs. Obligado,
70 Phil. 86 and others), and becomesindefeasible and incontrovertible
upon the expiration of one year from the dateof the issuance
thereof, ... a certificate of title, be it original or aduplicate, may only be
ordered cancelled under special circumstances, andone of them is
when the title is void. And a title will be consideredvoid if it is procured
through fraud, as when a person applies for theregistration of a land in
his name although he knows that the property belongsto another.

6. REPUBLIC OF THE PHILIPPINES VS. HON.SOFRONIO G. SAYO

G.R. No. L-60413

Facts:

Spouses Casiano Sandoval and Luz Marquez filed anoriginal application


for registration of 33,950 hectares tract of land but wasopposed by the
government including the Heirs of Liberato Bayaua . The land
wasformerly part of Santiago, Isabela, but had been transferred to
Nueva Vizcayain virtue of Republic Act No. 236.
Then, an order of general default was enteredagainst the whole world
except the oppositors.
After about 20 years, a compromise agreement wasentered into by the
parties. Under the compromise agreement, the Heirs ofCasiano
Sandoval renounced their claims and ceded
1) in favor ofthe Bureau of Lands, an area of 4,109 hectares;
2) in favor ofthe Bureau of Forest Development, 12,341 hectares;
3) in favor ofthe Heirs of Liberato Bayaua, 4,000 hectares; and
4) in favor ofPhilippine Cacao & Farm Products, Inc., 8,000 hectares.
The remaining area of 5,500 hectares was adjudicated to and
acknowledgedas owned by the Heirs of Sandoval, but out of this area,
1,500 hectares wereassigned to their counsel, Jose C. Reyes, in
payment of his attorney's fees.
The respondent Judge approved the compromise agreement and
confirmed thetitle and ownership of the parties in accordance with its
terms.
The applicants for registration presented a mere photocopy of
acertification of the National Library stating that the property in

question wasregistered under the Spanish system of land registration


as private property.But, that Spanish document cannotbe considered a
title to property, it not being one of the grants made duringthe Spanish
regime, and obviously not constituting primary evidence ofownership.

Issues:
Whether the Registration Court correctly rendereddecision based from
the
compromise agreement of the parties.
Whether informacion posesoria is a prima facieevidence of possession.

Ruling:
The compromise agreement and the judgmentapproving it is declared
null and void.
The decision of the Registration Court aquo is based solely on
thecompromise agreement of the parties but such included private
persons who hadnot adduced any competent evidence of their
ownership over the land subject ofthe registration proceeding. Portions
of the land in controversy were assignedto persons or entities who had
presented nothing whatever to prove their ownershipof any part of the
land. What was done was to consider the compromise agreementas
proof of title of the parties taking part therein, a totally
unacceptableproposition. The result has been the adjudication of lands
of no littleextension to persons who had not submitted any
substantiation at all of theirpretensions to ownership, founded on
nothing but the agreement among themselvesthat they had rights and
interests over the land.
Under the Spanish Mortgage Law, informacion posesoria was
considered a mode of acquiringtitle to public lands, subject to two
conditions: first, the inscriptionthereof in the Registry of Property, and
second, actual, public, adverse, anduninterrupted possession of the
land for 20 years; but where, as here, proof offulfillment of these
conditions is absent.

7. THE DIRECTOR OF LANDS et.al. VS HON. SALVADOR C.


REYES
G.R. No. L-27594

Facts:

On February 24, 1964,the applicant Alipio Alinsunurin, claiming


ownership in fee simple byinheritance from the late Maria Padilla,
sought the registration of title underAct 496, as amended, of a vast
tract of land, situated at the municipality ofLaur, province of Nueva
Ecija, admittedly inside the boundary of the militaryreservation of Fort
Magsaysay.
The Director of Lands,Director of Forestry, and the Armed Forces of the
Philippines opposed theapplication, claiming that the applicant was
without sufficient title and wasnot in open, exclusive, continuous and
notorious possession and occupation ofthe land in question for at least
thirty (30) years immediately preceding thefiling of the application;
that approximately 13,957 hectares of said landconsist of the military
reservation of Fort Magsaysay established underProclamation No. 237,
dated December 10, 1955 of the President.
The applicant AlipioAlinsunurin filed a motion for substitution of
parties, requesting that theParaaque Investment and Development
Corporation be considered as the applicantin his place, it having
acquired all his rights, interests, ownership anddominion over the
property subject matter of the application. The motion was granted by
the lowercourt.
It is beyond disputethat the land subject of the application is included
within the area reservedfor military purposes under Proclamation No.
237, dated December 19, 1955, ofthe President. The land is largely
uncultivated, mountainous and thicklyforested with a heavy growth of
timber of commercial quantities.

It is claimed by theapplicant that Melecio Padilla acquired the land by


virtue of a possessoryinformation title issued during the Spanish
regime on March 5, 1895, and uponhis death in 1900, he transmitted
the ownership and possession thereof to hisdaughter and sole heir,
Maria Padilla. The latter in turn continued tocultivate the land thru
tenants and utilized portions for pasture, until herdeath sometime in
1944.
On November 19, 1966,the lower court rendered decision holding that
the parcel of land applied foris adjudicated to and ordered to be
registered in favor of:
(a) Paraaque Investment and DevelopmentCorporation, a Philippine
corporation wholly owned by Filipino citizens, withaddress at Manila,
Philippines, two-thirds (2/3) portion, subject to the rightsof Ariosto
Santos and
(b) Roman C. Tamayo, Filipino citizen, married,resident of Cullit, Lallo,
Cagayan, one-third (1/3) portion of the saidproperty.

The oppositors Director of Lands, Director ofForestry and the Armed


Forces of the Philippines filed a Notice of Appeal fromthe said decision
to the Supreme Court.
By an order, the lower court required theProvincial Fiscal to file an
Amended Record on Appeal, so as to include thereincertain orders and
pleadings, within ten days from receipt of the order. OnMarch 16, 1967,
the Amended Record on Appeal was duly filed and copies servedupon
the appellees.
Pending the approval of the Record on Appeal, theapplicant Paraaque
Investment and Development Corporation filed a motion forthe
issuance of a decree of registration pending appeal. Likewise, Roman
C.Tamayo, thru counsel, filed a motion for the issuance of a decree
ofregistration. Both motions were opposed by the Government.
On March 11, 1967, the lower court, ruling thatits decision of
November 19, 1966 had become final as to the share of Roman
C.Tamayo, directed the issuance of a decree of registration of the
entire land,one-third (1/3) pro-indiviso in favor of Roman C. Tamayo,
andtwo-thirds (2/3) proindiviso in favor ofParaaque Investment and
Development Corporation, subject to the final outcomeof the appeal.

On March 14, 1967, the Commissioner of LandRegistration issued


Decree No. 113485 pursuant to the said order, and, on March15, 1967,
the Register of Deeds issued Original Certificate of Title No. 0-3151of
the Register of Deeds of the Province of Nueva Ecija.
On April 12, 1967, the lower court approved theAmended Record on
Appeal which, together with the evidence and transcripts,
wasforwarded to this Court in due course of appeal.
As the lower court denied reconsideration of theorder directing the
issuance of a decree of registration, the petitionersinstituted before the
Supreme Court a special civil action for certiorari andmandamus with
preliminary injunction, seeking to nullify the order dated March11,
1967, the decree of registration issued pursuant thereto and
OriginalCertificate of Title of the Register of Deeds for the province of
Nueva Ecija.

Issue:

Whether thelower court acted without jurisdiction or exceeded its


jurisdiction in orderingthe issuance of a decree of registration despite
the appeal timely taken fromthe entire decision a quo.

Ruling:

The appeal takenby the Government was from the entire decision,
which is not severable. Thus,the appeal affects the whole decision.
In any event, Werule that execution pending appeal is not applicable in
a land registrationproceeding. It is fraught with dangerous
consequences. Innocent purchasers maybe misled into purchasing real
properties upon reliance on a judgment which maybe reversed on
appeal.
A Torrens titleissued on the basis of a judgment that is not final is a
nullity, as it isviolative of the explicit provisions of the Land
Registration Act whichrequires that a decree shall be issued only after
the decision adjudicating thetitle becomes final and executory, and it is

on the basis of said decree thatthe Register of Deeds concerned issues


the corresponding certificate of title.
Consequently,the lower court acted without jurisdiction or exceeded its
jurisdiction inordering the issuance of a decree of registration despite
the appeal timelytaken from the entire decision a quo.

8. ATTY. JOSE S. GOMEZ et,al. VS HON. COURT OF APPEALS,

G.R. No. 77770

Facts:
Petitionersapplied for registration of several lots situated in
Bayambang, Pangasinan onAugust 30, 1968.The lots were among
those involved in the case of Governmentof the Philippine Islands vs.
Abran, whereinthe Supreme Court declared Consolacion M. Gomez
owner of certain lots in SitioPoponto Bayambang, Pangasinan.
Petitioners are the heirs of Teodoro Y. Gomez(father of Consolacion).
Afternotice and publication, and there being no opposition to the
application, thetrial court issued an order of general default. On 5
August 1981, the courtrendered its decision adjudicating the subject
lots in petitioners' favor.
OnOctober 6, 1981, the trial court issued an order expressly stating
that the decisionof August 5, 1981 had become final anddirected the
Chief of the General Land Registration Office to issue the
correspondingdecrees of registration over the lots adjudicated in the
decision of August 5,1981.
OnJuly 11, 1984, respondent Silverio G. Perez, Chief of the Division of
OriginalRegistration, Land Registration Commission submitted a report
to the court aquo stating that portionsof the land sought for
registration were covered by homestead patents issued in1928 and
1929 and registered under the Land Registration Act. He
recommendedthat the decision of August 5, 1981 and the order of
October 6, 1981 be setaside. Petitioners opposed the report, pointing

out that no opposition wasraised by the Bureau of Lands during the


registration proceedings and that thedecision of August 5, 1981 should
be implemented because it had long becomefinal and executory.
Afterhearing, the lower court rendered a second decision setting aside
the decisiondated August 5, 1981 and the order dated October 6, 1981
for the issuance ofdecrees. Petitionersmoved for reconsideration but
the motion was denied. AHence, this recourse.
Issue:
Whetherthe decision dated August 5, 1981 had become final and
executory, that it mayno longer be reopened, reviewed, or set aside.
Ruling:
Petitionersanchor their claim on section 30 of P.D. No. 1529 (Property
RegistrationDecree) which provides that, after judgment has become
final and executory, thecourt shall forthwith issue an order to the
Commissioner of Land Registrationfor the issuance of the decree of
registration and certificate of title.Petitioners contend that section 30
should be read in relation to section 32 ofP.D. 1529 in that, once the
judgment becomes final and executory under section30, the decree of
registration must issue as a matter of course. This being thelaw,
petitioners assert, when respondent Judge set aside in his decision,
dated25 March 1985, the decision of 5 August 1981 and the order of 6
October 1981,he clearly acted without jurisdiction.
Petitioners'contention is not correct. Unlike ordinary civil actions, the
adjudication ofland in a cadastral or land registration proceeding does
not become final, inthe sense of incontrovertibility until after the
expiration of one (1) yearafter the entry of the final decree of
registration.
Petition denied.

9. CAYANAN V. DE LOS SANTOS


21 SCRA 1348

Facts:

The title of appellee De Los Santos to Lot 56 of Porac Cadastrewas


confirmed by Judge Santos of the Court of First Instance of Pampanga.

In the same year, there was a petition for review for the saidlot. It was
alleged that said lot was registered in the name of appellee De
losSantos through actual fraud, through deceit and through
intentionalomission of facts. It was stated further that a simulated
Deed ofAbsolute Sale was executed in favor of the other respondent
appellee,Felix Camaya.

It prayed further for the opening of the decree ofregistration, the


cancellation of the Original Certificate of Title, aswell as the Transfer
Certificate of Title and the adjudication of said lotin favor of petitioners,
now appellant Cayanan, however it wasdenied.

Issue:

Whether the case may still be reopened and the decision set
asidewhen granted

Ruling:

Yes,it may.
As long as the final decree is not issued by theChief of the General
Land Registration Office in accordance with the law, andthe period of
one year filed for the review thereof has not elapsed, the titleis not
finally adjudicated and the decision therein rendered continues to
beunder the control and sound discretion of the court rendering it.

10. HEIRS OF CRISTOBAL MARCOS vs. DE BANUVAR


G.R. No. L-22110.
FACTS:
On March 24, 1938 the CFI rendered a decision confirming thetitles of
La Urbana, Inc. over lot 5 and lot 1, Psu-56145, with
certainreservations, and ordered the registration of these lots in favor
of thelatter.On May 17, 1960 de Erquiaga, one of the successors-ininterest of LaUrbana, Inc. filed a petition for reconstitution of the
decision of March 24,1938. During the pendency of the reconstitution
proceedings, the respondent DeBanuvar acquired lot 1 from de
Erquiaga, who was thus substituted as a partyfor the latter.
However, the petitionersopposed the application for the reconstitution,
on a claim that they have beenin actual, adverse, open and
uninterrupted possession and occupation of thesaid parcel in the
concept of owners since time immemorial, long before theWorld War II,
and have introduced improvements thereon. They prayed that
theapplication for the reconstitution of records be denied and that
"theparcels of land in question be ordered registered in the respective
names ofthe herein oppositors or declare the same as public land and
be subdivided tooppositors who are landless." OnJune 13, 1963 the
respondent court ordered the issuance of a decree in favor ofDe
Banuvar with respect to lot 1 only, after finding that the decision in
theland registration case had already become final and executory.
Moreover, thecourt issued another order granting a writ of possession
in favor of De Banuvarand "against all persons who have entered and
occupied portions of lot 1,Psu-56145 before the issuance of the
decree."

ISSUE:
Whether the issuanceof the order is proper.

RULING:
In the opposition to thepetition for reconstitution, the petitioners'
alleged possession and occupationof portions of lot 1 arose prior to or
during the registration proceedings. Forthis reason, the order of
granting a writ of possession in favor of De Banuvaragainst the
petitioners, is proper and justified. The petitioners hereinadmittedly
took possession and occupation of portions of lot 1 prior to July 1,1963
when the decree in question was issued. The fundamental rule is that
awrit of possession can be issued not only against the original
oppositors in aland registration case and their representatives and
successors-in-interest,but also against any person unlawfully and
adversely occupying said lot at anytime before and up to the issuance
of the final decree.
Furthermore, there is noprovision in the Land Registration Act similar
to Sec. 6, Rule 39, regardingthe execution of a judgment in civil action,
except the proceedings to placethe winner in possession by virtue of a
writ of possession. The decision in aland registration case unless the
adverse or losing party is in possession,becomes final without any
further action, upon the expiration of the period forperfecting an
appeal.
There is nothing in the law that limits the period within whichthe court
may order or issue a decree. The reason is ... that the judgment
ismerely declaratory in character and does not need to be asserted or
enforcedagainst the adverse party. Furthermore, the issuance of a
decree is aministerial duty both of the judge and of the Land
Registration Commission;failure of the court or of the clerk to issue the
decree for the reason that nomotion therefor has been filed cannot
prejudice the owner, or the person inwhom the land is ordered to be
registered.

11. EUFEMIA VILLANUEVA VDA. DE BARROGA,et al. VS.ANGEL


ALBANO,et al.
G.R. No. L-43445 January 20, 1988

Facts:

On July 31, 1941, CFI Ilocos Norte rendered a decision inCadastral


Proceeding No. 44 adjudicating a parcel of land known as Lot No.
9821in favor of Delfina Aquino.One of the oppositors was Ruperta
Pascual, who wasdeclared in default. However, thedecree of
registration was only issued on October 14, 1955; and it was only
onNovember 17, 1979, that an original certificate of title was issued in
DelfinaAquino's name.On August 11, 1970, EufemiaBarroga and
Saturnina Padaca,the children and heirs of Ruperta Pascual,
broughtsuit in the same Court of First Instance praying that Delfina
Aquino's title bevoided and cancelled, and that title be adjudicated in
their favor. Barroga'sand Padaca's complaint was denied by the court.
Thereafter, the Court of FirstInstance ordered execution of the
judgment on December 6, 1973. On August 8,1975, the Cadastral
Court promulgated an order granting the motion of AngelAlbano, et al.
for a writ of possession as regards Lot No. 9821. A writ ofpossession
dated August 28, 1975 was issued. The plaintiffs appealed. Themotion
was thereafter denied by the court by Order dated September 22,
1975. Hence, this petition.

Issue:

Whether theCourt could still issue orders despite lapse of long period
of time from entryof judgment.

Held:

The judgment of the cadastral court was ajudgment in rem,binding


generally upon the whole world, inclusive of persons not
partiesthereto, and particularly uponthose who had actually taken part
in the proceeding as well as their successorsin interest by title
subsequent to the commencement of the action or specialproceeding,
litigating for the same thing and under the same title and in thesame
capacity. The writ ofpossession could properly issue despite the not
inconsiderable period of timethat had elapsed from the date of the

registration decree, since the right tothe same does not prescribe. The
appellees are entitled to said writ ofpossession, despite the lapse of
many years, their right thereto beingimprescriptible at least as against
the persons who were parties to thecadastral case or their successorsin-interest.

12. MAMERTA DE LA MERCED VS.COURT OF APPEALS


5 SCRA 240
Facts:
Ezequiel Santos (and his wife) is claiming ownership of LotNo. 395 of
the Rizal Cadastre by virtue of an adjudication of the cadastralcourt in
favor of his father. Defendants, in their answer, resisted plaintiffs'claim
and asserted their ownership over said property as evidenced by an
OCTissued to their predecessor Juan de la Merced and their continuous
possessionof the land for more than 30 years. Mamerta de la Merced, a
legitimate daughterof Juan de la Merced, was allowed to intervene and
make common cause with thedefendants. The court rendered a
decision for the plaintiffs after making afinding the lot was part of the
OCT in the name of the spouses Inocencio de losSantos and Victorina
Macapagal, parents of plaintiff Ezequiel Santos. Thecadastral court
declared the lot a public land, as a consequence of which Juande la
Merced, after filing a homestead application therefor, was able to
obtainOriginal Certificate of Title. Holding that the cadastral court had
nojurisdiction to issue the order declaring the lot public land, and,
therefore,the same as well as the certificate of title issued thereafter
was null andvoid, the court ordered the cancellation of OCT in the
name of Juan de laMerced; directed defendants to vacate Lot. While
the court held that the landhaving ceased to be part of the public
domain, the Director of Lands no longerhad authority to grant the
homestead patent over the same to Juan de la Merced,Plaintiffs
interposed an appeal to the Court of Appeals.. Hence, the filing of
theinstant petition for review.
Issue:
Whether the property thereby affected still be lost byadverse
possession.
Ruling:

A decree of registration and a certificate of title, under Act496, are two


different things. And it is the decree of registration, to beissued by the
Land Registration Commissioner, which shall be the basis of
thecertificate of title to be issued subsequently by the corresponding
register ofdeeds, that quiets title to and bindsthe land. The title of
ownership on the land is vested upon the owner upon theexpiration of
the period to appeal from the decision or adjudication by thecadastral
court, without such an appeal having been perfected. The certificateof
title would then be necessary for purposes of effecting registration of
subsequentdisposition of the land where court proceedings would no
longer be necessary.
As we have here a decree issued by the cadastral court, orderingthe
issuance to Inocencio de los Santos of the certificate of title over
LotNo. 395 after the decision adjudicating ownership to him of the said
propertyhad already become final, and there being no imputation of
irregularity in thesaid cadastral proceedings, title of ownership on the
said adjudicatee wasvested as of the date of the issuance of such
judicial decree. The land, forall intents and purposes, had become,
from that time, registered property whichcould not be acquired by
adverse possession.

13(Douglas)

14(Daniel Eblahan)
Eland Philippines Inc. v. GarciaGR no. 173289, February 17,
2010Peralta, J.:Facts: The respondents owned a parcel of land in
Tagaytay City. They were occupying the possessing said land
continuously, publicly, and adversely for at least thirty years, under the
Public Land Act. They later found out that the petitioner has applied to
register the land with the Regional Trial Court and that the same court
has already issued a decree. They filed a Quieting of Title with the
same court against Eland Philippines. In a summary judgment, the
same trial court ruled in favor of the respondents as the absolute
owners and rightful possessors of the land and set aside its earlier
decree. The Court of Appeals dismissed the case. Issue: Whether or
not the summary judgment was properHeld: No. The Court reversed

the lower courts decision, ruling that summary judgment is


appropriate when there are no genuine issues of fact. In this case, a
genuine issue of fact has been raised by the respondents that tend to
prove the invalidity of the petitioners claim. These issues can only be
resolved in a full trial. Also, the Court ruled that the quieting of title
was not the proper remedy but a petition for review of the decree of
registration. This petition may be availed a person deprived of land or
any interest or estate by reason of fraud when the following requisites
are satisfied: 1. Petitioner must have an estate or interest in the land.
2. He must show actual fraud in the procurement of the decree of
registration 3. The petition must be filed within one year from the
issuance of the final decree by the Land Registration Authority 4. The
property has not yet passed to an innocent purchaser for value. In this
case, the period of one year before the decree becomes final and
incontrovertible has not yet lapsed when they filed the complaint for
the quieting of title and because the decree was not yet issued by the
Land Registration Administrator.Significance:A review of a decree is
allowed when the person is deprived of land or his interest or estate
within due to fraud. The Court also laid down the rules on how to avail
said remedy. T Court also noted that the petition may be filed after the
decision of the court up until before the one year expiration period
from the entry of the final decree issued by the Land Registration
Administrator.

15(Douglas)

16(Mel)

HEIRS OF CRISTOBAL MARCOS vs. DE BANUVAR


G.R. No. L-22110.
FACTS:
On March 24, 1938 the CFI rendered a decision confirming the titles of
La Urbana, Inc. over lot 5 and lot 1, Psu-56145, with certain
reservations, and ordered the registration of these lots in favor of the
latter.On May 17, 1960 de Erquiaga, one of the successors-in- interest
of La Urbana, Inc. filed a petition for reconstitution of the decision of
March 24, 1938. During the pendency of the reconstitution

proceedings, the respondent De Banuvar acquired lot 1 from de


Erquiaga, who was thus substituted as a party for the latter.
However, the petitioners opposed the application for the
reconstitution, on a claim that they have been in actual, adverse, open
and uninterrupted possession and occupation of the said parcel in the
concept of owners since time immemorial, long before the World War II,
and have introduced improvements thereon. They prayed that the
application for the reconstitution of records be denied and that "the
parcels of land in question be ordered registered in the respective
names of the herein oppositors or declare the same as public land and
be subdivided to oppositors who are landless." On June 13, 1963 the
respondent court ordered the issuance of a decree in favor of De
Banuvar with respect to lot 1 only, after finding that the decision in the
land registration case had already become final and executory.
Moreover, the court issued another order granting a writ of possession
in favor of De Banuvar and "against all persons who have entered and
occupied portions of lot 1, Psu-56145 before the issuance of the
decree."

ISSUE:
Whether the issuance of the order is proper.
RULING:
In the opposition to the petition for reconstitution, the petitioners'
alleged possession and occupation of portions of lot 1 arose prior to or
during the registration proceedings. For this reason, the order of
granting a writ of possession in favor of De Banuvar against the
petitioners, is proper and justified. The petitioners herein admittedly
took possession and occupation of portions of lot 1 prior to July 1, 1963
when the decree in question was issued. The fundamental rule is that a
writ of possession can be issued not only against the original
oppositors in a land registration case and their representatives and
successors-in-interest, but also against any person unlawfully and
adversely occupying said lot at any time before and up to the issuance
of the final decree.
Furthermore, there is no provision in the Land Registration Act similar
to Sec. 6, Rule 39, regarding the execution of a judgment in civil action,
except the proceedings to place the winner in possession by virtue of a
writ of possession. The decision in a land registration case unless the
adverse or losing party is in possession, becomes final without any

further action, upon the expiration of the period for perfecting an


appeal.
There is nothing in the law that limits the period within which the court
may order or issue a decree. The reason is ... that the judgment is
merely declaratory in character and does not need to be asserted or
enforced against the adverse party. Furthermore, the issuance of a
decree is a ministerial duty both of the judge and of the Land
Registration Commission; failure of the court or of the clerk to issue the
decree for the reason that no motion therefor has been filed cannot
prejudice the owner, or the person in whom the land is ordered to be
registered.

17(Douglas)

18(Maribeth)

Manotok vs. Barque, G.R.No. 162335, December 12, 2005


FACTS:
Respondents Barque, filed a petitionwith the LRA for administrative
reconstitution of the original copy of TCT No.210177 issued in the
name of Homer L. Barque, which was destroyed in the firethat gutted
the Quezon City Hall, including the Office of the Register of Deedsof
Quezon City. In support of said petition, they submitted the
ownersduplicate copy of TCT No. 210177, real estate tax receipts, tax
declarationsand the Plan FLS 3168 D covering the property.
Upon being notified of the petition foradministrative reconstitution,
Manotok filed their opposition thereto claimingthat the lot covered by
the title under reconstitution forms part of the landcovered by their
reconstituted title TCT No. RT-22481, and alleging that TCTNo. 210177
in the name of petitioners predecessors-in-interest is spurious.
On June 30, 1997, Atty. Benjamin M.Bustos, as reconstituting officer,
denied the reconstitution of TCT No.210177 on grounds that:

1. Lots 823-A and 823-B, Fls-3168-D,containing areas of 171,473 Sq.


Mtrs. and 171,472 Sq. Mtrs., respectively,covered by TCT No. 210177,
appear to duplicate Lot 823 Piedad Estate,containing an area of
342,945 Sq. Mtrs., covered by TCT No. 372302 registeredin the name
of Severino M. Manotok, et. al., reconstituted under Adm.Reconstitution
No. Q-213 dated February 01, 1991;
2. The submitted plan Fls-3168-D is aspurious document as
categorically stated by Engr. Privadi J.G. Dalire, Chief,Geodetic Surveys
Division, Land Management Bureau, in his letter dated February19,
1997.<a></a><a>6</a>
Respondents motion for reconsiderationwas denied hence they
appealed to the LRA which ruled that the reconstitutingofficer should
not have required the submission of documents other than theowners
duplicate certificate of title as bases in denying the petition andshould
have confined himself with the owners duplicate certificate oftitle. The
LRA further declared that based on the documents presented,
theBarques have established by clear and convincing evidence that
TCT NO. 210177,at the time of the destruction thereof, was valid,
genuine, authentic andeffective. They duly presented the original of
the owners duplicate copy ofTCT No. 210177. The logbook of the
Register of Deeds of Quezon City lists TCTNo. 210177 as among the
titles lost. The Register of Deeds of Quezon Cityhimself acknowledged
the existence and authenticity of TCT No. 210177 when heissued a
certification to the effect that TCT No. 210177 was one of the
titlesdestroyed and not salvaged from the fire that gutted the Quezon
City Hall.
It is likewise noteworthy that thetechnical description and boundaries
of the lot reflected in TCT No. 210177absolutely conform to the
technical description and boundaries of Lot 823Piedad Estate ... as
indicated in the B. L. Form No. 28-37-R dated 11-8-94 andB. L. Form
No. 31-10 duly issued by the Bureau of Lands.
It therefore becomes evident that theexistence, validity, authenticity
and effectivity of TCT No. 210177 wasestablished indubitably and
irrefutably by the petitioners. Under suchcircumstances, the
reconstitution thereof should be given due course and thesame is
mandatory.
Nevertheless, notwithstanding the conclusionthat Respondents title
was fraudulently reconstituted, the LRA noted that itis only the
Regional Trial Court (RTC) which can declare that the same wasindeed
fraudulently reconstituted. It thus opined that respondents title

mayonly be reconstituted after a judicial declaration that petitioners


title wasvoid and should therefore be cancelled.
Petitioners filed a motion forreconsideration which was opposed by
respondents with a prayer thatreconstitution be ordered immediately.
Both were denied.
From the foregoing, respondents filed apetition for review with the
Court of Appeals docketed as CA-G.R. SP No.66700 and praying that
the LRA be directed to immediately reconstitute TCT No.210177
without being subjected to the condition that petitioners TCT No.RT22481 [372302] should first be cancelled by a court of
competentjurisdiction. Petitioners likewise filed a petition for review
with theCourt of Appeals docketed as CA-G.R. SP No. 66642.
In CA-G.R. SP No. 66700, the SecondDivision of the Court of Appeals
rendered a Decision the dispositive portion ofwhich reads:
WHEREFORE, the foregoing premisesconsidered the assailed Resolution
of the LRA dated June 24, 1998 is AFFIRMEDin toto and the petition for
review is ordered DISMISSED.Respondents moved for reconsideration.
On November 7, 2003, the SpecialDivision of Five of the Former Second
Division rendered an Amended Decision inCA-G.R. SP No. 66700, the
dispositive portion of which reads:
WHEREFORE, our decision dated 13September 2002 is hereby
reconsidered. Accordingly, the Register of Deeds ofQuezon City is
hereby directed to cancel TCT No. RT-22481 of privaterespondents and
the LRA is hereby directed to reconstitute forthwithpetitioners valid,
genuine and existing Certificate of Title No. T-210177.
Petitioners motion for reconsiderationof the amended decision in CAG.R. SP No. 66700 was denied, hence, thepetition docketed as G.R. No.
162605.
Meanwhile, in CA-G.R. SP No. 66642, theThird Division of the Court of
Appeals rendered a Decision on October 29,2003, the dispositive
portion of which reads:
WHEREFORE, the petition is herebyDENIED. The Resolution of the LRA
dated 24 June 1998 is hereby AFFIRMED.
In so ruling, the Third Division of theCourt of Appeals declared that the
LRA correctly deferred in giving due courseto the petition for
reconstitution since there is yet no final judgmentupholding or
annulling respondents title.<a></a><a></a>

Respondents motion for reconsiderationwas granted by the Third


Division of the Court of Appeals on February 24, 2004,thus:
WHEREFORE, the Motion forReconsideration is hereby GRANTED. The
Decision of this Court dated 29 October2003 is RECONSIDERED and a
new one is entered ordering the Register of Deeds ofQuezon City to
cancel petitioners TCT No. RT-22481 and directing the LRA
toreconstitute forthwith respondents TCT No. T-210177.
From the foregoing decisions of theCourt of Appeals in CA-G.R. SP No.
66700 and CA-G.R. SP No. 66642, petitionersfiled separate petitions for
review before the Supreme Court docketed as G.R.No. 162605 and G.R.
No. 162335, respectively.
ISSUE: Whether or not the LRA could proceed with thereconstitution of
the tile?
HELD: Yes. The court held that the LRA properly ruled that the
reconstitutingofficer should have confined himself to the owners
duplicate certificate oftitle prior to the reconstitution. Section 3 of
Republic Act (RA) No. 26 clearlyprovides:
Section 3. Transfer certificates oftitle shall be reconstituted from such
of the sources hereunder enumerated asmay be available, in the
following order:
(a) The ownersduplicate of the certificate of title;
When respondents filed the petition forreconstitution, they submitted
in support thereof the owners duplicatecertificate of title, real estate
tax receipts and tax declaration. Plainly,the same should have more
than sufficed as sources for the reconstitutionpursuant to Section 3 of
RA No. 26 which explicitly mandates that thereconstitution shall be
made following the hierarchy of sources asenumerated by law. In
addition, Section 12 of the same law requires that thepetition shall be
accompanied with a plan and technical description of theproperty
only if the source of the reconstitution is Section 3(f)of RA No. 26. Thus:
Section 12. Provided, That in casethe reconstitution is to be made
exclusively from sourcesenumerated in section 2(f) or 3(f) of this Act,
the petition shall further beaccompanied with a plan and technical
description of the property duly approvedby the Chief of the General
Land Registration Office, or with a certified copyof the description
taken
from
a
prior
certificate
of
title
covering
the
sameproperty.<a></a><a>29</a>

Since respondents source ofreconstitution is the owners duplicate


certificate of title, there is no needfor the reconstituting officer to
require the submission of the plan, much lessdeny the petition on the
ground that the submitted plan appears to be spurious.By enumerating
the hierarchy of sources to be used for the reconstitution, itis the intent
of the law to give more weight and preference to the ownersduplicate
certificate of title over the other enumerated sources.

19(Ailyn)

NATURE OF RECLAIMED LANDS


G.R. No. 133250 July 9,2002
FRANCISCO I. CHAVEZ, PETITIONER, VS. PUBLIC ESTATES AUTHORITY
AND
AMARICOASTAL
BAY
DEVELOPMENT
CORPORATION,
RESPONDENTS
FACTS
On November 20, 1973, the government, through theCommissioner of
Public Highways, signed a contract with the Construction
andDevelopment Corporation of the Philippines (CDCP) to reclaim
certain foreshoreand offshore areas of Manila Bay. On February 4,
1977, then President FerdinandE. Marcos issued Presidential Decree
No. 1084 creating PEA - tasked to reclaim land, including foreshoreand
submerged areas, and to develop, improve, acquire, x x x lease and
sellany and all kinds of lands. On the same date, then President
Marcos issuedPresidential Decree No. 1085 transferring to PEA the
lands reclaimed in theforeshore and offshore of the Manila Bay under
the Manila-Cavite Coastal Roadand Reclamation Project (MCCRRP).
OnJanuary 19, 1988, then President Corazon C. Aquino issued Special
Patent No.3517, granting and transferring to PEA the parcels of land
so reclaimed underthe Manila-Cavite Coastal Road and Reclamation
Project (MCCRRP) .Subsequently,on April 9, 1988, the Register of Deeds
of the Municipality of Paraaque issuedTransfer Certificates of Title in
the name of PEA, covering the three reclaimedislands known as the
Freedom Islands located at the southern portion of theManila-Cavite
Coastal Road, Paraaque City. On April 25, 1995, PEA entered intoa

Joint Venture Agreement (JVA) with AMARI, a private corporation, to


developthe Freedom Islands. PEA and AMARI entered into the JVA
through negotiationwithout public bidding. On June 8, 1995, then
President Fidel V. Ramos, throughthen Executive Secretary Ruben
Torres, approved the JVA.
OnNovember 29, 1996, then Senate President Ernesto Maceda
delivered a privilegespeech in the Senate and denounced the JVA as
the grandmother of all scams.As a result, the Senate Committee on
Government Corporations and PublicEnterprises, and the Committee
on Accountability of Public Officers andInvestigations, conducted a joint
investigation. Among the conclusions of theirreport are: (1) the
reclaimed lands PEA seeks to transfer to AMARI under theJVA are lands
of the public domain which the government has not classified as
alienablelands and therefore PEA cannot alienate these lands; (2) the
certificates oftitle covering the Freedom Islands are thus void, and (3)
the JVA itself isillegal.
OnDecember 5, 1997, then President Fidel V. Ramos issued
PresidentialAdministrative Order No. 365 creating a Legal Task Force to
conduct a study onthe legality of the JVA. The Legal Task Force upheld
the legality of the JVA,contrary to the conclusions reached by the
Senate Committees.
On April27, 1998, petitioner Frank I. Chavez (Petitioner for brevity) as
a taxpayer,filed the instant Petition for Mandamus with Prayer for the
Issuance of aWrit of Preliminary Injunction and Temporary Restraining
Order. Petitionercontends the government stands to lose billions of
pesos in the sale by PEA ofthe reclaimed lands to AMARI. Petitioner
prays that PEA publicly disclose theterms of any renegotiation of the
JVA, invoking Section 28, Article II, andSection 7, Article III, of the 1987
Constitution on the right of the people toinformation on matters of
public concern. Petitioner assails the sale to AMARIof lands of the
public domain as a blatant violation of Section 3, Article XIIof the 1987
Constitution prohibiting the sale of alienable lands of the publicdomain
to private corporations. In a Resolution dated March 23, 1999, the
Courtgave due course to the petition and required the parties to file
theirrespective memoranda.
On March30, 1999, PEA and AMARI signed the Amended Joint Venture
Agreement (Amended JVA).On May 28, 1999, the Office of the President
under the administration of thenPresident Joseph E. Estrada approved
the Amended JVA.

Due tothe approval of the Amended JVA by the Office of the President,
petitioner nowprays that on constitutional and statutory grounds the
renegotiated contractbe declared null and void.
ISSUE
WHETHERTHE Lands reclaimed from foreshore andsubmerged areas
also form part of the public domain and are also inalienable
WHETHERTHE STIPULATIONS IN THE AMENDED JOINT VENTURE
AGREEMENT FOR THE TRANSFER TOAMARI OF CERTAIN LANDS,
RECLAIMED AND STILL TO BE RECLAIMED, VIOLATE THE
1987CONSTITUTION
RULING
The 1987 Constitution, like the 1935 and 1973 Constitutions before
it,has adopted the Regalian doctrine. The 1987 Constitution declares
that allnatural resources are owned by the State, and except
foralienable agricultural lands of the public domain, natural resources
cannot bealienated.
UnderSection 2, Article XII of the 1987 Constitution, the foreshore and
submergedareas of Manila Bay are part of the lands of the public
domain, waters x x xand other natural resources and consequently
owned by the State. As such,foreshore and submerged areas shall
not be alienated, unless they areclassified as agricultural lands of
the public domain. The mere reclamationof these areas does not
convert these inalienable natural resources of theState into alienable
or disposable lands of the public domain. There must be alaw or
presidential proclamation officially classifying these reclaimed landsas
alienable or disposable and open to disposition or concession.
Moreover,these reclaimed lands cannot be classified as alienable or
disposable if thelaw has reserved them for some public or quasi-public
use.
Landsreclaimed by the government are sui generis, not available
forsale to private parties unlike other alienable public lands. Reclaimed
landsretain their inherent potential as areas for public use or public
service.
Clearly,the Amended JVA violates glaringly Sections 2 and 3, Article XII
of the 1987Constitution. Under Article 1409 of the Civil Code, contracts
whose object orpurpose is contrary to law, or whose object is
outside the commerce of men,are inexistent and void from the
beginning. The Court must perform its dutyto defend and uphold the

Constitution, and therefore declares the Amended JVAnull and void


ab initio.

TAX DECLARATION
G.R. No. 177797, December 04, 2008
SPS. PEDRO TAN AND NENA ACERO TAN,PETITIONERS, VS. REPUBLIC
OF THE PHILIPPINES, RESPONDENT
FACTS
The spouses Pedro Tan and Nena AceroTan were natural-born Filipino
citizens, who became Australian citizens on 9February 1984. They seek
to have thesubject property registered in their names. The subject
property was declaredalienable and disposable on 31 December 1925,
as established by a Certificationdated 14 August 2000 issued by the
Department of Environment and NaturalResources (DENR), Community
Environment and Natural Resources Office (CENRO),Cagayan de Oro
City. Prior to the spouses Tan, the subject property was in
thepossession of Lucio and Juanito Neri and their respective spouses.
Lucio and Juanito Neri had declared thesubject property for taxation
purposes in their names.The spouses Tan acquiredthe subject property
from Lucio and Juanito Neri and their spouses by virtue ofa duly
notarized Deed of Sale of Unregistered Real Estate Property dated
26June 1970. The spouses Tan tookimmediate possession of the
subject property on which they planted rubber,gemelina, and other
fruit-bearing trees. They declared the subject property for taxation
purposes in their names andpaid realty taxes thereon.

However, a certain Patermateo Casio (Casio) claimed aportion of the


subject property, prompting the spouses Tan to file a Complaintfor
Quieting of Title against him before the RTC of Cagayan de Oro City. On
29August 1989, the RTC rendered a Decision favoring the spouses Tan
and declaringtheir title to the subject property thus "quieted." Casio
appealed the said RTC Decision to theCourt of Appeals . In a Resolution
the appellate court dismissed for lack ofinterest to prosecute. Casio
elevatedhis case to this Court via a Petition for Review on Certiorari. In
a Resolutiondated 13 March 1991 the Court deniedCasio's Petition for
being insufficient in form and substance. The said Resolution became

final andexecutory on 3 June 1991. Refusing to give up, Casio filed an


Application forFree Patent on the subject property before the Bureau of
Lands. On 8 December1999, Casio's application was ordered
cancelled by Officer Ruth G. Sabijon ofDENR-CENRO, Cagayan de Oro
City, upon the request of herein petitioner PedroTan, the declared
owner of the subject property pursuant to the 29 August 1989Decision
of the RTC. In 2000, the spouses Tan filed their Application
forRegistration of Title to the subject property before the RTC of
Cagayan de OroCity. The application of the spouses Tan invoked the
provisions of Act No. 496 and/orSection 48 of Commonwealth Act No.
141,as amended. In compliance with the request of the
LandRegistration Authority (LRA) dated 29 August 2000, the spouses
Tan filed on 5October 2000 an Amended Application for Registration of
Title to the subjectproperty.

The Office of the Solicitor General (OSG) entered itsappearance on


behalf of the Republic, but failed to submit a written oppositionto the
application of the spouses Tan.

When no opposition to the application of the spouses Tan wasfiled by


the time of the initial hearing the RTC issued on 23 April 2001 anorder
of general default, except as against the Republic. Thereafter, the
spouses Tan were allowed topresent their evidence ex-parte.

After the establishment of the jurisdictional facts, the RTCheard the


testimony of John B. Acero , nephew and lone witness of the
spousesTan. After Acero's testimony, the spouses Tan already made a
formal offer ofevidence, which was admitted by the court a quo. On 9
May 2001, the RTCrendered a Decision granting the application of the
spouses Tan. The Republicappealed the RTC Decision to the Court of
Appeals.

On 28 February 2006, the Court of Appeals rendered aDecision


granting the appeal of the Republic, and reversing and setting asidethe
9 May 2001 Decision of the RTC on the ground that the spouses Tan
failed tocomply with Section 48(b) of Commonwealth Act No. 141,
otherwise known as thePublic Land Act, as amended by Presidential
Decree No. 1073, which requirespossession of the subject property to

start on or prior to 12 June 1945. Hence,the appellate court ordered


the spouses Tan to return the subject property tothe Republic.

The spouses Tan filed a Motion for Reconsideration of theforegoing


Decision of the Court of Appeals. To refute the finding of the appellate
court that they and theirpredecessors-in-interest did not possess the
subject property by 12 June 1945or earlier, the spouses Tan attached
to their Motion a copy of Tax DeclarationNo. 4627 covering the subject
property issued in 1948 in the name of theirpredecessor-in-interest,
Lucio Neri. They called attention to the statement in Tax Declaration
No. 4627 thatit cancelled Tax Declaration No. 2948. Unfortunately, no
copy of Tax Declaration No. 2948 was available even inthe Office of the
Archive of the Province of Misamis Oriental. The spouses Tanasserted
that judicial notice may be taken of the fact that land assessment
isrevised by the government every four years; and since Tax
Declaration No. 4627was issued in the year 1948, it can be
presupposed that Tax Declaration No.2948 was issued in the year
1944.

The Court of Appeals denied the Motion for Reconsiderationof the


spouses Tan in a Resolution dated 12 April 2007.

The earliestevidence of possession and occupation of the subject


property can be tracedback to a tax declaration issued in the name of
their predecessors-in-interestonly in 1952. However, the spouses
Tanare now asking the kind indulgence of this Court to take into
account TaxDeclaration No. 4627 issued in 1948, which they had
attached to their Motionfor Reconsideration before the Court of
Appeals but which the appellate courtrefused to consider. Just as they
hadargued before the Court of Appeals, the spouses Tan point out that
TaxDeclaration No. 4627 was not newly issued but cancelled Tax
Declaration No.2948; and should the Court take judicial notice of the
fact that taxassessments are revised every four years, then Tax
Declaration No. 2948covering the subject property was issued as early
as 1944.

ISSUE

WHETHER TAXDECLARATIONS
EVIDENCE OF OWNERSHIP

AND

RECEIPTS

ARE

CONCLUSIVE

RULING

Tax declarations and receipts are not conclusive evidence ofownership.


At most, they constitute mere prima facie proofs of ownershipof the
property for which taxes have been paid. In the absence of
actual,public and adverse possession, the declaration of the
land for tax purposesdoes not prove ownership.They may be
good supporting or collaboratingevidence together with other acts of
possession and ownership; but bythemselves, tax declarations are
inadequate to establish possession of the propertyin the nature and for
the period required by statute for acquiring imperfect orincomplete
title to the land.

The spouses Tan purchased the subject property and came


intopossession of the same only in 1970. To justify their application for
registrationof title, they had to tack their possession of the subject
property to that oftheir predecessors-in-interest. Whilethe spouses Tan
undoubtedly possessed and occupied the subject property
openly,continuously, exclusively and notoriously, by immediately
introducingimprovements on the said property, in addition to declaring
the same and payingrealty tax thereon; in contrast, there was a dearth
of evidence that theirpredecessors-in-interest possessed and occupied
the subject property in thesame manner. The possession
andoccupation of the subject property by the predecessors-in-interest
of thespouses Tan were evidenced only by the tax declarations in the
names of theformer, the earliest of which, Tax Declaration No. 4627,
having been issued onlyin 1948. No other evidence was presentedby
the spouses Tan to show specific acts of ownership exercised by
theirpredecessors-in-interest over the subject property which may date
back to 12June 1945 or earlier.

For failure of the Spouses Tan to satisfy the requirementsprescribed by


Section 48(b) of the Public Land Act, as amended, this Court hasno
other option but to deny their application for judicial confirmation
andregistration of their title to the subject property.

DECREE OF REGISTRATION

G.R. No.123346, December 14, 2007

MANOTOK
REALTY,
INC.
CORPORATION,PETITIONERS,
VS.
CORPORATION, RESPONDENT

AND
CLT

MANOTOK
ESTATE
REALTY
DEVELOPMENT

FACTS

On 10August 1992, CLT Realty Development Corporation (CLT)


sought to recover fromManotok Realty, Inc. and Manotok Estate
Corporation (Manotoks) the possessionof Lot 26 of the Maysilo Estate
in an action filed before the Regional TrialCourt of Caloocan City. CLTs
claim was anchored on Transfer Certificate ofTitle (TCT) issued in its
name by the Caloocan City Register of Deeds, whichtitle in turn was
derived from Estelita Hipolito (Hipolito) by virtue of a Deedof Sale with
Real Estate Mortgage dated 10 December 1988. Hipolitos
titleemanated from Jose Dimsons (Dimson) TCT , a title issued
pursuant to an orderof the Court of First Instance (CFI) of Caloocan City.
Dimsons title appearsto have been sourced from OCT No. 994.

For their part, the Manotoks challenged the validity of thetitle relied on
by CLT, claiming that Dimsons title, the proximate source ofCLTs title,
was irregularly issued and, hence, the same and subsequent
titlesflowing therefrom are likewise void. The Manotoks asserted their

ownership overLot 26 and claimed that they derived it from several


awardees and/or vendees ofthe National Housing Authority. The
Manotok title likewise traced as itsprimary source OCT No. 994 which,
on 9 September 1918, was transferred toAlejandro Ruiz and Mariano
Leuterio who had previously acquired the property on21 August 1918
by virtue of an Escritura de Venta executed by Don TomasArguelles
and Don Enrique Llopis. On 3 March 1920, Ruiz and Leuterio sold
theproperty to Francisco Gonzalez who held title thereto until 22
August 1938 whenthe property was transferred to Jose Leon Gonzalez,
Consuelo Susana Gonzalez,Juana Francisca Gonzalez, Maria Clara
Gonzalez, Francisco Felipe Gonzalez andConcepcion Maria Gonzalez
under TCT No. 35486. The lot was then, per annotationdated 21
November 1946, subdivided into seven (7) parcels each in the name
ofeach of the Gonzalezes.

The trial court, ruling for CLT, adopted the factualfindings and
conclusions arrived at by the majority commissioners appointed
toresolve the conflict of titles. It was established that the entire
MaysiloEstate was registered under Act No. 496 by virtue of which OCT
No. 994 wasissued by the Register of Deeds of Rizal;that Lot 26 was
transferred to CLT byHipolito whose title was derived from the Dimson
title and that on the basis ofthe technical descriptions of the property
appearing in the Manotok titles, thelatters property indeed encroached
on the property described in CLTs title.

The Manotoks appealed to the Court of Appeals, whichaffirmed the


decision of the trial court. Their motion for reconsiderationhaving been
denied, they filed a petition for review with the Supreme
Court,ascribing error to the appellate court in upholding the trial
courts decisionwhich decided the case on the basis of the majority
commissioners report andoverlooked relevant facts in the minority
commissioners report.

ISSUE

whether the titles issued in thename of CLT IS valid

RULING

With respect to G.R. No. 123346, the Court upheld thevalidity of the
trial courts adoption of the commissioners majority report aspart of
the decision. The Court pointed out that the titles of respondents inall
three cases were derived from OCT No. 994 of the Registry of Deeds
ofCaloocan City registered on 19 April 1917. The Manotoks filed their
respectivemotions for reconsideration. On 5 June 2006, the cases were
elevated to theCourt en banc. In the Manotok petition, CLT
hadoriginally filed a complaint for annulment of the titles in the name
of theManotoks, alleging that it was the registered owner of Lot 26 of
the MaysiloEstate. It is evident from all three titles CCLTs, Hipolitos
andDimsonsthat the properties they purport to cover were
originally registeredon the 19th day April 1917 in the Registration
Book of the Office of theRegister of Deeds of Rizal. As earlier
established, there is no such OCT No.994 originally registered on 19
April 1917. None of these three titles can beaccorded recognition
simply because the original title commonly referred totherein never
existed. To conclude otherwise would constitute deliberatedisregard of
the truth. These titles could be affirmed only if it can be proventhat
OCT No. 994 registered on 19 April 1917 had actually existed. CLT and
theDimsons were given the opportunity to submit such proof before
this Court, butthey did not. In fact, CLT has specifically manifested that
the OCT No. 994they concede as true is also the one which the Office
of Solicitor Generalsubmitted as true, and that is OCT No. 994 issued
on 3 May 1917.Given thisessential clarification, there is no sense in
affirming the 2005 Decision whichsustained the complaints for
annulment of title and/or recovery of possessionfiled by CLT and the
Dimson when their causes of action are both founded on aninexistent
mother title.

From these premises, the Court is able to make the followingbinding


conclusions. First, there is only one OCT No. 994. As it appears on
therecord, that mother title was received for transcription by the
Register ofDeeds on 3 May 1917, and that should be the date which
should be reckoned asthe date of registration of the title. It may also
be acknowledged, as appearson the title, that OCT No. 994 resulted
from the issuance of the decree ofregistration on 17 April 1917,

although such date cannot be considered as thedate of the title or the


date when the title took effect. Second,any title thattraces its source to
OCT No. 994 dated 17 April 1917 is void, for such mothertitle is
inexistent. This error alone is, in fact, sufficient to invalidate theDimson
and CLT claims over the subject property if singular reliance is
placedby them on the dates appearing on their respective titles.

The land becomes a registered landonly upon the transcription of the


decree in the original registration book bythe register of deeds, the
date and time of such transcription being set forthin the process and
certified to at the foot of each entry or certificate oftitle. The issuance
of the original and owners duplicate certificates arebasic for the valid
existence of the title. Issuance of additional copies arepermissive and
their non-existence does not affect the status of title. Acertificate of
title is deemed as regularly issued with the issuance of theoriginal copy
and owners duplicate

7L%

20(Douglas)

21 (Douglas)

22(Douglas)

23(Cha)

24(Cha)

25(Cha)

26(Cha)

27(Cha)

28 (Cha)
29(Cha)

g
g
g
g

g
g

A writ of demolition is but a compliment of the writ of possession


and may be issued by a special order of the court.Gawaran v. IAC
162 SCRA 154
Facts:
Petitioners oppose the application for registration and
confirmation of title over Lot 2, PSU-173975 situated in Digman,
Bacoor, Cavite, on which petitioners had their residential house and a
"camarin." The trial court awarded the lot to the petitioners. The
respondents appealed to the CA which reversed the decision of the
trial court and confirmed the ownership of said Lot No. 2 in the names
of private respondents. On motion of private respondents, the
respondent RTC, on March 19, 1985, issued the questioned writ of
possession with the complimentary directive for the oppositors to
dismantle and remove their building and/or structure from Lot No. 2
under pain of demolition and to vacate the premises in favor of private
respondents within thirty (30) days. The petitioners appealed but the
petition was dismissed, and an order for the issuance of a decree in
favor of private respondents was issued.Issue:
WON the court is correct in issuing the writ of possession with a
special order of demolitionHeld:
The SC held that the respondent appellate court committed no
reversible error in holding that the writ of possession issued by the trial
court and it is a necessary consequence of the adjudication of
ownership and the corresponding issuance of the Original Certificate of

Title. In a registration case, the judgment confirming the title of the


applicant and ordering its registration in his name necessarily carries
with it the delivery of possession which is an inherent element of the
right of ownership. Hence, a writ of possession may be issued not only
against the person who has been defeated in a registration case, but
also against any one unlawfully and adversely occupying the land or
any portion thereof during the registration proceedings up to the
issuance of the final decree. It is the duty of the registration court to
issue said writ when asked for by the successful party.
g
g
g
g

9/7, 1:51pmMush TapaIn lieu of demolition, award of damages


proper.Ayala Corp. v. Ray Burton Development Corp.
294 SCRA 48Facts:
Petitioner Ayala Corporation (AYALA) is the owner of the Ayala
estate located in Makati City. The said estate was originally a raw land
which was subdivided for sale into different lots devoted for residential,
commercial and industrial purposes. On March 20, 1984, Karamfil
Import-Export Company Ltd. (KARAMFIL) bought from AYALA a piece of
land identified as Lot 26, Block 2 consisting of 1,188 square meters,
located at what is now known as H.V. de la Costa Street, Salcedo
Village, Makati City. The said land is now the subject of this case. The
transaction was documented in a Deed of Sale of even date, which
provides, among others, that the vendee would comply with certain
special conditions and restrictions on the use or occupancy of the land.
On February 18, 1988, KARAMFIL sold the lot to Palmcrest
Development and Realty Corporation (PALMCREST) under a Deed of
Absolute Sale of even date. AYALA gave its written conformity to the
sale
but
reflecting
in
its
approval
the
same
special
conditions/restrictions as in the previous sale.
PALMCREST in turn sold the lot to Ray Burton Development
Corporation (RBDC), now respondent, on April 11, 1988, with the
agreement that AYALA retains possession of the Owners Duplicate
copy of the title until a building is erected on said parcel of land in
accordance with the requirements and/or restrictions of AYALA. As in
the KARAMFIL-PALMCREST transaction, AYALA gave its conformity to
the sale, subject to RBDCs compliance with the special
conditions/restrictions which were annotated in the deed of sale.
Sometime in June of 1989, RBDC submitted to AYALA for approval a set
of architectural plans for the construction of a 5-storey office building
on the subject lot. The building was to be known as Trafalgar Tower
but later renamed Trafalgar Plaza. Since the building was well within
the 42-meter height restriction, AYALA approved the architectural
plans.Early in June of 1990, RBDC made another set of building plans
for Trafalgar Plaza and submitted the same for approval, this time to

g
g
g
g

the Building Official of the Makati City Engineers Office, not to AYALA.
In these plans, the building was to be 26-storey high, or a height of
98.60 meters, with a total gross floor area of 28,600 square meters.
After having obtained the necessary building permits from the City
Engineers Office, RBDC began to construct Trafalgar Plaza in
accordance with these new plans.Issue: WON award of damages is
proper, in lieu of demolitionHeld:
Yes.In sum, there is more than ample evidence on record
pinpointing RBDCs violation of the applicable FAR restrictions in the
Consolidated and Revised Deed Restrictions (CRDRs) when it
constructed the 27-storey Trafalgar Plaza. The prayer of petitioner is
that judgment be rendered, among others, ordering Ray Burton to
comply with its contractual obligations in the construction of Trafalgar
Plaza by removing or demolishing the portions of areas thereof
constructed beyond or in excess of the approved height, as shown by
the building plans submitted to, and approved by, Ayala, including any
other portion of the building constructed not in accordance with the
said building plans.However, the record reveals that construction of
Trafalgar Plaza began in 1990, and a certificate of completion thereof
was issued by the Makati City Engineers Office per ocular inspection
on November 7, 1996. Apparently Trafalgar Plaza has been fully built,
and we assume, is now fully tenanted. The alternative prayers of
petitioner under the CRDRs, i.e., the demolition of excessively built
space or to permanently restrict the use thereof, are no longer feasible.
Thus, we perforce instead rule that RBDC may only be held
alternatively liable for substitute performance of its obligations the
payment of damages. Ray Burton Development Corporation acted in
bad faith in constructing Trafalgar Plaza in excess of the applicable
restrictions upon a double submission of plans and exercising deceit
upon both AYALA and the Makati Engineer's Office, and thus by way of
example and correction, should be held liable to pay AYALA exemplary
damages in the sum of P2,500,000.00.

9/7, 1:52pmMush TapaJUDICIAL CONFIRMATION OF IMPERFECT


TITLERepublic v. Diloy
G.R. No. 174633, 26 August 2008, 563 SCRA 413Unless the land
has been classified as alienable and disposable, the rules on the
confirmation of imperfect title shall not apply.Facts:
Before this Court is a Petition for Review on Certiorari under Rule
45 of the 1997 Revised Rules of Civil Procedure seeking to set aside the
Decision and Resolution of the Court of Appeals. The CA Decision
denied the appeal filed before it by the Republic of the Philippines
(Republic) and affirmed the Decision of the 2nd MCTC of SilangAmadeo, Silang, Cavite, granting the application for registration of title

filed before it by the respondent Gregoria L. Diloy over a parcel of land


located in Barangay Dagatan, Municipality of Amadeo, Province of
Cavite, covering an area of 22,249 square meters. The Resolution
denied the MR filed by the Republic.The Republic persistently argues
that the respondent's Application for Registration of Title should have
been denied because the latter failed to comply with the period of
possession required by law, i.e., Section 14 of Presidential Decree No.
1529.30 The Republic reveals that the subject property was only
declared alienable and disposable on 15 March 1982 per Forestry
Administration Office (FAO) No. 4-1650. From 1982 when the property
was declared alienable and disposable to 1997, the respondent had
only been in adverse possession of the subject property for a period of
15 years. Thus, there was no compliance with Section 14, PD No. 1529
because the subject property was not yet alienable and disposable on
12 June 1945, and respondent's possession lacked the required number
of years (30 years) for her to acquire the same through prescription.
Hence, respondent did not acquire an imperfect title, which may be
confirmed through a judicial proceeding.Issue: WON the respondent
has acquired a registrable titleHeld:
The Petition is meritorious.While this Court agrees with the lower
courts that, indeed, respondent's possession of the subject property
was open, continuous, exclusive and notorious, however, we hold that
respondent failed to prove that she or her predecessors-in-interest
were already in possession of the subject property under a bona fide
claim of ownership since 12 June 1945 or earlier, which is the
reckoning period specifically provided in Section 14(1) of PD No. 1529.
As can be gleaned from the records, respondent's possession of the
subject property started only in the year 1979 when her mother
executed a Deed of Absolute Sale over the same in her favor. There
was also no showing that her predecessors-in-interest had already
been in possession or had already exercised acts of ownership over the
subject property since 12 June 1945 or prior thereto, as her
predecessors-in-interest declared the subject property for taxation
purposes only in the year 1948. What was clearly established by the
respondent was possession of the subject property by her
predecessors-in-interest beginning 1948, which was short of three
years from 12 June 1945. What is more telling is that the subject
property became alienable and disposable only on 15 March 1982.
Prior to its declaration as alienable land in 1982, any occupation or
possession thereof could not be considered in the counting of the 30year possession requirement. The period of possession by the
respondent of the subject property cannot be considered to have
started in 1979, when the same was conveyed to her by her mother.
Neither can her possession of the subject property be tacked to that of
her predecessors-in-interest, even if they had occupied and were in
possession of the same since 1948, because during those periods, the

subject property had not yet been classified as alienable and


disposable land capable of private appropriation. Possession of the
subject property could only start to ripen into ownership on 15 March
1982, when the same became alienable and disposable. Any period of
possession prior to the date when the subject lot was classified as
alienable and disposable is inconsequential and should be excluded
from the computation of the period of possession; such possession can
never ripen into ownership and, unless the land has been classified as
alienable and disposable, the rules on the confirmation of imperfect
title shall not apply thereto. The adverse possession which may be the
basis of a grant of title or confirmation of an imperfect title refers only
to alienable or disposable portions of the public domain. There can be
no imperfect title to be confirmed over lands not yet classified as
disposable or alienable. In the absence of such classification, the land
remains unclassified public land until released therefrom and open to
disposition. Possession of the land by the respondent under the
circumstances, whether spanning decades or centuries, can never
ripen into ownership.
g
g
g
g

g
g
g

9/7, 1:52pmMush TapaTan v. Republic


G.R. No. 177797, 04 December 2008, 573 SCRA 89Facts:
This case is a Petition for Review on Certiorari under Rule 45 of
the 1997 Revised Rules of Civil Procedure seeking to reverse and set
aside the Decision and Resolution of the CA. In its assailed Decision,
the appellate court reversed and set aside the Decision of the RTC of
Misamis Oriental, 10th Judicial Region, Branch 39, Cagayan de Oro City,
in LRC Case No. N-2000-055, and ordered petitioners, spouses Pedro
and Nena Tan (spouses Tan), to return the parcel of land known, with
an area of 215,698 square meters, located in Calingagan, Villanueva,
Misamis Oriental (subject property) to respondent, Republic of the
Philippines (Republic). In its assailed Resolution, the appellate court
denied the spouses Tans MR.Issue:
WON [the Spouses Tan] have been in open, continuous, exclusive
and notorious possession and occupation of the subject [property],
under a bona fide claim of acquisition or ownership, since [12 June
1945], or earlier, immediately preceding the filing of the application for
confirmation of titleHeld:
The Court rules in the negative and, thus, finds the present
Petition devoid of merit.
Through the years, Section 48(b) of the Public Land Act has been
amended several times. As amended, Section 48(b) now reads:
(b) Those who by themselves or through their predecessors-ininterest have been in open, continuous, exclusive, and notorious
possession and occupation of agricultural lands of the public domain,

g
g

g
g
g
g

under a bona fide claim of acquisition or ownership, since June 12,


1945 or earlier, immediately preceding the filing of the application for
confirmation of title except when prevented by wars or force majeure.
Those shall be conclusively presumed to have performed all the
conditions essential to a Government grant and shall be entitled to a
certificate of title under the provisions of this chapter.
Section 48(b) of the Public Land Act, as amended by PD No.
1073, presently requires, for judicial confirmation of an imperfect or
incomplete title, the possession and occupation of the piece of land by
the applicants, by themselves or through their predecessors-ininterest, since 12 June 1945 or earlier. This provision is in total
conformity with Section 14(1) of the Property Registration Decree
heretofore cited.
As the law now stands, a mere showing of possession for thirty
years or more is not sufficient. It must be shown, too, that possession
and occupation had started on 12 June 1945 or earlier.
For failure of the Spouses Tan to satisfy the requirements
prescribed by Section 48(b) of the Public Land Act, as amended, this
Court has no other option but to deny their application for judicial
confirmation and registration of their title to the subject property. Much
as this Court wants to conform to the States policy of encouraging and
promoting the distribution of alienable public lands to spur economic
growth and remain true to the ideal of social justice, our hands are tied
by the laws stringent safeguards against registering imperfect titles.

9/7, 1:53pmMush TapaRepublic v. Credit Corp.


G.R. No. 173088, 25 June 2008, 555 SCRA 315Facts:
Respondent Imperial Credit Corporation is a corporation duly
organized and existing under the laws of the Philippines. On 07 March
1966, respondent purchased from a certain Jose Tajon a parcel of land
situated in Barrio Colaique (now Barangay San Roque), Antipolo City,
Rizal for the sum of P17,986.00 as evidenced by a Deed of Sale with
Mortgage. Upon full payment of the balance of P1,909.00 through
judicial consignation, ownership of the property was consolidated in
the name of respondent and the mortgage constituted thereon
released in December 1997. The property was thereafter privately
surveyed under PSU-178075 and approved on 25 January 2000.
On 14 February 2000, respondent filed before the RTC of Antipolo
City an application for registration of a parcel of land, as shown on Plan
PSU-178075 containing an area of 8,993 square meters. The
application was docketed and raffled off to Branch 74 of said RTC. The
application alleged, among others, that respondent "subrogated former
owner Jose Tajon, who has been in open, continuous, exclusive and
notorious possession and occupation of the parcel of land, being a part

g
g

g
g

g
g
g

g
g

of the alienable and disposable lands of the public domain, under a


bona fide claim of ownership since 12 June 1945, by virtue of Deed of
Sale with Mortgage executed on 07 March 1966,
After respondent presented evidence establishing the jurisdiction
facts, the RTC issued an order of general default against the whole
world allowing respondent to present its evidence ex parte.
At the hearing, Ricardo Santos, respondents legal researcher
and duly authorized attorney-in-fact, testified on the fact of
respondents actual possession through its caretaker, Teodisia Palapus,
who had been overseeing said property since its acquisition from Jose
Tajon. Palapus also corroborated Santos testimony and added that
except for some trespassers, no one else had laid possessory claim on
the property. Aside from the transfer documents, the other
documentary evidence submitted consisted of a 1993 tax declaration,
the tracing cloth plan, survey description, a certification from the Land
Management Sector in lieu of the geodetic engineers certificate and
the report by the Community Environment and Natural Resources
Office that the property falls within the alienable and disposable zone.
On 21 November 2002, the RTC rendered judgment granting
respondents application for registration.
Petitioner Republic of the Philippines, through the Office of the
Solicitor General (OSG), seasonably appealed from the RTCs Decision
to the CA, contending that respondent failed to present
incontrovertible evidence that respondent and its predecessor-ininterest have been in open continuous, exclusive and notorious
possession and occupation of the property since 12 June 1945 or
earlier.
The CA rendered a Decision on 02 June 2006, dismissing the
appeal by the OSG.
Issue:
Although petitioner concedes that respondent was able to show
that the land applied for has been declassified from the forest or
timber zone and is an alienable public agricultural land, respondents
evidence failed to satisfy the requirement under paragraph (1) of
Section 14, P.D. No. 1529, that is, respondents possession and
occupation of the property for the length of time and in the manner
required by law. Held:
The petition is meritorious.
Under the Regalian doctrine, the State is the source of any
asserted right to ownership of land. This is premised on the basic
doctrine that all lands not otherwise appearing to be clearly within
private ownership are presumed to belong to the State. Any applicant
for confirmation of imperfect title bears the burden of proving that he
is qualified to have the land titled in his name.
The reckoning date under the Public Land Act for the acquisition
of ownership of public lands is June 12, 1945 or earlier, and that

g
g
g

evidence of possession from that date or earlier is essential for a grant


of an application for judicial confirmation of imperfect title.
While a tax declaration by itself is not sufficient to prove
ownership, it may serve as sufficient basis for inferring possession.

9/7, 1:54pmMush TapaMalabanan v. Republic


G.R. No. 179987, 29 April 2009The SC en banc upheld the ruling
of the CA denying the application of the petitioners for the registration
of a parcel of land situated in Barangay Tibig, Silang, Cavite on the
ground that they had not established by sufficient evidence their right
to the registration in accordance with either Section 14(1) or Section
14(2) of PD No. 1529 (Property Registration Decree).Facts:
On 20 February 1998, Mario Malabanan filed an application for
land registration covering a parcel of land in Silang Cavite. Malabanan
claimed that he had purchased the property from Eduardo Velazco, and
that he and his predecessors-in-interest had been in open, notorious,
and continuous adverse and peaceful possession of the land for more
than thirty years. The application was raffled to the RTC CaviteTagaytay City. The OSG duly designated the Jose Velazco, Jr. to appear
on behalf of the State. Apart from presenting documentary evidence,
Malabanan himself and his witness, Aristedes Velazco, who testified
that the property was originally belonged to a twenty-two hectare
property owned by his great-grandfather, Lino Velazco.Issues:
1.
In order that an alienable and disposable land of the public
domain may be registered under Section 14(1) of PD No. 1529,
otherwise known as the Property Registration Decree, should the land
be classified as alienable and disposable as of June 12, 1945 or is it
sufficient that such classification occur at any time prior to the filing of
the applicant for registration provided that it is established that the
applicant has been in open, continuous, exclusive and notorious
possession of the land under a bona fide claim of ownership since June
12, 1945 or earlier?2. For purposes of Section 14(2) of the Property
Registration Decree may a parcel of land classified as alienable and
disposable be deemed private land and therefore susceptible to
acquisition by prescription in accordance with the Civil Code?3. May a
parcel of land established as agricultural in character either because of
its use or because its slope is below that of forest lands be registrable
under Section 14(2) of the Property Registration Decree in relation to
the provisions of the Civil Code on acquisitive prescription?4.
Are
petitioners entitled to the registration of the subject land in their
names under Section 14(1) or Section 14(2) of the Property
Registration Decree or both?Held:
The Petition is denied.(1) In connection with Section 14(1) of the
Property Registration Decree, Section 48(b) of the Public Land Act

recognizes and confirms that those who by themselves or through


their predecessors in interest have been in open, continuous,
exclusive, and notorious possession and occupation of alienable and
disposable lands of the public domain, under a bona fide claim of
acquisition of ownership, since June 12, 1945 have acquired
ownership of, and registrable title to, such lands based on the length
and quality of their possession.(a) Since Section 48(b) merely requires
possession since 12 June 1945 and does not require that the lands
should have been alienable and disposable during the entire period of
possession, the possessor is entitled to secure judicial confirmation of
his title thereto as soon as it is declared alienable and disposable,
subject to the timeframe imposed by Section 47 of the Public Land Act.
(b) The right to register granted under Section 48(b) of the Public Land
Act is further confirmed by Section 14(1) of the Property Registration
Decree.(2) In complying with Section 14(2) of the Property Registration
Decree, consider that under the Civil Code, prescription is recognized
as a mode of acquiring ownership of patrimonial property. However,
public domain lands become only patrimonial property not only with a
declaration that these are alienable or disposable. There must also be
an express government manifestation that the property is already
patrimonial or no longer retained for public service or the development
of national wealth, under Article 422 of the Civil Code. And only when
the property has become patrimonial can the prescriptive period for
the acquisition of property of the public dominion begin to run.(a)
Patrimonial property is private property of the government. The person
acquires ownership of patrimonial property by prescription under the
Civil Code is entitled to secure registration thereof under Section 14(2)
of the Property Registration Decree.(b) There are two kinds of
prescription by which patrimonial property may be acquired, one
ordinary and other extraordinary. Under ordinary acquisitive
prescription, a person acquires ownership of a patrimonial property
through possession for at least ten (10) years, in good faith and with
just title. Under extraordinary acquisitive prescription, a persons
uninterrupted adverse possession of patrimonial property for at least
thirty (30) years, regardless of good faith or just title, ripens into
ownership.It is clear that the evidence of petitioners is insufficient to
establish that Malabanan has acquired ownership over the subject
property under Section 48(b) of the Public Land Act. There is no
substantive evidence to establish that Malabanan or petitioners as his
predecessors-in-interest have been in possession of the property since
12 June 1945 or earlier. The earliest that petitioners can date back
their possession, according to their own evidencethe Tax
Declarations they presented in particularis to the year 1948. Thus,
they cannot avail themselves of registration under Section 14(1) of the
Property Registration Decree.Neither can petitioners properly invoke
Section 14(2) as basis for registration. While the subject property was

declared as alienable or disposable in 1982, there is no competent


evidence that is no longer intended for public use service or for the
development of the national evidence, conformably with Article 422 of
the Civil Code. The classification of the subject property as alienable
and disposable land of the public domain does not change its status as
property of the public dominion under Article 420(2) of the Civil Code.
Thus, it is insusceptible to acquisition by prescription.
g
g

g
g

VAGILIDAD v. VAGILIDAD G.R. No. 161136


(Part 1)Facts:
That on January 20, 1987, the heirs of ZOILO, Loreto Labiao
(hereafter LORETO), Efren Labiao (hereafter EFREN) and Priscilla
Espanueva (hereafter PRISCILLA) executed an Extrajudicial Settlement
of Estate adjudicating the entire Lot No. 1253, covering 4,280 square
meters, to LORETO. That On January 29, 1987, Transfer Certificate of
Title (TCT) No. T-16693 was issued in favor of LORETO, EFREN and
PRISCILLA, but on even date, TCT No. T-16693 was cancelled and TCT
No. T-16694, covering the said property, was issued in the name of
LORETO alone.That on September 21, 1988, Gavino Vagilidad Jr
( hereafter GABINO JR.] paid real estate taxes on the land he bought
from LORETO as per Tax Declaration No. 1038 where the property was
specified as Lot No. 1253-B. GABINO JR. thereafter sold the same lot to
Wilfredo Vagilidad (hereafter WILFREDO) as per Deed of Absolute Sale
dated December 7, 1989. On even date, Deed of Absolute Sale of a
Portion of Land involving the opt-described property was also executed
by LORETO in favor of WILFREDO. The aforementioned deeds, which
were both executed on December 7, 1989 [and] notarized by Atty.
Warloo Cardenal[,] [appear] to have been given the same entry
number in his notarial books as both contained the designation
Document No. 236, Page No. 49, Book No. XI, Series of 1989[.]
That on February 14, 1990, the sale of Lot No. 1253-B to
WILFREDO was registered with the Registry of Deeds of the Province of
Antique under Entry No. 180425. Consequently, TCT No. T-18023,
cancelling TCT No. 16694, was issued in favor of WILFREDO pursuant to
the Deed of Absolute Sale dated December 7, 1989.That on September
29, 1995, spouses GABINO and Ma. Dorothy Vagilidad (hereafter
DOROTHY), as plaintiffs, filed a Complaint for Annulment of Document,
Reconveyance and Damages, with the Regional Trial Court of Antique,
Sixth Judicial Region, Branch 11, against spouses WILFREDO and Lolita
Vagilidad (hereafter LOLITA), docketed as Civil Case No. 2825. The
plaintiffs claimed that they are the lawful owners of Lot No. 1253-B
which was sold to him by LORETO in 1986. They alleged that [GABINO
JR.] is a nephew of defendant WILFREDO. They likewise raised that
when GABINO SR. died, defendant WILFREDO requested GABINO JR. to
transfer the ownership of Lot No. 1253-B in defendant WILFREDOs

name for loaning purposes with the agreement that the land will be
returned when the plaintiffs need the same. They added that, pursuant
to the mentioned agreement, plaintiff GABINO JR., without the
knowledge and consent of his spouse, DOROTHY, executed the Deed of
Sale dated December 7, 1989 in favor of defendant WILFREDO
receiving nothing as payment therefor. Issue:Whether the Deed of Sale
executed in favor of the petitioners ( spouses Gabino Jr and Dorothy
Vagilidad) validly convey Lot No. 1253-B to them.Held:the evidence
preponderates in favor of the petitioners ( spouses Gabino Jr and
Dorothy Vagilidad). The Deed of Sale executed by LORETO did validly
convey Lot No. 1253-B to GABINO, JR.First, the Deed of Absolute Sale of
Portion of Land dated December 7, 1989 between LORETO and
WILFREDO is tainted with blatant irregularities. Second, the testimony
of a disinterested witness, Febe Mabuhay, established the irregularity.
Mabuhay used to work as secretary for Atty. Cardenal and co-signed as
witness in both Deeds. Third, the testimony of Atty. Ernesto Estoya,
then Clerk of Court of the Regional Trial Court of Antique, supports the
claim that there was bad faith in the execution of the Deed of Absolute
Sale of Portion of Land. Fourth, we give credence to the testimony of
GABINO, JR. that LORETO and WILFREDO had employed the scheme to
deprive him and his wife of their lawful title to the subject property.
With these corroborating circumstances and the following irrefragable
documents on record, the evidence preponderates in favor of GABINO,
JR. One, he acquired Lot No.1253-B from LORETO on May 12, 1986[20]
by virtue of the Deed of Absolute Sale. Two, the Bureau of Internal
Revenue issued a Certification, also on May 12, 1986, for the
exemption from the payment of capital gains tax when LORETO sold to
him the subject parcel. Three, GABINO, JR. paid the real estate tax on
the subject parcel in 1987. Four, he filed a Petition for the Surrender of
LORETOs title on July 31, 1987 so he could transfer the title of the
property in his name.
g
g
g
g

9/7, 9:08pmMelodia Lawangen DagasenRepublic v. Marcos


No. L-32941 July 31, 1973, 52 SCRA 238
(Part 3)Issue:
Whether or a not the respondent judge has the power to reopen
Civil Registration Case No. 1 of the Court of Fisrt Instance of Baguio
establishing the Baguio Townsite Reservation in favor of the
registration of the private respondents.Facts:
That the respondent judge at the instance of private
respondents, did reopen Civil Registrartion Case No. 1 of the Court of
Fisrt Instance of Baguio establishing the Baguio Townsite Reservation,
promulgated as far back as November 13, 1922, thus enabling private
respondents the registration of an area of 74,017 square meters inside

the Camp John Hay Leave and Recreation Center.That on November 9,


1968, the respondent judge ordered in favor of private respondent for
the registration of the aforesaid area.That on August 22, 1969, the
Solicitor-General filed a motion to annul the decision based on the lack
of jUrisdiction over the subject matter of the proceedings as the land in
question is a part of a duly established military reservation.That on
December 8, 1969, the respondent judge denied the motion. His
decision was impressed by the claim that the private respondents had
been in the possession " since the Spanish regime," and thus within
the protection on the words annotated on all survey plans of Camp
John Hay, to wit: " subject to prior and existing private rights."Held.The
respondent judge was without power to reopen Civil Registration Case
No. 1. RA No. 931 is quiet explicit, only persons ' claiming title to
parcels of lands that have been the subject of cadastral proceedings'
are granted the right to petition for the re opening thereof if the other
conditions named therein are successfully met. The lots now disputed
was reserved for naval purposes dated back as far as October 10, 1910
through an executive order issued by then President William Taft and
thus could not have been the object of the cad astral proceeding,
which was decided on November 13, 1922.What is more,
it is
undeniable that the land in question, being a part of a duly established
military camp or reservation, cannot be thus ordered registered in
favor of private respondents.Wherefore, the writ of certiorari is granted
annulling and settling aside the decision of the respondent judge.
g
g
g
g
g

g
g

9/7, 9:19pmMelodia Lawangen Dagasen(Part5)


Sec. Of DENR vs. Yap, GR No. 167707, 8 October 2008, 568 SCRA
164Facts:
That on November 10, 1978, then President Ferdinand Marcos
issued Proclamation No. 1801 declaring Boracay Island, among other
islands, caves and peninsulas in the Philippines, as tourist zones and
marine reserves under the administration of the Philippine Tourism
Authority (PTA). President Marcos later approved the issuance of PTA
Circular 3-82 dated September 3, 1982, to implement Proclamation No.
1801.
That Claiming that Proclamation No. 1801 and PTA Circular No 382 precluded them from filing an application for judicial confirmation of
imperfect title or survey of land for titling purposes, respondentsclaimants
Mayor Jose S. Yap, Jr., Libertad Talapian, Mila Y. Sumndad, and
Aniceto Yap filed a petition for declaratory relief with the RTC in Kalibo,
Aklan.

g
g

g
g

g
g

g
g

g
g
g
g
g

That in their petition, respondents-claimants alleged that


Proclamation No. 1801 and PTA Circular No. 3-82 raised doubts on their
right to secure titles over their occupied lands. They declared that they
themselves, or through their predecessors-in-interest, had been in
open, continuous, exclusive, and notorious possession and occupation
in Boracay since June 12, 1945, or earlier since time immemorial. They
declared their lands for tax purposes and paid realty taxes on them.
That Respondents-claimants posited that Proclamation No. 1801
and its implementing Circular did not place Boracay beyond the
commerce of man. Since the Island was classified as a tourist zone, it
was susceptible of private ownership. Under Section 48(b) of
Commonwealth Act (CA) No. 141, otherwise known as the Public Land
Act, they had the right to have the lots registered in their names
through judicial confirmation of imperfect titles.
That the Republic, through the Office of the Solicitor General
(OSG), opposed the petition for declaratory relief. The OSG countered
that Boracay Island was an unclassified land of the public domain. It
formed part of the mass of lands classified as public forest, which was
not available for disposition pursuant to Section 3(a) of Presidential
Decree (PD) No. 705 or the Revised Forestry Code, as amended.
That the OSG maintained that respondents-claimants reliance on
PD No. 1801 and PTA Circular No. 3-82 was misplaced. Their right to
judicial confirmation of title was governed by CA No. 141 and PD No.
705. Since Boracay Island had not been classified as alienable and
disposable, whatever possession they had cannot ripen into ownership.
April 14, 1976, the Department of Environment and Natural Resources
(DENR) approved the National Reservation Survey of Boracay
Island, which identified several lots as being occupied or claimed
by named persons.Issue:
whether Proclamation No. 1801 and PTA Circular No. 3-82 can be
deemed the positive act needed to classify Boracay Island as alienable
and disposable land
Held:
Private claimants cannot rely on Proclamation No. 1801 as basis
for judicial confirmation of imperfect title. The proclamation did not
convert Boracay into an agricultural land. There is nothing in the law
or the Circular which made Boracay Island an agricultural land. The
reference in Circular No. 3-82 to private lands and areas declared as
alienable and disposable does not by itself classify the entire island as
agricultural.
Clearly, the reference in the Circular to both private and public

lands merely recognizes that the island can be classified by the


Executive department pursuant to its powers under CA No. 141.
Therefore, Proclamation No. 1801 cannot be deemed the positive
act needed to classify Boracay Island as alienable and disposable land.
If President Marcos intended to classify the island as alienable and
disposable or forest, or both, he would have identified the specific
limits of each, as President Arroyo did in Proclamation No. 1064. This
was not done in Proclamation No. 1801.

g
The Whereas clauses of Proclamation No. 1801 also explain
the rationale behind the declaration of Boracay Island, together with
other islands, caves and peninsulas in the Philippines, as a tourist zone
and marine reserve to be administered by the PTA to ensure the
concentrated efforts of the public and private sectors in the
development of the areas tourism potential with due regard for
ecological balance in the marine environment. Simply put, the
proclamation is aimed at administering the islands for tourism and
ecological purposes. It does not address the areas alienability.
5) ANGELITA F. BUENAVENTURA and PRECIOSA F. BUENAVENTURA,
Petitioners,
vs.
REPUBLIC OF THE PHILIPPINES, Respondent.
G.R. No. 166865 March 2, 2007
Facts:
Petitioners Angelita and Preciosa filed an Application for registration of
Title on June 5, 2000 before the RTC of Paranaque City for a particular
Lot situated in San Dionisio, Paranaque. Said petitioners are heirs of
Spouses Amado Buenaventura and Irene Flores from whom they
acquired the same. The property was acquired by the Spouses
Buenaventura from the Heirs of Lazaro De Leon even before World War
II. However, it was only on January 30 1948 that the deed of sale was
executed in favor of Spouses Buenaventura. After the Execution of the
said Deed of Sale, the Spouses Buenaventura transferred the tax
declaration in their name. In 1978, the Spouses Buenaventura
transferred by way of Deed of Sale, the subject property to their
children, among who are the petitioners in the instant case.
Petitioners contend that "they and their predecessors-in-interest
acquired title to the said parcel of land thru inheritance, transfer and
possession as owners of the same since time immemorial and/or within
the period provided for by law". To buttress their claim, the petitioners
presented 5 witnesses. They likewise presented and identified several
documents to establish further the following: 1.) petitioners' fee simple

title over the property; 2.) the nature of the possession and occupation
of the property; 3.) its classification as part of the alienable zone of the
government; and 4.) the improvements introduced thereon and the
taxes paid on the subject property.
The trial Court, on 29, October 2001 rendered a decision granting the
application for registration of title.
The Republic appealed the case with the Court of Appeals. Said court
reversed the trial courts decision and explained that the petitioners
failed to show possession and occupation of the subject property under
a bona fide claim of ownership since June 12, 1945 or earlier as
provided for in Section 14(1) of the Property Registration Decree. It was
stated in the decision of the Court of Appeals that the petitioners and
their predecessors-in-interest only had possession of the subject
property in 1948.
ISSUE:
Can the petitioners validly register the land under their names even
though their possession commenced only after June 12, 1945?
Ruling:
Yes. Even if the possession of alienable lands of the public domain
commenced only after June 12, 1945, application for registration of
said property is still possible by virtue of Section 14(2) of the Property
Registration Decree which speaks of prescription.
Under the Civil Code, Prescription is one of the modes of acquiring
ownership. Article 1137 of the Civil Code states that Ownership and
other real rights over immovables also prescribe through uninterrupted
adverse possession thereof for thirty years, without need of title or of
good faith".
In the present case, while petitioners' possession over the subject
property can be reckoned only on January 3, 1968, the date when
according to evidence, the subject property became alienable, they
can still have the said property in their names by virtue of Section 14
(2) of the Property Registration Decree.
The records reveal that they were in possession of the property for 32
years, reckoned from the year 1968, when the subject property was
finally declared alienable by the DENR to the time they filed an
application for registration of title on June 5, 2000. Petitioner's
possession of the subject property since 1968 has been characterized
as open, continuous, exclusive and notorious possession and
occupation in the concept of an owner. Furthermore, the voluntary

declaration of a piece of property for taxation purposes and the


presentation of their realty tax payments of said property are good
indicia of possession in concept of an owner, for no one in his right
mind would be paying taxes for a property that is not in his actual or
constructive possession. Such an act strengthens one's bona fide claim
of acquisition of ownership.
Case No. 24: Ching v. Malaya, supra
Facts: This is a petition for certiorari under Rule 65 of the Rules of
Court. The petitioners alleged in their complaint for ejectment that the
private respondents had forced their way into the disputed premises
and refused to vacate the same despite repeated demands. The
petitioners claim that they own the property by a virtue of valid sale
while the respondents claim that the property belonged to them by
right of inheritance. The municipal court rendered judgment ordering
the private respondents to vacate the disputed property. On appeal,
this decision was set aside by the respondent judge claiming that the
municipal court had no competence to resolve the case.
Issue: Whether the municipal court had the jurisdiction over the case.
Held: Yes. It is settled that the mere assertion of ownership by the
defendant in an ejectment case will not oust the municipal court of its
summary jurisdiction. "The mere circumstance that proof of title, or
evidence of ownership, had been introduced during the trial before the
Municipal Court would not deprive said court of jurisdiction to rule on
the question of who had the prior physical possession." There is one
exception, however, and that is where it appears during the trial that,
by the nature of the evidence presented, the issue of possession
cannot be decided without deciding the issue of ownership. In such a
case, the jurisdiction of the municipal court is lost and the action
should be dismissed. After examining the facts of this present case, the
Court finds that it does not come under the exception to the rule.
Finally, the fact that the petitioners themselves adduced evidence of
ownership over the property in question did not, as claimed, have the
effect of divesting the municipal court of its jurisdiction.
Acosta vs. Salazar
Facts:
On November 10, 1985, respondents Trinidad and Aniceta Salazar filed
a petition for the cancellation of the entries of the late spouses, Juan
Soriano and Vicenta Macaraeg, annotated at the back of the Original
Title of Certificate (OCT) No. 40287 on the ground that these entries

have no consolidation rights in the Registry of Deeds (RD) of Tarlac and


that the Transfer Certificate Title (TCT) No. 9297 which supposedly
cancelled the said OCT is non-existent. The RTC Branch 63 of Tarlac
granted their petition and ordered the RD to recall all titles, to cancel
all tax declarations of the said entries, and to direct the owners of the
said property to appear before the court for objections. The RD
complied with the RTCs orders and transferred the entries of the late
spouses under the Salazars names.
With this, the petitioners, together with the 27 titleholders of the
disputed property filed a complaint contending that they had acquired
their titles in good faith and that the court committed an abuse of
discretion acting as a land registration court because it had no
jurisdiction over issues of ownership. The Salazars then filed for
quieting of title with the RTC Branch 64 of Tarlac but was denied and
ruled in favour of the herein petitioners contending that the RTC
Branch 63 had no jurisdiction over the matter and that the TCT
registered under their names was null and void.
Unsatisfied, the Salazars then appealed to the Court of Appeals which
ruled in their favour contending that the RTC Branch 64 of Tarlac had
no authority to dismiss RTC Branch 63 of Tarlacs decision because it is
a court of equal rank. Moreover, CA also struck down the arguments
raised by the titleholders and contended that the proceeding was a
land registration proceeding, which is an action in rem, and requires no
personal notice to the owners or claimants of the land in order to vest
the court with jurisdiction.
Issue:
Whether the Salazars filing of action with the RTC Branch 63 of Tarlac
is valid
Held:
No. While it is true that a proceeding regarding a registration of land is
a proceeding in rem, requiring no personal notice to the owners to
acquire jurisdiction over the case, the proceedings instituted by the
Salazars with the RTC Branches 63 and 64 of Tarlac can hardly be
considered as actions in rem. This is because the petition for
cancellation of entries should have been directed against specific
persons: namely, the heirs of Juan Soriano and the successors-ininterest who have acquired different portions of the property. Since no
indispensable parties were included in the petition and were not
impleaded by the Salazars, the said petitions with the RTC are not
quasi in rem. Moreover, the absence of indispensable parties over the

issue should have been a ground for the RTC to dismiss the case.
Furthermore, the property in dispute is under the protection of the
Torrens system and the Salazars dont have the right to impugn its
legality. Rarely will the court allow another person to attack the validity
and indefeasibility of a Torrens certificate, unless there is compelling
reason to do so and only upon a direct action filed in court proceeded
in accordance with law.
With this, the Court of Appeals decision is set aside and the the RTC
Branch 64 of Tarlacs decision is reinstated.
G.R. No. L-21024 July 28, 1969
CENON MATEO, petitioner-appellant,
vs.
HON. FLORENCIO MORENO, in his capacity as SECRETARY OF PUBLIC
WORKS AND COMMUNICATIONS, defendant-appellee.
FACTS: Sometime in 1959 a number of residents of Guiguinto, Bulacan,
sent a letter-complaint to the Highway District Engineer of that
province asking that the Sapang Cabay, a public navigable stream,
which had been blocked by means of dikes and dams and converted
into fishponds, be ordered reopened and restored to its original
condition. The letter was referred to the Secretary of Public Works and
Communications, who caused an investigation to be conducted
pursuant to Republic Act No. 2056. Acting on the report which the
investigator submitted to him, the Secretary rendered his decision on
August 10, 1959, finding that the Sapang Cabay was a public navigable
stream and ordering Cenon Mateo, the herein petitioner-appellant, who
had in the meantime acquired the property inside which the said creek
is situated, to remove the dikes and dams therein constructed within
thirty days from notice; otherwise they would be removed at his
expense. Mateo moved to reconsider but was turned down, whereupon
he filed the basic petition to restrain the respondent Secretary from
enforcing his decision. The petition, as already stated, was dismissed
by the Court a quo. The certification of the appeal to us was upon
motion of both parties in view of the constitutional question involved.
ISSUE: whether or not Republic Act No. 2056 is unconstitutional
because it unduly delegates judicial power to the Secretary and
unlawfully deprives the appellant and others similarly situated of their
property without due process of law
HELD: The constitutionality of the aforesaid statute has been upheld by
this Court in Lovina vs. Moreno, G.R. No. L-17821, November 29, 1963,
shortly before the present appeal was submitted for decision. That

case held, furthermore, that the absence of any mention of a navigable


stream within a property covered by a certificate of title does not
preclude a subsequent investigation and determination of its existence
nor make it private property of the title holder; that the findings of fact
made by the Secretary of Public Works and Communications should be
respected in the absence of illegality, error of law, fraud or imposition,
as long as such findings are supported by substantial evidence; and
that the ownership of a navigable stream or of the bed thereof is not
subject to acquisitive prescription.
Rubie:
LEDESMA VS. VILLASEOR
13 SCRA 494
FACTS:
Felix Villaseor is the special administrator of the estate of his
deceased father, Eusebio Villaseor. He filed a petition with the RTC
(CFI) of Negros Occidental to prohibit the Register of Deeds of the
same province to register a Deed of Sale of two lots made by his father
in favor of Jose Ledesma. This was because said Deed of Sale was
allegedly fictitious and the fathers signature was forged. The court
then issued a writ of preliminary injunction. Thereafter, the RTC
eventually dismissed the petition and lifted the writ on the ground that
Ledesma had not been impleaded as a party-defendant and he only
intervened in the case.
Two days later, Ledesma filed his own petition asking that the Register
of Deeds be ordered to register the aforementioned Deed of Sale on
the ground that the earlier case was already dismissed and the writ
was dissolved. On the same day, the court granted the petition without
notice to the Register of Deeds and to Villaseor and issued the order
for registration.
The Register of Deeds hence cancelled the certificates of title and
issued new ones in Ledesmas name. Villaseor moved for
reconsideration but was denied. Hence, present action.
ISSUE:
Whether the RTC erred in issuing the order for registration.
RULING:
Yes.
We are of the opinion that the lower court did commit the error
attributed to it.
The court had no authority to issue the orders after just 2 days after
lifting the injunction and dismissing the civil case without notice to the
Register of Deeds or appellant considering that it was not yet final.
The least that the court could have done was to afford appellant proper

notice and hearing as it knew of the pendency of that case and that
the relief sought therein was precisely to prevent registration.
It is one thing for the Register of Deeds, in the exercise of his
ministerial duties under the law, to register an instrument which in his
opinion is registrable, and quite another thing for the court itself to
order the registration. The former does not contemplate notice to and
hearing of interested parties such as are required in a judicial
proceeding nor carry with it the solemnity and legal consequences of a
court judgment. The court a quo, in anticipating the action of the
Register of Deeds, unnecessarily took the matter out of his hands and
at the same time preempted the question of registration still pending
in the civil action filed by appellant.
The orders appealed from are hereby set aside, with costs against
appellee.
27.G.R. No. 164687 February 12, 2009
SM PRIME HOLDINGS, INC., Petitioner,
vs.
ANGELA V. MADAYAG, Respondent.
Facts:
Petitioner sought the Department of Environment and Natural
Resources for the suspension of the land registration proceedings filed
by Madayag over seven parcels of land in Urdaneta, Pangasinan.
During the pendency of the petition, SM Prime Holdings filed an Urgent
Motion to Suspend Proceedings, stating that the administrative case is
prejudicial to the land registration case. The RTC concurred with
petitioners claims, citing that since the survey plan is a mandatory
requirement in land registration proceedings, cancellation thereof
would be prejudicial to the petition for land registration. The Court of
Appeals upheld the lower courts ruling.
Issue:
Whether the appellate court acted in grave abuse of discretion in
ordering the suspension of the proceedings.
Ruling:
Yes, there was grave abuse of discretion. The power to stay
proceedings is an incident to the power inherent in every court to
control the disposition of the cases in its dockets, with economy of time
and effort for the court, counsel and litigants. Hence, every order
suspending proceedings must be guided by the following precepts: it
shall be done in order to avoid multiplicity of suits and prevent
vexatious litigations, conflicting judgments, confusion between litigants
and courts, or when the rights of parties to the second action cannot
be properly determined until the questions raised in the first action are
settled.
Furthermore, P.D. No. 1529 eliminated the distinction between general
jurisdiction vested in the RTC and the latters limited jurisdiction when

acting merely as a land registration court. Hence, the court now has
the power to hear and decide controversial cases which involve
substantial issues, such as the case at bar.
Case 7
B. Effect of the Regalian Doctrine: All lands of the public domain belong
to the State which is the source of any asserted right to an asserted
ownership of land. Property of the public domain is beyond the
commerce of man and not susceptible of the private appropriation and
acquisitive prescription.
REPUBLIC VS HEIRS OF LACHICA-SIN
GR. NO. 157485
Facts:
On August 26, 1991, respondent heirs instituted in the RTC of Kalibo,
Aklan a complaint against Lucio Arquisola (Superintendent of ANCF) for
recovery of possession, quieting of title, and declaration of ownership
with damages. Respondent heirs claim that a 41,231-square meterportion of the property they inherited had been usurped by ANCF,
creating a cloud of doubt with respect to their ownership over the
parcel of land they wish to remove from the ANCF reservation.
The ANCF Superintendent countered that the parcel of land being
claimed by respondents was the subject of Proclamation No. 2074 of
then President Ferdinand E. Marcos allocating 24.0551 hectares of land
within the area, which included said portion of private respondents
alleged property, as civil reservation for educational purposes of ANCF.
The ANCF Superintendent averred that the subject parcel of land is
timberland and therefore not susceptible of private ownership.
The RTC remanded the case to the MCTC of New Washington and
Batan, Aklan, in view of the enactment of Republic Act No. 7659 which
expanded the jurisdiction of first-level courts.
On June 19, 2000, the MCTC rendered its Decision in favor of
respondents. The MCTC ruled that the claim of respondent heirs over
the disputed land by virtue of their and their predecessors open,
continuous, exclusive and notorious possession amounts to an
imperfect title, which should be respected and protected.
Petitioner, through the Solicitor General, appealed to the RTC of Kalibo,
Aklan and the RTC rendered its Decision affirming the MCTC Judgment.
Petitioner Republic elevated the case to the Court of Appeals through a
Petition for Review and the Court of Appeals rendered its Decision
dismissing the petition for lack of merit.
Issue:

Whether or not the CA gravely erred on a question of law in upholding


respondents claim to supposed private rights over subject land
despite the DENR certification that it is classified as timberland.
Ruling:
The private right referred to is an alleged imperfect title, which
respondents supposedly acquired by possession of the subject
property, through their predecessors-in-interest, for 30 years before it
was declared as a timberland on December 22, 1960. It must be noted
that respondents have not filed an application for judicial confirmation
of imperfect title under the Public Land Act or the Property Registration
Decree.
The Court held that there are two requisites for judicial confirmation of
imperfect or incomplete title under CA No. 141, namely: (1) open,
continuous, exclusive, and notorious possession and occupation of the
subject land by himself or through his predecessors-in-interest under a
bona fide claim of ownership since time immemorial or from June 12,
1945; and (2) the classification of the land as alienable and disposable
land of the public domain. With respect to the second requisite, the
courts a quo held that the disputed property was alienable and
disposable before 1960, citing petitioners failure to show competent
evidence that the subject land was declared a timberland before its
formal classification as such on said year. In Heirs ofMalabanan vs
Republic, the members of this Court were in disagreement as to
whether lands declared alienable or disposable after June 12, 1945
may be subject to judicial confirmation of imperfect title. There was,
however, no disagreement that there must be a declaration to that
effect.
In the case at bar, it is therefore the respondents which have the
burden to identify a positive act of the government, such as an official
proclamation, declassifying inalienable public land into disposable land
for agricultural or other purposes. Since respondents failed to do so,
the alleged possession by them and by their predecessors-in-interest is
inconsequential and could never ripen into ownership. Accordingly,
respondents cannot be considered to have private rights within the
purview of Proclamation No. 2074 as to prevent the application of said
proclamation to the subject property. The SC grant the prayer of
petitioner Republic to dismiss the civil case for lack of merit.
G. R. No. 156888 November 20, 2006
PEDRO R. SANTIAGO, Petitioner,
vs.
SUBIC BAY METROPOLITAN AUTHORITY, Respondent.
FACTS:
This case stemmed from a Complaint for Recovery of Possession of

Property, filed by Victoria M. Rodriguez, Armando G. Mateo and herein


petitioner Pedro R. Santiago against respondent Subic Bay Metropolitan
Authority
Victoria M. Rodriguez, Armando G. Mateo and petitioner Pedro R.
Santiago, alleged that: Victoria M. Rodriguez is the sole heir and
administrator of the estate of Hermogenes Rodriguez who, in his
lifetime, was the owner of parcels of land registered in his name under
a Spanish title. Victoria leased two parcels of land to Santiago and
Mateo for a period of 50 years. By virtue of the lease, Santiago is
presently occupying the land. SBMA, on the other hand, is claiming
possessory, if not proprietary, rights over the parcels of land, by using
them for its own commercial and other purposes. It is now the desire of
plaintiff Victoria Rodriguez to recover possession of the property from
the defendant so that she could comply with her contractual
commitments to her co-plaintiffs.
RTC: Plaintiffs complaint is anchored on a Spanish title which they
claim is still a valid, subsisting and enforceable title. Despite the fact
that said title was never registered under Act 496, the land
Registration Act (later PD 1529), plaintiffs still claim that they have a
cause of action. The court is not convinced.
Petitioner Santiagos immediate resort is by way of a petition for review
on certiorari under Rule 45 of the Rules of Court.
ISSUE:
Whether or not Spanish Titles are still admissible as evidence of
ownership of lands
RULING:
No. Although PD 892 reads: Whereas, Spanish titles to lands which
have not yet been brought under the operation of the Torrens system,
being subject to Prescription, are now ineffective to prove ownership
unless accompanied by proof of actual possession, petitioners
cannot claim that they can still present the Spanish title as proof of
ownership since they were in actual possession. Actual proof of
possession only becomes necessary because Spanish titles are subject
to prescription. The holder of a Spanish title may still lose his
ownership of the real property to the occupant who actually possesses
the same for the required prescriptive period. Because of this inherent
weakness, the applicant for registration of his Spanish title under the
Torrens system must also submit proof that he is in actual possession
of the real property by virtue of prescription. Taking the law as a whole,
it has clearly set a deadline for the filing of applications for registration
of ALL Spanish titles under the Torrens system (i.e., 6 months from its

affectivity or on 14 August 1976), after which, the Spanish titles may


no longer be presented to prove ownership. However, if such land
registration proceeding was filed and initiated after 14 August 1976,
the applicant could no longer present his Spanish title to the court to
evidence his ownership of the real property, regardless of whether the
real property was in his actual possession.
Therefore, the fact that petitioners were in actual possession of the
property when they filed the complaint with the RTC on April 29, 1996
does not exclude them from the application of PD 892, and their
Spanish title remain inadmissible as evidence of their ownership of the
property, whether in a land registration proceeding or in an action to
remove a cloud on or to quiet title. However, this does not bar holders
of Spanish titles from claiming ownership of real property on some
other basis, such as those provided in PD 1529 or in the Public Land
Act. Petitioners though failed to allege any other basis for their titles in
their Complaint aside from possession of the Subject Property from
time immemorial, which this Court has already controverted; and the
Spanish title, which is already ineffective to prove ownership over the
Subject Property.
For sure, Spanish titles can no longer be countenanced as indubitable
evidence of land ownership. And, without legal or equitable title to the
subject property, Victoria M. Rodriguez, Armando G. Mateo and
petitioner Pedro R. Santiago lacked the personality to claim entitlement
to possession of the same. Title to real property refers to that upon
which ownership is based. It is the evidence of the right of the owner or
the extent of his interest, by which means he can maintain control and,
as a rule, assert right to exclusive possession and enjoyment of the
property.
Therefore, the RTC correctly dismissed the complaint.
5) SECRETARY OF DENR VS. YAP
G.R. NO. 167707
Facts:
The DENR identified several lots in Boracay as being claimed by named
persons. President Ferdinand Marcos issued Proclamation No. 1801
declaring Boracay Island, among other islands, caves and peninsulas
as tourist zones and marine reserves under administration of the
Philippine Tourism Authority. Respondents-claimants alleged that it
raised doubts on their right to secure titles over their occupied lands.
Respondents-claimants posited that Proclamation No. 1801 and its
implementing Circular did not place Boracay beyond the commerce of
man. Since the Island was classified as a tourist zone, it was

susceptible of private ownership. The Office of the Solicitor General


countered that Boracay Island was unclassified land of the public
domain which formed part of the public forest
.
Issue: Whether or not the Proclamation No. 1801 has posed any legal
impediment to the titling of lands in Boracay Islands.
Ruling: The RTC and CA ruled that the Proclamation No. 1801 was not a
legal impediment in the titling of lands in Boracay Islands. The CA held
that respondents-claimants could not be prejudiced by a declaration
that the lands they occupied since time immemorial were part of a
forest reserve.
Proclamation No. 1801 cannot be deemed the positive act needed to
classify Boracay Island as alienable and disposable land. If President
Marcos intended to classify the island as alienable and disposable or
forest, or both, he would have identified the specific limits of each, as
President Arroyo did in Proclamation No. 1064. This was not done in
Proclamation No. 1801.
The Whereas clauses of Proclamation No. 1801 also explain the
rationale behind the declaration of Boracay Island, together with other
islands, caves and peninsulas in the Philippines, as a tourist zone and
marine reserve to be administered by the PTA to ensure the
concentrated efforts of the public and private sectors in the
development of the areas tourism potential with due regard for
ecological balance in the marine environment. Simply put, the
proclamation is aimed at administering the islands for tourism and
ecological purposes. It does not address the areas alienability. Thus
the islands remain as public domain which is governed by the Regalian
doctrine.
CASE NO. 13: Republic of the Philippines vs. Court of Appeals
83 SCRA 453
FACTS:
Prudencio Tesalona died in 1905. He was survived by his two children
Maria and Lucila. The two heirs, without executing an extrajudicial
settlement of Prudencio's estate and adjudicating the said 29-hectare
land to themselves, executed an " absolute sale" of the land in favor of
Tarciana Morales-Maxin, the wife of applicant Prudencio Maxino who
was Maria's son and the grandson of Prudencio Tesalona. In Land
Registration Case No. 81-G of the Court of First Instance at Gumaca,
Quezon, Judge Vicente del Rosario on March 21, 1961 rendered a
decision, ordering the registration of said land, Lot 1, allegedly located
at Barrio Cambuga (Anonang), Mulanay, in the names of the spouses

Prudencio Maxino and Tarciana Morales. The decision became final and
executory. A decree and an original certificate of title were issued.
More than eight years later, or on June 20, 1969, the Republic of the
Philippines filed with the Gumaca court an amended petition to annul
the decision, decree and title on the ground that they are void because
the land in question was still a part of the unclassified public forest.
Moreover, the possessory information title relied upon by the Maxino
spouses covered only 29 hectares of land and not 885 hectares. The
petition was verified by the Acting Director of Forestry.
The Maxinos opposed the petition but it was denied by the Appellate
Court, hence this petition.
ISSUE: Whether or not the registration of the 885 hectares of land is
valid.
RULING:
No. It is incontestable that Lot 1, the 885-hectare area registered by
the Maxinos, is within the public forest, not alienable and disposable
nor susceptible of private appropriation.Its inclusion in the public forest
was certified by Director of Forestry Florencio Tamesis on July 6, 1940,
as per Land Classification Map No. 1386, Tayabas Project No. 16-E of
Mulanay, and as shown in the report and testimony of Lorenzo R. Tria, a
forest station warde. Tria recommended that the title of the Maxino
spouses be annulled.
It is axiomatic that public forestal land is not registerable. Its inclusion
in a title, whether the title be issued during the Spanish regime or
under the Torrens system, nullifies the title. Possession of public
forestal lands, however long, cannot ripen into private ownership.
Thus, the registration of the 885 hectares is not valid.
Case No. 24: Republic v. Reyes, 155 SCRA 313: GR No. L-30263-5
Facts: Godofredo R. Eusebio and Urbano C. Lara filed their Free Patent
Applications for their parcels of land situated in Napindan, Taguig, Rizal
at the Bureau of Lands. The applications were approved and registered
at the Register of Deeds of Sale. In an investigation conducted by the
Anti-Graft and Corrupt Board of the Bureau of Lands it was discovered
that the parcels of land patented were actually under water and form
part of the Laguna de Bay and neither of the respondents were able to
occupy or possess said lots. Respondents admitted their noncompliance with the requirements and agreed to have their patents be
cancelled. The petitioner, represented by the Director of Lands, filed
complaints at the Court of First Instance against Eusebio and Lara.
However, due to the respondents failure to answer the complaint, CFI
of Rizal rendered decisions where the respondents patent numbers
were declared null and void and ordering the Register of Deeds to
cancel the said patent titles. After the lapse of almost 5 years,
respondents filed for the annulment of the decision of the CFI of Rizal

alleging as grounds that the Court had no jurisdiction over their


persons and that the decision was procured through fraud. Director of
Lands had not been properly served with summons and thus, CFI of
Rizal allowed the respondents, now plaintiffs, to adduce their evidence
before the Special Clerk of Court. However, CFI of Rizal again rendered
a decision declaring the patent numbers null and void and ordered the
Director of Lands to reinstate the free patent numbers issued in the
names of the respondent. Notwithstanding the fact that their patents
and certificates were declared null and void, respondents executed
separate deeds of absolute sale involving the alleged lands.
Issue: Whether the land patented is entitled to an absolute deed of
sale.
Held: No. That it is well settled that any title issued on non-disposable
lots even in the hands of an alleged innocent purchaser for value, shall
be cancelled. In the case at bar, the free patents and certificates of
title issued to Eusebio and Lara cover areas which form parts of Laguna
de Bay. These are neither agricultural nor disposable. Subject patents
and titles were erroneously issued due to misrepresentations and false
reports and must therefore be cancelled. Any false statement in an
application for public land shall ipso facto produce the cancellation of
the title granted. This rule applies even after the issuance of the
certificate of title. A certificate of title cannot be used as a shield to
perpetuate fraud, and the doctrine of indefeasibility of torrens title
does not apply to free patent secured through fraud. Likewise, the
Court ruled that mere possession of land does not itself divest the land
of its public character. Void free patents and certificates of title do not
divest the state of its ownership of the land nor operate to change the
public character of the land to private. It is apparent that the law on
innocent purchasers for value does not apply insofar as non-disposable
public lands are concerned.
27.) G.R. No. L-37682 March 29, 1974
REPUBLIC OF THE PHILIPPINES, Represented by the DIRECTOR OF
LANDS, petitioner,
vs.
HON. PEDRO SAMSON ANIMAS, in his capacity as Judge of CFI South
Cotabato, Branch I, General Santos City, ISAGANI DU TIMBOL and the
REGISTER OF DEEDS OF GENERAL SANTOS CITY, respondent.
Facts:
The Republic assailed the decision of the Court of First instance
through the respondent judge which granted the conveyance of a land
to Isagani Timbul. The Court of First Instance, through respondent
judge, averred that the States right to assail had already prescribed
because over one year had already lapsed since the title transfer

pursuant to Section 38 of the Land Registration Act, and that the free
patent issued therefor is indefeasible. The petitioner argued that the
land is a forest land; hence, it is of public dominion and can never be
appropriated.
Issue:
Whether the land, even though issued as a free patent, may be
alienated and disposed.
Ruling:
Yes. The land in question is a forest land, thus it is part of the public
dominion and it may not be disposed and alienated. The title granted
to the private respondent by virtue of the land registration proceeding
is void. It is the Bureau of Forestry which has been vested the sole
prerogative over the demarcation, administration, protection and
occupancy of the countrys forestry. Moreover, since it is the State
which assails the title, prescription may not be invoked against the
same: The Statute of Limitations does not lie against the State.
Case 7
a. Properties of public dominion
REPUBLIC V. CA
131 SCRA 532
Facts:
Respondents Benjamin Tancinco, Azucena Tancinco Reyes, Marina
(should be "Maria") Tancinco Imperial and Mario C. Tancinco are
registered owners of a parcel of land covered by Transfer Certificate of
Title No. T-89709 situated at Barrio Ubihan, Meycauayan, Bulacan
bordering on the Meycauayan and Bocaue rivers.
They filed an application for the registration of three lots adjacent to
their fishpond property but the Bureau of Lands filed a written
opposition to the application for registration.
The private respondents filed a partial withdrawal of the application for
registration with respect to Lot 3 in line with the recommendation of
the Commissioner appointed by the Court, hence it was ordered
withdrawn from the application. and trial proceeded only with respect
to Lots 1 and 2 covered by Plan Psu-131892.
On June 26, 1976, the lower court rendered a decision granting the
application on the finding that the lands in question are accretions to
the private respondents' fishponds covered by Transfer Certificate of

Title No. 89709 however, the petitioner Republic appealed to the


respondent Court of Appeals.
On August, 19, 1982, the respondent Court rendered a decision
affirming in toto the decision of the lower court.
There are facts and circumstances in the record which render
untenable the findings of the trial court and the Court of Appeals that
the lands in question are accretions to the private respondents'
fishponds.
Issue:
Whether the registration of the lots valid.
Ruling:
No, the registration of the lots is not valid.
The lower court cannot validly order the registration of Lots 1 & 2 in
the names of the private respondents. These lots were portions of the
bed of the Meycauayan river and are therefore classified as property of
the public domain under Article 420 paragraph 1 and Article 502,
paragraph 1 of the Civil Code of the Philippines. They are not open to
registration under the Land Registration Act. The adjudication of the
lands in question as private property in the names of the private
respondents is null and void. The only valid conclusion therefore is that
the said areas could not have been there in 1939. They existed only
after the private respondents transferred their dikes towards the bed of
the Meycauayan river in 1951. What private respondents claim as
accretion is really an encroachment of a portion of the Meycauayan
river by reclamation.
# 29
[G.R. No. L-27873. November 29, 1983.]
HEIRS OF JOSE AMUNATEGUI, Petitioners, v. DIRECTOR OF FORESTRY,
Respondent.
Facts:
There were two petitions for review on certiorari questioning the
decision of the Court of Appeals which declared the disputed property
as forest land, not subject to titling in favor of private persons, Borre
and Amunategui.
The Director of Forestry, through the Provincial Fiscal of Capiz, also

filed an opposition to the application for registration of title claiming


that the land was mangrove swamp which was still classified as forest
land and part of the public domain.
Another oppositor, Emeterio Bereber filed his opposition insofar as a
portion of Lot No. 885 containing 117,956 square meters was
concerned and prayed that title to said portion be confirmed and
registered in his name.
During the progress of the trial, applicant-petitioner Roque Borre sold
whatever rights and interests he may have on Lot No. 885 to Angel
Alpasan. The latter also filed an opposition, claiming that he is entitled
to have said lot registered in his name.
Issue:
WON the lot in question can be subject of registration and confirmation
of title in the name of the private person.
Held:
NO. The opposition of the Director of Forestry was strengthened by the
appellate court's finding that timber licenses had to be issued to
certain licensees and even Jose Amunategui himself took the trouble to
ask for a license to cut timber within the area. It was only sometime in
1950 that the property was converted into fishpond but only after a
previous warning from the District Forester that the same could not be
done because it was classified as "public forest.
A forested area classified as forest land of the public domain does not
lose such classification simply because loggers or settlers may have
stripped it of its forest cover. "Forest lands" do not have to be on
mountains or in out of the way places. Swampy areas covered by
mangrove trees, nipa palms, and other trees growing in brackish or sea
water may also be classified as forest land. The classification is
descriptive of its legal nature or status and does not have to be
descriptive of what the land actually looks like. Unless and until the
land classified as "forest" is released in an official proclamation to that
effect so that it may form part of the disposable agricultural lands of
the public domain, the rules on confirmation of imperfect title do not
apply.
The possession of forest lands, no matter how long, cannot ripen into
private ownership. Therefore, the lot in question never ceased to be
classified as forest land of public domain.
5) REPUBLIC V. CA
131 SCRA 532

Facts:
Respondents Benjamin Tancinco, Azucena Tancinco Reyes, Marina
(should be "Maria") Tancinco Imperial and Mario C. Tancinco are
registered owners of a parcel of land covered by Transfer Certificate of
Title No. T-89709 situated at Barrio Ubihan, Meycauayan, Bulacan
bordering on the Meycauayan and Bocaue rivers.
They filed an application for the registration of three lots adjacent to
their fishpond property but the Bureau of Lands filed a written
opposition to the application for registration.
The private respondents filed a partial withdrawal of the application for
registration with respect to Lot 3 in line with the recommendation of
the Commissioner appointed by the Court, hence it was ordered
withdrawn from the application. and trial proceeded only with respect
to Lots 1 and 2 covered by Plan Psu-131892.
On June 26, 1976, the lower court rendered a decision granting the
application on the finding that the lands in question are accretions to
the private respondents' fishponds covered by Transfer Certificate of
Title No. 89709 however, the petitioner Republic appealed to the
respondent Court of Appeals.
On August, 19, 1982, the respondent Court rendered a decision
affirming in toto the decision of the lower court.
There are facts and circumstances in the record which render
untenable the findings of the trial court and the Court of Appeals that
the lands in question are accretions to the private respondents'
fishponds.
Issue:
Whether the registration of the lots valid.
Ruling:
No, the registration of the lots is not valid.
The lower court cannot validly order the registration of Lots 1 & 2 in
the names of the private respondents. These lots were portions of the
bed of the Meycauayan river and are therefore classified as property of
the public domain under Article 420 paragraph 1 and Article 502,
paragraph 1 of the Civil Code of the Philippines. They are not open to

registration under the Land Registration Act. The adjudication of the


lands in question as private property in the names of the private
respondents is null and void. The only valid conclusion therefore is that
the said areas could not have been there in 1939. They existed only
after the private respondents transferred their dikes towards the bed of
the Meycauayan river in 1951. What private respondents claim as
accretion is really an encroachment of a portion of the Meycauayan
river by reclamation.
Case No. 24: Republic v. Sangalang, 159 SCRA 515: GR No. L-58822
Facts: The subject property was inherited by Kiangs from their father,
old man Kiang who in turn inherited the land in possession of his
parents since Spanish times. Old man Kiang filed an application for
registration of the land but it was dismissed by the CFI of Banquet, Mt
Province due to Civil Reservation Case No. L- declaring as public lands
all lands within the limits of the Baguio Townsite Reservation, with the
exception of lands reserved for specific public purposes and those
claimed and adjudicated private property. About 31 years later, Kiangs
filed an application for registration of the parcels of land in question.
The respondent court presided by Judge Pio R. Marcos rendered a
decision adjudicating the land in favor of the respondents. The
republic, represented by the Solicitor General, filed a complaint for the
annulment of the decision of Judge Marcos.
Issue: Whether the Kiangs are entitled for the registration of the land.
Held: No. The exception provided for in the Section 79 of
Commonwealth Act No. 141 refer to lands "claimed by or belonging to
private parties." This exception cannot possibly apply to the
respondents Kiangs since the land which was the subject can no longer
be considered land "claimed by or belonging to private parties." It was
held that the CFI of Baguio and Banquet had no jurisdiction to reopen
the case on the ground that it did not par-take of the nature of
cadastral proceedings as contemplated in Republic Act No. 931, as
amended, and that lands within Government reservations cannot be
registered in favor of private individuals. Recognizing that before the
promulgation of said decision , large portions of the public land within
the Baguio Towns; Reservation had been illegally decreed in favor of
private individuals, PD 1271 was issued intended to protect title
holders who, before the promulgation of the Supreme Court decision
on July 31, 1973, had acted in good faith and relied, although
mistakenly, on the indefeasibility of torrens certificates of titles anal
had introduced substantial improvements on the land covered by said
certificates. The proviso in favor of bona-fide holders of titles issued on
or before July 31, 1973 does not apply to the case at bar, since the

certificate of title of the respondents Kiangs was issued only on June 5,


1975. Neither is there any showing that respondents complied with the
requirements of Section 1 of PD No. 1271 for the validation of their
title.
8. RAMIREZ VS C.A.
144 SCRA 292
Facts:
On September 15,1959, petitioner- spouses Hilario Ramirez and
Valentina Bonifacio filed an application for registration of a parcel of
Riceland in Pamplona, Las Pinas Rizal. The petitioners presented parol
evidence that they acquired the land in question by purchase from
Gregorio Pascual during the early part of the American regime but the
corresponding contract of sale was lost and no copy or record of the
same was available. Thereafter, the court ordered the issuance of OCT
No. 2273 in the petitioners names. On March 30, 1960, the private
respondents filed a petition to review the decree of registration on the
ground of fraud. They alleged, among others, that in 1938 respondents
obtained a loan of P400.00 from the petitioners which they secured
with a mortgage on the land in question by way of antichresis and that
for this reason, Tax Declaration No. 8777 was cancelled and substituted
by Tax Declaration Nos. 9522 and 2385 issued in the names of the
petitioners. In their answer, the spouses Ramirez denied the material
allegations of the petition, they based their claim to the land on two
deeds of sale allegedly executed on April 15, 1937 and April 23, 1937
which they allegedly found accidentally in March 1960. After trial, the
court found that the deeds of sale were spurious, and that the
respondents took possession of the land as owners after the death of
Agapita Bonifacio and in 1938, mortgaged it to the spouses Ramirez to
secure the payment of a loan in the amount of P400.00. It was agreed
that the respondents could not redeem the property within a period of
five years and that the petitioners would take possession of the land,
enjoy its fruits, and pay the land taxes thereon. Finding the claims of
the herein respondents sustained by the evidence, it ordered the
cancellation of Original Certificate of Title No. 2273 of the Register of
Deeds of Rizal in the names of herein petitioners and the issuance in
lieu thereof of another original certificate of title in the names of herein
respondents.
Issue:
Whether or not an antichretic creditor can acquire by prescription the
land surrendered to him by the debtor.
Ruling:
NO. The Court ruled that the issue was submitted to the appellate court
and was correctly resolved therein. The Court of Appeals stated:...The
petition alleged that 'the applicants Hilario Ramirez and Valentina

Bonifacio willfully and fraudulently suppressed the facts that the


petitioners are the legal and rightful owners of the rice field in question
and that they possess the said rice field merely as antichretic creditors
as security for the loan of P400.00; that the applicants are guilty of
fraudulent misrepresentation and concealment when they declared in
their application, in the case at bar, that no other person had any claim
or interest in the said land.' These we believe are sufficient allegations
of extrinsic fraud.
In the applicant's application for registration, which followed the form
required by the Land Registration Act, the applicants alleged that 'to
the best of our knowledge and belief, there is no mortgage or
incumbrance of any kind whatsoever affecting said land, nor any other
person having any estate or interest therein, legal or equitable, in
possession, remainder, reversion or expectancy.' This allegation is false
and made in bad faith, for, as We have found, the applicants are not
the owners of the land sought to be registered and they are in
possession thereof only as antichretic creditors.
Case No. 24: De Los Angeles v. Santos, 12 SCRA 622: GR No. L-19615
Facts: Leonor de los Angeles and seven co-applicants filed an
application for registration of title to 12 parcels of land in Ampid, San
Mateo, Rizal. It is alleged that the applicants are owners pro-indiviso
and in fee simple of the land. The Director of Lands filed an opposition
after the initial hearing stating that the land is a portion of the public
domain. The Province of Rizal also interposed asserting the required
3.00 meters strips of public easement on lots along Ampid river and
creek. Julio Hidalgo also filed an opposition claiming that they are the
lawful owners of the parcels of land in question for having acquired
homestead patents over said lots.
Issue: Whether the court erred in dismissing the application for
registration, over which a homestead patent was issued during the
pendency of the registration proceeding.
Held: Yes. If applicants were to successfully prove the averment and
show their alleged registrable title to the land, it could only result in
the finding that when Julio Hidalgos homestead patent was issued
over on the said lot was no longer public. The land registration court
would have to order a decree of title issued in applicants' favor and
declare the aforesaid homestead patent a nullity which vested no title
in the patentee as against the real owners. Since the existence or nonexistence of applicants' registrable title is decisive of the validity or
nullity of the homestead patent issued as aforestated on said lot the
court a quo's jurisdiction in the land registration proceedings could not
have been divested by the homestead patent's issuance. A homestead

patent, therefore, does not finally dispose of the public or private


character of the land as far as courts upon proceedings in rem are
concerned. Applicants should thus be given opportunity to prove
registrable title to Lot 11.
8. OMICO MINING AND INDUSTRIAL CORPORATION and FREDERICK G.
WEBBER vs. JUDGE AMADOR T. VALLEJOS, in his capacity as Judge of
the Court of First Instance of Cavite, ALFREDO CATOLICO, and
LEONARDO ALCID, in his capacity as City Sheriff of Manila
[G.R. No. L-38974, March 25, 1975]
Facts:
On June 1, 1973, Alfredo Catolico, filed against Omico Mining and
Industrial Corporation and Frederick G. Webber, the latter in his
personal capacity and as President and Chairman of the Board of
Directors of said corporation, alleging first, for the return of ten (10)
certificates of stock of the corporation borrowed from him by the
defendants, and the second, for the payment of his services as legal
counsel for the corporation. Defendants filed a motion to dismiss the
complaint on two grounds: namely (1) improper venue, in that the case
was filed in Cavite where plaintiff is not a resident, the truth being that
he is a resident of Quezon City where he has his permanent family
home; and, as to the second cause of action, the contract of personal
and professional services between plaintiff and defendants was
entered into in the City of Manila, and, therefore, the case should have
been filed in Manila in accordance with Section I of Rule 4 of the
Revised Rules of Court; and (2) lack of cause of action, in that with
regard to the stock certificates the same are in the name of Vicente
Resonda; and, with respect to the contract of personal and professional
services wherein it was agreed that the plaintiff shall head the legal
department of defendant Omico Mining & Industrial Corporation.
On June 16, 1973, the date set for the hearing of the motion to dismiss,
neither the parties nor their respective counsels appeared in court.
While the motion to dismiss was pending resolution by the court
because defendants had not yet presented to the court the required
proof of service, plaintiff, on January 11, 1974, filed a petition to
declare the defendants in default that defendants had been served
with summons and copies of the complaint on June 8, 1973; that as of
January 11, 1974, or after a lapse of seven (7) months from the service
of summons, defendants had not filed their answer to the complaint.
The court granted the petitionand, consequently, it received ex parte
the evidence of the plaintiff and rendered judgment in favor of
Catolico. Defendants filed a motion for reconsideration but Catolico file
a motion to postpone hearing of motion for reconsideration.

On May 31, 1974, while defendants' motion for reconsideration was


still pending before the court because the defendants had not filed yet
their reply to the opposition as they had not received a copy, Catolico
filed a motion for immediate execution of judgment, alleging that said
judgment had already become final and executory because the
defendants failed to have the order of default lifted; that the motion for
reconsideration was filed out of time; that there was a "manifest
attempt on the part of the defendants to delay the proceedings to
afford them an opportunity to have all their assets and shares
dissipated by continuous sale of the same to the prejudice". Thence,
the court denied the defendants motion for reconsideration.
Defendants filed their notice of appeal. On July 22, Pio R. Marcos, as
President and Chairman of the Board of Directors of defendant Omico
Mining and Industrial Corporation, wrote a letter to respondent Sheriff
asking that the defendants be given a little chance to exhaust the legal
remedies available to hold in abeyance the execution and garnishment
for the reasons that defendants were not given a chance to have their
day in court in the motion for immediate execution of judgment and
that they have already appealed from the lower court's decision and
order of immediate execution.

Issue:
Whether respondent Judge acted without or in excess of jurisdiction or
with grave abuse of discretion in declaring the defendants in default, in
receiving plaintiff's evidence ex parte and in rendering judgment.
Held:
The Supreme Court ruled that the respondent Judge acted with grave
abuse of discretion when he declared the petitioners in default. The
motion to dismiss was pending before the court when such declaration
was made, and it is generally irregular to enter an order of default
while a motion to dismiss remains pending and undisposed of. The
irregularity of the order of default is evident from the fact that when
the petitioners were declared in default, their time for filing an answer
had not yet commenced to run anew because on said date, their
counsel had not yet received any notice of the action taken by the
court on their motion to dismiss. There may be cases where the
attendance of certain circumstances "may be considered substantive
enough to truncate the adverse literal application of the pertinent rules
violated." Inasmuch as petitioners were declared in default while their
motion to dismiss was still pending resolution, they were, therefore,

incorrectly declared in default, and the holding of the trial of the case
on the merits, in their absence, without due notice to them of the date
of hearing, was a denial of due process. Consequently, the order of
default, the judgment and the order of execution are patent nullities.

Vous aimerez peut-être aussi